Site Loader

Π‘ΠΎΠ΄Π΅Ρ€ΠΆΠ°Π½ΠΈΠ΅

УравнСния МаксвСлла β€’ ДТСймс Π’Ρ€Π΅Ρ„ΠΈΠ», энциклопСдия «ДвСсти Π·Π°ΠΊΠΎΠ½ΠΎΠ² мироздания»

Шотландский Ρ„ΠΈΠ·ΠΈΠΊ, ΠΎΠ΄ΠΈΠ½ ΠΈΠ· самых Π²Ρ‹Π΄Π°ΡŽΡ‰ΠΈΡ…ΡΡ Ρ‚Π΅ΠΎΡ€Π΅Ρ‚ΠΈΠΊΠΎΠ² XIX столСтия. Родился Π² Π­Π΄ΠΈΠ½Π±ΡƒΡ€Π³Π΅, происходит ΠΈΠ· старинного дворянского Ρ€ΠΎΠ΄Π°. Учился Π² Эдинбургском ΠΈ ΠšΠ΅ΠΌΠ±Ρ€ΠΈΠ΄ΠΆΡΠΊΠΎΠΌ унивСрситСтах. ΠŸΠ΅Ρ€Π²ΡƒΡŽ Π½Π°ΡƒΡ‡Π½ΡƒΡŽ ΡΡ‚Π°Ρ‚ΡŒΡŽ (ΠΎ ΠΌΠ΅Ρ‚ΠΎΠ΄Π΅ начСртания идСального ΠΎΠ²Π°Π»Π°) ΠΎΠΏΡƒΠ±Π»ΠΈΠΊΠΎΠ²Π°Π» Π² возрастС 14Β Π»Π΅Ρ‚. МаксвСлл Π·Π°Π½ΠΈΠΌΠ°Π» Π΄ΠΎΠ»ΠΆΠ½ΠΎΡΡ‚ΡŒ профСссора ΠΊΠ°Ρ„Π΅Π΄Ρ€Ρ‹ ΡΠΊΡΠΏΠ΅Ρ€ΠΈΠΌΠ΅Π½Ρ‚Π°Π»ΡŒΠ½ΠΎΠΉ Ρ„ΠΈΠ·ΠΈΠΊΠΈ ΠšΠ΅ΠΌΠ±Ρ€ΠΈΠ΄ΠΆΡΠΊΠΎΠ³ΠΎ унивСрситСта, ΠΊΠΎΠ³Π΄Π° Π² 48Β Π»Π΅Ρ‚ Π±Π΅Π·Π²Ρ€Π΅ΠΌΠ΅Π½Π½ΠΎ скончался ΠΎΡ‚ Ρ€Π°ΠΊΠ°.

ΠŸΠ΅Ρ€Π²Ρ‹ΠΌ большим тСорСтичСским исслСдованиСм ΠšΠ»Π΅Ρ€ΠΊΠ° МаксвСлла, ΠΊΠ°ΠΊ Π΅Π³ΠΎ часто ΠΈΠΌΠ΅Π½ΡƒΡŽΡ‚, стала Ρ€Π°Π±ΠΎΡ‚Π° ΠΏΠΎ Ρ‚Π΅ΠΎΡ€ΠΈΠΈ Ρ†Π²Π΅Ρ‚Π° ΠΈ Ρ†Π²Π΅Ρ‚Π½ΠΎΠ³ΠΎ зрСния. Он ΠΏΠ΅Ρ€Π²Ρ‹ΠΌ ΠΏΠΎΠΊΠ°Π·Π°Π», Ρ‡Ρ‚ΠΎ вся Π³Π°ΠΌΠΌΠ° Π²ΠΈΠ΄ΠΈΠΌΡ‹Ρ… Ρ†Π²Π΅Ρ‚ΠΎΠ² ΠΌΠΎΠΆΠ΅Ρ‚ Π±Ρ‹Ρ‚ΡŒ ΠΏΠΎΠ»ΡƒΡ‡Π΅Π½Π° ΠΏΡƒΡ‚Π΅ΠΌ смСшСния Ρ‚Ρ€Π΅Ρ… основных Ρ†Π²Π΅Ρ‚ΠΎΠ²Β β€” красного, ΠΆΠ΅Π»Ρ‚ΠΎΠ³ΠΎ ΠΈ синСго; объяснил ΠΏΡ€ΠΈΡ€ΠΎΠ΄Ρƒ Π΄Π°Π»ΡŒΡ‚ΠΎΠ½ΠΈΠ·ΠΌΠ° (Π΄Π΅Ρ„Π΅ΠΊΡ‚Π° зрСния, приводящСго ΠΊ Π½Π°Ρ€ΡƒΡˆΠ΅Π½ΠΈΡŽ восприятия Ρ†Π²Π΅Ρ‚ΠΎΠ²ΠΎΠΉ Π³Π°ΠΌΠΌΡ‹) Π²Ρ€ΠΎΠΆΠ΄Π΅Π½Π½Ρ‹ΠΌ ΠΈΠ»ΠΈ ΠΏΡ€ΠΈΠΎΠ±Ρ€Π΅Ρ‚Π΅Π½Π½Ρ‹ΠΌ Π΄Π΅Ρ„Π΅ΠΊΡ‚ΠΎΠΌ Ρ€Π΅Ρ†Π΅ΠΏΡ‚ΠΎΡ€ΠΎΠ² сСтчатки Π³Π»Π°Π·Π°. Он ΠΏΠ΅Ρ€Π²Ρ‹ΠΌ ΠΈΠ·ΠΎΠ±Ρ€Π΅Π» Ρ€Π΅Π°Π»ΡŒΠ½ΠΎ Ρ€Π°Π±ΠΎΡ‚Π°ΡŽΡ‰ΠΈΠΉ Ρ†Π²Π΅Ρ‚Π½ΠΎΠΉ Ρ„ΠΎΡ‚ΠΎΠ°ΠΏΠΏΠ°Ρ€Π°Ρ‚ (с использованиСм Ρ‚Π°Ρ€Ρ‚Π°Π½ΠΎΠ²ΠΎΠΉ Π»Π΅Π½Ρ‚Ρ‹ Π² качСствС ΡΠ²Π΅Ρ‚ΠΎΡ‡ΡƒΠ²ΡΡ‚Π²ΠΈΡ‚Π΅Π»ΡŒΠ½ΠΎΠ³ΠΎ ΠΌΠ°Ρ‚Π΅Ρ€ΠΈΠ°Π»Π°) ΠΈ продСмонстрировал Π΅Π³ΠΎ Ρ€Π°Π±ΠΎΡ‚Ρƒ Π½Π° собрании Лондонского королСвского общСства Π² 1861Β Π³ΠΎΠ΄Ρƒ.

Как Π±Ρ‹ ΠΌΠ΅ΠΆΠ΄Ρƒ Π΄Π΅Π»ΠΎΠΌ Ρ‚Ρ‰Π°Ρ‚Π΅Π»ΡŒΠ½ΠΎ рассчитал Π²ΠΎΠ·ΠΌΠΎΠΆΠ½ΡƒΡŽ структуру ΠΊΠΎΠ»Π΅Ρ† Π‘Π°Ρ‚ΡƒΡ€Π½Π° ΠΈ Π΄ΠΎΠΊΠ°Π·Π°Π», Ρ‡Ρ‚ΠΎ ΠΎΠ½ΠΈ Π½Π΅ ΠΌΠΎΠ³ΡƒΡ‚ Π±Ρ‹Ρ‚ΡŒ ΠΆΠΈΠ΄ΠΊΠΈΠΌΠΈ, ΠΊΠ°ΠΊ Ρ€Π°Π½Π΅Π΅ ΡΡ‡ΠΈΡ‚Π°Π»ΠΎΡΡŒ, Π° Π΄ΠΎΠ»ΠΆΠ½Ρ‹ ΡΠΎΡΡ‚ΠΎΡΡ‚ΡŒ ΠΈΠ· Ρ‚Π²Π΅Ρ€Π΄Ρ‹Ρ… частиц.

МаксвСлл внСс Π²Π°ΠΆΠ½Ρ‹ΠΉ Π²ΠΊΠ»Π°Π΄ Π² Ρ€Π°Π·Π²ΠΈΡ‚ΠΈΠ΅ ΠΌΠ½ΠΎΠ³ΠΈΡ… отраслСй СстСствознания. Но, ΠΏΠΎΠΆΠ°Π»ΡƒΠΉ, наиваТнСйшСС Π΅Π³ΠΎ достиТСниС состоит Π² Ρ€Π°Π·Π²ΠΈΡ‚ΠΈΠΈ Ρ‚Π΅ΠΎΡ€ΠΈΠΈ элСктромагнСтизма ΠΈ постановкС Π΅Π΅ Π½Π° ΠΏΡ€ΠΎΡ‡Π½ΡƒΡŽ ΠΌΠ°Ρ‚Π΅ΠΌΠ°Ρ‚ΠΈΡ‡Π΅ΡΠΊΡƒΡŽ основу. Π—Π°Π½ΠΈΠΌΠ°Ρ‚ΡŒΡΡ этим вопросом МаксвСлл Π½Π°Ρ‡Π°Π» Π² сСрСдинС 1850-Ρ…Β Π³ΠΎΠ΄ΠΎΠ². По ΠΈΡ€ΠΎΠ½ΠΈΠΈ ΡΡƒΠ΄ΡŒΠ±Ρ‹ МаксвСлл Ρ‚Π²Π΅Ρ€Π΄ΠΎ Π²Π΅Ρ€ΠΈΠ» Π² сущСствованиС свСтоносного эфира, ΠΈ всС свои уравнСния Π²Ρ‹Π²ΠΎΠ΄ΠΈΠ» исходя ΠΈΠ· Ρ‚ΠΎΠ³ΠΎ, Ρ‡Ρ‚ΠΎ эфир сущСствуСт, ΠΈ

Π² Π½Π΅ΠΌ Π²ΠΎΠ·Π±ΡƒΠΆΠ΄Π°ΡŽΡ‚ΡΡ элСктромагнитныС Π²ΠΎΠ»Π½Ρ‹, ΠΈΠΌΠ΅ΡŽΡ‰ΠΈΠ΅, ΠΊΠ°ΠΊ слСдствиС, ΠΊΠΎΠ½Π΅Ρ‡Π½ΡƒΡŽ ΡΠΊΠΎΡ€ΠΎΡΡ‚ΡŒ распространСния. Π”ΠΎ Ρ€Π΅Π·ΡƒΠ»ΡŒΡ‚Π°Ρ‚ΠΎΠ² ΠΎΠΏΡ‹Ρ‚Π° ΠœΠ°ΠΉΠΊΠ΅Π»ΡŒΡΠΎΠ½Π°β€”ΠœΠΎΡ€Π»ΠΈ, ΠΎΠΏΡ€ΠΎΠ²Π΅Ρ€Π³Π°ΡŽΡ‰ΠΈΡ… Ρ‚Π΅ΠΎΡ€ΠΈΡŽ сущСствования эфира, МаксвСлл Π½Π΅ Π΄ΠΎΠΆΠΈΠ». (Как Π½Π΅ Π΄ΠΎΠΆΠΈΠ» ΠΎΠ½ ΠΈ Π΄ΠΎ Π±Π΅Π·ΠΎΠ³ΠΎΠ²ΠΎΡ€ΠΎΡ‡Π½ΠΎΠ³ΠΎ признания своСй Ρ‚Π΅ΠΎΡ€ΠΈΠΈ. ΠžΠΊΠΎΠ½Ρ‡Π°Ρ‚Π΅Π»ΡŒΠ½ΠΎ волновая ΠΏΡ€ΠΈΡ€ΠΎΠ΄Π° свСта ΠΈ ΠΏΡ€Π°Π²ΠΈΠ»ΡŒΠ½ΠΎΡΡ‚ΡŒ ΡƒΡ€Π°Π²Π½Π΅Π½ΠΈΠΉ МаксвСлла Π±Ρ‹Π»ΠΈ ΠΏΠΎΠ΄Ρ‚Π²Π΅Ρ€ΠΆΠ΄Π΅Π½Ρ‹ ΠΎΠΏΡ‹Ρ‚Π°ΠΌΠΈ Π“Π΅Ρ€Ρ†Π° лишь Π²Β 1888Β Π³ΠΎΠ΄Ρƒ, Π° Π΄ΠΎ Ρ‚ΠΎΠ³ΠΎ Π²Ρ€Π΅ΠΌΠ΅Π½ΠΈ Π±ΠΎΠ»ΡŒΡˆΠΈΠ½ΡΡ‚Π²ΠΎ Ρ„ΠΈΠ·ΠΈΠΊΠΎΠ², Π²ΠΊΠ»ΡŽΡ‡Π°Ρ самого Π“Π΅Ρ€Ρ†Π°, с Π½Π΅Π΄ΠΎΠ²Π΅Ρ€ΠΈΠ΅ΠΌ ΠΎΡ‚Π½ΠΎΡΠΈΠ»ΠΈΡΡŒ ΠΊ ΡΡ‚ΠΎΠ»ΡŒ смСлой Ρ‚Π΅ΠΎΡ€ΠΈΠΈ. Β β€” ΠŸΡ€ΠΈΠΌΠ΅Ρ‡Π°Π½ΠΈΠ΅ ΠΏΠ΅Ρ€Π΅Π²ΠΎΠ΄Ρ‡ΠΈΠΊΠ°.) ΠšΒ ΡΡ‡Π°ΡΡ‚ΡŒΡŽ для Π½Π΅Π³ΠΎ ΠΈ для нас, Ρ‚Π΅ΠΎΡ€ΠΈΡŽ МаксвСлла этот ΠΎΠΏΡ‹Ρ‚ Π½Π΅ ΠΎΡ‚ΠΌΠ΅Π½ΠΈΠ», ΠΏΠΎΡΠΊΠΎΠ»ΡŒΠΊΡƒ уравнСния МаксвСлла Π²Ρ‹ΠΏΠΎΠ»Π½ΡΡŽΡ‚ΡΡ нСзависимо ΠΎΡ‚ наличия ΠΈΠ»ΠΈ отсутствия эфира.

НаконСц, МаксвСлл внСс ΠΎΠ³Ρ€ΠΎΠΌΠ½Ρ‹ΠΉ Π²ΠΊΠ»Π°Π΄ Π² становлСниС статистичСской ΠΌΠ΅Ρ…Π°Π½ΠΈΠΊΠΈ, найдя распрСдСлСниС ΠΌΠΎΠ»Π΅ΠΊΡƒΠ» Π³Π°Π·Π° ΠΏΠΎ скоростям, ΡΡ‚Π°Π²ΡˆΠ΅Π΅ ΠΊΡ€Π°Π΅ΡƒΠ³ΠΎΠ»ΡŒΠ½Ρ‹ΠΌ ΠΊΠ°ΠΌΠ½Π΅ΠΌ молСкулярно-кинСтичСской Ρ‚Π΅ΠΎΡ€ΠΈΠΈ. НаконСц, сам ΠΆΠ΅ МаксвСлл ΠΈ ΠΏΠΎΠ΄ΠΌΠ΅Ρ‚ΠΈΠ» Π½Π΅ΡΠΎΠ²Π΅Ρ€ΡˆΠ΅Π½ΡΡ‚Π²ΠΎ этой Ρ‚Π΅ΠΎΡ€ΠΈΠΈ, сформулировав парадокс, ΠΏΠΎΠ·ΠΆΠ΅ ΠΏΠΎΠ»ΡƒΡ‡ΠΈΠ²ΡˆΠΈΠΉ Π½Π°Π·Π²Π°Π½ΠΈΠ΅ Π΄Π΅ΠΌΠΎΠ½Π° МаксвСлла.

ΠŸΠΎΡ‡Π΅ΠΌΡƒ Ρ‚Π΅ΠΎΡ€ΠΈΡŽ МаксвСлла Ρ‚Π°ΠΊ Ρ‚Ρ€ΡƒΠ΄Π½ΠΎ ΠΏΠΎΠ½ΡΡ‚ΡŒ? / Π₯Π°Π±Ρ€

Π‘ΠΊΡ€ΠΎΠΌΠ½ΠΎΡΡ‚ΡŒ Π½Π΅ всСгда Π΄ΠΎΠ±Ρ€ΠΎΠ΄Π΅Ρ‚Π΅Π»ΡŒ

Π’ 1865 Π³ΠΎΠ΄Ρƒ ДТСймс ΠšΠ»Π΅Ρ€ΠΊ МаксвСлл ΠΎΠΏΡƒΠ±Π»ΠΈΠΊΠΎΠ²Π°Π» свою ΡΡ‚Π°Ρ‚ΡŒΡŽ β€œΠ”ΠΈΠ½Π°ΠΌΠΈΡ‡Π΅ΡΠΊΠ°Ρ тСория элСктромагнитного поля» Π² «Π€ΠΈΠ»ΠΎΡΠΎΡ„ских Ρ‚Ρ€ΡƒΠ΄Π°Ρ… ΠšΠΎΡ€ΠΎΠ»Π΅Π²ΡΠΊΠΎΠ³ΠΎ общСства». Π•ΠΌΡƒ Π±Ρ‹Π»ΠΎ Ρ‚ΠΎΠ³Π΄Π° Ρ‚Ρ€ΠΈΠ΄Ρ†Π°Ρ‚ΡŒ Ρ‡Π΅Ρ‚Ρ‹Ρ€Π΅ Π³ΠΎΠ΄Π°. ΠžΠ³Π»ΡΠ΄Ρ‹Π²Π°ΡΡΡŒ Π½Π°Π·Π°Π΄, ΠΌΡ‹ ΠΌΠΎΠΆΠ΅ΠΌ Π·Π°ΠΌΠ΅Ρ‚ΠΈΡ‚ΡŒ, Ρ‡Ρ‚ΠΎ Ρ€Π°Π±ΠΎΡ‚Π° МаксвСлла Π±Ρ‹Π»Π° самым Π²Π°ΠΆΠ½Ρ‹ΠΌ событиСм дСвятнадцатого Π²Π΅ΠΊΠ° Π² истории физичСских Π½Π°ΡƒΠΊ. Если Π³ΠΎΠ²ΠΎΡ€ΠΈΡ‚ΡŒ Π² ΠΎΠ±Ρ‰Π΅ΠΌ ΠΎ СстСствСнных Π½Π°ΡƒΠΊΠ°Ρ…, Ρ‚ΠΎ ΡΡ‚Π°Ρ‚ΡŒΡ МаксвСлла Π±Ρ‹Π»Π° Π²Ρ‚ΠΎΡ€ΠΎΠΉ ΠΏΠΎ значимости послС «ΠŸΡ€ΠΎΠΈΡΡ…оТдСния Π²ΠΈΠ΄ΠΎΠ²» Π”Π°Ρ€Π²ΠΈΠ½Π°. Но Π²Π°ΠΆΠ½ΠΎΡΡ‚ΡŒ Ρ€Π°Π±ΠΎΡ‚ МаксвСлла Π½Π΅ Π±Ρ‹Π»Π° ΠΎΡ‡Π΅Π²ΠΈΠ΄Π½Π° для Π΅Π³ΠΎ соврСмСнников. Π‘ΠΎΠ»Π΅Π΅ Π΄Π²Π°Π΄Ρ†Π°Ρ‚ΠΈ Π»Π΅Ρ‚ Π΅Π³ΠΎ тСория элСктромагнСтизма Π² основном ΠΈΠ³Π½ΠΎΡ€ΠΈΡ€ΠΎΠ²Π°Π»Π°ΡΡŒ. Π€ΠΈΠ·ΠΈΠΊΠ°ΠΌ Π±Ρ‹Π»ΠΎ Ρ‚Ρ€ΡƒΠ΄Π½ΠΎ Π΅Π΅ ΠΏΠΎΠ½ΡΡ‚ΡŒ ΠΈΠ·-Π·Π° обилия слоТных ΡƒΡ€Π°Π²Π½Π΅Π½ΠΈΠΉ. ΠœΠ°Ρ‚Π΅ΠΌΠ°Ρ‚ΠΈΠΊΠ°ΠΌ Π±Ρ‹Π»ΠΎ Ρ‚Ρ€ΡƒΠ΄Π½ΠΎ Π΅Π΅ ΠΏΠΎΠ½ΡΡ‚ΡŒ, ΠΏΠΎΡ‚ΠΎΠΌΡƒ Ρ‡Ρ‚ΠΎ МаксвСлл использовал для объяснСний физичСский язык. Π­Ρ‚ΠΎΡ‚ Ρ‚Ρ€ΡƒΠ΄ Π±Ρ‹Π» расцСнСн ΠΊΠ°ΠΊ нСясноС ΠΏΡ€Π΅Π΄ΠΏΠΎΠ»ΠΎΠΆΠ΅Π½ΠΈΠ΅ Π±Π΅Π· Π΄ΠΎΠ»ΠΆΠ½ΠΎΠ³ΠΎ количСства ΡΠΊΡΠΏΠ΅Ρ€ΠΈΠΌΠ΅Π½Ρ‚Π°Π»ΡŒΠ½Ρ‹Ρ… Π΄ΠΎΠΊΠ°Π·Π°Ρ‚Π΅Π»ΡŒΡΡ‚Π². Π€ΠΈΠ·ΠΈΠΊ ΠœΠΈΡ…Π°ΠΈΠ» ΠŸΡƒΠΏΠΈΠ½ Π² своСй Π°Π²Ρ‚ΠΎΠ±ΠΈΠΎΠ³Ρ€Π°Ρ„ΠΈΠΈ «ΠžΡ‚ ΠΈΠΌΠΌΠΈΠ³Ρ€Π°Π½Ρ‚Π° ΠΊ ΠΈΠ·ΠΎΠ±Ρ€Π΅Ρ‚Π°Ρ‚Π΅Π»ΡŽ» описываСт, ΠΊΠ°ΠΊ ΠΎΠ½ ΠΏΡƒΡ‚Π΅ΡˆΠ΅ΡΡ‚Π²ΠΎΠ²Π°Π» ΠΈΠ· АмСрики Π² Π•Π²Ρ€ΠΎΠΏΡƒ Π² 1883 Π³ΠΎΠ΄Ρƒ Π² поисках Ρ‚ΠΎΠ³ΠΎ, ΠΊΡ‚ΠΎ ΠΏΠΎΠ½ΠΈΠΌΠ°Π» МаксвСлла. Он отправился ΠΈΠ·ΡƒΡ‡Π°Ρ‚ΡŒ Ρ‚Π΅ΠΎΡ€ΠΈΡŽ МаксвСлла, ΠΊΠ°ΠΊ Ρ€Ρ‹Ρ†Π°Ρ€ΡŒ Π² поисках Бвятого Грааля.

ΠŸΡƒΠΏΠΈΠ½ сначала поступил Π² ΠšΠ΅ΠΌΠ±Ρ€ΠΈΠ΄ΠΆ с Ρ‚Π²Π΅Ρ€Π΄Ρ‹ΠΌ Π½Π°ΠΌΠ΅Ρ€Π΅Π½ΠΈΠ΅ΠΌ ΠΈΠ·ΡƒΡ‡ΠΈΡ‚ΡŒ Ρ‚Π΅ΠΎΡ€ΠΈΡŽ Ρƒ самого МаксвСлла. Он Π½Π΅ Π·Π½Π°Π», Ρ‡Ρ‚ΠΎ МаксвСлл ΡƒΠΌΠ΅Ρ€ Ρ‡Π΅Ρ‚Ρ‹Ρ€Π΅ Π³ΠΎΠ΄Π° Π½Π°Π·Π°Π΄. Π£Π·Π½Π°Π², Ρ‡Ρ‚ΠΎ МаксвСлл ΡƒΠΌΠ΅Ρ€, ΠΎΠ½ остался Π² ΠšΠ΅ΠΌΠ±Ρ€ΠΈΠ΄ΠΆΠ΅ ΠΈ Π±Ρ‹Π» Π½Π°Π·Π½Π°Ρ‡Π΅Π½ ΠΏΡ€Π΅ΠΏΠΎΠ΄Π°Π²Π°Ρ‚Π΅Π»Π΅ΠΌ ΠΊΠΎΠ»Π»Π΅Π΄ΠΆΠ°. Но Π΅Π³ΠΎ наставник Π·Π½Π°Π» ΠΎ Ρ‚Π΅ΠΎΡ€ΠΈΠΈ МаксвСлла мСньшС, Ρ‡Π΅ΠΌ ΠΎΠ½ сам, ΠΈ Π±Ρ‹Π» заинтСрСсован Ρ‚ΠΎΠ»ΡŒΠΊΠΎ Π² Ρ‚ΠΎΠΌ, Ρ‡Ρ‚ΠΎΠ±Ρ‹ Π½Π°ΡƒΡ‡ΠΈΡ‚ΡŒ ΠœΠΈΡ…Π°ΠΈΠ»Π° Ρ€Π΅ΡˆΠ°Ρ‚ΡŒ матСматичСскиС Π·Π°Π΄Π°Ρ‡ΠΈ трипоса. ΠœΠΈΡ…Π°ΠΈΠ» ΠŸΡƒΠΏΠΈΠ½ Π±Ρ‹Π» ΠΏΠΎΡ€Π°ΠΆΠ΅Π½, ΠΎΠ±Π½Π°Ρ€ΡƒΠΆΠΈΠ², ΠΊΠ°ΠΊ ΠΎΠ½ Π³ΠΎΠ²ΠΎΡ€ΠΈΡ‚, «ΠΊΠ°ΠΊ ΠΌΠ°Π»ΠΎ Π±Ρ‹Π»ΠΎ Ρ„ΠΈΠ·ΠΈΠΊΠΎΠ², ΠΊΠΎΡ‚ΠΎΡ€Ρ‹Π΅ ΡƒΠ»ΠΎΠ²ΠΈΠ»ΠΈ смысл Ρ‚Π΅ΠΎΡ€ΠΈΠΈ, Π΄Π°ΠΆΠ΅ Ρ‡Π΅Ρ€Π΅Π· Π΄Π²Π°Π΄Ρ†Π°Ρ‚ΡŒ Π»Π΅Ρ‚ послС Ρ‚ΠΎΠ³ΠΎ, ΠΊΠ°ΠΊ ΠΎΠ½Π° Π±Ρ‹Π»Π° сформулирована МаксвСллом Π² 1865 Π³ΠΎΠ΄Ρƒ». Π’ ΠΊΠΎΠ½Ρ†Π΅ ΠΊΠΎΠ½Ρ†ΠΎΠ² ΠΎΠ½ Π±Π΅ΠΆΠ°Π» ΠΈΠ· ΠšΠ΅ΠΌΠ±Ρ€ΠΈΠ΄ΠΆΠ° Π² Π‘Π΅Ρ€Π»ΠΈΠ½ ΠΈ поступил студСнтом ΠΊ Π“Π΅Ρ€ΠΌΠ°Π½Ρƒ Ρ„ΠΎΠ½ Π“Π΅Π»ΡŒΠΌΠ³ΠΎΠ»ΡŒΡ†Ρƒ. Π“Π΅Π»ΡŒΠΌΠ³ΠΎΠ»ΡŒΡ† ΠΏΠΎΠ½ΠΈΠΌΠ°Π» Ρ‚Π΅ΠΎΡ€ΠΈΡŽ ΠΈ ΡƒΡ‡ΠΈΠ» ΠŸΡƒΠΏΠΈΠ½Π° Ρ‚ΠΎΠΌΡƒ, Ρ‡Ρ‚ΠΎ Π·Π½Π°Π» сам. ΠŸΡƒΠΏΠΈΠ½ вСрнулся Π² Нью-Π™ΠΎΡ€ΠΊ, стал профСссором ΠšΠΎΠ»ΡƒΠΌΠ±ΠΈΠΉΡΠΊΠΎΠ³ΠΎ унивСрситСта ΠΈ ΠΎΠ±ΡƒΡ‡Π°Π» ΠΏΠΎΡΠ»Π΅Π΄ΡƒΡŽΡ‰ΠΈΠ΅ поколСния студСнтов, ΠΊΠΎΡ‚ΠΎΡ€Ρ‹Π΅ впослСдствии распространили Π•Π²Π°Π½Π³Π΅Π»ΠΈΠ΅ МаксвСлла ΠΏΠΎ всСй АмСрикС.

ΠžΡ‚ΠΊΡ€Ρ‹Ρ‚ΠΊΠ° ΠΎΡ‚ МаксвСлла ΠŸΠΈΡ‚Π΅Ρ€Ρƒ Π’Π΅ΠΉΡ‚Ρƒ

Как ΡΠ»ΡƒΡ‡ΠΈΠ»ΠΎΡΡŒ, Ρ‡Ρ‚ΠΎ тСория МаксвСлла Π±Ρ‹Π»Π° Ρ‚Π°ΠΊ ΡˆΠΈΡ€ΠΎΠΊΠΎ ΠΏΡ€ΠΎΠΈΠ³Π½ΠΎΡ€ΠΈΡ€ΠΎΠ²Π°Π½Π°? Π’ ΠΊΠΎΠ½Ρ†Π΅ ΠΊΠΎΠ½Ρ†ΠΎΠ², МаксвСлл Π½Π΅ Π±Ρ‹Π» ΠΏΠΎΡ…ΠΎΠΆ Π½Π° своСго соврСмСнника Π“Ρ€Π΅Π³ΠΎΡ€Π° МСндСля, ΠΌΠΎΠ½Π°Ρ…Π°, Ρ€Π°Π±ΠΎΡ‚Π°Π²ΡˆΠ΅Π³ΠΎ Π² бСзвСстном монастырском саду Π² Π‘ΠΎΠ³Π΅ΠΌΠΈΠΈ. МаксвСлл Π±Ρ‹Π» извСстным профСссором, Π΄ΠΈΡ€Π΅ΠΊΡ‚ΠΎΡ€ΠΎΠΌ КавСндишской Π»Π°Π±ΠΎΡ€Π°Ρ‚ΠΎΡ€ΠΈΠΈ Π² ΠšΠ΅ΠΌΠ±Ρ€ΠΈΠ΄ΠΆΠ΅, Π²Π΅Π΄ΡƒΡ‰Π΅ΠΉ Ρ„ΠΈΠ³ΡƒΡ€ΠΎΠΉ Π² британском Π½Π°ΡƒΡ‡Π½ΠΎΠΌ сообщСствС. Π‘Π²ΠΈΠ΄Π΅Ρ‚Π΅Π»ΡŒΡΡ‚Π²ΠΎΠΌ Π΅Π³ΠΎ высокого полоТСния ΠΌΠΎΠΆΠ½ΠΎ ΡΡ‡ΠΈΡ‚Π°Ρ‚ΡŒ Ρ‚ΠΎ, Ρ‡Ρ‚ΠΎ ΠΎΠ½ Π±Ρ‹Π» ΠΏΡ€Π΅Π·ΠΈΠ΄Π΅Π½Ρ‚ΠΎΠΌ сСкции А (матСматичСскиС ΠΈ физичСскиС Π½Π°ΡƒΠΊΠΈ) Британской ассоциации содСйствия Ρ€Π°Π·Π²ΠΈΡ‚ΠΈΡŽ Π½Π°ΡƒΠΊΠΈ, ΠΊΠΎΠ³Π΄Π° ассоциация ΠΏΡ€ΠΎΠ²Π΅Π»Π° своС Π΅ΠΆΠ΅Π³ΠΎΠ΄Π½ΠΎΠ΅ собраниС Π² Π›ΠΈΠ²Π΅Ρ€ΠΏΡƒΠ»Π΅ Π² 1870 Π³ΠΎΠ΄Ρƒ. Он выступил с прСзидСнтской Ρ€Π΅Ρ‡ΡŒΡŽ Π² Π›ΠΈΠ²Π΅Ρ€ΠΏΡƒΠ»Π΅, которая Π±Ρ‹Π»Π° ΠΎΠΏΡƒΠ±Π»ΠΈΠΊΠΎΠ²Π°Π½Π° Π²ΠΎ Π²Ρ‚ΠΎΡ€ΠΎΠΌ Ρ‚ΠΎΠΌΠ΅ Π½Π΅Π΄Π°Π²Π½ΠΎ основанного ΠΆΡƒΡ€Π½Π°Π»Π° «Nature». Π‘Ρ‚ΠΈΠ»ΡŒ Π΅Π³ΠΎ выступлСния ΠΏΠΎΠΊΠ°Π·Ρ‹Π²Π°Π΅Ρ‚ Π½Π°ΠΌ, ΠΏΠΎΡ‡Π΅ΠΌΡƒ Π΅Π³ΠΎ Ρ‚Π΅ΠΎΡ€ΠΈΡŽ Π½Π΅ воспринимали Π²ΡΠ΅Ρ€ΡŒΠ΅Π·. МоТно Π±Ρ‹Π»ΠΎ ΠΎΠΆΠΈΠ΄Π°Ρ‚ΡŒ, Ρ‡Ρ‚ΠΎ ΠΎΠ½ Π²ΠΎΡΠΏΠΎΠ»ΡŒΠ·ΡƒΠ΅Ρ‚ΡΡ Π²ΠΎΠ·ΠΌΠΎΠΆΠ½ΠΎΡΡ‚ΡŒΡŽ, прСдоставлСнной прСзидСнтской ΠΏΠ»Π°Ρ‚Ρ„ΠΎΡ€ΠΌΠΎΠΉ, Ρ‡Ρ‚ΠΎΠ±Ρ‹ ΠΎΠ±ΡŠΡΠ²ΠΈΡ‚ΡŒ ΠΌΠΈΡ€Ρƒ ΠΎ ваТности ΠΎΡ‚ΠΊΡ€Ρ‹Ρ‚ΠΈΠΉ, ΠΊΠΎΡ‚ΠΎΡ€Ρ‹Π΅ ΠΎΠ½ сдСлал ΠΏΡΡ‚ΡŒ Π»Π΅Ρ‚ Π½Π°Π·Π°Π΄. Он Π½Π΅ сдСлал Π½ΠΈΡ‡Π΅Π³ΠΎ ΠΏΠΎΠ΄ΠΎΠ±Π½ΠΎΠ³ΠΎ. Он Π±Ρ‹Π» абсурдно ΠΈ Ρ€Π°Π·Π΄Ρ€Π°ΠΆΠ°ΡŽΡ‰Π΅ скромСн.

МаксвСлл Π² ΠΏΠ΅Ρ€Π²ΡƒΡŽ ΠΎΡ‡Π΅Ρ€Π΅Π΄ΡŒ объявил Ρ‚Π΅ΠΌΡƒ своСго выступлСния — ΠΎΠ±Π·ΠΎΡ€ послСдних достиТСний, ΠΊΠΎΡ‚ΠΎΡ€Ρ‹Π΅ Π±Ρ‹Π»ΠΈ сдСланы Π½Π° Π³Ρ€Π°Π½ΠΈΡ†Π΅ ΠΌΠ΅ΠΆΠ΄Ρƒ ΠΌΠ°Ρ‚Π΅ΠΌΠ°Ρ‚ΠΈΠΊΠΎΠΉ ΠΈ Ρ„ΠΈΠ·ΠΈΠΊΠΎΠΉ. Π—Π°Ρ‚Π΅ΠΌ ΠΎΠ½ с большим энтузиазмом рассказал ΠΎ Π²ΠΈΡ…Ρ€Π΅Π²ΠΎΠΉ Ρ‚Π΅ΠΎΡ€ΠΈΠΈ ΠΌΠΎΠ»Π΅ΠΊΡƒΠ», Π½Π΅Π΄Π°Π²Π½ΠΎ ΠΏΡ€Π΅Π΄Π»ΠΎΠΆΠ΅Π½Π½ΠΎΠΉ сэром Уильямом Вомсоном (впослСдствии ΡΡ‚Π°Π²ΡˆΠΈΠΌ Π»ΠΎΡ€Π΄ΠΎΠΌ КСльвином).

ВСория, ΠΊΠΎΡ‚ΠΎΡ€ΡƒΡŽ сэр Уильям основал Π½Π° Π²Π΅Π»ΠΈΠΊΠΎΠ»Π΅ΠΏΠ½Ρ‹Ρ… гидродинамичСских Ρ‚Π΅ΠΎΡ€Π΅ΠΌΠ°Ρ… Π“Π΅Π»ΡŒΠΌΠ³ΠΎΠ»ΡŒΡ†Π°, ΠΈΡ‰Π΅Ρ‚ свойства ΠΌΠΎΠ»Π΅ΠΊΡƒΠ» Π² ΠΊΠΎΠ»ΡŒΡ†Π΅Π²Ρ‹Ρ… вихрях ΠΎΠ΄Π½ΠΎΡ€ΠΎΠ΄Π½ΠΎΠΉ нСсТимаСмой Тидкости Π±Π΅Π· трСния. Π“Π΅Π»ΡŒΠΌΠ³ΠΎΠ»ΡŒΡ† ΠΏΠΎΠΊΠ°Π·Π°Π», Ρ‡Ρ‚ΠΎ Π² идСальной Тидкости Ρ‚Π°ΠΊΠΎΠ΅ круТащССся ΠΊΠΎΠ»ΡŒΡ†ΠΎ, Ссли ΠΎΠ½ΠΎ ΠΎΠ΄Π½Π°ΠΆΠ΄Ρ‹ Π²ΠΎΠ·Π½ΠΈΠΊΠ»ΠΎ, Π±ΡƒΠ΄Π΅Ρ‚ ΠΏΡ€ΠΎΠ΄ΠΎΠ»ΠΆΠ°Ρ‚ΡŒ ΠΊΡ€ΡƒΠΆΠΈΡ‚ΡŒΡΡ Π²Π΅Ρ‡Π½ΠΎ, всСгда Π±ΡƒΠ΄Π΅Ρ‚ ΡΠΎΡΡ‚ΠΎΡΡ‚ΡŒ ΠΈΠ· Ρ‚ΠΎΠΉ ΠΆΠ΅ самой части Тидкости, которая Π±Ρ‹Π»Π° сначала Π·Π°ΠΊΡ€ΡƒΡ‡Π΅Π½Π°, ΠΈ Π½ΠΈΠΊΠΎΠ³Π΄Π° Π½Π΅ ΠΌΠΎΠΆΠ΅Ρ‚ Π±Ρ‹Ρ‚ΡŒ Ρ€Π°Π·Ρ€Π΅Π·Π°Π½Π° Π½Π°Π΄Π²ΠΎΠ΅ ΠΊΠ°ΠΊΠΎΠΉ-Π»ΠΈΠ±ΠΎ СстСствСнной ΠΏΡ€ΠΈΡ‡ΠΈΠ½ΠΎΠΉ. Π­Ρ‚ΠΈ ΠΊΠΎΠ»ΡŒΡ†Π΅Π²Ρ‹Π΅ Π²ΠΈΡ…Ρ€ΠΈ способны ΠΊ Ρ‚Π°ΠΊΠΈΠΌ Ρ€Π°Π·Π½ΠΎΠΎΠ±Ρ€Π°Π·Π½Ρ‹ΠΌ связям ΠΈ ΡƒΠ·Π»ΠΎΠ²Π°Ρ‚Ρ‹ΠΌ ΡΠ°ΠΌΠΎΠΈΠ½Π²ΠΎΠ»ΡŽΡ†ΠΈΡΠΌ, Ρ‡Ρ‚ΠΎ свойства Ρ€Π°Π·Π»ΠΈΡ‡Π½Ρ‹Ρ… ΡƒΠ·Π»ΠΎΠ²Π°Ρ‚Ρ‹Ρ… Π²ΠΈΡ…Ρ€Π΅ΠΉ Π΄ΠΎΠ»ΠΆΠ½Ρ‹ Π±Ρ‹Ρ‚ΡŒ ΡΡ‚ΠΎΠ»ΡŒ ΠΆΠ΅ Ρ€Π°Π·Π»ΠΈΡ‡Π½Ρ‹, ΠΊΠ°ΠΊ ΠΈ свойства Ρ€Π°Π·Π»ΠΈΡ‡Π½Ρ‹Ρ… Π²ΠΈΠ΄ΠΎΠ² ΠΌΠΎΠ»Π΅ΠΊΡƒΠ».

И Ρ‚Π°ΠΊ Π΄Π°Π»Π΅Π΅. МаксвСлл объяснил, ΠΊΠ°ΠΊ дрСвняя тСория ΠΎ Ρ‚ΠΎΠΌ, Ρ‡Ρ‚ΠΎ матСрия состоит ΠΈΠ· Π°Ρ‚ΠΎΠΌΠΎΠ², ΡΡ‚ΠΎΠ»ΠΊΠ½ΡƒΠ»Π°ΡΡŒ с логичСским парадоксом. Π‘ ΠΎΠ΄Π½ΠΎΠΉ стороны, Π°Ρ‚ΠΎΠΌΡ‹ Π΄ΠΎΠ»ΠΆΠ½Ρ‹ Π±Ρ‹Π»ΠΈ Π±Ρ‹Ρ‚ΡŒ Ρ‚Π²Π΅Ρ€Π΄Ρ‹ΠΌΠΈ, Π½Π΅ΠΏΡ€ΠΎΠ½ΠΈΡ†Π°Π΅ΠΌΡ‹ΠΌΠΈ ΠΈ Π½Π΅Ρ€Π°Π·Ρ€ΡƒΡˆΠΈΠΌΡ‹ΠΌΠΈ. Π‘ Π΄Ρ€ΡƒΠ³ΠΎΠΉ стороны, Π΄Π°Π½Π½Ρ‹Π΅ спСктроскопии ΠΈ Ρ…ΠΈΠΌΠΈΠΈ ΠΏΠΎΠΊΠ°Π·Π°Π»ΠΈ, Ρ‡Ρ‚ΠΎ Π°Ρ‚ΠΎΠΌΡ‹ ΠΈΠΌΠ΅ΡŽΡ‚ Π²Π½ΡƒΡ‚Ρ€Π΅Π½Π½ΡŽΡŽ структуру ΠΈ находятся ΠΏΠΎΠ΄ влияниСм Π²Π½Π΅ΡˆΠ½ΠΈΡ… сил. Π­Ρ‚ΠΎΡ‚ парадокс Π² Ρ‚Π΅Ρ‡Π΅Π½ΠΈΠ΅ ΠΌΠ½ΠΎΠ³ΠΈΡ… Π»Π΅Ρ‚ Π±Π»ΠΎΠΊΠΈΡ€ΠΎΠ²Π°Π» прогрСсс Π² ΠΏΠΎΠ½ΠΈΠΌΠ°Π½ΠΈΠΈ ΠΏΡ€ΠΈΡ€ΠΎΠ΄Ρ‹ ΠΌΠ°Ρ‚Π΅Ρ€ΠΈΠΈ. Π’Π΅ΠΏΠ΅Ρ€ΡŒ, Π½Π°ΠΊΠΎΠ½Π΅Ρ†, вихрСвая тСория ΠΌΠΎΠ»Π΅ΠΊΡƒΠ» Ρ€Π°Π·Ρ€Π΅ΡˆΠΈΠ»Π° парадокс. Π’ΠΈΡ…Ρ€ΠΈ Π² эфирС мягкиС ΠΈ ΠΈΠΌΠ΅ΡŽΡ‚ Π²Π½ΡƒΡ‚Ρ€Π΅Π½Π½ΡŽΡŽ структуру, ΠΈ Ρ‚Π΅ΠΌ Π½Π΅ ΠΌΠ΅Π½Π΅Π΅, согласно Π“Π΅Π»ΡŒΠΌΠ³ΠΎΠ»ΡŒΡ†Ρƒ, ΠΎΠ½ΠΈ ΠΈΠ½Π΄ΠΈΠ²ΠΈΠ΄ΡƒΠ°Π»ΡŒΠ½Ρ‹ ΠΈ Π½Π΅Ρ€Π°Π·Ρ€ΡƒΡˆΠΈΠΌΡ‹. ΠžΡΡ‚Π°Π²Π°Π»ΠΎΡΡŒ Ρ‚ΠΎΠ»ΡŒΠΊΠΎ вывСсти Ρ„Π°ΠΊΡ‚Ρ‹ спСктроскопии ΠΈ Ρ…ΠΈΠΌΠΈΠΈ ΠΈΠ· Π·Π°ΠΊΠΎΠ½ΠΎΠ² взаимодСйствия Π²ΠΈΡ…Ρ€Π΅ΠΉ, прСдсказанных Π³ΠΈΠ΄Ρ€ΠΎΠ΄ΠΈΠ½Π°ΠΌΠΈΠΊΠΎΠΉ идСальной Тидкости. МаксвСлл считал эту Π²ΠΈΡ…Ρ€Π΅Π²ΡƒΡŽ Ρ‚Π΅ΠΎΡ€ΠΈΡŽ ΠΌΠ°Ρ‚Π΅Ρ€ΠΈΠΈ Π·Π°ΠΌΠ΅Ρ‡Π°Ρ‚Π΅Π»ΡŒΠ½Ρ‹ΠΌ ΠΏΡ€ΠΈΠΌΠ΅Ρ€ΠΎΠΌ ΠΏΠ»ΠΎΠ΄ΠΎΡ‚Π²ΠΎΡ€Π½ΠΎΠ³ΠΎ взаимодСйствия ΠΌΠ°Ρ‚Π΅ΠΌΠ°Ρ‚ΠΈΠΊΠΈ ΠΈ Ρ„ΠΈΠ·ΠΈΠΊΠΈ.

НСясно, Π²Π΅Ρ€ΠΈΠ» Π»ΠΈ МаксвСлл Π²ΡΠ΅Ρ€ΡŒΠ΅Π· Π² Ρ‚ΠΎ, Ρ‡Ρ‚ΠΎ Π³ΠΎΠ²ΠΎΡ€ΠΈΠ» ΠΎ Π²ΠΈΡ…Ρ€Π΅Π²ΠΎΠΉ Ρ‚Π΅ΠΎΡ€ΠΈΠΈ. Π’ΠΎΠ·ΠΌΠΎΠΆΠ½ΠΎ, ΠΎΠ½ Ρ…ΠΎΡ‚Π΅Π», Ρ‡Ρ‚ΠΎΠ±Ρ‹ Π΅Π³ΠΎ Ρ€Π΅Ρ‡ΡŒ Ρ€Π°Π·Π²Π»Π΅ΠΊΠ°Π»Π° ΡΠ»ΡƒΡˆΠ°Ρ‚Π΅Π»Π΅ΠΉ, Π° Π½Π΅ просвСщала ΠΈΡ…. Π£ Π½Π΅Π³ΠΎ Π±Ρ‹Π»ΠΎ Ρ…ΠΈΡ‚Ρ€ΠΎΠ΅ чувство ΡŽΠΌΠΎΡ€Π°, ΠΈ Π²ΠΏΠΎΠ»Π½Π΅ Π²ΠΎΠ·ΠΌΠΎΠΆΠ½ΠΎ, Ρ‡Ρ‚ΠΎ ΠΎΠ½ Ρ…Π²Π°Π»ΠΈΠ» Ρ‚Π΅ΠΎΡ€ΠΈΡŽ вихря, зная, Ρ‡Ρ‚ΠΎ Π±ΠΎΠ»Π΅Π΅ ΠΏΡ€ΠΎΠ½ΠΈΡ†Π°Ρ‚Π΅Π»ΡŒΠ½Ρ‹Π΅ Ρ‡Π»Π΅Π½Ρ‹ Π°ΡƒΠ΄ΠΈΡ‚ΠΎΡ€ΠΈΠΈ ΠΏΠΎΠΉΠΌΡƒΡ‚, Ρ‡Ρ‚ΠΎ тСория Π±Ρ‹Π»Π° ΡˆΡƒΡ‚ΠΊΠΎΠΉ. Волько Π² ΠΊΠΎΠ½Ρ†Π΅ своСго выступлСния МаксвСлл ΠΊΡ€Π°Ρ‚ΠΊΠΎ упомянул ΠΎ своСй Ρ‚Π΅ΠΎΡ€ΠΈΠΈ элСктромагнСтизма.

Другая тСория элСктричСства, ΠΊΠΎΡ‚ΠΎΡ€ΡƒΡŽ я ΠΏΡ€Π΅Π΄ΠΏΠΎΡ‡ΠΈΡ‚Π°ΡŽ, ΠΎΡ‚Ρ€ΠΈΡ†Π°Π΅Ρ‚ дСйствиС Π½Π° расстоянии ΠΈ приписываСт элСктричСскоС дСйствиС напряТСниям ΠΈ давлСниям Π²ΠΎ Π²ΡΠ΅ΠΏΡ€ΠΎΠ½ΠΈΠΊΠ°ΡŽΡ‰Π΅ΠΉ срСдС, ΠΏΡ€ΠΈΡ‡Π΅ΠΌ эти напряТСния ΠΎΠ΄ΠΈΠ½Π°ΠΊΠΎΠ²Ρ‹ ΠΏΠΎ Ρ…Π°Ρ€Π°ΠΊΡ‚Π΅Ρ€Ρƒ с Ρ‚Π΅ΠΌΠΈ, ΠΊΠΎΡ‚ΠΎΡ€Ρ‹Π΅ извСстны ΠΈΠ½ΠΆΠ΅Π½Π΅Ρ€Π°ΠΌ, ΠΈ срСда ΠΈΠ΄Π΅Π½Ρ‚ΠΈΡ‡Π½Π° Ρ‚ΠΎΠΉ, Π² ΠΊΠΎΡ‚ΠΎΡ€ΠΎΠΉ прСдполагаСтся распространСниС свСта.

Π€Ρ€Π°Π·Π° «Π”ругая тСория элСктричСства, ΠΊΠΎΡ‚ΠΎΡ€ΡƒΡŽ я ΠΏΡ€Π΅Π΄ΠΏΠΎΡ‡ΠΈΡ‚Π°ΡŽ», каТСтся, Π½Π°ΠΌΠ΅Ρ€Π΅Π½Π½ΠΎ скрываСт Ρ‚ΠΎΡ‚ Ρ„Π°ΠΊΡ‚, Ρ‡Ρ‚ΠΎ это Π±Ρ‹Π»Π° Π΅Π³ΠΎ собствСнная тСория. ΠΠ΅ΡƒΠ΄ΠΈΠ²ΠΈΡ‚Π΅Π»ΡŒΠ½ΠΎ, Ρ‡Ρ‚ΠΎ Π²ΠΈΡ…Ρ€ΠΈ КСльвина ΠΏΡ€ΠΎΠΈΠ·Π²Π΅Π»ΠΈ Π½Π° Π΅Π³ΠΎ ΡΠ»ΡƒΡˆΠ°Ρ‚Π΅Π»Π΅ΠΉ большСС Π²ΠΏΠ΅Ρ‡Π°Ρ‚Π»Π΅Π½ΠΈΠ΅, Ρ‡Π΅ΠΌ уравнСния МаксвСлла.

ΠœΠΎΡ€Π°Π»ΡŒ этой истории Π·Π°ΠΊΠ»ΡŽΡ‡Π°Π΅Ρ‚ΡΡ Π² Ρ‚ΠΎΠΌ, Ρ‡Ρ‚ΠΎ ΡΠΊΡ€ΠΎΠΌΠ½ΠΎΡΡ‚ΡŒ Π½Π΅ всСгда являСтся Π΄ΠΎΠ±Ρ€ΠΎΠ΄Π΅Ρ‚Π΅Π»ΡŒΡŽ. МаксвСлл ΠΈ МСндСль ΠΎΠ±Π° Π±Ρ‹Π»ΠΈ Ρ‡Ρ€Π΅Π·ΠΌΠ΅Ρ€Π½ΠΎ скромны. Π‘ΠΊΡ€ΠΎΠΌΠ½ΠΎΡΡ‚ΡŒ МСндСля Π·Π°Π΄Π΅Ρ€ΠΆΠ°Π»Π° прогрСсс Π±ΠΈΠΎΠ»ΠΎΠ³ΠΈΠΈ Π½Π° ΠΏΡΡ‚ΡŒΠ΄Π΅ΡΡΡ‚ Π»Π΅Ρ‚. Π‘ΠΊΡ€ΠΎΠΌΠ½ΠΎΡΡ‚ΡŒ МаксвСлла Π·Π°ΠΌΠ΅Π΄Π»ΠΈΠ»Π° прогрСсс Ρ„ΠΈΠ·ΠΈΠΊΠΈ Π½Π° Π΄Π²Π°Π΄Ρ†Π°Ρ‚ΡŒ Π»Π΅Ρ‚. Для прогрСсса Π½Π°ΡƒΠΊΠΈ Π±ΡƒΠ΄Π΅Ρ‚ Π»ΡƒΡ‡ΡˆΠ΅, Ссли люди, Π΄Π΅Π»Π°ΡŽΡ‰ΠΈΠ΅ Π²Π΅Π»ΠΈΠΊΠΈΠ΅ открытия, Π½Π΅ Π±ΡƒΠ΄ΡƒΡ‚ слишком скромны, Ρ‡Ρ‚ΠΎΠ±Ρ‹ Ρ‚Ρ€ΡƒΠ±ΠΈΡ‚ΡŒ Π² свои собствСнныС Ρ‚Ρ€ΡƒΠ±Ρ‹. Если Π±Ρ‹ Ρƒ МаксвСлла Π±Ρ‹Π»ΠΎ Ρ‚Π°ΠΊΠΎΠ΅ ΠΆΠ΅ эго, ΠΊΠ°ΠΊ Ρƒ ГалилСя ΠΈΠ»ΠΈ ΠΡŒΡŽΡ‚ΠΎΠ½Π°, ΠΎΠ½ Π±Ρ‹ позаботился ΠΎ Ρ‚ΠΎΠΌ, Ρ‡Ρ‚ΠΎΠ±Ρ‹ Π΅Π³ΠΎ Ρ€Π°Π±ΠΎΡ‚Ρ‹ Π½Π΅ ΠΈΠ³Π½ΠΎΡ€ΠΈΡ€ΠΎΠ²Π°Π»ΠΈΡΡŒ. МаксвСлл Π±Ρ‹Π» Ρ‚Π°ΠΊΠΈΠΌ ΠΆΠ΅ Π²Π΅Π»ΠΈΠΊΠΈΠΌ ΡƒΡ‡Π΅Π½Ρ‹ΠΌ, ΠΊΠ°ΠΊ ΠΡŒΡŽΡ‚ΠΎΠ½, ΠΈ Π³ΠΎΡ€Π°Π·Π΄ΠΎ Π±ΠΎΠ»Π΅Π΅ приятным Ρ‡Π΅Π»ΠΎΠ²Π΅ΠΊΠΎΠΌ. Но, ΠΊ соТалСнию, ΠΎΠ½ Π½Π΅ Π½Π°Ρ‡Π°Π» ΠΏΡ€Π΅Π·ΠΈΠ΄Π΅Π½Ρ‚ΡΠΊΡƒΡŽ Ρ€Π΅Ρ‡ΡŒ Π² Π›ΠΈΠ²Π΅Ρ€ΠΏΡƒΠ»Π΅ словами, ΠΏΠΎΠ΄ΠΎΠ±Π½Ρ‹ΠΌΠΈ Ρ‚Π΅ΠΌ, ΠΊΠΎΡ‚ΠΎΡ€Ρ‹Π΅ ΠΡŒΡŽΡ‚ΠΎΠ½ использовал, Ρ‡Ρ‚ΠΎΠ±Ρ‹ ΠΏΡ€Π΅Π΄ΡΡ‚Π°Π²ΠΈΡ‚ΡŒ Ρ‚Ρ€Π΅Ρ‚ΠΈΠΉ Ρ‚ΠΎΠΌ своСй Principia Mathematica: «… исходя ΠΈΠ· Ρ‚Π΅Ρ… ΠΆΠ΅ ΠΏΡ€ΠΈΠ½Ρ†ΠΈΠΏΠΎΠ², я Ρ‚Π΅ΠΏΠ΅Ρ€ΡŒ Π΄Π΅ΠΌΠΎΠ½ΡΡ‚Ρ€ΠΈΡ€ΡƒΡŽ структуру систСмы ΠΌΠΈΡ€Π°». ΠΡŒΡŽΡ‚ΠΎΠ½ Π½Π΅ Π½Π°Π·Ρ‹Π²Π°Π» свой Π·Π°ΠΊΠΎΠ½ всСмирного тяготСния «ΠΎΡ‡Π΅Ρ€Π΅Π΄Π½ΠΎΠΉ Ρ‚Π΅ΠΎΡ€ΠΈΠ΅ΠΉ тяготСния, ΠΊΠΎΡ‚ΠΎΡ€ΡƒΡŽ я ΠΏΡ€Π΅Π΄ΠΏΠΎΡ‡ΠΈΡ‚Π°ΡŽ».

ВСория МаксвСлла ΠΈ квантовая ΠΌΠ΅Ρ…Π°Π½ΠΈΠΊΠ°

Помимо скромности МаксвСлла, Π±Ρ‹Π»ΠΈ ΠΈ Π΄Ρ€ΡƒΠ³ΠΈΠ΅ ΠΏΡ€ΠΈΡ‡ΠΈΠ½Ρ‹, ΠΏΠΎ ΠΊΠΎΡ‚ΠΎΡ€Ρ‹ΠΌ Π΅Π³ΠΎ Ρ‚Π΅ΠΎΡ€ΠΈΡŽ Π±Ρ‹Π»ΠΎ Ρ‚Ρ€ΡƒΠ΄Π½ΠΎ ΠΏΠΎΠ½ΡΡ‚ΡŒ. Он Π·Π°ΠΌΠ΅Π½ΠΈΠ» Π½ΡŒΡŽΡ‚ΠΎΠ½ΠΎΠ²ΡΠΊΡƒΡŽ Π²ΡΠ΅Π»Π΅Π½Π½ΡƒΡŽ ΠΌΠ°Ρ‚Π΅Ρ€ΠΈΠ°Π»ΡŒΠ½Ρ‹Ρ… ΠΎΠ±ΡŠΠ΅ΠΊΡ‚ΠΎΠ², Π²Π·Π°ΠΈΠΌΠΎΠ΄Π΅ΠΉΡΡ‚Π²ΡƒΡŽΡ‰ΠΈΡ… Π΄Ρ€ΡƒΠ³ с Π΄Ρ€ΡƒΠ³ΠΎΠΌ Π½Π° расстоянии, всСлСнной ΠΏΠΎΠ»Π΅ΠΉ, ΠΏΡ€ΠΎΡΡ‚ΠΈΡ€Π°ΡŽΡ‰ΠΈΡ…ΡΡ Ρ‡Π΅Ρ€Π΅Π· пространство ΠΈ Π²Π·Π°ΠΈΠΌΠΎΠ΄Π΅ΠΉΡΡ‚Π²ΡƒΡŽΡ‰ΠΈΡ… Ρ‚ΠΎΠ»ΡŒΠΊΠΎ локально с ΠΌΠ°Ρ‚Π΅Ρ€ΠΈΠ°Π»ΡŒΠ½Ρ‹ΠΌΠΈ ΠΎΠ±ΡŠΠ΅ΠΊΡ‚Π°ΠΌΠΈ. ΠŸΠΎΠ½ΡΡ‚ΠΈΠ΅ поля Π±Ρ‹Π»ΠΎ Ρ‚Ρ€ΡƒΠ΄Π½ΠΎ ΠΏΠΎΠ½ΡΡ‚ΡŒ, ΠΏΠΎΡ‚ΠΎΠΌΡƒ Ρ‡Ρ‚ΠΎ поля нСосязаСмы. Π£Ρ‡Π΅Π½Ρ‹Π΅ Ρ‚ΠΎΠ³ΠΎ Π²Ρ€Π΅ΠΌΠ΅Π½ΠΈ, Π²ΠΊΠ»ΡŽΡ‡Π°Ρ самого МаксвСлла, ΠΏΡ‹Ρ‚Π°Π»ΠΈΡΡŒ ΠΏΡ€Π΅Π΄ΡΡ‚Π°Π²ΠΈΡ‚ΡŒ поля ΠΊΠ°ΠΊ мСханичСскиС структуры, состоящиС ΠΈΠ· мноТСства ΠΌΠ°Π»Π΅Π½ΡŒΠΊΠΈΡ… колСсиков ΠΈ Π²ΠΈΡ…Ρ€Π΅ΠΉ, ΠΏΡ€ΠΎΡΡ‚ΠΈΡ€Π°ΡŽΡ‰ΠΈΡ…ΡΡ Π² пространствС. Π­Ρ‚ΠΈ структуры Π΄ΠΎΠ»ΠΆΠ½Ρ‹ Π±Ρ‹Π»ΠΈ ΠΏΠ΅Ρ€Π΅Π½ΠΎΡΠΈΡ‚ΡŒ мСханичСскиС напряТСния, ΠΊΠΎΡ‚ΠΎΡ€Ρ‹Π΅ элСктричСскиС ΠΈ ΠΌΠ°Π³Π½ΠΈΡ‚Π½Ρ‹Π΅ поля ΠΏΠ΅Ρ€Π΅Π΄Π°Π²Π°Π»ΠΈ ΠΌΠ΅ΠΆΠ΄Ρƒ элСктричСскими зарядами ΠΈ Ρ‚ΠΎΠΊΠ°ΠΌΠΈ. Π§Ρ‚ΠΎΠ±Ρ‹ поля удовлСтворяли уравнСниям МаксвСлла, систСма колСс ΠΈ Π²ΠΈΡ…Ρ€Π΅ΠΉ Π΄ΠΎΠ»ΠΆΠ½Π° Π±Ρ‹Π»Π° Π±Ρ‹Ρ‚ΡŒ Ρ‡Ρ€Π΅Π·Π²Ρ‹Ρ‡Π°ΠΉΠ½ΠΎ слоТной.

Если ΠΏΠΎΠΏΡ‹Ρ‚Π°Ρ‚ΡŒΡΡ Π²ΠΈΠ·ΡƒΠ°Π»ΠΈΠ·ΠΈΡ€ΠΎΠ²Π°Ρ‚ΡŒ Ρ‚Π΅ΠΎΡ€ΠΈΡŽ МаксвСлла с ΠΏΠΎΠΌΠΎΡ‰ΡŒΡŽ Ρ‚Π°ΠΊΠΈΡ… мСханичСских ΠΌΠΎΠ΄Π΅Π»Π΅ΠΉ, ΠΎΠ½Π° выглядит ΠΊΠ°ΠΊ Π²ΠΎΠ·Π²Ρ€Π°Ρ‚ ΠΊ астрономии ΠŸΡ‚ΠΎΠ»Π΅ΠΌΠ΅Ρ с ΠΏΠ»Π°Π½Π΅Ρ‚Π°ΠΌΠΈ, двиТущимися ΠΏΠΎ Ρ†ΠΈΠΊΠ»Π°ΠΌ ΠΈ эпициклам Π² Π½Π΅Π±Π΅. Π­Ρ‚ΠΎ Π½Π΅ ΠΏΠΎΡ…ΠΎΠΆΠ΅ Π½Π° ΠΈΠ·ΡΡ‰Π½ΡƒΡŽ Π°ΡΡ‚Ρ€ΠΎΠ½ΠΎΠΌΠΈΡŽ ΠΡŒΡŽΡ‚ΠΎΠ½Π°. УравнСния МаксвСлла, записанныС Π² Π½Π΅ΡƒΠΊΠ»ΡŽΠΆΠΈΡ… обозначСниях, ΠΊΠΎΡ‚ΠΎΡ€Ρ‹ΠΌΠΈ пользовался МаксвСлл, Π±Ρ‹Π»ΠΈ ΠΏΡƒΠ³Π°ΡŽΡ‰Π΅ слоТными, Π° Π΅Π³ΠΎ мСханичСскиС ΠΌΠΎΠ΄Π΅Π»ΠΈ — Π΅Ρ‰Π΅ Ρ…ΡƒΠΆΠ΅. Для соврСмСнников тСория МаксвСлла Π±Ρ‹Π»Π° лишь ΠΎΠ΄Π½ΠΎΠΉ ΠΈΠ· ΠΌΠ½ΠΎΠ³ΠΈΡ… Ρ‚Π΅ΠΎΡ€ΠΈΠΉ элСктричСства ΠΈ ΠΌΠ°Π³Π½Π΅Ρ‚ΠΈΠ·ΠΌΠ°. Π•Π΅ Π±Ρ‹Π»ΠΎ Ρ‚Ρ€ΡƒΠ΄Π½ΠΎ ΠΎΡΠΌΡ‹ΡΠ»ΠΈΡ‚ΡŒ, ΠΈ, казалось, Π½Π΅ Π±Ρ‹Π»ΠΎ явного прСимущСства ΠΏΠ΅Ρ€Π΅Π΄ Π΄Ρ€ΡƒΠ³ΠΈΠΌΠΈ тСориями, ΠΊΠΎΡ‚ΠΎΡ€Ρ‹Π΅ описывали элСктричСскиС ΠΈ ΠΌΠ°Π³Π½ΠΈΡ‚Π½Ρ‹Π΅ силы Π² Π½ΡŒΡŽΡ‚ΠΎΠ½ΠΎΠ²ΡΠΊΠΎΠΌ стилС ΠΊΠ°ΠΊ Π΄Π°Π»ΡŒΠ½ΠΎΠ΄Π΅ΠΉΡΡ‚Π²ΠΈΠ΅ ΠΌΠ΅ΠΆΠ΄Ρƒ зарядами ΠΈ ΠΌΠ°Π³Π½ΠΈΡ‚Π°ΠΌΠΈ. ΠΠ΅ΡƒΠ΄ΠΈΠ²ΠΈΡ‚Π΅Π»ΡŒΠ½ΠΎ, Ρ‡Ρ‚ΠΎ ΠΌΠ°Π»ΠΎ ΠΊΡ‚ΠΎ ΠΈΠ· соврСмСнников МаксвСлла ΠΏΡ€ΠΈΠ»Π°Π³Π°Π» усилия, Ρ‡Ρ‚ΠΎΠ±Ρ‹ ΠΈΠ·ΡƒΡ‡ΠΈΡ‚ΡŒ Π΅Π³ΠΎ Ρ‚Π΅ΠΎΡ€ΠΈΡŽ.

ВСория МаксвСлла становится простой ΠΈ понятной Ρ‚ΠΎΠ»ΡŒΠΊΠΎ Ρ‚ΠΎΠ³Π΄Π°, ΠΊΠΎΠ³Π΄Π° Π²Ρ‹ ΠΎΡ‚ΠΊΠ°Π·Ρ‹Π²Π°Π΅Ρ‚Π΅ΡΡŒ ΠΌΡ‹ΡΠ»ΠΈΡ‚ΡŒ Π² Ρ‚Π΅Ρ€ΠΌΠΈΠ½Π°Ρ… мСханичСских ΠΌΠΎΠ΄Π΅Π»Π΅ΠΉ. ВмСсто Ρ‚ΠΎΠ³ΠΎ Ρ‡Ρ‚ΠΎΠ±Ρ‹ Π΄ΡƒΠΌΠ°Ρ‚ΡŒ ΠΎ мСханичСских ΠΎΠ±ΡŠΠ΅ΠΊΡ‚Π°Ρ… ΠΊΠ°ΠΊ ΠΎ ΠΏΠ΅Ρ€Π²ΠΈΡ‡Π½Ρ‹Ρ…, Π° ΠΎΠ± элСктромагнитных напряТСниях ΠΊΠ°ΠΊ ΠΎ Π²Ρ‚ΠΎΡ€ΠΈΡ‡Π½Ρ‹Ρ… слСдствиях, Π²Ρ‹ Π΄ΠΎΠ»ΠΆΠ½Ρ‹ Π΄ΡƒΠΌΠ°Ρ‚ΡŒ ΠΎΠ± элСктромагнитном ΠΏΠΎΠ»Π΅ ΠΊΠ°ΠΊ ΠΎ ΠΏΠ΅Ρ€Π²ΠΈΡ‡Π½ΠΎΠΌ, Π° ΠΎ мСханичСских силах ΠΊΠ°ΠΊ ΠΎ Π²Ρ‚ΠΎΡ€ΠΈΡ‡Π½ΠΎΠΌ конструктС. ΠœΡ‹ΡΠ»ΡŒ ΠΎ Ρ‚ΠΎΠΌ, Ρ‡Ρ‚ΠΎ ΠΏΠ΅Ρ€Π²ΠΈΡ‡Π½Ρ‹ΠΌΠΈ ΡΠΎΡΡ‚Π°Π²Π»ΡΡŽΡ‰ΠΈΠΌΠΈ ВсСлСнной ΡΠ²Π»ΡΡŽΡ‚ΡΡ поля, Π½Π΅ сразу ΠΏΡ€ΠΈΡˆΠ»Π° Π² Π³ΠΎΠ»ΠΎΠ²Ρƒ Ρ„ΠΈΠ·ΠΈΠΊΠ°ΠΌ поколСния МаксвСлла. Поля — это абстрактноС понятиС, Π΄Π°Π»Π΅ΠΊΠΎΠ΅ ΠΎΡ‚ ΠΏΡ€ΠΈΠ²Ρ‹Ρ‡Π½ΠΎΠ³ΠΎ ΠΌΠΈΡ€Π° Π²Π΅Ρ‰Π΅ΠΉ ΠΈ сил. УравнСния поля МаксвСлла ΡΠ²Π»ΡΡŽΡ‚ΡΡ уравнСниями Π² частных ΠΏΡ€ΠΎΠΈΠ·Π²ΠΎΠ΄Π½Ρ‹Ρ…. Они Π½Π΅ ΠΌΠΎΠ³ΡƒΡ‚ Π±Ρ‹Ρ‚ΡŒ Π²Ρ‹Ρ€Π°ΠΆΠ΅Π½Ρ‹ простыми словами, Ρ‚Π°ΠΊΠΈΠΌΠΈ ΠΊΠ°ΠΊ Π·Π°ΠΊΠΎΠ½ двиТСния ΠΡŒΡŽΡ‚ΠΎΠ½Π°: сила Ρ€Π°Π²Π½Π° массС, ΡƒΠΌΠ½ΠΎΠΆΠ΅Π½Π½ΠΎΠΉ Π½Π° ускорСниС. ВСория МаксвСлла Π΄ΠΎΠ»ΠΆΠ½Π° Π±Ρ‹Π»Π° ΠΆΠ΄Π°Ρ‚ΡŒ ΡΠ»Π΅Π΄ΡƒΡŽΡ‰Π΅Π³ΠΎ поколСния Ρ„ΠΈΠ·ΠΈΠΊΠΎΠ², Π“Π΅Ρ€Ρ†Π°, Π›ΠΎΡ€Π΅Π½Ρ†Π° ΠΈ Π­ΠΉΠ½ΡˆΡ‚Π΅ΠΉΠ½Π°, Ρ‡Ρ‚ΠΎΠ±Ρ‹ Ρ€Π°ΡΠΊΡ€Ρ‹Ρ‚ΡŒ свою силу ΠΈ ΠΏΡ€ΠΎΡΡΠ½ΠΈΡ‚ΡŒ свои ΠΊΠΎΠ½Ρ†Π΅ΠΏΡ†ΠΈΠΈ. Π‘Π»Π΅Π΄ΡƒΡŽΡ‰Π΅Π΅ ΠΏΠΎΠΊΠΎΠ»Π΅Π½ΠΈΠ΅ выросло с уравнСниями МаксвСлла ΠΈ чувствовало сСбя ΠΊΠ°ΠΊ Π΄ΠΎΠΌΠ° Π²ΠΎ всСлСнной, построСнной ΠΈΠ· ΠΏΠΎΠ»Π΅ΠΉ. ΠŸΠ΅Ρ€Π²Π΅Π½ΡΡ‚Π²ΠΎ ΠΏΠΎΠ»Π΅ΠΉ Π±Ρ‹Π»ΠΎ Ρ‚Π°ΠΊ ΠΆΠ΅ СстСствСнно для Π­ΠΉΠ½ΡˆΡ‚Π΅ΠΉΠ½Π°, ΠΊΠ°ΠΊ пСрвСнство мСханичСских структур — для МаксвСлла.

https://ddcolrs.wordpress.com/2018/01/17/maxwells-equations-from-20-to-4/

Π‘ΠΎΠ²Ρ€Π΅ΠΌΠ΅Π½Π½Ρ‹ΠΉ взгляд Π½Π° ΠΌΠΈΡ€, возникший ΠΈΠ· Ρ‚Π΅ΠΎΡ€ΠΈΠΈ МаксвСлла, — это ΠΌΠΈΡ€ с двумя слоями. ΠŸΠ΅Ρ€Π²Ρ‹ΠΉ слой, слой Ρ„ΡƒΠ½Π΄Π°ΠΌΠ΅Π½Ρ‚Π°Π»ΡŒΠ½Ρ‹Ρ… ΡΠΎΡΡ‚Π°Π²Π»ΡΡŽΡ‰ΠΈΡ… ΠΌΠΈΡ€Π°, состоит ΠΈΠ· ΠΏΠΎΠ»Π΅ΠΉ, ΡƒΠ΄ΠΎΠ²Π»Π΅Ρ‚Π²ΠΎΡ€ΡΡŽΡ‰ΠΈΡ… простым Π»ΠΈΠ½Π΅ΠΉΠ½Ρ‹ΠΌ уравнСниям. Π’Ρ‚ΠΎΡ€ΠΎΠΉ слой, слой Π²Π΅Ρ‰Π΅ΠΉ, ΠΊΠΎΡ‚ΠΎΡ€Ρ‹Π΅ ΠΌΡ‹ ΠΌΠΎΠΆΠ΅ΠΌ нСпосрСдствСнно ΠΏΠΎΡ‚Ρ€ΠΎΠ³Π°Ρ‚ΡŒ ΠΈ ΠΈΠ·ΠΌΠ΅Ρ€ΠΈΡ‚ΡŒ, состоит ΠΈΠ· мСханичСских напряТСний, энСргий ΠΈ сил. Π­Ρ‚ΠΈ Π΄Π²Π° слоя связаны, ΠΏΠΎΡ‚ΠΎΠΌΡƒ Ρ‡Ρ‚ΠΎ Π²Π΅Π»ΠΈΡ‡ΠΈΠ½Ρ‹ Π²ΠΎ Π²Ρ‚ΠΎΡ€ΠΎΠΌ слоС ΡΠ²Π»ΡΡŽΡ‚ΡΡ ΠΊΠ²Π°Π΄Ρ€Π°Ρ‚ΠΈΡ‡Π½Ρ‹ΠΌΠΈ ΠΈΠ»ΠΈ Π±ΠΈΠ»ΠΈΠ½Π΅ΠΉΠ½Ρ‹ΠΌΠΈ комбинациями Π²Π΅Π»ΠΈΡ‡ΠΈΠ½ Π² ΠΏΠ΅Ρ€Π²ΠΎΠΌ слоС. Π§Ρ‚ΠΎΠ±Ρ‹ Π²Ρ‹Ρ‡ΠΈΡΠ»ΠΈΡ‚ΡŒ энСргии ΠΈΠ»ΠΈ напряТСния, Π²Ρ‹ Π±Π΅Ρ€Π΅Ρ‚Π΅ ΠΊΠ²Π°Π΄Ρ€Π°Ρ‚ напряТСнности элСктричСского поля ΠΈΠ»ΠΈ ΡƒΠΌΠ½ΠΎΠΆΠ°Π΅Ρ‚Π΅ ΠΎΠ΄Π½Ρƒ ΡΠΎΡΡ‚Π°Π²Π»ΡΡŽΡ‰ΡƒΡŽ поля Π½Π° Π΄Ρ€ΡƒΠ³ΡƒΡŽ. Двухслойная структура ΠΌΠΈΡ€Π° — основная ΠΏΡ€ΠΈΡ‡ΠΈΠ½Π°, ΠΏΠΎ ΠΊΠΎΡ‚ΠΎΡ€ΠΎΠΉ тСория МаксвСлла казалась Π·Π°Π³Π°Π΄ΠΎΡ‡Π½ΠΎΠΉ ΠΈ Ρ‚Ρ€ΡƒΠ΄Π½ΠΎΠΉ. ΠžΠ±ΡŠΠ΅ΠΊΡ‚Ρ‹ Π½Π° ΠΏΠ΅Ρ€Π²ΠΎΠΌ ΡƒΡ€ΠΎΠ²Π½Π΅, ΠΎΠ±ΡŠΠ΅ΠΊΡ‚Ρ‹, ΠΊΠΎΡ‚ΠΎΡ€Ρ‹Π΅ Π΄Π΅ΠΉΡΡ‚Π²ΠΈΡ‚Π΅Π»ΡŒΠ½ΠΎ Ρ„ΡƒΠ½Π΄Π°ΠΌΠ΅Π½Ρ‚Π°Π»ΡŒΠ½Ρ‹, ΡΠ²Π»ΡΡŽΡ‚ΡΡ абстракциями, Π½Π΅ доступными нСпосрСдствСнно нашим чувствам. ΠžΠ±ΡŠΠ΅ΠΊΡ‚Ρ‹, ΠΊΠΎΡ‚ΠΎΡ€Ρ‹Π΅ ΠΌΡ‹ ΠΌΠΎΠΆΠ΅ΠΌ Ρ‡ΡƒΠ²ΡΡ‚Π²ΠΎΠ²Π°Ρ‚ΡŒ ΠΈ ΠΎΡΡΠ·Π°Ρ‚ΡŒ, находятся Π½Π° Π²Ρ‚ΠΎΡ€ΠΎΠΌ слоС, ΠΈ ΠΈΡ… ΠΏΠΎΠ²Π΅Π΄Π΅Π½ΠΈΠ΅ лишь косвСнно опрСдСляСтся уравнСниями, Π΄Π΅ΠΉΡΡ‚Π²ΡƒΡŽΡ‰ΠΈΠΌΠΈ Π½Π° ΠΏΠ΅Ρ€Π²ΠΎΠΌ слоС. Двухслойная структура ΠΌΠΈΡ€Π° ΠΏΠΎΠ΄Ρ€Π°Π·ΡƒΠΌΠ΅Π²Π°Π΅Ρ‚, Ρ‡Ρ‚ΠΎ основныС процСссы ΠΏΡ€ΠΈΡ€ΠΎΠ΄Ρ‹ скрыты ΠΎΡ‚ нашСго взгляда.

Π’Π΅ΠΏΠ΅Ρ€ΡŒ ΠΌΡ‹ считаСм само собой Ρ€Π°Π·ΡƒΠΌΠ΅ΡŽΡ‰ΠΈΠΌΡΡ, Ρ‡Ρ‚ΠΎ элСктричСскиС ΠΈ ΠΌΠ°Π³Π½ΠΈΡ‚Π½Ρ‹Π΅ поля ΡΠ²Π»ΡΡŽΡ‚ΡΡ абстракциями, Π½Π΅ сводимыми ΠΊ мСханичСским модСлям. Π§Ρ‚ΠΎΠ±Ρ‹ ΡƒΠ±Π΅Π΄ΠΈΡ‚ΡŒΡΡ Π² этом, достаточно Π²Π·Π³Π»ΡΠ½ΡƒΡ‚ΡŒ Π½Π° Π΅Π΄ΠΈΠ½ΠΈΡ†Ρ‹ измСрСния элСктричСского ΠΈ ΠΌΠ°Π³Π½ΠΈΡ‚Π½ΠΎΠ³ΠΎ ΠΏΠΎΠ»Π΅ΠΉ. Условная Π΅Π΄ΠΈΠ½ΠΈΡ†Π° напряТСнности элСктричСского поля — ΠΊΠ²Π°Π΄Ρ€Π°Ρ‚Π½Ρ‹ΠΉ ΠΊΠΎΡ€Π΅Π½ΡŒ ΠΈΠ· дТоуля Π½Π° кубичСский ΠΌΠ΅Ρ‚Ρ€. Π”ΠΆΠΎΡƒΠ»ΡŒ — это Π΅Π΄ΠΈΠ½ΠΈΡ†Π° энСргии, Π° ΠΌΠ΅Ρ‚Ρ€ — Π΅Π΄ΠΈΠ½ΠΈΡ†Π° Π΄Π»ΠΈΠ½Ρ‹, Π½ΠΎ ΠΊΠ²Π°Π΄Ρ€Π°Ρ‚Π½Ρ‹ΠΉ ΠΊΠΎΡ€Π΅Π½ΡŒ ΠΈΠ· дТоуля — это Π½Π΅ Π΅Π΄ΠΈΠ½ΠΈΡ†Π° Ρ‡Π΅Π³ΠΎ-Ρ‚ΠΎ осязаСмого. ΠœΡ‹ Π½Π΅ ΠΌΠΎΠΆΠ΅ΠΌ ΠΏΡ€Π΅Π΄ΡΡ‚Π°Π²ΠΈΡ‚ΡŒ сСбС, ΠΊΠ°ΠΊ ΠΌΠΎΠΆΠ½ΠΎ ΠΈΠ·ΠΌΠ΅Ρ€ΠΈΡ‚ΡŒ нСпосрСдствСнно ΠΊΠ²Π°Π΄Ρ€Π°Ρ‚Π½Ρ‹ΠΉ ΠΊΠΎΡ€Π΅Π½ΡŒ ΠΈΠ· дТоуля. Π•Π΄ΠΈΠ½ΠΈΡ†Π° измСрСния напряТСнности элСктричСского поля — это матСматичСская абстракция, выбранная Ρ‚Π°ΠΊΠΈΠΌ ΠΎΠ±Ρ€Π°Π·ΠΎΠΌ, Ρ‡Ρ‚ΠΎ ΠΊΠ²Π°Π΄Ρ€Π°Ρ‚ напряТСнности поля Ρ€Π°Π²Π΅Π½ плотности энСргии, ΠΊΠΎΡ‚ΠΎΡ€ΡƒΡŽ ΠΌΠΎΠΆΠ½ΠΎ ΠΈΠ·ΠΌΠ΅Ρ€ΠΈΡ‚ΡŒ с ΠΏΠΎΠΌΠΎΡ‰ΡŒΡŽ Ρ€Π΅Π°Π»ΡŒΠ½Ρ‹Ρ… ΠΏΡ€ΠΈΠ±ΠΎΡ€ΠΎΠ². Π•Π΄ΠΈΠ½ΠΈΡ†Π΅ΠΉ плотности энСргии являСтся Π΄ΠΆΠΎΡƒΠ»ΡŒ Π½Π° кубичСский ΠΌΠ΅Ρ‚Ρ€, ΠΈ поэтому ΠΌΡ‹ Π³ΠΎΠ²ΠΎΡ€ΠΈΠΌ, Ρ‡Ρ‚ΠΎ Π΅Π΄ΠΈΠ½ΠΈΡ†Π΅ΠΉ напряТСнности поля являСтся ΠΊΠ²Π°Π΄Ρ€Π°Ρ‚Π½Ρ‹ΠΉ ΠΊΠΎΡ€Π΅Π½ΡŒ ΠΈΠ· дТоуля Π½Π° кубичСский ΠΌΠ΅Ρ‚Ρ€. Π­Ρ‚ΠΎ Π½Π΅ ΠΎΠ·Π½Π°Ρ‡Π°Π΅Ρ‚, Ρ‡Ρ‚ΠΎ Π½Π°ΠΏΡ€ΡΠΆΠ΅Π½Π½ΠΎΡΡ‚ΡŒ элСктричСского поля ΠΌΠΎΠΆΠ½ΠΎ ΠΈΠ·ΠΌΠ΅Ρ€ΠΈΡ‚ΡŒ с ΠΏΠΎΠΌΠΎΡ‰ΡŒΡŽ ΠΊΠ²Π°Π΄Ρ€Π°Ρ‚Π½ΠΎΠ³ΠΎ корня ΠΊΠ°Π»ΠΎΡ€ΠΈΠΌΠ΅Ρ‚Ρ€Π°. Π­Ρ‚ΠΎ ΠΎΠ·Π½Π°Ρ‡Π°Π΅Ρ‚, Ρ‡Ρ‚ΠΎ Π½Π°ΠΏΡ€ΡΠΆΠ΅Π½Π½ΠΎΡΡ‚ΡŒ элСктричСского поля — абстрактная Π²Π΅Π»ΠΈΡ‡ΠΈΠ½Π°, нСсоизмСримая с Π»ΡŽΠ±Ρ‹ΠΌΠΈ Π²Π΅Π»ΠΈΡ‡ΠΈΠ½Π°ΠΌΠΈ, ΠΊΠΎΡ‚ΠΎΡ€Ρ‹Π΅ ΠΌΡ‹ ΠΌΠΎΠΆΠ΅ΠΌ ΠΈΠ·ΠΌΠ΅Ρ€ΠΈΡ‚ΡŒ нСпосрСдствСнно.

Π§Π΅Ρ€Π΅Π· ΡˆΠ΅ΡΡ‚ΡŒΠ΄Π΅ΡΡΡ‚ Π»Π΅Ρ‚ послС Ρ‚ΠΎΠ³ΠΎ, ΠΊΠ°ΠΊ МаксвСлл ΠΎΠΏΡƒΠ±Π»ΠΈΠΊΠΎΠ²Π°Π» свою Ρ‚Π΅ΠΎΡ€ΠΈΡŽ, Π¨Ρ€Π΅Π΄ΠΈΠ½Π³Π΅Ρ€, Π“Π΅ΠΉΠ·Π΅Π½Π±Π΅Ρ€Π³ ΠΈ Π”ΠΈΡ€Π°ΠΊ ΠΈΠ·ΠΎΠ±Ρ€Π΅Π»ΠΈ ΠΊΠ²Π°Π½Ρ‚ΠΎΠ²ΡƒΡŽ ΠΌΠ΅Ρ…Π°Π½ΠΈΠΊΡƒ. ΠšΠ²Π°Π½Ρ‚ΠΎΠ²Π°Ρ ΠΌΠ΅Ρ…Π°Π½ΠΈΠΊΠ° Π±Ρ‹Π»Π° принята Π³ΠΎΡ€Π°Π·Π΄ΠΎ быстрСС, Ρ‡Π΅ΠΌ тСория МаксвСлла, ΠΏΠΎΡ‚ΠΎΠΌΡƒ Ρ‡Ρ‚ΠΎ ΠΎΠ½Π° сдСлала мноТСство ΠΎΠΏΡ€Π΅Π΄Π΅Π»Π΅Π½Π½Ρ‹Ρ… прСдсказаний ΠΎΠ± Π°Ρ‚ΠΎΠΌΠ½Ρ‹Ρ… процСссах ΠΈ экспСримСнты ΠΏΠΎΠΊΠ°Π·Π°Π»ΠΈ, Ρ‡Ρ‚ΠΎ всС прСдсказания Π±Ρ‹Π»ΠΈ ΠΏΡ€Π°Π²ΠΈΠ»ΡŒΠ½Ρ‹ΠΌΠΈ. Π§Π΅Ρ€Π΅Π· Π³ΠΎΠ΄-Π΄Π²Π° всС ΠΏΠΎΠ²Π΅Ρ€ΠΈΠ»ΠΈ Π² ΠΊΠ²Π°Π½Ρ‚ΠΎΠ²ΡƒΡŽ ΠΌΠ΅Ρ…Π°Π½ΠΈΠΊΡƒ ΠΊΠ°ΠΊ Π² практичСский инструмСнт для расчСта основных процСссов Ρ„ΠΈΠ·ΠΈΠΊΠΈ ΠΈ Ρ…ΠΈΠΌΠΈΠΈ. ΠŸΡ€ΠΈΡ€ΠΎΠ΄Π°, ΠΎΡ‡Π΅Π²ΠΈΠ΄Π½ΠΎ, ΠΏΠΎΠ΄Ρ‡ΠΈΠ½ΡΠ»Π°ΡΡŒ Π·Π°ΠΊΠΎΠ½Π°ΠΌ ΠΊΠ²Π°Π½Ρ‚ΠΎΠ²ΠΎΠΉ ΠΌΠ΅Ρ…Π°Π½ΠΈΠΊΠΈ. Но Π·Π½Π°Ρ‡Π΅Π½ΠΈΠ΅ ΠΊΠ²Π°Π½Ρ‚ΠΎΠ²ΠΎΠΉ ΠΌΠ΅Ρ…Π°Π½ΠΈΠΊΠΈ ΠΎΡΡ‚Π°Π²Π°Π»ΠΎΡΡŒ спорным. Π₯отя квантовая ΠΌΠ΅Ρ…Π°Π½ΠΈΠΊΠ° Π±Ρ‹Π»Π° быстро принята, ΠΎΠ½Π° Π½Π΅ Π±Ρ‹Π»Π° быстро понята. Π Π΅Π·ΠΊΠΈΠ΅ расхоТдСния Π²ΠΎ мнСниях ΠΏΠΎ ΠΏΠΎΠ²ΠΎΠ΄Ρƒ ΠΈΠ½Ρ‚Π΅Ρ€ΠΏΡ€Π΅Ρ‚Π°Ρ†ΠΈΠΈ ΠΊΠ²Π°Π½Ρ‚ΠΎΠ²ΠΎΠΉ ΠΌΠ΅Ρ…Π°Π½ΠΈΠΊΠΈ ΡΠΎΡ…Ρ€Π°Π½ΡΡŽΡ‚ΡΡ Π½Π° протяТСнии сСмидСсяти Π»Π΅Ρ‚.

И ΠΏΠΎΡ‡Π΅ΠΌΡƒ ΠΈΡ… Π½ΠΈΠΊΡ‚ΠΎ Π½Π΅ ΠΏΠΎΠ½ΠΈΠΌΠ°Π»?

ΠŸΡ€ΠΎΡˆΠ»ΠΎ ΠΎΠΊΠΎΠ»ΠΎ Ρ‚Ρ€ΠΈΠ΄Ρ†Π°Ρ‚ΠΈ Π»Π΅Ρ‚ послС МаксвСлла, ΠΏΡ€Π΅ΠΆΠ΄Π΅ Ρ‡Π΅ΠΌ Π΅Π³ΠΎ уравнСния стали понятны всСм. Для достиТСния согласованного понимания ΠΊΠ²Π°Π½Ρ‚ΠΎΠ²ΠΎΠΉ ΠΌΠ΅Ρ…Π°Π½ΠΈΠΊΠΈ потрСбуСтся ΠΏΠΎ мСньшСй ΠΌΠ΅Ρ€Π΅ Π²Π΄Π²ΠΎΠ΅ большС Π²Ρ€Π΅ΠΌΠ΅Π½ΠΈ. ΠœΡ‹ всС Π΅Ρ‰Π΅ Π²Π΅Π΄Π΅ΠΌ страстныС споры ΠΌΠ΅ΠΆΠ΄Ρƒ сторонниками Ρ€Π°Π·Π»ΠΈΡ‡Π½Ρ‹Ρ… ΠΈΠ½Ρ‚Π΅Ρ€ΠΏΡ€Π΅Ρ‚Π°Ρ†ΠΈΠΉ ΠΊΠ²Π°Π½Ρ‚ΠΎΠ²ΠΎΠΉ ΠΌΠ΅Ρ…Π°Π½ΠΈΠΊΠΈ, КопСнгагСнской ΠΈΠ½Ρ‚Π΅Ρ€ΠΏΡ€Π΅Ρ‚Π°Ρ†ΠΈΠΈ, ΠΌΠ½ΠΎΠ³ΠΎΠΌΠΈΡ€ΠΎΠ²ΠΎΠΉ ΠΈΠ½Ρ‚Π΅Ρ€ΠΏΡ€Π΅Ρ‚Π°Ρ†ΠΈΠΈ, Π΄Π΅ΠΊΠΎΠ³Π΅Ρ€Π΅Π½Ρ‚Π½ΠΎΠΉ ΠΈΠ½Ρ‚Π΅Ρ€ΠΏΡ€Π΅Ρ‚Π°Ρ†ΠΈΠΈ, ΠΈΠ½Ρ‚Π΅Ρ€ΠΏΡ€Π΅Ρ‚Π°Ρ†ΠΈΠΉ скрытых ΠΏΠ΅Ρ€Π΅ΠΌΠ΅Π½Π½Ρ‹Ρ… ΠΈ ΠΌΠ½ΠΎΠ³ΠΈΡ… Π΄Ρ€ΡƒΠ³ΠΈΡ…. ΠŸΡ€ΠΈΡ‡ΠΈΠ½Π° этих споров Π·Π°ΠΊΠ»ΡŽΡ‡Π°Π΅Ρ‚ΡΡ Π² Ρ‚ΠΎΠΌ, Ρ‡Ρ‚ΠΎ Ρ€Π°Π·Π»ΠΈΡ‡Π½Ρ‹Π΅ ΠΈΠ½Ρ‚Π΅Ρ€ΠΏΡ€Π΅Ρ‚Π°Ρ‚ΠΎΡ€Ρ‹ ΠΏΡ‹Ρ‚Π°ΡŽΡ‚ΡΡ ΠΎΠΏΠΈΡΠ°Ρ‚ΡŒ ΠΊΠ²Π°Π½Ρ‚ΠΎΠ²Ρ‹ΠΉ ΠΌΠΈΡ€ словами повсСднСвного языка, Π° язык Π½Π΅ ΠΏΠΎΠ΄Ρ…ΠΎΠ΄ΠΈΡ‚ для этой Ρ†Π΅Π»ΠΈ. ΠŸΠΎΠ²ΡΠ΅Π΄Π½Π΅Π²Π½Ρ‹ΠΉ язык описываСт ΠΌΠΈΡ€ Ρ‚Π°ΠΊΠΈΠΌ, ΠΊΠ°ΠΊΠΈΠΌ Π΅Π³ΠΎ видят люди. НашС восприятиС ΠΌΠΈΡ€Π° Ρ†Π΅Π»ΠΈΠΊΠΎΠΌ связано с макроскопичСскими ΠΎΠ±ΡŠΠ΅ΠΊΡ‚Π°ΠΌΠΈ, ΠΊΠΎΡ‚ΠΎΡ€Ρ‹Π΅ Π²Π΅Π΄ΡƒΡ‚ сСбя Π² соотвСтствии с ΠΏΡ€Π°Π²ΠΈΠ»Π°ΠΌΠΈ классичСской Ρ„ΠΈΠ·ΠΈΠΊΠΈ. ВсС понятия, ΠΊΠΎΡ‚ΠΎΡ€Ρ‹Π΅ ΠΏΠΎΡΠ²Π»ΡΡŽΡ‚ΡΡ Π² нашСм языкС, ΡΠ²Π»ΡΡŽΡ‚ΡΡ классичСскими. КаТдая ΠΈΠ· ΠΈΠ½Ρ‚Π΅Ρ€ΠΏΡ€Π΅Ρ‚Π°Ρ†ΠΈΠΉ ΠΊΠ²Π°Π½Ρ‚ΠΎΠ²ΠΎΠΉ ΠΌΠ΅Ρ…Π°Π½ΠΈΠΊΠΈ — это ΠΏΠΎΠΏΡ‹Ρ‚ΠΊΠ° ΠΎΠΏΠΈΡΠ°Ρ‚ΡŒ ΠΊΠ²Π°Π½Ρ‚ΠΎΠ²ΡƒΡŽ ΠΌΠ΅Ρ…Π°Π½ΠΈΠΊΡƒ Π½Π° языкС, Π² ΠΊΠΎΡ‚ΠΎΡ€ΠΎΠΌ ΠΎΡ‚ΡΡƒΡ‚ΡΡ‚Π²ΡƒΡŽΡ‚ ΡΠΎΠΎΡ‚Π²Π΅Ρ‚ΡΡ‚Π²ΡƒΡŽΡ‰ΠΈΠ΅ понятия. Π‘ΠΈΡ‚Π²Ρ‹ ΠΌΠ΅ΠΆΠ΄Ρƒ ΡΠΎΠΏΠ΅Ρ€Π½ΠΈΡ‡Π°ΡŽΡ‰ΠΈΠΌΠΈ интСрпрСтациями ΠΏΡ€ΠΎΠ΄ΠΎΠ»ΠΆΠ°ΡŽΡ‚ΡΡ бСзостановочно, ΠΈ ΠΊΠΎΠ½Ρ†Π° ΠΈΠΌ Π½Π΅ Π²ΠΈΠ΄Π½ΠΎ.

Для понимания ΠΊΠ²Π°Π½Ρ‚ΠΎΠ²ΠΎΠΉ ΠΌΠ΅Ρ…Π°Π½ΠΈΠΊΠΈ ΠΌΠΎΠΆΠ΅Ρ‚ ΠΎΠΊΠ°Π·Π°Ρ‚ΡŒΡΡ ΠΏΠΎΠ»Π΅Π·Π½Ρ‹ΠΌ ΠΏΠΎΠ΄Ρ‡Π΅Ρ€ΠΊΠ½ΡƒΡ‚ΡŒ сходство ΠΌΠ΅ΠΆΠ΄Ρƒ ΠΊΠ²Π°Π½Ρ‚ΠΎΠ²ΠΎΠΉ ΠΌΠ΅Ρ…Π°Π½ΠΈΠΊΠΎΠΉ ΠΈ Ρ‚Π΅ΠΎΡ€ΠΈΠ΅ΠΉ МаксвСлла. Π’ Π΄Π²ΡƒΡ… ΠΎΡ‚Π½ΠΎΡˆΠ΅Π½ΠΈΡΡ… тСория МаксвСлла ΠΌΠΎΠΆΠ΅Ρ‚ Π΄Π°Ρ‚ΡŒ ΠΊΠ»ΡŽΡ‡ ΠΊ Ρ‚Π°ΠΉΠ½Π°ΠΌ ΠΊΠ²Π°Π½Ρ‚ΠΎΠ²ΠΎΠΉ ΠΌΠ΅Ρ…Π°Π½ΠΈΠΊΠΈ.

Π’ΠΎ-ΠΏΠ΅Ρ€Π²Ρ‹Ρ…, ΠΏΠΎΠΏΡ‹Ρ‚ΠΊΠΈ ΠΏΠΎΠ½ΡΡ‚ΡŒ ΠΊΠ²Π°Π½Ρ‚ΠΎΠ²ΡƒΡŽ ΠΌΠ΅Ρ…Π°Π½ΠΈΠΊΡƒ Π² Ρ‚Π΅Ρ€ΠΌΠΈΠ½Π°Ρ… языка, основанного Π½Π° классичСских понятиях, Π°Π½Π°Π»ΠΎΠ³ΠΈΡ‡Π½Ρ‹ ΠΏΠΎΠΏΡ‹Ρ‚ΠΊΠ°ΠΌ ΠΏΠΎΠ½ΡΡ‚ΡŒ Ρ‚Π΅ΠΎΡ€ΠΈΡŽ МаксвСлла Π² Ρ‚Π΅Ρ€ΠΌΠΈΠ½Π°Ρ… мСханичСских ΠΌΠΎΠ΄Π΅Π»Π΅ΠΉ. ВСория МаксвСлла стала изящной ΠΈ понятной Ρ‚ΠΎΠ»ΡŒΠΊΠΎ послС Ρ‚ΠΎΠ³ΠΎ, ΠΊΠ°ΠΊ Π±Ρ‹Π»ΠΈ оставлСны ΠΏΠΎΠΏΡ‹Ρ‚ΠΊΠΈ ΠΏΡ€Π΅Π΄ΡΡ‚Π°Π²ΠΈΡ‚ΡŒ элСктромагнитныС поля с ΠΏΠΎΠΌΠΎΡ‰ΡŒΡŽ мСханичСских ΠΌΠΎΠ΄Π΅Π»Π΅ΠΉ. Π’ΠΎΡ‡Π½ΠΎ Ρ‚Π°ΠΊ ΠΆΠ΅ квантовая ΠΌΠ΅Ρ…Π°Π½ΠΈΠΊΠ° становится изящной ΠΈ понятной Ρ‚ΠΎΠ»ΡŒΠΊΠΎ послС Ρ‚ΠΎΠ³ΠΎ, ΠΊΠ°ΠΊ ΠΏΠΎΠΏΡ‹Ρ‚ΠΊΠΈ ΠΎΠΏΠΈΡΠ°Ρ‚ΡŒ Π΅Π΅ словами ΠΏΡ€Π΅ΠΊΡ€Π°Ρ‰Π°ΡŽΡ‚ΡΡ. Π§Ρ‚ΠΎΠ±Ρ‹ ΡƒΠ²ΠΈΠ΄Π΅Ρ‚ΡŒ красоту Ρ‚Π΅ΠΎΡ€ΠΈΠΈ МаксвСлла, Π½Π΅ΠΎΠ±Ρ…ΠΎΠ΄ΠΈΠΌΠΎ ΠΎΡ‚ΠΎΠΉΡ‚ΠΈ ΠΎΡ‚ мСханичСских ΠΌΠΎΠ΄Π΅Π»Π΅ΠΉ ΠΈ ΠΏΠΎΠ³Ρ€ΡƒΠ·ΠΈΡ‚ΡŒΡΡ Π² абстрактный ΠΌΠΈΡ€ ΠΏΠΎΠ»Π΅ΠΉ. Π§Ρ‚ΠΎΠ±Ρ‹ ΡƒΠ²ΠΈΠ΄Π΅Ρ‚ΡŒ красоту ΠΊΠ²Π°Π½Ρ‚ΠΎΠ²ΠΎΠΉ ΠΌΠ΅Ρ…Π°Π½ΠΈΠΊΠΈ, Π½Π΅ΠΎΠ±Ρ…ΠΎΠ΄ΠΈΠΌΠΎ ΠΎΡ‚ΠΎΠΉΡ‚ΠΈ ΠΎΡ‚ словСсных описаний ΠΈ ΠΏΠΎΠ³Ρ€ΡƒΠ·ΠΈΡ‚ΡŒΡΡ Π² абстрактный ΠΌΠΈΡ€ Π³Π΅ΠΎΠΌΠ΅Ρ‚Ρ€ΠΈΠΈ. ΠœΠ°Ρ‚Π΅ΠΌΠ°Ρ‚ΠΈΠΊΠ° — это язык, Π½Π° ΠΊΠΎΡ‚ΠΎΡ€ΠΎΠΌ Π³ΠΎΠ²ΠΎΡ€ΠΈΡ‚ ΠΏΡ€ΠΈΡ€ΠΎΠ΄Π°. Π―Π·Ρ‹ΠΊ ΠΌΠ°Ρ‚Π΅ΠΌΠ°Ρ‚ΠΈΠΊΠΈ Π΄Π΅Π»Π°Π΅Ρ‚ ΠΌΠΈΡ€ максвСлловских ΠΏΠΎΠ»Π΅ΠΉ ΠΈ ΠΌΠΈΡ€ ΠΊΠ²Π°Π½Ρ‚ΠΎΠ²Ρ‹Ρ… процСссов ΠΎΠ΄ΠΈΠ½Π°ΠΊΠΎΠ²ΠΎ ΠΏΡ€ΠΎΠ·Ρ€Π°Ρ‡Π½Ρ‹ΠΌΠΈ.

Вторая связь ΠΌΠ΅ΠΆΠ΄Ρƒ Ρ‚Π΅ΠΎΡ€ΠΈΠ΅ΠΉ МаксвСлла ΠΈ ΠΊΠ²Π°Π½Ρ‚ΠΎΠ²ΠΎΠΉ ΠΌΠ΅Ρ…Π°Π½ΠΈΠΊΠΎΠΉ Π·Π°ΠΊΠ»ΡŽΡ‡Π°Π΅Ρ‚ΡΡ Π² Π³Π»ΡƒΠ±ΠΎΠΊΠΎΠΌ сходствС структуры. Подобно Ρ‚Π΅ΠΎΡ€ΠΈΠΈ МаксвСлла, квантовая ΠΌΠ΅Ρ…Π°Π½ΠΈΠΊΠ° Π΄Π΅Π»ΠΈΡ‚ Π’ΡΠ΅Π»Π΅Π½Π½ΡƒΡŽ Π½Π° Π΄Π²Π° слоя. ΠŸΠ΅Ρ€Π²Ρ‹ΠΉ слой содСрТит Π²ΠΎΠ»Π½ΠΎΠ²Ρ‹Π΅ Ρ„ΡƒΠ½ΠΊΡ†ΠΈΠΈ Π¨Ρ€Π΅Π΄ΠΈΠ½Π³Π΅Ρ€Π°, ΠΌΠ°Ρ‚Ρ€ΠΈΡ†Ρ‹ Π“Π΅ΠΉΠ·Π΅Π½Π±Π΅Ρ€Π³Π° ΠΈ Π²Π΅ΠΊΡ‚ΠΎΡ€Ρ‹ состояний Π”ΠΈΡ€Π°ΠΊΠ°. Π’Π΅Π»ΠΈΡ‡ΠΈΠ½Ρ‹ Π² ΠΏΠ΅Ρ€Π²ΠΎΠΌ слоС ΠΏΠΎΠ΄Ρ‡ΠΈΠ½ΡΡŽΡ‚ΡΡ простым Π»ΠΈΠ½Π΅ΠΉΠ½Ρ‹ΠΌ уравнСниям. Π˜Ρ… ΠΏΠΎΠ²Π΅Π΄Π΅Π½ΠΈΠ΅ ΠΌΠΎΠΆΠ½ΠΎ Ρ‚ΠΎΡ‡Π½ΠΎ Ρ€Π°ΡΡΡ‡ΠΈΡ‚Π°Ρ‚ΡŒ. Но ΠΈΡ… нСльзя Π½Π°Π±Π»ΡŽΠ΄Π°Ρ‚ΡŒ нСпосрСдствСнно. Π’Ρ‚ΠΎΡ€ΠΎΠΉ слой содСрТит вСроятности столкновСний ΠΈ ΠΏΡ€Π΅Π²Ρ€Π°Ρ‰Π΅Π½ΠΈΠΉ частиц, интСнсивности ΠΈ поляризации излучСния, матСматичСскиС оТидания энСргий ΠΈ спинов частиц. Π’Π΅Π»ΠΈΡ‡ΠΈΠ½Ρ‹ Π²ΠΎ Π²Ρ‚ΠΎΡ€ΠΎΠΌ слоС ΠΌΠΎΠ³ΡƒΡ‚ Π±Ρ‹Ρ‚ΡŒ нСпосрСдствСнно Π½Π°Π±Π»ΡŽΠ΄Π°Π΅ΠΌΡ‹, Π½ΠΎ Π½Π΅ ΠΌΠΎΠ³ΡƒΡ‚ Π±Ρ‹Ρ‚ΡŒ нСпосрСдствСнно вычислСны. Они Π½Π΅ ΠΏΠΎΠ΄Ρ‡ΠΈΠ½ΡΡŽΡ‚ΡΡ простым уравнСниям. Π­Ρ‚ΠΎ Π»ΠΈΠ±ΠΎ ΠΊΠ²Π°Π΄Ρ€Π°Ρ‚Ρ‹ Π²Π΅Π»ΠΈΡ‡ΠΈΠ½ ΠΏΠ΅Ρ€Π²ΠΎΠ³ΠΎ слоя, Π»ΠΈΠ±ΠΎ произвСдСния ΠΎΠ΄Π½ΠΎΠΉ Π²Π΅Π»ΠΈΡ‡ΠΈΠ½Ρ‹ ΠΏΠ΅Ρ€Π²ΠΎΠ³ΠΎ слоя Π½Π° Π΄Ρ€ΡƒΠ³ΡƒΡŽ. Π’ ΠΊΠ²Π°Π½Ρ‚ΠΎΠ²ΠΎΠΉ ΠΌΠ΅Ρ…Π°Π½ΠΈΠΊΠ΅, ΠΊΠ°ΠΊ ΠΈ Π² Ρ‚Π΅ΠΎΡ€ΠΈΠΈ МаксвСлла, ΠŸΡ€ΠΈΡ€ΠΎΠ΄Π° ΠΆΠΈΠ²Π΅Ρ‚ Π² абстрактном матСматичСском ΠΌΠΈΡ€Π΅ ΠΏΠ΅Ρ€Π²ΠΎΠ³ΠΎ слоя, Π½ΠΎ ΠΌΡ‹, люди, ΠΆΠΈΠ²Π΅ΠΌ Π² ΠΊΠΎΠ½ΠΊΡ€Π΅Ρ‚Π½ΠΎΠΌ мСханичСском ΠΌΠΈΡ€Π΅ Π²Ρ‚ΠΎΡ€ΠΎΠ³ΠΎ слоя. ΠœΡ‹ ΠΌΠΎΠΆΠ΅ΠΌ ΠΎΠΏΠΈΡΠ°Ρ‚ΡŒ ΠŸΡ€ΠΈΡ€ΠΎΠ΄Ρƒ Ρ‚ΠΎΠ»ΡŒΠΊΠΎ абстрактным матСматичСским языком, ΠΏΠΎΡ‚ΠΎΠΌΡƒ Ρ‡Ρ‚ΠΎ наш Π²Π΅Ρ€Π±Π°Π»ΡŒΠ½Ρ‹ΠΉ язык находится Π΄ΠΎΠΌΠ° Ρ‚ΠΎΠ»ΡŒΠΊΠΎ Π²ΠΎ Π²Ρ‚ΠΎΡ€ΠΎΠΌ слоС.

Как ΠΈ Π² случаС с Ρ‚Π΅ΠΎΡ€ΠΈΠ΅ΠΉ МаксвСлла, абстрактноС качСство Π²Π΅Π»ΠΈΡ‡ΠΈΠ½ ΠΏΠ΅Ρ€Π²ΠΎΠ³ΠΎ слоя проявляСтся Π² Π΅Π΄ΠΈΠ½ΠΈΡ†Π°Ρ…, Π² ΠΊΠΎΡ‚ΠΎΡ€Ρ‹Ρ… ΠΎΠ½ΠΈ Π²Ρ‹Ρ€Π°ΠΆΠ°ΡŽΡ‚ΡΡ. НапримСр, волновая функция Π¨Ρ€Π΅Π΄ΠΈΠ½Π³Π΅Ρ€Π° выраТаСтся Π² Π΅Π΄ΠΈΠ½ΠΈΡ†Π΅, которая являСтся ΠΊΠ²Π°Π΄Ρ€Π°Ρ‚Π½Ρ‹ΠΌ ΠΊΠΎΡ€Π½Π΅ΠΌ ΠΈΠ· ΠΎΠ±Ρ€Π°Ρ‚Π½ΠΎΠ³ΠΎ кубичСского ΠΌΠ΅Ρ‚Ρ€Π°. Π£ΠΆΠ΅ ΠΎΠ΄ΠΈΠ½ этот Ρ„Π°ΠΊΡ‚ ясно ΠΏΠΎΠΊΠ°Π·Ρ‹Π²Π°Π΅Ρ‚, Ρ‡Ρ‚ΠΎ волновая функция — это абстракция, навсСгда скрытая ΠΎΡ‚ нашСго взгляда. Никто Π½ΠΈΠΊΠΎΠ³Π΄Π° Π½Π΅ ΠΈΠ·ΠΌΠ΅Ρ€ΠΈΡ‚ Π½Π°ΠΏΡ€ΡΠΌΡƒΡŽ ΠΊΠ²Π°Π΄Ρ€Π°Ρ‚Π½Ρ‹ΠΉ ΠΊΠΎΡ€Π΅Π½ΡŒ ΠΈΠ· кубичСского ΠΌΠ΅Ρ‚Ρ€Π°. ΠšΠΎΠ½Π΅Ρ‡Π½Π°Ρ Π²Π°ΠΆΠ½ΠΎΡΡ‚ΡŒ Ρ‚Π΅ΠΎΡ€ΠΈΠΈ МаксвСлла Π³ΠΎΡ€Π°Π·Π΄ΠΎ большС, Ρ‡Π΅ΠΌ Π΅Π΅ нСпосрСдствСнныС достиТСния Π² объяснСнии ΠΈ объСдинСнии явлСний элСктричСства ΠΈ ΠΌΠ°Π³Π½Π΅Ρ‚ΠΈΠ·ΠΌΠ°. Π•Π³ΠΎ конСчная Π²Π°ΠΆΠ½ΠΎΡΡ‚ΡŒ состоит Π² Ρ‚ΠΎΠΌ, Ρ‡Ρ‚ΠΎΠ±Ρ‹ ΡΡ‚Π°Ρ‚ΡŒ ΠΏΡ€ΠΎΡ‚ΠΎΡ‚ΠΈΠΏΠΎΠΌ для всСх Π²Π΅Π»ΠΈΠΊΠΈΡ… Ρ‚Ρ€ΠΈΡƒΠΌΡ„ΠΎΠ² Ρ„ΠΈΠ·ΠΈΠΊΠΈ Π΄Π²Π°Π΄Ρ†Π°Ρ‚ΠΎΠ³ΠΎ Π²Π΅ΠΊΠ°. Π­Ρ‚ΠΎ ΠΏΡ€ΠΎΡ‚ΠΎΡ‚ΠΈΠΏ Ρ‚Π΅ΠΎΡ€ΠΈΠΈ ΠΎΡ‚Π½ΠΎΡΠΈΡ‚Π΅Π»ΡŒΠ½ΠΎΡΡ‚ΠΈ Π­ΠΉΠ½ΡˆΡ‚Π΅ΠΉΠ½Π°, ΠΊΠ²Π°Π½Ρ‚ΠΎΠ²ΠΎΠΉ ΠΌΠ΅Ρ…Π°Π½ΠΈΠΊΠΈ, Ρ‚Π΅ΠΎΡ€ΠΈΠΈ ΠΎΠ±ΠΎΠ±Ρ‰Π΅Π½Π½ΠΎΠΉ ΠΊΠ°Π»ΠΈΠ±Ρ€ΠΎΠ²ΠΎΡ‡Π½ΠΎΠΉ инвариантности Π―Π½Π³Π°-Миллса ΠΈ Π΅Π΄ΠΈΠ½ΠΎΠΉ Ρ‚Π΅ΠΎΡ€ΠΈΠΈ ΠΏΠΎΠ»Π΅ΠΉ ΠΈ частиц.

ВсС эти Ρ‚Π΅ΠΎΡ€ΠΈΠΈ основаны Π½Π° ΠΊΠΎΠ½Ρ†Π΅ΠΏΡ†ΠΈΠΈ динамичСских ΠΏΠΎΠ»Π΅ΠΉ, Π²Π²Π΅Π΄Π΅Π½Π½ΠΎΠΉ МаксвСллом Π² 1865 Π³ΠΎΠ΄Ρƒ. ВсС ΠΎΠ½ΠΈ ΠΈΠΌΠ΅ΡŽΡ‚ ΠΎΠ΄ΠΈΠ½Π°ΠΊΠΎΠ²ΡƒΡŽ Π΄Π²ΡƒΡ…ΡΠ»ΠΎΠΉΠ½ΡƒΡŽ структуру, ΠΎΡ‚Π΄Π΅Π»ΡΡŽΡ‰ΡƒΡŽ ΠΌΠΈΡ€ простых динамичСских ΡƒΡ€Π°Π²Π½Π΅Π½ΠΈΠΉ ΠΎΡ‚ ΠΌΠΈΡ€Π° чСловСчСского наблюдСния. ВсС ΠΎΠ½ΠΈ Π²ΠΎΠΏΠ»ΠΎΡ‰Π°ΡŽΡ‚ Π² сСбС Ρ‚ΠΎ ΠΆΠ΅ качСство матСматичСской абстракции, ΠΊΠΎΡ‚ΠΎΡ€ΠΎΠ΅ сдСлало Ρ‚Π΅ΠΎΡ€ΠΈΡŽ МаксвСлла Ρ‚Ρ€ΡƒΠ΄Π½ΠΎΠΉ для понимания Π΅Π³ΠΎ соврСмСнниками. ΠœΡ‹ ΠΌΠΎΠΆΠ΅ΠΌ Π½Π°Π΄Π΅ΡΡ‚ΡŒΡΡ, Ρ‡Ρ‚ΠΎ Π³Π»ΡƒΠ±ΠΎΠΊΠΎΠ΅ ΠΏΠΎΠ½ΠΈΠΌΠ°Π½ΠΈΠ΅ Ρ‚Π΅ΠΎΡ€ΠΈΠΈ МаксвСлла ΠΏΡ€ΠΈΠ²Π΅Π΄Π΅Ρ‚ ΠΊ Ρ€Π°ΡΡΠ΅ΠΈΠ²Π°Π½ΠΈΡŽ Ρ‚ΡƒΠΌΠ°Π½Π° нСпонимания, ΠΊΠΎΡ‚ΠΎΡ€Ρ‹ΠΉ всС Π΅Ρ‰Π΅ ΠΎΠΊΡ€ΡƒΠΆΠ°Π΅Ρ‚ ΠΈΠ½Ρ‚Π΅Ρ€ΠΏΡ€Π΅Ρ‚Π°Ρ†ΠΈΡŽ ΠΊΠ²Π°Π½Ρ‚ΠΎΠ²ΠΎΠΉ ΠΌΠ΅Ρ…Π°Π½ΠΈΠΊΠΈ. И ΠΌΡ‹ ΠΌΠΎΠΆΠ΅ΠΌ Π½Π°Π΄Π΅ΡΡ‚ΡŒΡΡ, Ρ‡Ρ‚ΠΎ Π³Π»ΡƒΠ±ΠΎΠΊΠΎΠ΅ ΠΏΠΎΠ½ΠΈΠΌΠ°Π½ΠΈΠ΅ Ρ‚Π΅ΠΎΡ€ΠΈΠΈ МаксвСлла ΠΏΠΎΠΌΠΎΠΆΠ΅Ρ‚ ΠΏΡ€ΠΎΠ»ΠΎΠΆΠΈΡ‚ΡŒ ΠΏΡƒΡ‚ΡŒ ΠΊ дальнСйшим Ρ‚Ρ€ΠΈΡƒΠΌΡ„Π°ΠΌ Ρ„ΠΈΠ·ΠΈΠΊΠΈ Π² XXI Π²Π΅ΠΊΠ΅.

УравнСния МаксвСлла

УравнСния МаксвСлла


Maxwell’s equations

Β Β Β  УравнСния МаксвСлла βˆ’ уравнСния классичСской элСктродинамики, ΠΎΠΏΠΈΡΡ‹Π²Π°ΡŽΡ‰ΠΈΠ΅ Π΄ΠΈΠ½Π°ΠΌΠΈΠΊΡƒ элСктромагнитного поля ΠΈ Π΅Π³ΠΎ связь с зарядами ΠΈ Ρ‚ΠΎΠΊΠ°ΠΌΠΈ.

УравнСния МаксвСлла явились тСорСтичСским ΠΎΠ±ΠΎΠ±Ρ‰Π΅Π½ΠΈΠ΅ΠΌ ΡΠΊΡΠΏΠ΅Ρ€ΠΈΠΌΠ΅Π½Ρ‚Π°Π»ΡŒΠ½Ρ‹Ρ… Π·Π°ΠΊΠΎΠ½ΠΎΠ²: ΠšΡƒΠ»ΠΎΠ½Π°, АмпСра, Π·Π°ΠΊΠΎΠ½ΠΎΠ² элСктромагнитной ΠΈΠ½Π΄ΡƒΠΊΡ†ΠΈΠΈ ΠΈ Π΄Ρ€ΡƒΠ³ΠΈΡ….
Β Β Β  УравнСния МаксвСлла Π² гауссовой систСмС Π΅Π΄ΠΈΠ½ΠΈΡ† ΠΈΠΌΠ΅ΡŽΡ‚ Π²ΠΈΠ΄

div B = 0,
div D = 4πρ,

Π³Π΄Π΅ E βˆ’ Π½Π°ΠΏΡ€ΡΠΆΡ‘Π½Π½ΠΎΡΡ‚ΡŒ элСктричСского поля, H βˆ’ Π½Π°ΠΏΡ€ΡΠΆΡ‘Π½Π½ΠΎΡΡ‚ΡŒ ΠΌΠ°Π³Π½ΠΈΡ‚Π½ΠΎΠ³ΠΎ поля, D βˆ’ элСктричСская индукция, B βˆ’ магнитная индукция, ρ βˆ’ ΠΏΠ»ΠΎΡ‚Π½ΠΎΡΡ‚ΡŒ элСктричСского заряда, j βˆ’ ΠΏΠ»ΠΎΡ‚Π½ΠΎΡΡ‚ΡŒ элСктричСского Ρ‚ΠΎΠΊΠ°.

Β Β Β  Для Ρ‚ΠΎΠ³ΠΎ, Ρ‡Ρ‚ΠΎΠ±Ρ‹ ΠΈΡΠΏΠΎΠ»ΡŒΠ·ΠΎΠ²Π°Ρ‚ΡŒ уравнСния МаксвСлла для Ρ€Π΅ΡˆΠ΅Π½ΠΈΡ Π·Π°Π΄Π°Ρ‡ элСктродинамики Π² Ρ€Π°Π·Π»ΠΈΡ‡Π½Ρ‹Ρ… срСдах, Π½Π΅ΠΎΠ±Ρ…ΠΎΠ΄ΠΈΠΌΠΎ ΡƒΡ‡Π΅ΡΡ‚ΡŒ ΠΈΠ½Π΄ΠΈΠ²ΠΈΠ΄ΡƒΠ°Π»ΡŒΠ½Ρ‹Π΅ свойства срСды.

D = Ξ΅E,
B = ΞΌH,

Ξ΅ βˆ’ диэлСктричСская ΠΏΡ€ΠΎΠ½ΠΈΡ†Π°Π΅ΠΌΠΎΡΡ‚ΡŒ срСды, ΞΌ βˆ’ магнитная ΠΏΡ€ΠΎΠ½ΠΈΡ†Π°Π΅ΠΌΠΎΡΡ‚ΡŒ срСды, Οƒ — ΡΠ»Π΅ΠΊΡ‚Ρ€ΠΎΠΏΡ€ΠΎΠ²ΠΎΠ΄Π½ΠΎΡΡ‚ΡŒ срСды.
Β Β Β  Π’ Π²Π°ΠΊΡƒΡƒΠΌΠ΅ Π±Π΅Π· зарядов ΠΈ Ρ‚ΠΎΠΊΠΎΠ²

D = Ξ΅0E,B = ΞΌ0H,
div E = 0,div H = 0,

Β Β Β  Π­Ρ‚Π° систСма Π΄ΠΈΡ„Ρ„Π΅Ρ€Π΅Π½Ρ†ΠΈΠ°Π»ΡŒΠ½Ρ‹Ρ… ΡƒΡ€Π°Π²Π½Π΅Π½ΠΈΠΉ ΠΈΠΌΠ΅Π΅Ρ‚ Ρ€Π΅ΡˆΠ΅Π½ΠΈΠ΅ — Π³Π°Ρ€ΠΌΠΎΠ½ΠΈΡ‡Π΅ΡΠΊΡƒΡŽ ΠΏΠ»ΠΎΡΠΊΡƒΡŽ Π²ΠΎΠ»Π½Ρƒ.

Π’Π΅ΠΊΡ‚ΠΎΡ€Ρ‹ элСктричСского ΠΈ ΠΌΠ°Π³Π½ΠΈΡ‚Π½ΠΎΠ³ΠΎ ΠΏΠΎΠ»Π΅ΠΉ пСрпСндикулярны Π½Π°ΠΏΡ€Π°Π²Π»Π΅Π½ΠΈΡŽ распространСния Π²ΠΎΠ»Π½Ρ‹ ΠΈ Π΄Ρ€ΡƒΠ³ Π΄Ρ€ΡƒΠ³Ρƒ ΠΈ находятся Π² Ρ„Π°Π·Π΅. Π’ΠΎΠ»Π½Π° распространяСтся со ΡΠΊΠΎΡ€ΠΎΡΡ‚ΡŒΡŽ

c = (ΞΌ0Ξ΅0)-1/2.

c βˆ’ ΡΠΊΠΎΡ€ΠΎΡΡ‚ΡŒ свСта Π² Π²Π°ΠΊΡƒΡƒΠΌΠ΅, c = 2.99792458Β·108 ΠΌ/с,
Ξ΅0 βˆ’ элСктричСская постоянная, Ξ΅0 = 8.85418782Β·10-12 Π€/ΠΌ,
ΞΌ0 βˆ’ магнитная постоянная, ΞΌ0 = 1.25663706Β·10-6 Π“Π½/ΠΌ.


Β 

Β 

ΠœΠΠšΠ‘Π’Π•Π›Π›Π Π£Π ΠΠ’ΠΠ•ΠΠ˜Π― β€’ Π‘ΠΎΠ»ΡŒΡˆΠ°Ρ российская энциклопСдия

ΠœΠΜΠšΠ‘Π’Π•Π›Π›Π Π£Π ΠΠ’ΠΠ•ΜΠΠ˜Π―, ΠΎΡΒ­Π½ΠΎΒ­Π²ΠΎΒ­ΠΏΠΎΒ­Π»Π°Β­Π³Π°ΡŽΒ­Ρ‰ΠΈΠ΅ урав­нС­ния клас­сич. мак­ро­ско­пич. элСк­тро­ди­на­ми­ки, ΠΎΠΏΠΈΒ­ΡΡ‹Β­Π²Π°ΡŽΒ­Ρ‰ΠΈΠ΅ за­ко­но­мСр­но­сти элСк­тро­маг­нит­ных яв­лС­ний Π² сплош­ной срС­дС ΠΈΠ»ΠΈ Π²Π°Β­ΠΊΡƒΡƒΒ­ΠΌΠ΅ (Π² ΠΏΡ€Π΅Β­Π½Π΅Π±Β­Ρ€Π΅Β­ΠΆΠ΅Β­Π½ΠΈΠΈ ΠΊΠ²Π°Π½Β­Ρ‚ΠΎΒ­Π²Ρ‹Β­ΠΌΠΈ яв­лС­ния­ми). ВСо­рия элСк­тро­маг­нит­но­го поля Π±Ρ‹Β­Π»Π° Ρ€Π°Π·Β­Ρ€Π°Β­Π±ΠΎΒ­Ρ‚Π°Β­Π½Π° Π”ΠΆ. К. Мак­свСл­лом Π² 1856–73. Π’Β Πœ.Β Ρƒ. ΠΎΠ±ΠΎΠ±Β­Ρ‰Π΅Β­Π½Ρ‹ Ρ€Π°Β­Π½Π΅Π΅ ус­та­нов­лСн­ныС ΠΎΠΏΡ‹Ρ‚Β­Π½Ρ‹Π΅ Π·Π°Β­ΠΊΠΎΒ­Π½Ρ‹ элСк­трич.

ΠΈ ΠΌΠ°Π³Β­Π½ΠΈΡ‚Β­Π½Ρ‹Ρ… яв­лС­ний, ΠΈ эти Π·Π°Β­ΠΊΠΎΒ­Π½Ρ‹ ΠΎΠ±ΡŠΒ­Π΅Β­Π΄ΠΈΒ­Π½Π΅Β­Π½Ρ‹ с ΠΊΠΎΠ½Β­Ρ†Π΅ΠΏΒ­Ρ†ΠΈΒ­Π΅ΠΉ М. Ѐа­ра­дСя ΠΎΠ± элСк­тро­маг­нит­ном ΠΏΠΎΒ­Π»Π΅, ΠΎΠ±Π΅ΡΒ­ΠΏΠ΅Β­Ρ‡ΠΈΒ­Π²Π°ΡŽΒ­Ρ‰Π΅ΠΌ взаи­мо­дСй­ст­виС ΠΌΠ΅Β­ΠΆΒ­Π΄Ρƒ ΡƒΠ΄Π°Β­Π»Ρ‘Π½Β­Π½Ρ‹Β­ΠΌΠΈ за­ря­ТСн­ны­ми Ρ‚Π΅Β­Π»Π°Β­ΠΌΠΈ (Ρ‚.Β Π½. тСо­рия близ­ко­дСй­ст­вия). Π’Β ΠΎΡ€ΠΈΒ­Π³ΠΈΒ­Π½Π°Π»ΡŒΒ­Π½ΠΎΠΌ ΠΈΠ·Β­Π»ΠΎΒ­ΠΆΠ΅Β­Π½ΠΈΠΈ Мак­свСл­ла Π±Ρ‹Β­Π»ΠΎ ΡΠΎΠ·Β­Π½Π°Β­Ρ‚Π΅Π»ΡŒΒ­Π½ΠΎ ΠΏΡ€ΠΈΒ­Π²Π΅Β­Π΄Π΅Β­Π½ΠΎ ΠΈΠ·Β­Π±Ρ‹Β­Ρ‚ΠΎΡ‡Β­Π½ΠΎΠ΅ чис­ло ΡƒΡ€Π°Π²Β­Π½Π΅Β­Π½ΠΈΠΉ; ΠΏΡ€ΠΈ этом Мак­свСлл ис­поль­зо­вал ΠΌΠ°Β­Ρ‚Π΅Β­ΠΌΠ°Β­Ρ‚ΠΈΡ‡. Π°ΠΏΒ­ΠΏΠ°Β­Ρ€Π°Ρ‚ ΠΊΠ²Π°Β­Ρ‚Π΅Ρ€Β­Π½ΠΈΠΎΒ­Π½ΠΎΠ² Π“Π°Β­ΠΌΠΈΠ»ΡŒΒ­Ρ‚ΠΎΒ­Π½Π°. Π‘ΠΎΠ²Ρ€. Ρ„ΠΎΡ€Β­ΠΌΡƒ М.Β Ρƒ. с ис­поль­зо­ва­ни­См Π²Π΅ΠΊΒ­Ρ‚ΠΎΡ€Β­Π½ΠΎΒ­Π³ΠΎ ис­чис­лС­ния ΠΏΡ€ΠΈΒ­Π΄Π°Β­Π»ΠΈ Π“.Β Π .Β Π“Π΅Ρ€Ρ† ΠΈ О.Β Π₯С­ви­сайд. М.Β Ρƒ. ΡΠ²ΡΒ­Π·Ρ‹Β­Π²Π°Β­ΡŽΡ‚ Π²Π΅ΠΊΒ­Ρ‚ΠΎΡ€Β­Π½Ρ‹Π΅ ΠΏΠΎΒ­Π»Π΅Β­Π²Ρ‹Π΅ Π²Π΅Β­Π»ΠΈΒ­Ρ‡ΠΈΒ­Π½Ρ‹ (ΡΠ²Β­Π»ΡΡŽΒ­Ρ‰ΠΈΠ΅Β­ΡΡ функ­ция­ми ΠΊΠΎΒ­ΠΎΡ€Β­Π΄ΠΈΒ­Π½Π°Ρ‚ ΠΈ Π²Ρ€Π΅Β­ΠΌΠ΅Β­Π½ΠΈ) с ис­точ­ни­ка­ми элСк­тро­маг­нит­но­го по­ля – рас­прС­дС­лён­ны­ми Π² про­стран­ст­вС ΠΈ ΠΈΠ·Β­ΠΌΠ΅Β­Π½ΡΡŽΒ­Ρ‰ΠΈΒ­ΠΌΠΈΒ­ΡΡ Π²ΠΎ Π²Ρ€Π΅Β­ΠΌΠ΅Β­Π½ΠΈ элСк­трич. за­ря­да­ми ΠΈ Ρ‚ΠΎΒ­ΠΊΠ°Β­ΠΌΠΈ. М.Β Ρƒ. ΠΈΠΌΠ΅Β­ΡŽΡ‚ Π²ΠΈΠ΄ (Π΄ΠΈΡ„Β­Ρ„Π΅Β­Ρ€Π΅Π½Β­Ρ†ΠΈΒ­Π°Π»ΡŒΒ­Π½Π°Ρ Ρ„ΠΎΡ€Β­ΠΌΠ° М.Β Ρƒ. Π² БИ): $$\textrm{rot}\,\boldsymbol E=-\frac{\partial \boldsymbol B}{\partial t},\quad \textrm{rot}\,\boldsymbol H=\boldsymbol j+\frac{\partial \boldsymbol D}{\partial t},\\ \textrm{div}\,\boldsymbol D=ρ,\quad \textrm{div}\,\boldsymbol B=0,$$ Π³Π΄Π΅ $\boldsymbolΒ E$ – Π½Π°Β­ΠΏΡ€ΡΒ­ΠΆΡ‘Π½Β­Π½ΠΎΡΡ‚ΡŒ элСк­трич.
по­ля,Β $\boldsymbol B$ – маг­нит­ная ин­дук­ция,Β $\boldsymbolΒ H$ – Π½Π°Β­ΠΏΡ€ΡΒ­ΠΆΡ‘Π½Β­Π½ΠΎΡΡ‚ΡŒ ΠΌΠ°Π³Β­Π½ΠΈΡ‚Β­Π½ΠΎΒ­Π³ΠΎ по­ля,Β $\boldsymbolΒ D$ – элСк­трич. ин­дук­ция,Β $\boldsymbol j$ – ΠΏΠ»ΠΎΡ‚Β­Π½ΠΎΡΡ‚ΡŒ элСк­трич. Ρ‚ΠΎΒ­ΠΊΠ°, $ρ$  – ΠΎΠ±ΡŠΒ­Ρ‘ΠΌΒ­Π½Π°Ρ ΠΏΠ»ΠΎΡ‚Β­Π½ΠΎΡΡ‚ΡŒ элСк­трич. за­ря­да. ДСй­ст­виС Π΄ΠΈΡ„Β­Ρ„Π΅Β­Ρ€Π΅Π½Β­Ρ†ΠΈΒ­Π°Π»ΡŒΒ­Π½Ρ‹Ρ… ΠΎΠΏΠ΅Β­Ρ€Π°Β­Ρ‚ΠΎΒ­Ρ€ΠΎΠ² $\textrm{rot}$ ΠΈ $\textrm{div}$ Π½Π° Π²Π΅ΠΊΒ­Ρ‚ΠΎΒ­Ρ€Ρ‹ элСк­тро­маг­нит­но­го по­ля ΠΌΠΎΒ­ΠΆΠ΅Ρ‚ Π±Ρ‹Ρ‚ΡŒ Π²Ρ‹Β­Ρ€Π°Β­ΠΆΠ΅Β­Π½ΠΎ Ρ‡Π΅Β­Ρ€Π΅Π· Π²Π΅ΠΊΒ­Ρ‚ΠΎΡ€Β­Π½ΠΎΠ΅ ΠΈ ска­ляр­ноС про­из­вС­дС­ния ΠΎΠΏΠ΅Β­Ρ€Π°Β­Ρ‚ΠΎΒ­Ρ€Π° Π“Π°Β­ΠΌΠΈΠ»ΡŒΒ­Ρ‚ΠΎΒ­Π½Π° $\nabla$Β (Π½Π°Β­Π±Π»Π°) ΠΈ ΡΠΎΒ­ΠΎΡ‚Β­Π²Π΅Ρ‚Β­ΡΡ‚Β­Π²ΡƒΡŽΒ­Ρ‰Π΅Β­Π³ΠΎ ΠΏΠΎΒ­Π»Π΅Β­Π²ΠΎΒ­Π³ΠΎ Π²Π΅ΠΊΒ­Ρ‚ΠΎΒ­Ρ€Π°; Π² Π΄Π΅Β­ΠΊΠ°Ρ€Β­Ρ‚ΠΎΒ­Π²ΠΎΠΉ сис­тС­мС ΠΊΠΎΒ­ΠΎΡ€Β­Π΄ΠΈΒ­Π½Π°Ρ‚$$\nabla=\boldsymbol e_x\frac{\partial}{\partial x}+\boldsymbol e_y\frac{\partial}{\partial y}+\boldsymbol e_z\frac{\partial}{\partial z}$$(Π³Π΄Π΅ $\boldsymbolΒ e_x,Β \boldsymbolΒ e_y,Β \boldsymbolΒ e_z$ – Π΅Π΄ΠΈΒ­Π½ΠΈΡ‡Β­Π½Ρ‹Π΅ Π²Π΅ΠΊΒ­Ρ‚ΠΎΒ­Ρ€Ρ‹ ΡΠΎΠΎΡ‚Β­Π²Π΅Ρ‚Β­ΡΡ‚Β­Π²ΡƒΡŽΒ­Ρ‰ΠΈΡ… ΠΊΠΎΒ­ΠΎΡ€Β­Π΄ΠΈΒ­Π½Π°Ρ‚Β­Π½Ρ‹Ρ… осСй), ΠΈ для ΠΏΡ€ΠΎΒ­ΠΈΠ·Β­Π²ΠΎΠ»ΡŒΒ­Π½ΠΎΠΉ Π²Π΅ΠΊΒ­Ρ‚ΠΎΡ€Β­Π½ΠΎΠΉ Ρ„ΡƒΠ½ΠΊΒ­Ρ†ΠΈΠΈ $\boldsymbolΒ f=\boldsymbolΒ e_xf_x+\boldsymbolΒ e_yf_y+\boldsymbolΒ e_zf_z$ ΠΏΠΎΒ­Π»ΡƒΒ­Ρ‡Π°Β­Π΅ΠΌ:$$\textrm{rot}\,\boldsymbol f=[\nabla \boldsymbol f]=\boldsymbol e_x \left( \frac{\partial f_z}{\partial y}-\frac{\partial f_y}{\partial z} \right) + \boldsymbol e_y \left( \frac{\partial f_x}{\partial z}-\frac{\partial f_z}{\partial x} \right) + \boldsymbol e_z \left( \frac{\partial f_y}{\partial x}-\frac{\partial f_x}{\partial y} \right),\\ \textrm{div}\,\boldsymbol f=\nabla \boldsymbol f=\frac{\partial f_x}{\partial x} + \frac{\partial f_y}{\partial y} + \frac{\partial f_z}{\partial z}.
$$

Для Ρ‚ΠΎΒ­Π³ΠΎ Ρ‡Ρ‚ΠΎΒ­Π±Ρ‹ М.Β Ρƒ. ΠΎΠ±Β­Ρ€Π°Β­Π·ΠΎΒ­Π²Π°Β­Π»ΠΈ ма­тС­ма­ти­чС­ски ΠΏΠΎΠ»Β­Π½ΡƒΡŽ сис­тС­му ΡƒΡ€Π°Π²Β­Π½Π΅Β­Π½ΠΈΠΉ, ΠΎΠ½ΠΈ Π΄ΠΎΠ»ΠΆΒ­Π½Ρ‹ Π±Ρ‹Ρ‚ΡŒ Π΄ΠΎΒ­ΠΏΠΎΠ»Β­Π½Π΅Β­Π½Ρ‹ Ρ„ΠΈΒ­Π·ΠΈΡ‡. урав­нС­ния­ми свя­зи ΠΌΠ΅Β­ΠΆΒ­Π΄Ρƒ ΠΏΠΎΒ­Π»Π΅Β­Π²Ρ‹Β­ΠΌΠΈ Π²Π΅ΠΊΒ­Ρ‚ΠΎΒ­Ρ€Π°Β­ΠΌΠΈ $\boldsymbolΒ E$Β ΠΈΒ $\boldsymbolΒ B$Β (дос­та­точ­ны­ми для опи­са­ния элСк­тро­маг­нит­но­го по­ля Π² Π²Π°Β­ΠΊΡƒΡƒΒ­ΠΌΠ΅) ΠΈ ΠΏΠΎΒ­Π»Π΅Β­Π²Ρ‹Β­ΠΌΠΈ Π²Π΅ΠΊΒ­Ρ‚ΠΎΒ­Ρ€Π°Β­ΠΌΠΈΒ $\boldsymbolΒ D$Β ΠΈΒ $\boldsymbolΒ H$, за­ви­ся­щи­ми ΠΎΡ‚ элСк­трич. ΠΈ ΠΌΠ°Π³Β­Π½ΠΈΡ‚Β­Π½Ρ‹Ρ… свойств ΠΌΠ°Β­Ρ‚Π΅Β­Ρ€ΠΈΒ­Π°Π»ΡŒΒ­Π½ΠΎΠΉ срС­ды, Π³Π΄Π΅ рас­смат­ри­ва­Ст­ся элСк­тро­маг­нит­ноС ΠΏΠΎΒ­Π»Π΅, Π° Ρ‚Π°ΠΊΒ­ΠΆΠ΅ урав­нС­ния­ми свя­зи плот­но­сти Ρ‚ΠΎΒ­ΠΊΠ°Β $\boldsymbolΒ j$, ΠΏΡ€ΠΎΒ­Ρ‚Π΅Β­ΠΊΠ°ΡŽΒ­Ρ‰Π΅Β­Π³ΠΎ Π² ΠΌΠ°Β­Ρ‚Π΅Β­Ρ€ΠΈΒ­Π°Π»ΡŒΒ­Π½ΠΎΠΉ срС­дС, с элСк­тро­маг­нит­ным ΠΏΠΎΒ­Π»Π΅ΠΌ. Π’ ΠΎΠ±Β­Ρ‰Π΅ΠΌ слу­чаС эти урав­нС­ния ΡΠ²Β­Π»ΡΒ­ΡŽΡ‚Β­ΡΡ слоТ­ны­ми ΠΈΠ½Β­Ρ‚Π΅Β­Π³Ρ€Π°Π»ΡŒΒ­Π½Ρ‹Β­ΠΌΠΈ ΡΠΎΒ­ΠΎΡ‚Β­Π½ΠΎΒ­ΡˆΠ΅Β­Π½ΠΈΡΒ­ΠΌΠΈ, ΡƒΡ‡ΠΈΒ­Ρ‚Ρ‹Β­Π²Π°ΡŽΒ­Ρ‰ΠΈΒ­ΠΌΠΈ, Ρ‡Ρ‚ΠΎ ис­ко­мыС ΠΏΠΎΒ­Π»Π΅Β­Π²Ρ‹Π΅ Π²Π΅ΠΊΒ­Ρ‚ΠΎΒ­Ρ€Ρ‹ Π²Β Π΄Π°Π½Β­Π½ΠΎΠΉ Ρ‚ΠΎΡ‡Β­ΠΊΠ΅ про­стран­ст­ва ΠΈ Π² Π΄Π°Π½Β­Π½Ρ‹ΠΉ ΠΌΠΎΒ­ΠΌΠ΅Π½Ρ‚ Π²Ρ€Π΅Β­ΠΌΠ΅Β­Π½ΠΈ ΠΌΠΎΒ­Π³ΡƒΡ‚ Π·Π°Β­Π²ΠΈΒ­ΡΠ΅Ρ‚ΡŒ ΠΎΡ‚ элСк­тро­маг­нит­но­го по­ля Π²ΠΎ всём про­стран­ст­вС ΠΈ Π²ΠΎ всС ΠΏΡ€Π΅Π΄Β­ΡˆΠ΅Β­ΡΡ‚Β­Π²ΡƒΡŽΒ­Ρ‰ΠΈΠ΅ ΠΌΠΎΒ­ΠΌΠ΅Π½Β­Ρ‚Ρ‹ Π²Ρ€Π΅Β­ΠΌΠ΅Β­Π½ΠΈ с ΡƒΡ‡Ρ‘Β­Ρ‚ΠΎΠΌ за­паз­ды­ва­ния, Π²Ρ‹Β­Π·Π²Π°Π½Β­Π½ΠΎΒ­Π³ΠΎ ΠΊΠΎΒ­Π½Π΅Ρ‡Β­Π½ΠΎΠΉ ΡΠΊΠΎΒ­Ρ€ΠΎΒ­ΡΡ‚ΡŒΡŽ рас­про­стра­нС­ния элСк­трич.

{–7}$Β Π“Π½/м – маг­нит­ная по­сто­ян­ная, $c$ – ΡΠΊΠΎΒ­Ρ€ΠΎΡΡ‚ΡŒ рас­про­стра­нС­ния элСк­тро­маг­нит­ных Π²ΠΎΠ»Π½ (ΡΠΊΠΎΒ­Ρ€ΠΎΡΡ‚ΡŒ свС­та) Π²Β Π²Π°Β­ΠΊΡƒΡƒΒ­ΠΌΠ΅, $Ξ΅$ – ди­элСк­трич. ΠΏΡ€ΠΎΒ­Π½ΠΈΒ­Ρ†Π°Π΅Β­ΠΌΠΎΡΡ‚ΡŒ, $ΞΌ$ – маг­нит­ная ΠΏΡ€ΠΎΒ­Π½ΠΈΒ­Ρ†Π°Π΅Β­ΠΌΠΎΡΡ‚ΡŒ, $Οƒ$ – ΡΠ»Π΅ΠΊΒ­Ρ‚Ρ€ΠΎΒ­ΠΏΡ€ΠΎΒ­Π²ΠΎΠ΄Β­Π½ΠΎΡΡ‚ΡŒ ΠΌΠ°Β­Ρ‚Π΅Β­Ρ€ΠΈΒ­Π°Π»ΡŒΒ­Π½ΠΎΠΉ срС­ды, $\boldsymbolΒ j_{стор}$ – ΠΏΠ»ΠΎΡ‚Β­Π½ΠΎΡΡ‚ΡŒ элСк­трич. Ρ‚ΠΎΒ­ΠΊΠ° (ΠΏΠΎΒ­Ρ‚ΠΎΒ­ΠΊΠ° за­ря­ТСн­ных час­тиц), Π²Ρ‹Β­Π·Π²Π°Π½Β­Π½ΠΎΒ­Π³ΠΎ нС­элСк­три­чСс­ки­ми (сто­рон­ни­ми) ΠΏΡ€ΠΈΒ­Ρ‡ΠΈΒ­Π½Π°Β­ΠΌΠΈ. ΠœΠ°Β­Ρ‚Π΅Β­Ρ€ΠΈΒ­Π°Π»ΡŒΒ­Π½Ρ‹Π΅ ко­эф­фи­ци­Сн­ты $Ξ΅$, $ΞΌ$ ΠΈ $Οƒ$ Ρ€Π°Π·Β­Π»ΠΈΒ­Ρ‡Π°Β­ΡŽΡ‚Β­ΡΡ для Ρ€Π°Π·Β­Π½Ρ‹Ρ… ΠΌΠ°Β­Ρ‚Π΅Β­Ρ€ΠΈΒ­Π°Π»ΡŒΒ­Π½Ρ‹Ρ… срСд ΠΈ для ΠΊΠΎΠ½Β­ΠΊΡ€Π΅Ρ‚Β­Π½ΠΎΠΉ срС­ды ΠΌΠΎΒ­Π³ΡƒΡ‚ Π±Ρ‹Ρ‚ΡŒ кон­стан­та­ми ΠΈΠ»ΠΈ функ­ция­ми ΠΊΠΎΒ­ΠΎΡ€Β­Π΄ΠΈΒ­Π½Π°Ρ‚ ΠΈ Π²Ρ€Π΅Β­ΠΌΠ΅Β­Π½ΠΈ (Π»ΠΈΒ­Π½Π΅ΠΉΒ­Π½Ρ‹Π΅ срС­ды) ΠΈΠ»ΠΈ ΠΆΠ΅ Π΄ΠΎΒ­ΠΏΠΎΠ»Β­Π½ΠΈΒ­Ρ‚Π΅Π»ΡŒΒ­Π½ΠΎ Π·Π°Β­Π²ΠΈΒ­ΡΠ΅Ρ‚ΡŒ ΠΎΡ‚ Π²Π΅Β­Π»ΠΈΒ­Ρ‡ΠΈΠ½ на­пря­Тён­но­стСй $\boldsymbolΒ E$Β ΠΈ $\boldsymbolΒ H$Β (Π½Π΅Β­Π»ΠΈΒ­Π½Π΅ΠΉΒ­Π½Ρ‹Π΅ срС­ды). Для ΠΈΠ·ΠΎΒ­Ρ‚Ρ€ΠΎΠΏΒ­Π½Ρ‹Ρ… срСд ΠΌΠ°Β­Ρ‚Π΅Β­Ρ€ΠΈΒ­Π°Π»ΡŒΒ­Π½Ρ‹Π΅ ко­эф­фи­ци­Сн­ты ΡΠ²Β­Π»ΡΒ­ΡŽΡ‚Β­ΡΡ ска­ля­ра­ми, для Π°Π½ΠΈΒ­Π·ΠΎΒ­Ρ‚Ρ€ΠΎΠΏΒ­Π½Ρ‹Ρ… (Π½Π°ΠΏΡ€., кри­стал­ли­чС­ских) – Ρ‚Π΅Π½Β­Π·ΠΎΡ€Β­Π½Ρ‹Β­ΠΌΠΈ Π²Π΅Β­Π»ΠΈΒ­Ρ‡ΠΈΒ­Π½Π°Β­ΠΌΠΈ; для элСк­тро­маг­нит­но­го по­ля Π² Π²Π°Β­ΠΊΡƒΡƒΒ­ΠΌΠ΅ $Ξ΅=ΞΌ=1, Οƒ=0$. ΠœΠΈΠΊΒ­Ρ€ΠΎΒ­ΡΠΊΠΎΒ­ΠΏΠΈΡ‡. смысл ΠΌΠ°Β­Ρ‚Π΅Β­Ρ€ΠΈΒ­Π°Π»ΡŒΒ­Π½Ρ‹Ρ… ко­эф­фи­ци­Сн­тов ΠΈ ΠΏΠΎΒ­Π»Π΅Β­Π²Ρ‹Ρ… Π²Π΅ΠΊΒ­Ρ‚ΠΎΒ­Ρ€ΠΎΠ² $\boldsymbolΒ D$ ΠΈ $\boldsymbolΒ H$, ΡƒΡ‡ΠΈΒ­Ρ‚Ρ‹Β­Π²Π°ΡŽΒ­Ρ‰ΠΈΡ… элСк­тро­маг­нит­ныС свой­ст­ва ΠΊΠΎΠ½Β­ΠΊΡ€Π΅Ρ‚Β­Π½ΠΎΠΉ ΠΌΠ°Β­Ρ‚Π΅Β­Ρ€ΠΈΒ­Π°Π»ΡŒΒ­Π½ΠΎΠΉ срС­ды, вы­яв­ля­Ст­ся ΠΏΡ€ΠΈ ус­рСд­нС­нии Ло­рСн­ца – Мак­свСл­ла ΡƒΡ€Π°Π²Β­Π½Π΅Β­Π½ΠΈΠΉ, Ρ€Π°ΡΒ­ΡΠΌΠ°Ρ‚Β­Ρ€ΠΈΒ­Π²Π°Β­ΡŽΒ­Ρ‰ΠΈΡ… ΠΌΠ°Β­Ρ‚Π΅Β­Ρ€ΠΈΒ­Π°Π»ΡŒΒ­Π½Ρ‹Π΅ срС­ды ΠΊΠ°ΠΊ ΡΠΎΒ­Π²ΠΎΒ­ΠΊΡƒΠΏΒ­Π½ΠΎΡΡ‚ΡŒ мик­ро­ско­пич. за­ря­ТСн­ных час­тиц.

ΠŸΡ€ΠΈΒ­ΠΌΠ΅Β­Π½ΡΡ Ρ‚Π΅ΠΎΒ­Ρ€Π΅Β­ΠΌΡƒ Π“Ρ€ΠΈΒ­Π½Π° ΠΈ Ρ„ΠΎΡ€Β­ΠΌΡƒΒ­Π»Ρƒ Га­ус­са – ΠžΡΡ‚Β­Ρ€ΠΎΒ­Π³Ρ€Π°Π΄Β­ΡΠΊΠΎΒ­Π³ΠΎ ΠΊ М.Β Ρƒ. Π² Π΄ΠΈΡ„Β­Ρ„Π΅Β­Ρ€Π΅Π½Β­Ρ†ΠΈΒ­Π°Π»ΡŒΒ­Π½ΠΎΠΉ Ρ„ΠΎΡ€Β­ΠΌΠ΅, ΠΌΠΎΠΆΒ­Π½ΠΎ ΠΏΠΎΒ­Π»ΡƒΒ­Ρ‡ΠΈΡ‚ΡŒ М.Β Ρƒ. Π² ΠΈΠ½Β­Ρ‚Π΅Β­Π³Ρ€Π°Π»ΡŒΒ­Π½ΠΎΠΉ Ρ„ΠΎΡ€Β­ΠΌΠ΅:$$\oint\limits_L \boldsymbol E d \boldsymbol l =-\frac{d}{dt}\int\limits_S \boldsymbol Bd\boldsymbol S,\qquad (1)\\ \oint\limits_L \boldsymbol Hd\boldsymbol l=\int\limits_S \left( \boldsymbol j+\frac{\partial \boldsymbol D}{dt}\right)d\boldsymbol S,\qquad(2)\\ \oint\limits_S \boldsymbol D d \boldsymbol S=\int\limits_V ρdV,\qquad(3)\\ \oint\limits_S \boldsymbol B d \boldsymbol S=0\qquad(4)$$ Π’ урав­нС­ни­ях (1) ΠΈ (2) $S$ – ΠΏΠΎΒ­Π²Π΅Ρ€Ρ…Β­Π½ΠΎΡΡ‚ΡŒ ΠΏΡ€ΠΎΒ­ΠΈΠ·Β­Π²ΠΎΠ»ΡŒΒ­Π½ΠΎΠΉ Ρ„ΠΎΡ€Β­ΠΌΡ‹, ог­ра­ни­чСн­ная Π·Π°ΠΌΠΊΒ­Π½ΡƒΒ­Ρ‚Ρ‹ΠΌ ΠΊΠΎΠ½Β­Ρ‚ΡƒΒ­Ρ€ΠΎΠΌ $L, d\boldsymbolΒ l$ – Π²Π΅ΠΊΒ­Ρ‚ΠΎΡ€ элС­мСн­тар­ной час­ти ΠΊΠΎΠ½Β­Ρ‚ΡƒΒ­Ρ€Π°, Π½Π°Β­ΠΏΡ€Π°Π²Β­Π»Π΅Π½Β­Π½Ρ‹ΠΉ ΠΏΠΎ Π½Π°Β­ΠΏΡ€Π°Π²Β­Π»Π΅Β­Π½ΠΈΡŽ Π΅Π³ΠΎ ΠΎΠ±Β­Ρ…ΠΎΒ­Π΄Π° Π² про­цСс­сС ин­тСг­ри­ро­ва­ния, $d\boldsymbolΒ S$ – Π²Π΅ΠΊΒ­Ρ‚ΠΎΡ€ элС­мСн­тар­ной ΠΏΠ»ΠΎΒ­Ρ‰Π°Π΄Β­ΠΊΠΈ по­вСрх­но­сти $S$, чис­лСн­но Ρ€Π°Π²Β­Π½Ρ‹ΠΉ ΠΏΠ»ΠΎΒ­Ρ‰Π°Β­Π΄ΠΈ ΠΏΠ»ΠΎΒ­Ρ‰Π°Π΄Β­ΠΊΠΈ ΠΈ Π½Π°Β­ΠΏΡ€Π°Π²Β­Π»Π΅Π½Β­Π½Ρ‹ΠΉ пСр­пСн­ди­ку­ляр­но по­вСрх­но­сти Π² Π½Π°Β­ΠΏΡ€Π°Π²Β­Π»Π΅Β­Π½ΠΈΠΈ, со­гла­со­ван­ном с Π½Π°Β­ΠΏΡ€Π°Π²Β­Π»Π΅Β­Π½ΠΈΒ­Π΅ΠΌ ΠΎΠ±Β­Ρ…ΠΎΒ­Π΄Π° ΠΏΠΎ ΠΏΡ€Π°Β­Π²ΠΈΒ­Π»Ρƒ Π²ΠΈΠ½Β­Ρ‚Π°. Π’ урав­нС­ни­ях (3) ΠΈ (4) $S$ – замк­ну­тая ΠΏΠΎΒ­Π²Π΅Ρ€Ρ…Β­Π½ΠΎΡΡ‚ΡŒ, ΠΎΡ…Β­Π²Π°Β­Ρ‚Ρ‹Β­Π²Π°ΡŽΒ­Ρ‰Π°Ρ ΠΎΠ±ΡŠΒ­Ρ‘ΠΌ $V, d\boldsymbolΒ S$ – Π²Π΅ΠΊΒ­Ρ‚ΠΎΡ€ элС­мСн­тар­ной ΠΏΠ»ΠΎΒ­Ρ‰Π°Π΄Β­ΠΊΠΈ, Π½Π°Β­ΠΏΡ€Π°Π²Β­Π»Π΅Π½Β­Π½Ρ‹ΠΉ пСр­пСн­ди­ку­ляр­но по­вСрх­но­сти Π½Π°Β­Ρ€ΡƒΒ­ΠΆΡƒ ΠΎΡ‚ ΠΎΡ…Β­Π²Π°Β­Ρ‚Ρ‹Β­Π²Π°Π΅Β­ΠΌΠΎΒ­Π³ΠΎ ΠΎΠ±ΡŠΒ­Ρ‘Β­ΠΌΠ°.

М.Β Ρƒ. Π² ΠΈΠ½Β­Ρ‚Π΅Β­Π³Ρ€Π°Π»ΡŒΒ­Π½ΠΎΠΉ Ρ„ΠΎΡ€Β­ΠΌΠ΅ ΠΈΠΌΠ΅Β­ΡŽΡ‚ нС­по­срСд­ст­вСн­ный Ρ„ΠΈΒ­Π·ΠΈΡ‡. смысл, пС­рС­но­си­мый ΠΈ Π½Π° ΡΠΎΒ­ΠΎΡ‚Β­Π²Π΅Ρ‚Β­ΡΡ‚Β­Π²ΡƒΡŽΒ­Ρ‰ΠΈΠ΅ М.Β Ρƒ. Π² Π΄ΠΈΡ„Β­Ρ„Π΅Β­Ρ€Π΅Π½Β­Ρ†ΠΈΒ­Π°Π»ΡŒΒ­Π½ΠΎΠΉ Ρ„ΠΎΡ€Β­ΠΌΠ΅. Π£Ρ€Π°Π²Β­Π½Π΅Β­Π½ΠΈΠ΅Β (1) ΠΎΠ±ΠΎΠ±Β­Ρ‰Π°Β­Π΅Ρ‚ Π·Π°Β­ΠΊΠΎΠ½ элСк­тро­маг­нит­ной ΠΈΠ½Β­Π΄ΡƒΠΊΒ­Ρ†ΠΈΠΈ Ѐа­ра­дСя, ΡΠ²ΡΒ­Π·Ρ‹Β­Π²Π°ΡŽΒ­Ρ‰ΠΈΠΉ ΡΠΊΠΎΒ­Ρ€ΠΎΡΡ‚ΡŒ из­мС­нС­ния ΠΌΠ°Π³Β­Π½ΠΈΡ‚Β­Π½ΠΎΒ­Π³ΠΎ ΠΏΠΎΒ­Ρ‚ΠΎΒ­ΠΊΠ° (ΠΏΠΎΒ­Ρ‚ΠΎΒ­ΠΊΠ° Π²Π΅ΠΊΒ­Ρ‚ΠΎΒ­Ρ€Π° ΠΌΠ°Π³Β­Π½ΠΈΡ‚Β­Π½ΠΎΠΉ ΠΈΠ½Β­Π΄ΡƒΠΊΒ­Ρ†ΠΈΠΈ $\boldsymbol B$), сцС­п­лСн­но­го с Π½Π΅Β­ΠΊΠΎΒ­Ρ‚ΠΎΒ­Ρ€Ρ‹ΠΌ ΠΊΠΎΠ½Β­Ρ‚ΡƒΒ­Ρ€ΠΎΠΌ, с эдс ΠΈΠ½Β­Π΄ΡƒΠΊΒ­Ρ†ΠΈΠΈ, Π½Π°Β­Π²Π΅Β­Π΄Ρ‘Π½Β­Π½ΠΎΠΉ Π² этом ΠΊΠΎΠ½Β­Ρ‚ΡƒΒ­Ρ€Π΅. Π’ ΠΎΡ‚Β­Π»ΠΈΒ­Ρ‡ΠΈΠ΅ ΠΎΡ‚ ΠΎΠΏΡ‹Β­Ρ‚ΠΎΠ² М. Ѐа­ра­дСя, Π³Π΄Π΅ ΠΊΠΎΠ½Β­Ρ‚ΡƒΡ€ прСд­став­лял со­бой ΠΌΠ΅Β­Ρ‚Π°Π»Β­Π»ΠΈΡ‡. ΠΏΡ€ΠΎΒ­Π²ΠΎΠ΄Β­Π½ΠΈΠΊ, ΠΏΠΎ ΠΊΠΎΒ­Ρ‚ΠΎΒ­Ρ€ΠΎΒ­ΠΌΡƒ ΠΏΡ€ΠΎΒ­Ρ‚Π΅Β­ΠΊΠ°Π» рС­ги­ст­ри­руС­мый ΠΈΠ½Β­Π΄ΡƒΠΊΒ­Ρ†ΠΈΒ­ΠΎΠ½Β­Π½Ρ‹ΠΉ Ρ‚ΠΎΠΊ, Мак­свСлл сфор­му­ли­ро­вал ΡƒΡ‚Β­Π²Π΅Ρ€Β­ΠΆΠ΄Π΅Β­Π½ΠΈΠ΅, Ρ‡Ρ‚ΠΎ эдс ΠΈΠ½Β­Π΄ΡƒΠΊΒ­Ρ†ΠΈΠΈ Π±ΡƒΒ­Π΄Π΅Ρ‚ Ρ‚Π°ΠΊΒ­ΠΆΠ΅ Π²ΠΎΠ·Β­Π½ΠΈΒ­ΠΊΠ°Ρ‚ΡŒ ΠΈ ΠΏΡ€ΠΈ ΠΈΠ·Β­ΠΌΠ΅Β­Π½Π΅Β­Π½ΠΈΠΈ ΠΌΠ°Π³Β­Π½ΠΈΡ‚Β­Π½ΠΎΒ­Π³ΠΎ ΠΏΠΎΒ­Ρ‚ΠΎΒ­ΠΊΠ° Π² Π²Π°Β­ΠΊΡƒΡƒΒ­ΠΌΠ΅ ΠΈΠ»ΠΈ ΠΈΠ½ΠΎΠΉ нС­про­во­дя­щСй срС­дС. Π’.Β ΠΎ., со­глас­но ΡƒΡ€Π°Π²Β­Π½Π΅Β­Π½ΠΈΡŽ (1), ΠΈΠ·Β­ΠΌΠ΅Β­Π½Π΅Β­Π½ΠΈΠ΅ ΠΌΠ°Π³Β­Π½ΠΈΡ‚Β­Π½ΠΎΒ­Π³ΠΎ по­ля Π²ΠΎ Π²Ρ€Π΅Β­ΠΌΠ΅Β­Π½ΠΈ Π²Ρ‹Β­Π·Ρ‹Β­Π²Π°Β­Π΅Ρ‚ Π²ΠΎΠ·Β­Π½ΠΈΠΊΒ­Π½ΠΎΒ­Π²Π΅Β­Π½ΠΈΠ΅ элСк­трич. по­ля (Ρ‚Π°ΠΊΒ­ΠΆΠ΅ ΠΈΠ·Β­ΠΌΠ΅Β­Π½ΡΡŽΒ­Ρ‰Π΅Β­Π³ΠΎΒ­ΡΡ Π²ΠΎ Π²Ρ€Π΅Β­ΠΌΠ΅Β­Π½ΠΈ).

Π£Ρ€Π°Π²Β­Π½Π΅Β­Π½ΠΈΠ΅ (2) яв­ля­Ст­ся ΠΎΠ±ΠΎΠ±Β­Ρ‰Π΅Β­Π½ΠΈΒ­Π΅ΠΌ Био – Π‘Π°Β­Π²Π°Β­Ρ€Π° Π·Π°Β­ΠΊΠΎΒ­Π½Π° ΠΎ Π²ΠΎΠ·Β­Π±ΡƒΒ­ΠΆΒ­Π΄Π΅Β­Π½ΠΈΠΈ ΠΌΠ°Π³Β­Π½ΠΈΡ‚Β­Π½ΠΎΒ­Π³ΠΎ по­ля элСк­трич. Ρ‚ΠΎΒ­ΠΊΠΎΠΌ. Ана­ли­зи­руя ΠΏΡ€ΠΎΒ­Ρ…ΠΎΒ­ΠΆΒ­Π΄Π΅Β­Π½ΠΈΠ΅ ΠΏΠ΅Β­Ρ€Π΅Β­ΠΌΠ΅Π½Β­Π½ΠΎΒ­Π³ΠΎ Ρ‚ΠΎΒ­ΠΊΠ° ΠΏΠΎ Ρ†Π΅Β­ΠΏΠΈ с кон­дСн­са­то­ром, Мак­свСлл ΠΏΡ€Π΅Π΄Β­ΠΏΠΎΒ­Π»ΠΎΒ­ΠΆΠΈΠ», Ρ‡Ρ‚ΠΎ для замк­ну­то­сти элСк­трич. Ρ‚ΠΎΒ­ΠΊΠ°, ΠΊΡ€ΠΎΒ­ΠΌΠ΅ Ρ‚ΠΎΒ­ΠΊΠ° про­во­ди­мо­сти, обу­слов­лСн­но­го Π΄Π²ΠΈΒ­ΠΆΠ΅Β­Π½ΠΈΒ­Π΅ΠΌ за­ря­дов ΠΏΠΎ ΠΏΡ€ΠΎΒ­Π²ΠΎΠ΄Β­Π½ΠΈΒ­ΠΊΡƒ, Π΄ΠΎΠ»Β­ΠΆΠ΅Π½ ΡΡƒΒ­Ρ‰Π΅Β­ΡΡ‚Β­Π²ΠΎΒ­Π²Π°Ρ‚ΡŒ Π΄ΠΎΒ­ΠΏΠΎΠ»Β­Π½ΠΈΡ‚. Ρ‚ΠΎΠΊ (Π½Π°Β­Π·Π²Π°Π½Β­Π½Ρ‹ΠΉ ΠΈΠΌ Ρ‚ΠΎΒ­ΠΊΠΎΠΌ смС­щС­ния), ΠΏΠ»ΠΎΡ‚Β­Π½ΠΎΡΡ‚ΡŒ ΠΊΠΎΒ­Ρ‚ΠΎΒ­Ρ€ΠΎΒ­Π³ΠΎ Ρ€Π°Π²Β­Π½Π° $πœ•\boldsymbol D/πœ•t$ ΠΈ ΠΊΠΎΒ­Ρ‚ΠΎΒ­Ρ€Ρ‹ΠΉ Ρ‚Π°ΠΊΒ­ΠΆΠ΅ Π΄ΠΎΠ»Β­ΠΆΠ΅Π½ ΡΠΎΠ·Β­Π΄Π°Β­Π²Π°Ρ‚ΡŒ ΠΌΠ°Π³Β­Π½ΠΈΡ‚Β­Π½ΠΎΠ΅ ΠΏΠΎΒ­Π»Π΅. Π­ΠΊΒ­Π²ΠΈΒ­Π²Π°Β­Π»Π΅Π½Ρ‚Β­Π½ΠΎΡΡ‚ΡŒ ΠΌΠ°Π³Β­Π½ΠΈΡ‚Β­Π½ΠΎΒ­Π³ΠΎ дСй­ст­вия Ρ‚ΠΎΒ­ΠΊΠ° про­во­ди­мо­сти ΠΈ Ρ‚ΠΎΒ­ΠΊΠ° смС­щС­ния Π±Ρ‹Β­Π»Π° ус­та­нов­лС­на ΡΠΊΡΒ­ΠΏΠ΅Β­Ρ€ΠΈΠΌΠ΅Π½Β­Ρ‚Π°Π»ΡŒΒ­Π½ΠΎ А. А. Π­ΠΉΒ­Ρ…Π΅Π½Β­Π²Π°Π»ΡŒΒ­Π΄ΠΎΠΌ Π² 1904 (см.Β Π­ΠΉΒ­Ρ…Π΅Π½Β­Π²Π°Π»ΡŒΒ­Π΄Π° ΠΎΠΏΡ‹Ρ‚). Π’.Β ΠΎ., со­глас­но ΡƒΡ€Π°Π²Β­Π½Π΅Β­Π½ΠΈΡŽΒ (2), ΠΌΠ°Π³Β­Π½ΠΈΡ‚Β­Π½ΠΎΠ΅ ΠΏΠΎΒ­Π»Π΅ Π²ΠΎΠ·Β­Π½ΠΈΒ­ΠΊΠ°Β­Π΅Ρ‚ Π½Π΅ Ρ‚ΠΎΠ»ΡŒΒ­ΠΊΠΎ Π² слу­чаС про­тС­ка­ния элСк­трич. Ρ‚ΠΎΒ­ΠΊΠ°, Π½ΠΎ ΠΈ ΠΏΡ€ΠΈ ΠΈΠ·Β­ΠΌΠ΅Β­Π½Π΅Β­Π½ΠΈΠΈ элСк­трич. по­ля Π²ΠΎ Π²Ρ€Π΅Β­ΠΌΠ΅Β­Π½ΠΈ; ΠΏΡ€ΠΈ этом Π²ΠΎΠ·Β­Π½ΠΈΒ­ΠΊΠ°ΡŽΒ­Ρ‰Π΅Π΅Β ΠΌΠ°Π³Β­Π½ΠΈΡ‚Β­Π½ΠΎΠ΅ ΠΏΠΎΒ­Π»Π΅ Ρ‚Π°ΠΊΒ­ΠΆΠ΅ из­мС­ня­Ст­ся Π²ΠΎ Π²Ρ€Π΅Β­ΠΌΠ΅Β­Π½ΠΈ.

Π£Ρ€Π°Π²Β­Π½Π΅Β­Π½ΠΈΠ΅Β (3) (Га­ус­са Ρ‚Π΅ΠΎΒ­Ρ€Π΅Β­ΠΌΠ°) вы­во­дит­ся с ΠΏΠΎΒ­ΠΌΠΎΒ­Ρ‰ΡŒΡŽ ΠšΡƒΒ­Π»ΠΎΒ­Π½Π° Π·Π°Β­ΠΊΠΎΒ­Π½Π° (спра­вСд­ли­во­го Ρ‚ΠΎΠ»ΡŒΒ­ΠΊΠΎ для Π½Π΅Β­ΠΏΠΎΠ΄Β­Π²ΠΈΠΆΒ­Π½Ρ‹Ρ… за­ря­дов) ΠΈ яв­ля­Ст­ся Π΅Π³ΠΎ ΠΎΠ±ΠΎΠ±Β­Ρ‰Π΅Β­Π½ΠΈΒ­Π΅ΠΌ. Π€ΠΈΒ­Π·ΠΈΡ‡. смысл урав­нС­ния (3) – ис­точ­ни­ком элСк­трич. по­ля ΡΠ²Β­Π»ΡΒ­ΡŽΡ‚Β­ΡΡ элСк­трич. за­ря­ды [на­ря­ду с ΠΏΠ΅Β­Ρ€Π΅Β­ΠΌΠ΅Π½Π½Ρ‹ΠΌ ΠΌΠ°Π³Β­Π½ΠΈΡ‚Β­Π½Ρ‹ΠΌ ΠΏΠΎΒ­Π»Π΅ΠΌ, см. ΡƒΡ€Π°Π²Β­Π½Π΅Π½ΠΈΠ΅Β (1)]. Π£Ρ€Π°Π²Β­Π½Π΅Β­Π½ΠΈΠ΅Β (4) Π°Π½Π°Β­Π»ΠΎΒ­Π³ΠΈΡ‡Β­Π½ΠΎ ΡƒΡ€Π°Π²Β­Π½Π΅Β­Π½ΠΈΡŽΒ (3) ΠΈ яв­ля­Ст­ся ΠΌΠ°Β­Ρ‚Π΅Β­ΠΌΠ°Β­Ρ‚ΠΈΡ‡. Π²Ρ‹Β­Ρ€Π°Β­ΠΆΠ΅Β­Π½ΠΈΒ­Π΅ΠΌ ΡΠΊΡΒ­ΠΏΠ΅Β­Ρ€ΠΈΒ­ΠΌΠ΅Π½Β­Ρ‚Π°Π»ΡŒΒ­Π½ΠΎ обос­но­вы­ваС­мо­го ут­вСр­ТдС­ния, Ρ‡Ρ‚ΠΎ ис­точ­ни­ком ΠΌΠ°Π³Β­Π½ΠΈΡ‚Β­Π½ΠΎΒ­Π³ΠΎ по­ля ΠΌΠΎΒ­Π³ΡƒΡ‚ Π±Ρ‹Ρ‚ΡŒ Ρ‚ΠΎΠ»ΡŒΒ­ΠΊΠΎ элСк­трич. Ρ‚ΠΎΒ­ΠΊΠΈ (про­во­ди­мо­сти ΠΈ смС­щС­ния), Π° ΠΌΠ°Π³Β­Π½ΠΈΡ‚Β­Π½Ρ‹Π΅ за­ря­ды (Π°Π½Π°Β­Π»ΠΎΒ­Π³ΠΈΡ‡Β­Π½Ρ‹Π΅ элСк­трич. за­ря­дам – ис­точ­ни­кам ΠΏΠΎΒ­Π»Π΅ΠΉ Π² Ρ‚Π΅ΠΎΒ­Ρ€Π΅Β­ΠΌΠ΅ Га­ус­са) Π² ΠΏΡ€ΠΈΒ­Ρ€ΠΎΒ­Π΄Π΅ ΠΎΡ‚Β­ΡΡƒΡ‚Β­ΡΡ‚Β­Π²ΡƒΒ­ΡŽΡ‚. ΠŸΡ€Π΅Π΄Β­ΡΠΊΠ°Β­Π·Π°Β­Π½ΠΈΡ Π½Π΅Β­ΠΊΠΎΒ­Ρ‚ΠΎΒ­Ρ€Ρ‹Ρ… Ρ„ΠΈΒ­Π·ΠΈΡ‡. Ρ‚Π΅ΠΎΒ­Ρ€ΠΈΠΉ ΠΎ су­щС­ст­во­ва­нии ΠΎΡ‚Π΄. ΠΌΠ°Π³Β­Π½ΠΈΡ‚Β­Π½Ρ‹Ρ… за­ря­дов (ΠΌΠ°Π³Β­Π½ΠΈΡ‚Β­Π½Ρ‹Ρ… ΠΌΠΎΒ­Π½ΠΎΒ­ΠΏΠΎΒ­Π»Π΅ΠΉ) ΠΏΠΎΒ­ΠΊΠ° Π½Π΅ ΠΏΠΎΒ­Π»ΡƒΒ­Ρ‡ΠΈΒ­Π»ΠΈ экс­пС­рим. под­твСр­Т­дС­ния.

Как слС­ду­Ст ΠΈΠ· Ρ„ΠΈΒ­Π·ΠΈΡ‡. смыс­ла ΡƒΡ€Π°Π²Β­Π½Π΅Β­Π½ΠΈΠΉΒ (1) ΠΈ (2), ΠΏΠ΅Β­Ρ€Π΅Β­ΠΌΠ΅Π½Β­Π½ΠΎΠ΅ ΠΌΠ°Π³Β­Π½ΠΈΡ‚Β­Π½ΠΎΠ΅ ΠΏΠΎΒ­Π»Π΅ Π²Ρ‹Β­Π·Ρ‹Β­Π²Π°Β­Π΅Ρ‚ Π²ΠΎΠ·Β­Π½ΠΈΠΊΒ­Π½ΠΎΒ­Π²Π΅Β­Π½ΠΈΠ΅ ΠΏΠ΅Β­Ρ€Π΅Β­ΠΌΠ΅Π½Β­Π½ΠΎΒ­Π³ΠΎ элСк­трич. по­ля, Π° ΠΏΠ΅Β­Ρ€Π΅Β­ΠΌΠ΅Π½Β­Π½ΠΎΠ΅ элСк­трич. по­лС – Π²ΠΎΠ·Β­Π½ΠΈΠΊΒ­Π½ΠΎΒ­Π²Π΅Β­Π½ΠΈΠ΅ ΠΏΠ΅Β­Ρ€Π΅Β­ΠΌΠ΅Π½Β­Π½ΠΎΒ­Π³ΠΎ ΠΌΠ°Π³Β­Π½ΠΈΡ‚Β­Π½ΠΎΒ­Π³ΠΎ по­ля ΠΈ, Ρ‚.Β ΠΎ., ΠΏΠ΅Β­Ρ€Π΅Β­ΠΌΠ΅Π½Β­Π½Ρ‹Π΅ элСк­трич. ΠΈ ΠΌΠ°Π³Β­Π½ΠΈΡ‚Β­Π½Ρ‹Π΅ по­ля ΠΌΠΎΒ­Π³ΡƒΡ‚ ΠΏΠΎΠ΄Β­Π΄Π΅Ρ€Β­ΠΆΠΈΒ­Π²Π°Ρ‚ΡŒ Π΄Ρ€ΡƒΠ³ Π΄Ρ€ΡƒΒ­Π³Π°, об­ра­зуя ΡΠ°Β­ΠΌΠΎΒ­ΡΡ‚ΠΎΒ­ΡΡ‚Π΅Π»ΡŒΒ­Π½Ρ‹ΠΉ Ρ„ΠΈΒ­Π·ΠΈΡ‡. ΠΎΠ±ΡŠΒ­Π΅ΠΊΡ‚Β β€“ ΡΠ»Π΅ΠΊΒ­Ρ‚Ρ€ΠΎΒ­ΠΌΠ°Π³Β­Π½ΠΈΡ‚Β­Π½ΡƒΡŽ Π²ΠΎΠ»Β­Π½Ρƒ, ΡΡƒΒ­Ρ‰Π΅Β­ΡΡ‚Β­Π²ΡƒΡŽΒ­Ρ‰ΡƒΡŽ ΡƒΠΆΠ΅ нС­за­ви­си­мо ΠΎΡ‚ ΠΏΠ΅Ρ€Β­Π²ΠΈΡ‡Β­Π½Ρ‹Ρ… ис­точ­ни­ков элСк­трич. ΠΈ ΠΌΠ°Π³Β­Π½ΠΈΡ‚Β­Π½ΠΎΒ­Π³ΠΎ ΠΏΠΎΒ­Π»Π΅ΠΉ. Π”ΠΆ. К. Мак­свСлл Π²ΠΏΠ΅Ρ€Β­Π²Ρ‹Π΅ ΠΏΠΎΒ­Π»ΡƒΒ­Ρ‡ΠΈΠ» ΠΈΠ· М.Β Ρƒ. Π²ΠΎΠ»Β­Π½ΠΎΒ­Π²ΠΎΠ΅ ΡƒΡ€Π°Π²Β­Π½Π΅Β­Π½ΠΈΠ΅ для элСк­тро­маг­нит­ной Π²ΠΎΠ»Β­Π½Ρ‹ ΠΈ ус­та­но­вил, Ρ‡Ρ‚ΠΎ элСк­тро­маг­нит­ная Π²ΠΎΠ»Β­Π½Π° рас­про­стра­ня­Ст­ся Π² Π²Π°Β­ΠΊΡƒΡƒΒ­ΠΌΠ΅ со ΡΠΊΠΎΒ­Ρ€ΠΎΒ­ΡΡ‚ΡŒΡŽ, ко­то­рая сов­па­да­Ст ΠΏΠΎ Π²Π΅Β­Π»ΠΈΒ­Ρ‡ΠΈΒ­Π½Π΅ с элСк­тро­ди­на­мич. по­сто­ян­ной, вхо­дя­щСй Π² ΠΈΡΒ­ΠΏΠΎΠ»ΡŒΒ­Π·ΠΎΒ­Π²Π°Π½Β­Π½ΡƒΡŽ Мак­свСл­лом Π°Π±Β­ΡΠΎΒ­Π»ΡŽΡ‚Β­Π½ΡƒΡŽ га­ус­со­ву сис­тС­му Π΅Π΄ΠΈΒ­Π½ΠΈΡ†. Π’.Β Π­.Β Π’Π΅Β­Π±Π΅Ρ€ ΠΈ Π½Π΅ΠΌ. Ρ„ΠΈΒ­Π·ΠΈΠΊ Π .Β ΠšΠΎΠ»ΡŒΒ­Ρ€Π°Β­ΡƒΡˆ Π² 1856 ус­та­но­ви­ли, Ρ‡Ρ‚ΠΎ элСк­тро­ди­на­мич. по­сто­ян­ная Ρ€Π°Π²Β­Π½Π° ско­ро­сти свС­та Π² Π²Π°Β­ΠΊΡƒΡƒΒ­ΠΌΠ΅; это ΠΏΠΎΒ­Π·Π²ΠΎΠ»ΠΈΒ­Π»ΠΎ ΠœΠ°ΠΊΒ­ΡΠ²Π΅Π»Β­Π»Ρƒ ΠΏΡ€Π΅Π΄Β­ΠΏΠΎΒ­Π»ΠΎΒ­ΠΆΠΈΡ‚ΡŒ, Ρ‡Ρ‚ΠΎ свСт прСд­став­ля­Ст со­бой элСк­тро­маг­нит­ныС Π²ΠΎΠ»Β­Π½Ρ‹. Π­Ρ‚ΠΎ ΠΏΡ€Π΅Π΄Β­ΠΏΠΎΒ­Π»ΠΎΒ­ΠΆΠ΅Β­Π½ΠΈΠ΅ на­шло своё ΠΏΠΎΠ΄Β­Ρ‚Π²Π΅Ρ€Β­ΠΆΠ΄Π΅Β­Π½ΠΈΠ΅ Π² даль­нСй­шСм Ρ€Π°Π·Β­Π²ΠΈΒ­Ρ‚ΠΈΠΈ учС­ния ΠΎ свС­тС.

Π‘ ΠΏΠΎΒ­ΠΌΠΎΒ­Ρ‰ΡŒΡŽ М.Β Ρƒ. Π±Ρ‹Β­Π»ΠΎ ус­та­нов­лС­но, Ρ‡Ρ‚ΠΎ элСк­тро­маг­нит­ноС ΠΏΠΎΒ­Π»Π΅ ΠΎΠ±Β­Π»Π°Β­Π΄Π°Β­Π΅Ρ‚ энСр­ги­Сй ΠΈ ΠΈΠΌΒ­ΠΏΡƒΠ»ΡŒΒ­ΡΠΎΠΌ. На­ли­чиС ΠΈΠΌΒ­ΠΏΡƒΠ»ΡŒΒ­ΡΠ° Ρƒ элСк­тро­маг­нит­ной Π²ΠΎΠ»Β­Π½Ρ‹ ΠΈ, ΡΠ»Π΅Β­Π΄ΠΎΒ­Π²Π°Β­Ρ‚Π΅Π»ΡŒΒ­Π½ΠΎ, Π΅Π³ΠΎ ΠΈΠ·Β­ΠΌΠ΅Β­Π½Π΅Β­Π½ΠΈΠ΅ ΠΏΡ€ΠΈ ΠΏΠΎΒ­Π³Π»ΠΎΒ­Ρ‰Π΅Β­Π½ΠΈΠΈ ΠΈΠ»ΠΈ ΠΎΡ‚Β­Ρ€Π°Β­ΠΆΠ΅Β­Π½ΠΈΠΈ ΠΏΡ€ΠΈΒ­Π²ΠΎΒ­Π΄ΠΈΡ‚ ΠΊ воз­ник­но­вС­нию дав­лС­ния элСк­тро­маг­нит­ной Π²ΠΎΠ»Β­Π½Ρ‹ Π½Π° ΠΏΠΎΒ­Π³Π»ΠΎΒ­Ρ‰Π°ΡŽΒ­Ρ‰ΡƒΡŽ ΠΈΠ»ΠΈ ΠΎΡ‚Β­Ρ€Π°Β­ΠΆΠ°ΡŽΒ­Ρ‰ΡƒΡŽ ΠΏΠΎΒ­Π²Π΅Ρ€Ρ…Β­Π½ΠΎΡΡ‚ΡŒ. ВСо­рС­ти­чС­ски прСд­ска­зан­ноС ΠΈ ко­ли­чС­ст­вСн­но рас­счи­тан­ноС Π”ΠΆ. К. Мак­свСл­лом Π΄Π°Π²Β­Π»Π΅Β­Π½ΠΈΠ΅ свС­та Π²ΠΏΠ΅Ρ€Β­Π²Ρ‹Π΅ Π±Ρ‹Β­Π»ΠΎ ΡΠΊΡΒ­ΠΏΠ΅Β­Ρ€ΠΈΒ­ΠΌΠ΅Π½Β­Ρ‚Π°Π»ΡŒΒ­Π½ΠΎ ΠΎΠ±Β­Π½Π°Β­Ρ€ΡƒΒ­ΠΆΠ΅Β­Π½ΠΎ ΠΈ ΠΈΠ·Β­ΠΌΠ΅Β­Ρ€Π΅Β­Π½ΠΎ П. Н. Π›Π΅Β­Π±Π΅Β­Π΄Π΅Β­Π²Ρ‹ΠΌ Π² 1899. Π Π΅Β­Π·ΡƒΠ»ΡŒΒ­Ρ‚Π°Β­Ρ‚Ρ‹ экс­пС­ри­мСн­тов Π›Π΅Β­Π±Π΅Β­Π΄Π΅Β­Π²Π°, ΠΊΠ°ΠΊ ΠΈ ΠΏΠΎΒ­ΡΠ»Π΅Β­Π΄ΡƒΡŽΒ­Ρ‰ΠΈΠ΅ экс­пС­рим. ис­слС­до­ва­ния свС­то­во­го дав­лС­ния, ΠΏΠΎΠ»Β­Π½ΠΎΒ­ΡΡ‚ΡŒΡŽ ΠΏΠΎΠ΄Β­Ρ‚Π²Π΅Ρ€Β­Π΄ΠΈΒ­Π»ΠΈ Π³ΠΈΒ­ΠΏΠΎΒ­Ρ‚Π΅Β­Π·Ρƒ Мак­свСл­ла ΠΎΠ± элСк­тро­маг­нит­ном Ρ…Π°Β­Ρ€Π°ΠΊΒ­Ρ‚Π΅Β­Ρ€Π΅ свС­то­вых Π²ΠΎΠ»Π½.

Осн. ха­рак­тС­ри­сти­кой, ΠΎΠΏΠΈΒ­ΡΡ‹Β­Π²Π°ΡŽΒ­Ρ‰Π΅ΠΉ про­цСсс рас­про­стра­нС­ния энСр­гии ΠΈ ΠΈΠΌΒ­ΠΏΡƒΠ»ΡŒΒ­ΡΠ° элСк­тро­маг­нит­но­го по­ля Π² про­стран­ст­вС, яв­ля­Ст­ся ΠŸΠΎΠΉΠ½Β­Ρ‚ΠΈΠ½Β­Π³Π° Π²Π΅ΠΊΒ­Ρ‚ΠΎΡ€Β $\it {\mathbf Ξ }=[\boldsymbol E \boldsymbol H]$, Π½Π°Β­ΠΏΡ€Π°Π²Β­Π»Π΅Β­Π½ΠΈΠ΅ ΠΊΠΎΒ­Ρ‚ΠΎΒ­Ρ€ΠΎΒ­Π³ΠΎ сов­па­да­Ст с Π½Π°Β­ΠΏΡ€Π°Π²Β­Π»Π΅Β­Π½ΠΈΒ­Π΅ΠΌ ΠΈΠΌΒ­ΠΏΡƒΠ»ΡŒΒ­ΡΠ° элСк­тро­маг­нит­но­го по­ля ΠΈ Π½Π°Β­ΠΏΡ€Π°Π²Β­Π»Π΅Β­Π½ΠΈΒ­Π΅ΠΌ рас­про­стра­нС­ния Π΅Π³ΠΎ энСр­гии, Π° Π²Π΅Β­Π»ΠΈΒ­Ρ‡ΠΈΒ­Π½Π° Ρ€Π°Π²Β­Π½Π° плот­но­сти ΠΏΠΎΒ­Ρ‚ΠΎΒ­ΠΊΠ° мощ­но­сти элСк­тро­маг­нит­но­го по­ля – энСр­гии, пС­рС­но­си­мой Π² Π΅Π΄ΠΈΒ­Π½ΠΈΒ­Ρ†Ρƒ Π²Ρ€Π΅Β­ΠΌΠ΅Β­Π½ΠΈ Ρ‡Π΅Β­Ρ€Π΅Π· Π΅Π΄ΠΈΒ­Π½ΠΈΡ‡Β­Π½ΡƒΡŽ ΠΏΠ»ΠΎΒ­Ρ‰Π°Π΄Β­ΠΊΡƒ, ΠΏΠ΅Ρ€Β­ΠΏΠ΅Π½Β­Π΄ΠΈΒ­ΠΊΡƒΒ­Π»ΡΡ€Β­Π½ΡƒΡŽ Π²Π΅ΠΊΒ­Ρ‚ΠΎΒ­Ρ€Ρƒ $\bf Ξ $ (ΠΈΠ»ΠΈ Π½Π°Β­ΠΏΡ€Π°Π²Β­Π»Π΅Β­Π½ΠΈΡŽ рас­про­стра­нС­ния энСр­гии).

Π­Π»Π΅ΠΊΒ­Ρ‚Ρ€ΠΎΒ­Π΄ΠΈΒ­Π½Π°Β­ΠΌΠΈΒ­ΠΊΠ° Мак­свСл­ла ока­за­лась ис­то­ри­чС­ски ΠΏΠ΅Ρ€Β­Π²ΠΎΠΉ рС­ля­ти­ви­ст­ской Ρ‚Π΅ΠΎΒ­Ρ€ΠΈΒ­Π΅ΠΉ. ИмСн­но Π°Π½Π°Β­Π»ΠΈΠ· М.Β Ρƒ. ΠΈ ис­слС­до­ва­ниС ΠΌΠ΅Β­Ρ‚ΠΎΒ­Π΄ΠΎΠ² ΠΈΡ… при­мС­нС­ния ΠΊ дви­Ту­щим­ся срС­дам ΠΏΡ€ΠΈΒ­Π²Π΅Β­Π»ΠΈ ΠΊ нС­об­хо­ди­мо­сти пС­рС­строй­ки клас­сич. Ρ„ΠΈΒ­Π·ΠΈΡ‡. прСд­став­лС­ний ΠΎ про­стран­ст­вС ΠΈ Π²Ρ€Π΅Β­ΠΌΠ΅Β­Π½ΠΈ ΠΈ соз­да­нию ча­ст­ной (ΡΠΏΠ΅Β­Ρ†ΠΈΒ­Π°Π»ΡŒΒ­Π½ΠΎΠΉ) Ρ‚Π΅ΠΎΒ­Ρ€ΠΈΠΈ ΠΎΡ‚Β­Π½ΠΎΒ­ΡΠΈΒ­Ρ‚Π΅Π»ΡŒΒ­Π½ΠΎΒ­ΡΡ‚ΠΈ. РС­ля­ти­ви­ст­ский Ρ…Π°Β­Ρ€Π°ΠΊΒ­Ρ‚Π΅Ρ€ М.Β Ρƒ. по­зво­ля­Ст Π·Π°Β­ΠΏΠΈΒ­ΡΠ°Ρ‚ΡŒ ΠΈΡ… Π² рС­ля­ти­вист­ски ΠΊΠΎΒ­Π²Π°Β­Ρ€ΠΈΒ­Π°Π½Ρ‚Β­Π½ΠΎΠΉ (ΠΎΠ΄ΠΈΒ­Π½Π°Β­ΠΊΠΎΒ­Π²ΠΎΠΉ Π²ΠΎ всСх ΠΈΠ½Π΅Ρ€Β­Ρ†ΠΈΒ­Π°Π»ΡŒΒ­Π½Ρ‹Ρ… сис­тС­мах от­счё­та) Ρ‚Π΅Π½Β­Π·ΠΎΡ€Β­Π½ΠΎΠΉ Ρ„ΠΎΡ€Β­ΠΌΠ΅, ΠΎΡ‚Β­ΠΊΡƒΒ­Π΄Π° ΠΌΠΎΒ­Π³ΡƒΡ‚ Π±Ρ‹Ρ‚ΡŒ ΠΏΠΎΒ­Π»ΡƒΒ­Ρ‡Π΅Β­Π½Ρ‹ Ρ„ΠΎΡ€Β­ΠΌΡƒΒ­Π»Ρ‹ прС­об­ра­зо­ва­ния ΠΏΠΎΒ­Π»Π΅Β­Π²Ρ‹Ρ… Π²Π΅ΠΊΒ­Ρ‚ΠΎΒ­Ρ€ΠΎΠ² $\boldsymbolΒ E,Β \boldsymbolΒ B,Β \boldsymbolΒ D$ ΠΈ $\boldsymbolΒ H$, Π° Ρ‚Π°ΠΊΒ­ΠΆΠ΅ $\boldsymbolΒ j$ ΠΈ $ρ$ ΠΏΡ€ΠΈ ΠΏΠ΅Β­Ρ€Π΅Β­Ρ…ΠΎΒ­Π΄Π΅ ΠΎΡ‚ ΠΎΠ΄Β­Π½ΠΎΠΉ ΠΈΠ½Π΅Ρ€Β­Ρ†ΠΈΒ­Π°Π»ΡŒΒ­Π½ΠΎΠΉ сис­тС­мы от­счё­та ΠΊ Π΄Ρ€ΡƒΒ­Π³ΠΎΠΉ.

М.Β Ρƒ. по­слу­Ти­ли Ρ‚Π΅ΠΎΒ­Ρ€Π΅Β­Ρ‚ΠΈΡ‡. ос­но­вой для соз­да­ния ΠΈ раз­ви­тия Ρ‚Π΅Ρ…Β­Π½ΠΈΒ­ΠΊΠΈ ра­дио­свя­зи ΠΈ тС­лС­ви­дС­ния, элСк­тро­тСх­ни­ки, элСк­тро­ни­ки ΠΈ Π΄Ρ€. Π½Π°Β­ΠΏΡ€Π°Π²Β­Π»Π΅Β­Π½ΠΈΠΉ совр. Π½Π°ΡƒΒ­ΠΊΠΈ ΠΈ Ρ‚Π΅Ρ…Β­Π½ΠΈΒ­ΠΊΠΈ.

1,2,3,4 уравнСния МаксвСлла для элСктромагнитного поля, смысл, ΡΡƒΡ‚ΡŒ, Ρ€Π΅ΡˆΠ΅Π½ΠΈΠ΅

УравнСния МаксвСлла Π² элСктродинамикС – это ΠΊΠ°ΠΊ Π·Π°ΠΊΠΎΠ½Ρ‹ ΠΡŒΡŽΡ‚ΠΎΠ½Π° Π² классичСской ΠΌΠ΅Ρ…Π°Π½ΠΈΠΊΠ΅ ΠΈΠ»ΠΈ ΠΊΠ°ΠΊ постулаты Π­ΠΉΠ½ΡˆΡ‚Π΅ΠΉΠ½Π° Π² Ρ‚Π΅ΠΎΡ€ΠΈΠΈ ΠΎΡ‚Π½ΠΎΡΠΈΡ‚Π΅Π»ΡŒΠ½ΠΎΡΡ‚ΠΈ. Π€ΡƒΠ½Π΄Π°ΠΌΠ΅Π½Ρ‚Π°Π»ΡŒΠ½Ρ‹Π΅ уравнСния, Π² сущности ΠΊΠΎΡ‚ΠΎΡ€Ρ‹Ρ… ΠΌΡ‹ сСгодня Π±ΡƒΠ΄Π΅ΠΌ Ρ€Π°Π·Π±ΠΈΡ€Π°Ρ‚ΡŒΡΡ, Ρ‡Ρ‚ΠΎΠ±Ρ‹ Π½Π΅ Π²ΠΏΠ°Π΄Π°Ρ‚ΡŒ Π² ступор ΠΎΡ‚ ΠΎΠ΄Π½ΠΎΠ³ΠΎ ΠΈΡ… упоминания.

ПолСзная ΠΈ интСрСсная информация ΠΏΠΎ Π΄Ρ€ΡƒΠ³ΠΈΠΌ тСмам – у нас Π² Ρ‚Π΅Π»Π΅Π³Ρ€Π°ΠΌ.

УравнСния МаксвСлла – это систСма ΡƒΡ€Π°Π²Π½Π΅Π½ΠΈΠΉ Π² Π΄ΠΈΡ„Ρ„Π΅Ρ€Π΅Π½Ρ†ΠΈΠ°Π»ΡŒΠ½ΠΎΠΉ ΠΈΠ»ΠΈ ΠΈΠ½Ρ‚Π΅Π³Ρ€Π°Π»ΡŒΠ½ΠΎΠΉ Ρ„ΠΎΡ€ΠΌΠ΅, ΠΎΠΏΠΈΡΡ‹Π²Π°ΡŽΡ‰Π°Ρ Π»ΡŽΠ±Ρ‹Π΅ элСктромагнитныС поля, связь ΠΌΠ΅ΠΆΠ΄Ρƒ Ρ‚ΠΎΠΊΠ°ΠΌΠΈ ΠΈ элСктричСскими зарядами Π² Π»ΡŽΠ±Ρ‹Ρ… срСдах.

УравнСния МаксвСлла Π½Π΅ΠΎΡ…ΠΎΡ‚Π½ΠΎ ΠΏΡ€ΠΈΠ½ΠΈΠΌΠ°Π»ΠΈΡΡŒ ΠΈ критичСски Π²ΠΎΡΠΏΡ€ΠΈΠ½ΠΈΠΌΠ°Π»ΠΈΡΡŒ ΡƒΡ‡Π΅Π½Ρ‹ΠΌΠΈ-соврСмСнниками МаксвСлла. ВсС ΠΏΠΎΡ‚ΠΎΠΌΡƒ, Ρ‡Ρ‚ΠΎ эти уравнСния Π½Π΅ Π±Ρ‹Π»ΠΈ ΠΏΠΎΡ…ΠΎΠΆΠΈ Π½ΠΈ Π½Π° Ρ‡Ρ‚ΠΎ ΠΈΠ· извСстного людям Ρ€Π°Π½Π΅Π΅.

Π’Π΅ΠΌ Π½Π΅ ΠΌΠ΅Π½Π΅Π΅, ΠΈ ΠΏΠΎ сСй дСнь Π½Π΅Ρ‚ Π½ΠΈΠΊΠ°ΠΊΠΈΡ… сомнСний Π² ΠΏΡ€Π°Π²ΠΈΠ»ΡŒΠ½ΠΎΡΡ‚ΠΈ ΡƒΡ€Π°Π²Π½Π΅Π½ΠΈΠΉ МаксвСлла, ΠΎΠ½ΠΈ Β«Ρ€Π°Π±ΠΎΡ‚Π°ΡŽΡ‚Β» Π½Π΅ Ρ‚ΠΎΠ»ΡŒΠΊΠΎ Π² ΠΏΡ€ΠΈΠ²Ρ‹Ρ‡Π½ΠΎΠΌ Π½Π°ΠΌ ΠΌΠ°ΠΊΡ€ΠΎΠΌΠΈΡ€Π΅, Π½ΠΎ ΠΈ Π² области ΠΊΠ²Π°Π½Ρ‚ΠΎΠ²ΠΎΠΉ ΠΌΠ΅Ρ…Π°Π½ΠΈΠΊΠΈ.

УравнСния МаксвСлла ΡΠΎΠ²Π΅Ρ€ΡˆΠΈΠ»ΠΈ настоящий ΠΏΠ΅Ρ€Π΅Π²ΠΎΡ€ΠΎΡ‚ Π² восприятии людьми Π½Π°ΡƒΡ‡Π½ΠΎΠΉ ΠΊΠ°Ρ€Ρ‚ΠΈΠ½Ρ‹ ΠΌΠΈΡ€Π°. Π’Π°ΠΊ, ΠΎΠ½ΠΈ прСдвосхитили ΠΎΡ‚ΠΊΡ€Ρ‹Ρ‚ΠΈΠ΅ Ρ€Π°Π΄ΠΈΠΎΠ²ΠΎΠ»Π½ ΠΈ ΠΏΠΎΠΊΠ°Π·Π°Π»ΠΈ, Ρ‡Ρ‚ΠΎ свСт ΠΈΠΌΠ΅Π΅Ρ‚ ΡΠ»Π΅ΠΊΡ‚Ρ€ΠΎΠΌΠ°Π³Π½ΠΈΡ‚Π½ΡƒΡŽ ΠΏΡ€ΠΈΡ€ΠΎΠ΄Ρƒ.

Β 

ΠšΡΡ‚Π°Ρ‚ΠΈ! Для всСх Π½Π°ΡˆΠΈΡ… Ρ‡ΠΈΡ‚Π°Ρ‚Π΅Π»Π΅ΠΉ сСйчас дСйствуСт скидка 10% на любой Π²ΠΈΠ΄ Ρ€Π°Π±ΠΎΡ‚Ρ‹.Β 

По порядку запишСм ΠΈ поясним всС 4 уравнСния. Π‘Ρ€Π°Π·Ρƒ ΡƒΡ‚ΠΎΡ‡Π½ΠΈΠΌ, Ρ‡Ρ‚ΠΎ Π·Π°ΠΏΠΈΡΡ‹Π²Π°Ρ‚ΡŒ ΠΈΡ… Π±ΡƒΠ΄Π΅ΠΌ Π² систСмС БИ.

ΠŸΠ΅Ρ€Π²ΠΎΠ΅ ΡƒΡ€Π°Π²Π½Π΅Π½ΠΈΠ΅ МаксвСлла

Π‘ΠΎΠ²Ρ€Π΅ΠΌΠ΅Π½Π½Ρ‹ΠΉ Π²ΠΈΠ΄ ΠΏΠ΅Ρ€Π²ΠΎΠ³ΠΎ уравнСния МаксвСлла Ρ‚Π°ΠΊΠΎΠ²:

Π’ΡƒΡ‚ Π½ΡƒΠΆΠ½ΠΎ ΠΏΠΎΡΡΠ½ΠΈΡ‚ΡŒ, Ρ‡Ρ‚ΠΎ Ρ‚Π°ΠΊΠΎΠ΅ дивСргСнция. ДивСргСнция – это Π΄ΠΈΡ„Ρ„Π΅Ρ€Π΅Π½Ρ†ΠΈΠ°Π»ΡŒΠ½Ρ‹ΠΉ ΠΎΠΏΠ΅Ρ€Π°Ρ‚ΠΎΡ€, ΠΎΠΏΡ€Π΅Π΄Π΅Π»ΡΡŽΡ‰ΠΈΠΉ ΠΏΠΎΡ‚ΠΎΠΊ ΠΊΠ°ΠΊΠΎΠ³ΠΎ-Ρ‚ΠΎ поля Ρ‡Π΅Ρ€Π΅Π· ΠΎΠΏΡ€Π΅Π΄Π΅Π»Π΅Π½Π½ΡƒΡŽ ΠΏΠΎΠ²Π΅Ρ€Ρ…Π½ΠΎΡΡ‚ΡŒ. УмСстным Π±ΡƒΠ΄Π΅Ρ‚ сравнСниС с ΠΊΡ€Π°Π½ΠΎΠΌ ΠΈΠ»ΠΈ с Ρ‚Ρ€ΡƒΠ±ΠΎΠΉ. НапримСр, Ρ‡Π΅ΠΌ большС Π΄ΠΈΠ°ΠΌΠ΅Ρ‚Ρ€ носика ΠΊΡ€Π°Π½Π° ΠΈ Π½Π°ΠΏΠΎΡ€ Π² Ρ‚Ρ€ΡƒΠ±Π΅, Ρ‚Π΅ΠΌ большим Π±ΡƒΠ΄Π΅Ρ‚ ΠΏΠΎΡ‚ΠΎΠΊ Π²ΠΎΠ΄Ρ‹ Ρ‡Π΅Ρ€Π΅Π· ΠΏΠΎΠ²Π΅Ρ€Ρ…Π½ΠΎΡΡ‚ΡŒ, ΠΊΠΎΡ‚ΠΎΡ€ΡƒΡŽ прСдставляСт собой носик.

Π’ ΠΏΠ΅Ρ€Π²ΠΎΠΌ ΡƒΡ€Π°Π²Π½Π΅Π½ΠΈΠΈ МаксвСлла E – это Π²Π΅ΠΊΡ‚ΠΎΡ€Π½ΠΎΠ΅ элСктричСскоС ΠΏΠΎΠ»Π΅, Π° грСчСская Π±ΡƒΠΊΠ²Π° Β«Ρ€ΠΎΒ» – суммарный заряд, Π·Π°ΠΊΠ»ΡŽΡ‡Π΅Π½Π½Ρ‹ΠΉ Π²Π½ΡƒΡ‚Ρ€ΠΈ Π·Π°ΠΌΠΊΠ½ΡƒΡ‚ΠΎΠΉ повСрхности.

Π’Π°ΠΊ Π²ΠΎΡ‚, ΠΏΠΎΡ‚ΠΎΠΊ элСктричСского поля E Ρ‡Π΅Ρ€Π΅Π· Π»ΡŽΠ±ΡƒΡŽ Π·Π°ΠΌΠΊΠ½ΡƒΡ‚ΡƒΡŽ ΠΏΠΎΠ²Π΅Ρ€Ρ…Π½ΠΎΡΡ‚ΡŒ зависит ΠΎΡ‚ суммарного заряда Π²Π½ΡƒΡ‚Ρ€ΠΈ этой повСрхности. Π”Π°Π½Π½ΠΎΠ΅ ΡƒΡ€Π°Π²Π½Π΅Π½ΠΈΠ΅ прСдставляСт собой Π·Π°ΠΊΠΎΠ½ (Ρ‚Π΅ΠΎΡ€Π΅ΠΌΡƒ) Гаусса.

Π’Ρ€Π΅Ρ‚ΡŒΠ΅ ΡƒΡ€Π°Π²Π½Π΅Π½ΠΈΠ΅ МаксвСлла

БСйчас ΠΌΡ‹ пропустим Π²Ρ‚ΠΎΡ€ΠΎΠ΅ ΡƒΡ€Π°Π²Π½Π΅Π½ΠΈΠ΅, Ρ‚Π°ΠΊ ΠΊΠ°ΠΊ Ρ‚Ρ€Π΅Ρ‚ΡŒΠ΅ ΡƒΡ€Π°Π²Π½Π΅Π½ΠΈΠ΅ МаксвСлла – это Ρ‚ΠΎΠΆΠ΅ Π·Π°ΠΊΠΎΠ½ Гаусса, Ρ‚ΠΎΠ»ΡŒΠΊΠΎ ΡƒΠΆΠ΅ Π½Π΅ для элСктричСского поля, Π½ΠΎ для ΠΌΠ°Π³Π½ΠΈΡ‚Π½ΠΎΠ³ΠΎ.

Оно ΠΈΠΌΠ΅Π΅Ρ‚ Π²ΠΈΠ΄:

Π§Ρ‚ΠΎ это Π·Π½Π°Ρ‡ΠΈΡ‚? ΠŸΠΎΡ‚ΠΎΠΊ ΠΌΠ°Π³Π½ΠΈΡ‚Π½ΠΎΠ³ΠΎ поля Ρ‡Π΅Ρ€Π΅Π· Π·Π°ΠΌΠΊΠ½ΡƒΡ‚ΡƒΡŽ ΠΏΠΎΠ²Π΅Ρ€Ρ…Π½ΠΎΡΡ‚ΡŒ Ρ€Π°Π²Π΅Π½ Π½ΡƒΠ»ΡŽ. Если элСктричСскиС заряды (ΠΏΠΎΠ»ΠΎΠΆΠΈΡ‚Π΅Π»ΡŒΠ½Ρ‹Π΅ ΠΈ ΠΎΡ‚Ρ€ΠΈΡ†Π°Ρ‚Π΅Π»ΡŒΠ½Ρ‹Π΅) Π²ΠΏΠΎΠ»Π½Π΅ ΠΌΠΎΠ³ΡƒΡ‚ ΡΡƒΡ‰Π΅ΡΡ‚Π²ΠΎΠ²Π°Ρ‚ΡŒ ΠΏΠΎ ΠΎΡ‚Π΄Π΅Π»ΡŒΠ½ΠΎΡΡ‚ΠΈ, пороТдая Π²ΠΎΠΊΡ€ΡƒΠ³ сСбя элСктричСскоС ΠΏΠΎΠ»Π΅, Ρ‚ΠΎ ΠΌΠ°Π³Π½ΠΈΡ‚Π½Ρ‹Ρ… зарядов Π² ΠΏΡ€ΠΈΡ€ΠΎΠ΄Π΅ просто Π½Π΅ сущСствуСт.

Π’Ρ‚ΠΎΡ€ΠΎΠ΅ ΡƒΡ€Π°Π²Π½Π΅Π½ΠΈΠ΅ МаксвСлла

Π’Ρ‚ΠΎΡ€ΠΎΠ΅ ΡƒΡ€Π°Π²Π½Π΅Π½ΠΈΠ΅ МаксвСлла прСдставляСт собой Π½ΠΈ Ρ‡Ρ‚ΠΎ ΠΈΠ½ΠΎΠ΅, ΠΊΠ°ΠΊ Π·Π°ΠΊΠΎΠ½ ЀарадСя. Π•Π³ΠΎ Π²ΠΈΠ΄:

Π ΠΎΡ‚ΠΎΡ€ элСктричСского поля (ΠΈΠ½Ρ‚Π΅Π³Ρ€Π°Π» Ρ‡Π΅Ρ€Π΅Π· Π·Π°ΠΌΠΊΠ½ΡƒΡ‚ΡƒΡŽ ΠΏΠΎΠ²Π΅Ρ€Ρ…Π½ΠΎΡΡ‚ΡŒ) Ρ€Π°Π²Π΅Π½ скорости измСнСния ΠΌΠ°Π³Π½ΠΈΡ‚Π½ΠΎΠ³ΠΎ ΠΏΠΎΡ‚ΠΎΠΊΠ°, ΠΏΡ€ΠΎΠ½ΠΈΠ·Ρ‹Π²Π°ΡŽΡ‰Π΅Π³ΠΎ эту ΠΏΠΎΠ²Π΅Ρ€Ρ…Π½ΠΎΡΡ‚ΡŒ. Π§Ρ‚ΠΎΠ±Ρ‹ Π»ΡƒΡ‡ΡˆΠ΅ ΠΏΠΎΠ½ΡΡ‚ΡŒ, возьмСм Π²ΠΎΠ΄Ρƒ Π² Π²Π°Π½Π½ΠΎΠΉ, которая сливаСтся Ρ‡Π΅Ρ€Π΅Π· отвСрстиС. Π’ΠΎΠΊΡ€ΡƒΠ³ отвСрстия образуСтся Π²ΠΎΡ€ΠΎΠ½ΠΊΠ°. Π ΠΎΡ‚ΠΎΡ€ – это сумма (ΠΈΠ½Ρ‚Π΅Π³Ρ€Π°Π») Π²Π΅ΠΊΡ‚ΠΎΡ€ΠΎΠ² скоростСй частиц Π²ΠΎΠ΄Ρ‹, ΠΊΠΎΡ‚ΠΎΡ€Ρ‹Π΅ Π²Ρ€Π°Ρ‰Π°ΡŽΡ‚ΡΡ Π²ΠΎΠΊΡ€ΡƒΠ³ отвСрстия.

Как Π’Ρ‹ ΠΏΠΎΠΌΠ½ΠΈΡ‚Π΅, Π½Π° основС Π·Π°ΠΊΠΎΠ½Π° ЀарадСя Ρ€Π°Π±ΠΎΡ‚Π°ΡŽΡ‚ элСктродвигатСли: Π²Ρ€Π°Ρ‰Π°ΡŽΡ‰ΠΈΠΉΡΡ ΠΌΠ°Π³Π½ΠΈΡ‚ ΠΏΠΎΡ€ΠΎΠΆΠ΄Π°Π΅Ρ‚ Ρ‚ΠΎΠΊ Π² ΠΊΠ°Ρ‚ΡƒΡˆΠΊΠ΅.

Π§Π΅Ρ‚Π²Π΅Ρ€Ρ‚ΠΎΠ΅ ΡƒΡ€Π°Π²Π½Π΅Π½ΠΈΠ΅ МаксвСлла

Π§Π΅Ρ‚Π²Π΅Ρ€Ρ‚ΠΎΠ΅ — самоС Π²Π°ΠΆΠ½ΠΎΠ΅ ΠΈΠ· всСх ΡƒΡ€Π°Π²Π½Π΅Π½ΠΈΠΉ МаксвСлла. ИмСнно Π² Π½Π΅ΠΌ ΡƒΡ‡Π΅Π½Ρ‹ΠΉ Π²Π²Π΅Π» понятиС Ρ‚ΠΎΠΊΠ° смСщСния.

Π­Ρ‚ΠΎ ΡƒΡ€Π°Π²Π½Π΅Π½ΠΈΠ΅ Π΅Ρ‰Π΅ называСтся Ρ‚Π΅ΠΎΡ€Π΅ΠΌΠΎΠΉ ΠΎ циркуляции Π²Π΅ΠΊΡ‚ΠΎΡ€Π° ΠΌΠ°Π³Π½ΠΈΡ‚Π½ΠΎΠΉ ΠΈΠ½Π΄ΡƒΠΊΡ†ΠΈΠΈ. Оно Π³ΠΎΠ²ΠΎΡ€ΠΈΡ‚ Π½Π°ΠΌ ΠΎ Ρ‚ΠΎΠΌ, Ρ‡Ρ‚ΠΎ элСктричСский Ρ‚ΠΎΠΊ ΠΈ ΠΈΠ·ΠΌΠ΅Π½Π΅Π½ΠΈΠ΅ элСктричСского поля ΠΏΠΎΡ€ΠΎΠΆΠ΄Π°ΡŽΡ‚ Π²ΠΈΡ…Ρ€Π΅Π²ΠΎΠ΅ ΠΌΠ°Π³Π½ΠΈΡ‚Π½ΠΎΠ΅ ΠΏΠΎΠ»Π΅.

ΠŸΡ€ΠΈΠ²Π΅Π΄Π΅ΠΌ Ρ‚Π΅ΠΏΠ΅Ρ€ΡŒ всю систСму ΡƒΡ€Π°Π²Π½Π΅Π½ΠΈΠΉ ΠΈ ΠΊΡ€Π°Ρ‚ΠΊΠΎ ΠΎΠ±ΠΎΠ·Π½Π°Ρ‡ΠΈΠΌ ΡΡƒΡ‚ΡŒ ΠΊΠ°ΠΆΠ΄ΠΎΠ³ΠΎ ΠΈΠ· Π½ΠΈΡ…:

ΠŸΠ΅Ρ€Π²ΠΎΠ΅ ΡƒΡ€Π°Π²Π½Π΅Π½ΠΈΠ΅: элСктричСский заряд ΠΏΠΎΡ€ΠΎΠΆΠ΄Π°Π΅Ρ‚ элСктричСскоС ΠΏΠΎΠ»Π΅

Π’Ρ‚ΠΎΡ€ΠΎΠ΅ ΡƒΡ€Π°Π²Π½Π΅Π½ΠΈΠ΅: ΠΈΠ·ΠΌΠ΅Π½ΡΡŽΡ‰Π΅Π΅ΡΡ ΠΌΠ°Π³Π½ΠΈΡ‚Π½ΠΎΠ΅ ΠΏΠΎΠ»Π΅ ΠΏΠΎΡ€ΠΎΠΆΠ΄Π°Π΅Ρ‚ Π²ΠΈΡ…Ρ€Π΅Π²ΠΎΠ΅ элСктричСскоС ΠΏΠΎΠ»Π΅

Π’Ρ€Π΅Ρ‚ΡŒΠ΅ ΡƒΡ€Π°Π²Π½Π΅Π½ΠΈΠ΅: ΠΌΠ°Π³Π½ΠΈΡ‚Π½Ρ‹Ρ… зарядов Π½Π΅ сущСствуСт

Π§Π΅Ρ‚Π²Π΅Ρ€Ρ‚ΠΎΠ΅ ΡƒΡ€Π°Π²Π½Π΅Π½ΠΈΠ΅: элСктричСский Ρ‚ΠΎΠΊ ΠΈ ΠΈΠ·ΠΌΠ΅Π½Π΅Π½ΠΈΠ΅ элСктричСской ΠΈΠ½Π΄ΡƒΠΊΡ†ΠΈΠΈ ΠΏΠΎΡ€ΠΎΠΆΠ΄Π°ΡŽΡ‚ Π²ΠΈΡ…Ρ€Π΅Π²ΠΎΠ΅ ΠΌΠ°Π³Π½ΠΈΡ‚Π½ΠΎΠ΅ ΠΏΠΎΠ»Π΅

РСшая уравнСния МаксвСлла для свободной элСктромагнитной Π²ΠΎΠ»Π½Ρ‹, ΠΌΡ‹ ΠΏΠΎΠ»ΡƒΡ‡ΠΈΠΌ ΡΠ»Π΅Π΄ΡƒΡŽΡ‰ΡƒΡŽ ΠΊΠ°Ρ€Ρ‚ΠΈΠ½Ρƒ Π΅Π΅ распространСния Π² пространствС:

НадССмся, эта ΡΡ‚Π°Ρ‚ΡŒΡ ΠΏΠΎΠΌΠΎΠΆΠ΅Ρ‚Β ΡΠΈΡΡ‚Π΅ΠΌΠ°Ρ‚ΠΈΠ·ΠΈΡ€ΠΎΠ²Π°Ρ‚ΡŒ знания ΠΎΠ± уравнСниях МаксвСлла. А Ссли ΠΏΠΎΠ½Π°Π΄ΠΎΠ±ΠΈΡ‚ΡŒΡΡ Ρ€Π΅ΡˆΠΈΡ‚ΡŒ Π·Π°Π΄Π°Ρ‡Ρƒ ΠΏΠΎ элСктродинамикС с ΠΏΡ€ΠΈΠΌΠ΅Π½Π΅Π½ΠΈΠ΅ΠΌ этих ΡƒΡ€Π°Π²Π½Π΅Π½ΠΈΠΉ, ΠΌΠΎΠΆΠ΅Ρ‚Π΅ смСло ΠΎΠ±Ρ€Π°Ρ‚ΠΈΡ‚ΡŒΡΡ Π·Π° ΠΏΠΎΠΌΠΎΡ‰ΡŒΡŽ Π² студСнчСский сСрвис. ΠŸΠΎΠ΄Ρ€ΠΎΠ±Π½ΠΎΠ΅ объяснСниС любого задания ΠΈ отличная ΠΎΡ†Π΅Π½ΠΊΠ°Β Π³Π°Ρ€Π°Π½Ρ‚ΠΈΡ€ΠΎΠ²Π°Π½Ρ‹.

ЭлСктричСство ΠΈ ΠΌΠ°Π³Π½Π΅Ρ‚ΠΈΠ·ΠΌ

Π§Π΅Ρ‚Ρ‹Ρ€Π΅ уравнСния, ΡΠΎΠΎΡ‚Π²Π΅Ρ‚ΡΡ‚Π²ΡƒΡŽΡ‰ΠΈΠ΅ нашим (ΠΌΠΎΠ΄ΠΈΡ„ΠΈΡ†ΠΈΡ€ΠΎΠ²Π°Π½Π½Ρ‹ΠΌ) утвСрТдСниям, Π½Π°Π·Ρ‹Π²Π°ΡŽΡ‚ΡΡ уравнСниями МаксвСлла Π² ΠΈΠ½Ρ‚Π΅Π³Ρ€Π°Π»ΡŒΠ½ΠΎΠΉ Ρ„ΠΎΡ€ΠΌΠ΅.

Π’Ρ‹ΠΏΠΈΡˆΠ΅ΠΌ ΠΈΡ… всС рядом Π΅Ρ‰Π΅ Ρ€Π°Π·:

Β 

Π§Ρ‚ΠΎΠ±Ρ‹ ΠΏΠΎΠ»ΡƒΡ‡ΠΈΡ‚ΡŒ уравнСния МаксвСлла Π² срСдС, Π½Π°Π΄ΠΎ произвСсти Π·Π°ΠΌΠ΅Π½Ρƒ:

,

Ρ‚ΠΎ Π΅ΡΡ‚ΡŒ ΡƒΠΊΠ°Π·Π°Ρ‚ΡŒ связь (Ρ‚Π°ΠΊ Π½Π°Π·Ρ‹Π²Π°Π΅ΠΌΡ‹Π΅ Β«ΠΌΠ°Ρ‚Π΅Ρ€ΠΈΠ°Π»ΡŒΠ½Ρ‹Π΅Β» уравнСния) ΠΌΠ΅ΠΆΠ΄Ρƒ напряТСнностями ΠΈ индукциями: Β ΠΈ Β ΠΈ Π΄ΠΎΠΏΠΎΠ»Π½ΠΈΡ‚ΡŒ систСму ΡƒΡ€Π°Π²Π½Π΅Π½ΠΈΠ΅ΠΌ Π·Π°ΠΊΠΎΠ½Π° Ома

.

ΠžΡ‚ΠΌΠ΅Ρ‚ΠΈΠΌ, Ρ‡Ρ‚ΠΎ ΠΏΡ€ΠΈΠ²Π΅Π΄Π΅Π½Π½Ρ‹ΠΌΠΈ Π²Ρ‹ΡˆΠ΅ ΠΏΡ€ΠΎΡΡ‚Π΅ΠΉΡˆΠΈΠΌΠΈ ΡΠΎΠΎΡ‚Π½ΠΎΡˆΠ΅Π½ΠΈΡΠΌΠΈ ΠΌΠΎΠΆΠ½ΠΎ ΠΏΠΎΠ»ΡŒΠ·ΠΎΠ²Π°Ρ‚ΡŒΡΡ Π½Π΅ всСгда. Битуация Π·Π°ΠΌΠ΅Ρ‚Π½ΠΎ слоТнСС Π² присутствии Ρ‚Π°ΠΊΠΈΡ… вСщСств ΠΊΠ°ΠΊ сСгнСтоэлСктрики, ΠΏΡŒΠ΅Π·ΠΎΡΠ»Π΅ΠΊΡ‚Ρ€ΠΈΠΊΠΈ, Ρ„Π΅Ρ€Ρ€ΠΎΠΌΠ°Π³Π½Π΅Ρ‚ΠΈΠΊΠΈ, вСщСства Π°Π½ΠΈΠ·ΠΎΡ‚Ρ€ΠΎΠΏΠ½Ρ‹Π΅ ΠΈ Ρ‚ΠΎΠΌΡƒ ΠΏΠΎΠ΄ΠΎΠ±Π½ΠΎΠ΅. Π—Π΄Π΅ΡΡŒ наша Ρ†Π΅Π»ΡŒ ΠΏΠΎΠΊΠ°Π·Π°Ρ‚ΡŒ, ΠΊΠ°ΠΊ формируСтся полная систСма ΡƒΡ€Π°Π²Π½Π΅Π½ΠΈΠΉ, ΠΏΠΎΠ·Π²ΠΎΠ»ΡΡŽΡ‰Π°Ρ (с ΡƒΡ‡Π΅Ρ‚ΠΎΠΌ Π½Π°Ρ‡Π°Π»ΡŒΠ½Ρ‹Ρ… ΠΈ Π³Ρ€Π°Π½ΠΈΡ‡Π½Ρ‹Ρ… условий, разумССтся) Ρ€Π°ΡΡΡ‡ΠΈΡ‚Π°Ρ‚ΡŒ элСктромагнитноС ΠΏΠΎΠ»Π΅.Β 

ΠžΡ‚ ΡƒΡ€Π°Π²Π½Π΅Π½ΠΈΠΉ Π² ΠΈΠ½Ρ‚Π΅Π³Ρ€Π°Π»ΡŒΠ½ΠΎΠΉ Ρ„ΠΎΡ€ΠΌΠ΅ ΠΌΠΎΠΆΠ½ΠΎ с ΠΏΠΎΠΌΠΎΡ‰ΡŒΡŽ Ρ‚Π΅ΠΎΡ€Π΅ΠΌ Π²Π΅ΠΊΡ‚ΠΎΡ€Π½ΠΎΠ³ΠΎ Π°Π½Π°Π»ΠΈΠ·Π° ΠΏΠ΅Ρ€Π΅ΠΉΡ‚ΠΈ ΠΊ уравнСниям Π² Π΄ΠΈΡ„Ρ„Π΅Ρ€Π΅Π½Ρ†ΠΈΠ°Π»ΡŒΠ½ΠΎΠΉ Ρ„ΠΎΡ€ΠΌΠ΅, ΡΠ²ΡΠ·Ρ‹Π²Π°ΡŽΡ‰ΠΈΠΌ значСния ΠΏΠΎΠ»Π΅ΠΉ Β ΠΈ Β ΠΈ ΠΈΡ… пространствСнных ΠΈ Π²Ρ€Π΅ΠΌΠ΅Π½Π½Ρ‹Ρ… ΠΏΡ€ΠΎΠΈΠ·Π²ΠΎΠ΄Π½Ρ‹Ρ… со значСниями плотностСй заряда ΠΈ Ρ‚ΠΎΠΊΠ°. Π­Ρ‚ΠΈΠΌΠΈ уравнСниями ΠΌΡ‹ ΠΏΠΎΠ»ΡŒΠ·ΠΎΠ²Π°Ρ‚ΡŒΡΡ Π½Π΅ Π±ΡƒΠ΄Π΅ΠΌ, Π½ΠΎ всС ΠΆΠ΅ ΠΏΡ€ΠΈΠ²Π΅Π΄Π΅ΠΌ ΠΈΡ… хотя Π±Ρ‹ ΠΊΠ°ΠΊ Ρ‡Π°ΡΡ‚ΡŒ ΡˆΡƒΡ‚ΠΊΠΈ, ΠΎΠΏΡƒΠ±Π»ΠΈΠΊΠΎΠ²Π°Π½Π½ΠΎΠΉ Π² ΠΎΠ΄Π½ΠΎΠΌ ΠΈΠ· ΠΆΡƒΡ€Π½Π°Π»ΠΎΠ² Π² Π΄Π½ΠΈ юбилСя МаксвСлла:Β 

«И сказал Π‘ΠΎΠ³:

И стал свСт».Β 

НСпонятныС Π·Π½Π°Ρ‡ΠΊΠΈ div (читаСтся «дивСргСнция») ΠΈ rot (читаСтся Β«Ρ€ΠΎΡ‚ΠΎΡ€Β») β€” это особыС ΠΎΠΏΠ΅Ρ€Π°Ρ†ΠΈΠΈ диффСрСнцирования, выполняСмыС Π½Π°Π΄ Π²Π΅ΠΊΡ‚ΠΎΡ€Π½Ρ‹ΠΌΠΈ полями. ДивСргСнция β€” ΠΏΠΎ Π»Π°Ρ‚Ρ‹Π½ΠΈ «расхоТдСниС». Π­Ρ‚Π° опСрация описываСт ΠΊΠΎΠ½Ρ„ΠΈΠ³ΡƒΡ€Π°Ρ†ΠΈΡŽ силовых Π»ΠΈΠ½ΠΈΠΉ Ρ‚ΠΈΠΏΠ° Β«Π΅ΠΆΠ°Β», расходящихся ΠΈΠ· Ρ‚ΠΎΡ‡Π΅ΠΊ, Π³Π΄Π΅ ΠΈΠΌΠ΅ΡŽΡ‚ΡΡ элСктричСскиС заряды. Π‘Π»ΠΎΠ²ΠΎ Β«Ρ€ΠΎΡ‚ΠΎΡ€Β» Π² ΠΏΠ΅Ρ€Π΅Π²ΠΎΠ΄Π΅ Π½Π΅ нуТдаСтся, ΠΎΠ½ΠΎ явно ассоциируСтся с Π²Ρ€Π°Ρ‰Π΅Π½ΠΈΠ΅ΠΌ. Π­Ρ‚Π° опСрация описываСт Π²ΠΈΡ…Ρ€Π΅Π²Ρ‹Π΅ поля (ΠΊΠΎΠ»ΡŒΡ†Π΅ΠΎΠ±Ρ€Π°Π·Π½Ρ‹Π΅ β€” Π·Π°ΠΌΠΊΠ½ΡƒΡ‚Ρ‹Π΅ силовыС Π»ΠΈΠ½ΠΈΠΈ) Π²ΠΎΠΊΡ€ΡƒΠ³ ΠΈΡ… источников β€” Ρ‚ΠΎΠΊΠΎΠ² ΠΈΠ»ΠΈ Π΄Ρ€ΡƒΠ³ΠΈΡ… ΠΏΠΎΠ»Π΅ΠΉ, ΠΌΠ΅Π½ΡΡŽΡ‰ΠΈΡ…ΡΡ Π²ΠΎ Π²Ρ€Π΅ΠΌΠ΅Π½ΠΈ.Β 

Π§Π΅Ρ‚Ρ‹Ρ€Π΅ ΠΈΠ½Ρ‚Π΅Π³Ρ€Π°Π»ΡŒΠ½Ρ‹Ρ… уравнСния ΠΈ Ρ‡Π΅Ρ‚Ρ‹Ρ€Π΅ Π΄ΠΈΡ„Ρ„Π΅Ρ€Π΅Π½Ρ†ΠΈΠ°Π»ΡŒΠ½Ρ‹Ρ… эквивалСнтны. МаксвСлл ΠΏΠΎΠΊΠ°Π·Π°Π», Ρ‡Ρ‚ΠΎ всС явлСния элСктромагнСтизма ΠΌΠΎΠΆΠ½ΠΎ ΠΏΠΎΠ»Π½ΠΎΡΡ‚ΡŒΡŽ ΠΎΠΏΠΈΡΠ°Ρ‚ΡŒ этими Ρ‡Π΅Ρ‚Ρ‹Ρ€ΡŒΠΌΡ уравнСниями, ΡΠ²Π»ΡΡŽΡ‰ΠΈΠΌΠΈΡΡ ΠΎΠ±ΠΎΠ±Ρ‰Π΅Π½ΠΈΠ΅ΠΌ ΡΠΊΡΠΏΠ΅Ρ€ΠΈΠΌΠ΅Π½Ρ‚Π°Π»ΡŒΠ½Ρ‹Ρ… Ρ„Π°ΠΊΡ‚ΠΎΠ².

Π’ ΠΏΡ€ΠΈΠ²Π΅Π΄Π΅Π½Π½ΠΎΠΉ ΡˆΡƒΡ‚ΠΊΠ΅ упоминался свСт. Π”Π΅ΠΉΡΡ‚Π²ΠΈΡ‚Π΅Π»ΡŒΠ½ΠΎ, свСт β€” это элСктромагнитноС ΠΈΠ·Π»ΡƒΡ‡Π΅Π½ΠΈΠ΅ ΠΎΠΏΡ€Π΅Π΄Π΅Π»Π΅Π½Π½ΠΎΠ³ΠΎ Π΄ΠΈΠ°ΠΏΠ°Π·ΠΎΠ½Π° частот. ΠŸΡ€Π΅Π΄ΡΠΊΠ°Π·Π°Π½ΠΈΠ΅ элСктромагнитных Π²ΠΎΠ»Π½ стало ΠΎΠ΄Π½ΠΈΠΌ ΠΈΠ· Π²Π΅Π»ΠΈΡ‡Π°ΠΉΡˆΠΈΡ… достиТСний Ρ‚Π΅ΠΎΡ€ΠΈΠΈ МаксвСлла. ΠŸΡ€Π΅Π΄ΡΡ‚Π°Π²ΠΈΠΌ сСбС, Ρ‡Ρ‚ΠΎ заряды ΠΈ Ρ‚ΠΎΠΊΠΈ ΠΎΡ‚ΡΡƒΡ‚ΡΡ‚Π²ΡƒΡŽΡ‚. ΠŸΠΎΡΠΌΠΎΡ‚Ρ€ΠΈΠΌ Π½Π° уравнСния МаксвСлла Π² Π΄ΠΈΡ„Ρ„Π΅Ρ€Π΅Π½Ρ†ΠΈΠ°Π»ΡŒΠ½ΠΎΠΉ Ρ„ΠΎΡ€ΠΌΠ΅. Π’ΠΈΠ΄Π½ΠΎ, Ρ‡Ρ‚ΠΎ Ссли поля Π½Π΅ статичСскиС, Π½ΠΎ зависят ΠΎΡ‚ Π²Ρ€Π΅ΠΌΠ΅Π½ΠΈ, Ρ‚ΠΎ имССтся Π²ΠΈΡ…Ρ€Π΅Π²ΠΎΠ΅ элСктричСскоС ΠΈ ΠΌΠ°Π³Π½ΠΈΡ‚Π½Ρ‹Π΅ поля (ΡΠΎΠΎΡ‚Π²Π΅Ρ‚ΡΡ‚Π²ΡƒΡŽΡ‰ΠΈΠ΅ Ρ€ΠΎΡ‚ΠΎΡ€Ρ‹ ΠΎΡ‚Π»ΠΈΡ‡Π½Ρ‹ ΠΎΡ‚ нуля). РаспространСниС ΠΏΠΎΠ»Π΅ΠΉ Π±Π΅Π· зарядов ΠΈ Ρ‚ΠΎΠΊΠΎΠ² β€” это ΠΈ Π΅ΡΡ‚ΡŒ элСктромагнитныС Π²ΠΎΠ»Π½Ρ‹. И ΠΌΠΎΠΆΠ½ΠΎ ΡƒΠ³Π»ΡΠ΄Π΅Ρ‚ΡŒ Π² уравнСниях Π½Π°ΠΌΠ΅ΠΊ Π½Π° ΡΠΊΠΎΡ€ΠΎΡΡ‚ΡŒ ΠΈΡ… распространСния: Ρ‚ΡƒΠ΄Π° Π²Ρ…ΠΎΠ΄ΠΈΡ‚ комбинация e0m0, Ρ‡Π΅Ρ€Π΅Π· ΠΊΠΎΡ‚ΠΎΡ€ΡƒΡŽ ΠΌΠΎΠΆΠ΅Ρ‚ Π±Ρ‹Ρ‚ΡŒ Π²Ρ‹Ρ€Π°ΠΆΠ΅Π½Π° ΡΠΊΠΎΡ€ΠΎΡΡ‚ΡŒ свСта Π² Π²Π°ΠΊΡƒΡƒΠΌΠ΅ (см.Β (6.3))

Но ΠΎΠ± этом β€” ΠΏΠΎΠ·ΠΆΠ΅, Π² ΡΠ»Π΅Π΄ΡƒΡŽΡ‰Π΅ΠΉ части нашСго курса.Β 

Π’ Π·Π°ΠΊΠ»ΡŽΡ‡Π΅Π½ΠΈΠ΅ ΠΆΠ΅ этой части ΠΏΡ€ΠΎΡ†ΠΈΡ‚ΠΈΡ€ΡƒΠ΅ΠΌ слова Π“. Π“Π΅Ρ€Ρ†Π° ΠΎΠ± уравнСниях МаксвСлла:

Β«Π’Ρ€ΡƒΠ΄Π½ΠΎ ΠΈΠ·Π±Π°Π²ΠΈΡ‚ΡŒΡΡ ΠΎΡ‚ чувства, Ρ‡Ρ‚ΠΎ эти матСматичСскиС Ρ„ΠΎΡ€ΠΌΡƒΠ»Ρ‹ ΠΆΠΈΠ²ΡƒΡ‚ нСзависимой Тизнью ΠΈ ΠΎΠ±Π»Π°Π΄Π°ΡŽΡ‚ своим собствСнным ΠΈΠ½Ρ‚Π΅Π»Π»Π΅ΠΊΡ‚ΠΎΠΌ, Ρ‡Ρ‚ΠΎ ΠΎΠ½ΠΈ ΠΌΡƒΠ΄Ρ€Π΅Π΅, Ρ‡Π΅ΠΌ ΠΌΡ‹ сами, ΠΌΡƒΠ΄Ρ€Π΅Π΅ Π΄Π°ΠΆΠ΅, Ρ‡Π΅ΠΌ ΠΈΡ… ΠΏΠ΅Ρ€Π²ΠΎΠΎΡ‚ΠΊΡ€Ρ‹Π²Π°Ρ‚Π΅Π»ΠΈ, ΠΈ Ρ‡Ρ‚ΠΎ ΠΌΡ‹ ΠΈΠ·Π²Π»Π΅ΠΊΠ°Π΅ΠΌ ΠΈΠ· Π½ΠΈΡ… большС, Ρ‡Π΅ΠΌ Π±Ρ‹Π»ΠΎ Π·Π°Π»ΠΎΠΆΠ΅Π½ΠΎ Π² Π½ΠΈΡ… ΠΏΠ΅Ρ€Π²ΠΎΠ½Π°Ρ‡Π°Π»ΡŒΠ½ΠΎΒ».

УравнСния МаксвСлла для элСктромагнитного поля

Π’ линиях элСктропСрСдач, элСктротСхничСских ΠΈ элСктронных устройствах ΠΈΠΌΠ΅ΡŽΡ‚ мСсто Ρ†Π΅Π»Ρ‹Π΅ систСмы заряТСнных частиц, Ρ‚Π΅Π» ΠΈ ΠΊΠΎΠ½Ρ‚ΡƒΡ€ΠΎΠ² с Ρ‚ΠΎΠΊΠ°ΠΌΠΈ, Π²Π·Π°ΠΈΠΌΠΎΠ΄Π΅ΠΉΡΡ‚Π²ΡƒΡŽΡ‰ΠΈΡ… Π΄Ρ€ΡƒΠ³ с Π΄Ρ€ΡƒΠ³ΠΎΠΌ. Π’ Ρ€Π΅Π·ΡƒΠ»ΡŒΡ‚Π°Ρ‚Π΅ этих взаимодСйствий Π²ΠΎΠ·Π½ΠΈΠΊΠ°ΡŽΡ‚ элСктромагнитныС явлСния, ΠΊΠΎΡ‚ΠΎΡ€Ρ‹Π΅ зависят Π½Π΅ Ρ‚ΠΎΠ»ΡŒΠΊΠΎ ΠΎΡ‚ процСссов Π² заряТСнных частицах, Ρ‚Π΅Π»Π°Ρ… ΠΈ ΠΊΠΎΠ½Ρ‚ΡƒΡ€Π°Ρ… с Ρ‚ΠΎΠΊΠ°ΠΌΠΈ, Π½ΠΎ ΠΈ ΠΎΠΊΡ€ΡƒΠΆΠ°ΡŽΡ‰Π΅ΠΉ срСды, Π² ΠΊΠΎΡ‚ΠΎΡ€ΠΎΠΉ распространяСтся элСктромагнитноС ΠΏΠΎΠ»Π΅. ЭлСктричСскоС ΠΏΠΎΠ»Π΅ принято Ρ…Π°Ρ€Π°ΠΊΡ‚Π΅Ρ€ΠΈΠ·ΠΎΠ²Π°Ρ‚ΡŒ Π²Π΅ΠΊΡ‚ΠΎΡ€ΠΎΠΌ элСктричСской напряТённости- Π•, Π° ΠΌΠ°Π³Π½ΠΈΡ‚Π½ΠΎΠ΅ ΠΏΠΎΠ»Π΅ Π²Π΅ΠΊΡ‚ΠΎΡ€ΠΎΠΌ ΠΌΠ°Π³Π½ΠΈΡ‚Π½ΠΎΠΉ напряТённости – H.

Π’ макроскопичСском ΠΏΠΎΠ½ΠΈΠΌΠ°Π½ΠΈΠΈ (Ρ‚.Π΅. Π² Ρ†Π΅Π»ΠΎΠΌ) процСссы Π² элСктромагнитных полях ΠΎΠΏΠΈΡΡ‹Π²Π°ΡŽΡ‚ уравнСниями МаксвСлла, ΠΊΠΎΡ‚ΠΎΡ€Ρ‹Π΅ основаны Π½Π° ΠΎΠ±ΠΎΠ±Ρ‰Π΅Π½ΠΈΠΈ Π·Π°ΠΊΠΎΠ½ΠΎΠ² АмпСра, ЀарадСя ΠΈ Гаусса. УравнСния МаксвСлла, Π² ΠΈΠ½Ρ‚Π΅Π³Ρ€Π°Π»ΡŒΠ½ΠΎΠΉ Ρ„ΠΎΡ€ΠΌΠ΅, ΠΏΡ€ΠΈΠ²Π΅Π΄Π΅Π½Ρ‹ Π½ΠΈΠΆΠ΅, Π² Ρ‚Π°Π±Π»ΠΈΡ†Π΅ 1.1.

ЭнСргия, носитСлСм ΠΊΠΎΡ‚ΠΎΡ€ΠΎΠΉ выступаСт элСктромагнитноС ΠΏΠΎΠ»Π΅:

Π³Π΄Π΅Β  Ξ΅Π° = ΡΡ0, ΞΌΠ° = ΞΌΞΌ0 β€” Π°Π±ΡΠΎΠ»ΡŽΡ‚Π½Ρ‹Π΅ элСктричСская ΠΈ магнитная проницаСмости срСды, Π² ΠΊΠΎΡ‚ΠΎΡ€ΠΎΠΉ находится элСктромагнитноС ΠΏΠΎΠ»Π΅ ; Β Ξ΅ = Ξ΅Π° / Ξ΅0, ΞΌ = ΞΌΠ° / ΞΌ0 β€” значСния ΠΎΡ‚Π½ΠΎΡΠΈΡ‚Π΅Π»ΡŒΠ½ΠΎΠΉ элСктричСской ΠΈ ΠΌΠ°Π³Π½ΠΈΡ‚Π½ΠΎΠΉ проницаСмости ; Β Ξ΅0 = 8,854Β·10-12 Π€/ΠΌ, ΞΌ0 = 1,257Β·10-6 Π“Π½/ΠΌ β€” элСктричСская ΠΈ магнитная ΠΏΡ€ΠΎΠ½ΠΈΡ†Π°Π΅ΠΌΠΎΡΡ‚ΡŒ Π²Π°ΠΊΡƒΡƒΠΌΠ°. Π’ ΡΠΎΠΎΡ‚Π²Π΅Ρ‚ΡΡ‚Π²ΡƒΡŽΡ‰ΠΈΡ… физичСских Ρ‚Π°Π±Π»ΠΈΡ†Π°Ρ…, Π² зависимости ΠΎΡ‚ Ρ€Π΅Π΄Π°ΠΊΡ†ΠΈΠΈ, ΠΌΠΎΠ³ΡƒΡ‚ ΠΏΡ€ΠΈΠ²ΠΎΠ΄ΠΈΡ‚ΡŒΡΡ ΠΊΠ°ΠΊ ΠΎΡ‚Π½ΠΎΡΠΈΡ‚Π΅Π»ΡŒΠ½Ρ‹Π΅, Ρ‚Π°ΠΊ ΠΈ Π°Π±ΡΠΎΠ»ΡŽΡ‚Π½Ρ‹Π΅ значСния Π²Ρ‹ΡˆΠ΅ΡƒΠΏΠΎΠΌΡΠ½ΡƒΡ‚Ρ‹Ρ… Π²Π΅Π»ΠΈΡ‡ΠΈΠ½.

Π’ Π²Π°ΠΊΡƒΡƒΠΌΠ΅, элСктромагнитныС Π²ΠΎΠ»Π½Ρ‹ ΠΏΠ΅Ρ€Π΅ΠΌΠ΅Ρ‰Π°ΡŽΡ‚ΡΡ со ΡΠΊΠΎΡ€ΠΎΡΡ‚ΡŒΡŽ с = (Ξ΅0ΞΌ0)-1/2 =2,999Β·108 ΠΌ/с, Π° Π² срСдС  со ΡΠΊΠΎΡ€ΠΎΡΡ‚ΡŒΡŽ Ο… = с(Ρ·μ)-1/2. ΠžΡ‚Π½ΠΎΡˆΠ΅Π½ΠΈΠ΅ скоростСй распространСния элСктромагнитных Π²ΠΎΠ»Π½ Π² Π²Π°ΠΊΡƒΡƒΠΌΠ΅ ΠΈ срСдС  Β  n = c/Ο…=(Ρ·μ)1/2 принято Π½Π°Π·Ρ‹Π²Π°Ρ‚ΡŒ ΠΏΠΎΠΊΠ°Π·Π°Ρ‚Π΅Π»Π΅ΠΌ прСломлСния срСды.

Π’ уравнСниях МаксвСлла, элСктромагнитныС свойства срСды, Π² ΠΎΠ±Ρ‰Π΅ΠΌ случаС, ΡƒΡ‡ΠΈΡ‚Ρ‹Π²Π°ΡŽΡ‚ ΠΈΠ· ΠΎΠΏΡ‹Ρ‚Π° значСниями ΡƒΠ΄Π΅Π»ΡŒΠ½ΠΎΠΉ ΠΎΠ±ΡŠΡ‘ΠΌΠ½ΠΎΠΉ проводимости Ξ³, элСктричСской Ξ΅ ΠΈ ΠΌΠ°Π³Π½ΠΈΡ‚Π½ΠΎΠΉ ΞΌ проницаСмостСй Π² соотвСтствии с уравнСниями: Ξ΄ΠΏΡ€ = Ξ³Π•, D = Ξ΅E, Β B = ΞΌH.

Π“Π΄Π΅;Β  Ξ΄ΠΏΡ€ β€” ΠΏΠ»ΠΎΡ‚Π½ΠΎΡΡ‚ΡŒ элСктричСского Ρ‚ΠΎΠΊΠ° проводимости;

D – Π²Π΅ΠΊΡ‚ΠΎΡ€ ΠΈΠ½Π΄ΡƒΠΊΡ†ΠΈΠΈ элСктричСского поля;

B – Π²Π΅ΠΊΡ‚ΠΎΡ€ ΠΈΠ½Π΄ΡƒΠΊΡ†ΠΈΠΈ ΠΌΠ°Π³Π½ΠΈΡ‚Π½ΠΎΠ³ΠΎ поля.

Π”ΠΎ настоящСго Π²Ρ€Π΅ΠΌΠ΅Π½ΠΈ (с 1873Π³.) Π½Π΅ Π±Ρ‹Π»ΠΎ Π½ΠΈ ΠΎΠ΄Π½ΠΎΠ³ΠΎ Ρ„Π°ΠΊΡ‚Π° ставящСго ΠΏΠΎΠ΄ сомнСния уравнСния МаксвСлла, хотя срСди Π½Π΅ΠΊΠΎΡ‚ΠΎΡ€Ρ‹Ρ… Ρ„ΠΈΠ·ΠΈΠΊΠΎΠ² ΠΈ сущСствуСт ΠΌΠ½Π΅Π½ΠΈΠ΅ ΠΎΠ± ΠΈΡ… нСкоррСктности. ΠŸΡ€ΠΈΠΌΠ΅Π½ΡΡŽΡ‚ уравнСния МаксвСлла Π½Π΅ Ρ‚ΠΎΠ»ΡŒΠΊΠΎ для описания явлСний элСктромагнитного поля. Они вошли ΠΈ Π² ΠΊΠ²Π°Π½Ρ‚ΠΎΠ²ΡƒΡŽ ΠΌΠ΅Ρ…Π°Π½ΠΈΠΊΡƒ, ΠΏΠΎΠ»ΠΎΠΆΠΈΠ² Π½Π°Ρ‡Π°Π»ΠΎ ΠΊΠ²Π°Π½Ρ‚ΠΎΠ²ΠΎΠΉ элСктродинамикС, Π° Ρ‚Π°ΠΊΠΆΠ΅ Π±Ρ‹Π»ΠΈ ΠΎΠ΄Π½ΠΎΠΉ ΠΈΠ· ΠΎΡ‚ΠΏΡ€Π°Π²Π½Ρ‹Ρ… Ρ‚ΠΎΡ‡Π΅ΠΊ ΠΏΡ€ΠΈ создании ΠΎΠ±Ρ‰Π΅ΠΉ Ρ‚Π΅ΠΎΡ€ΠΈΠΈ ΠΎΡ‚Π½ΠΎΡΠΈΡ‚Π΅Π»ΡŒΠ½ΠΎΡΡ‚ΠΈ ΠΠ»ΡŒΠ±Π΅Ρ€Ρ‚Π° Π­ΠΉΠ½ΡˆΡ‚Π΅ΠΉΠ½Π° (отчасти ΠΎΡ‚ Π½ΠΈΡ… ΠΏΠΎΠΏΠ°Π»Π° Π² Ρ„ΠΎΡ€ΠΌΡƒΠ»Ρ‹ Ρ‚Π΅ΠΎΡ€ΠΈΠΈ ΠΎΡ‚Π½ΠΎΡΠΈΡ‚Π΅Π»ΡŒΠ½ΠΎΡΡ‚ΠΈ ΡΠΊΠΎΡ€ΠΎΡΡ‚ΡŒ свСта).

УравнСния МаксвСлла

УравнСния МаксвСлла — это Π½Π°Π±ΠΎΡ€ ΠΈΠ· 4 слоТных ΡƒΡ€Π°Π²Π½Π΅Π½ΠΈΠΉ, ΠΎΠΏΠΈΡΡ‹Π²Π°ΡŽΡ‰ΠΈΡ… ΠΌΠΈΡ€ элСктромагнСтизма. Π­Ρ‚ΠΈ уравнСния ΠΎΠΏΠΈΡΡ‹Π²Π°ΡŽΡ‚, ΠΊΠ°ΠΊ элСктричСскиС ΠΈ ΠΌΠ°Π³Π½ΠΈΡ‚Π½Ρ‹Π΅ поля Ρ€Π°ΡΠΏΡ€ΠΎΡΡ‚Ρ€Π°Π½ΡΡŽΡ‚ΡΡ, Π²Π·Π°ΠΈΠΌΠΎΠ΄Π΅ΠΉΡΡ‚Π²ΡƒΡŽΡ‚ ΠΈ ΠΊΠ°ΠΊ Π½Π° Π½ΠΈΡ… Π²Π»ΠΈΡΡŽΡ‚. ΠΏΠΎ ΠΎΠ±ΡŠΠ΅ΠΊΡ‚Π°ΠΌ.

ДТСймс ΠšΠ»Π΅Ρ€ΠΊ МаксвСлл [1831-1879] Π±Ρ‹Π» Π³Π΅Π½ΠΈΠ΅ΠΌ уровня Π­ΠΉΠ½ΡˆΡ‚Π΅ΠΉΠ½Π° / ΠΡŒΡŽΡ‚ΠΎΠ½Π°, ΠΊΠΎΡ‚ΠΎΡ€Ρ‹ΠΉ взял Π½Π°Π±ΠΎΡ€ извСстных ΡΠΊΡΠΏΠ΅Ρ€ΠΈΠΌΠ΅Π½Ρ‚Π°Π»ΡŒΠ½Ρ‹Ρ… Π·Π°ΠΊΠΎΠ½ΠΎΠ² (Π·Π°ΠΊΠΎΠ½ ЀарадСя, Π·Π°ΠΊΠΎΠ½ АмпСра) ΠΈ объСдинил ΠΈΡ… Π² симмСтричный согласованный Π½Π°Π±ΠΎΡ€ ΡƒΡ€Π°Π²Π½Π΅Π½ΠΈΠΉ, извСстных ΠΊΠ°ΠΊ УравнСния МаксвСлла.МаксвСлл ΠΎΠ΄Π½ΠΈΠΌ ΠΈΠ· ΠΏΠ΅Ρ€Π²Ρ‹Ρ… ΠΎΠΏΡ€Π΅Π΄Π΅Π»ΠΈΠ» ΡΠΊΠΎΡ€ΠΎΡΡ‚ΡŒ распространСния элСктромагнитных Π²ΠΎΠ»Π½. (ЭМ) Π²ΠΎΠ»Π½Ρ‹ Π±Ρ‹Π»ΠΈ Ρ‚Π°ΠΊΠΈΠΌΠΈ ΠΆΠ΅, ΠΊΠ°ΠΊ ΡΠΊΠΎΡ€ΠΎΡΡ‚ΡŒ свСта — ΠΈ, ΡΠ»Π΅Π΄ΠΎΠ²Π°Ρ‚Π΅Π»ΡŒΠ½ΠΎ, ΠΌΠΎΠΆΠ½ΠΎ Π·Π°ΠΊΠ»ΡŽΡ‡ΠΈΡ‚ΡŒ, Ρ‡Ρ‚ΠΎ ЭМ-Π²ΠΎΠ»Π½Ρ‹ ΠΈ Π²ΠΈΠ΄ΠΈΠΌΡ‹Π΅ свСт Π±Ρ‹Π»ΠΈ Π΄Π΅ΠΉΡΡ‚Π²ΠΈΡ‚Π΅Π»ΡŒΠ½ΠΎ Ρ‚ΠΎ ΠΆΠ΅ самоС.

УравнСния МаксвСлла ΠΈΠΌΠ΅ΡŽΡ‚ Ρ€Π΅ΡˆΠ°ΡŽΡ‰Π΅Π΅ Π·Π½Π°Ρ‡Π΅Π½ΠΈΠ΅ для понимания Π°Π½Ρ‚Π΅Π½Π½ ΠΈ Π­Π»Π΅ΠΊΡ‚Ρ€ΠΎΠΌΠ°Π³Π½Π΅Ρ‚ΠΈΠ·ΠΌ. На Π½ΠΈΡ… слоТно ΡΠΌΠΎΡ‚Ρ€Π΅Ρ‚ΡŒ — Π½Π°ΡΡ‚ΠΎΠ»ΡŒΠΊΠΎ слоТно, Ρ‡Ρ‚ΠΎ Π±ΠΎΠ»ΡŒΡˆΠΈΠ½ΡΡ‚Π²ΠΎ ΠΈΠ½ΠΆΠ΅Π½Π΅Ρ€ΠΎΠ²-элСктриков ΠΈ Ρ„ΠΈΠ·ΠΈΠΊΠΎΠ² Π΄Π°ΠΆΠ΅ Ρ‚ΠΎΠ»ΠΊΠΎΠΌ Π½Π΅ понимаю, Ρ‡Ρ‚ΠΎ ΠΎΠ½ΠΈ ΠΎΠ·Π½Π°Ρ‡Π°ΡŽΡ‚. ΠžΠΊΡƒΡ‚Π°Π½Π½Ρ‹Π΅ слоТной ΠΌΠ°Ρ‚Π΅ΠΌΠ°Ρ‚ΠΈΠΊΠΎΠΉ (которая, вСроятно, позволяСт «ΠΈΠ½Ρ‚Π΅Π»Π»Π΅ΠΊΡ‚ΡƒΠ°Π»ΡŒΠ½Ρ‹ΠΌ» людям Ρ‡ΡƒΠ²ΡΡ‚Π²ΠΎΠ²Π°Ρ‚ΡŒ своС прСвосходство ΠΏΡ€ΠΈ ΠΈΡ… обсуТдСнии) Ρ‚Ρ€ΡƒΠ΄Π½ΠΎ ΠΏΡ€ΠΈΠΉΡ‚ΠΈ ΠΊ истинному пониманию этих ΡƒΡ€Π°Π²Π½Π΅Π½ΠΈΠΉ.

Π­Ρ‚ΠΎ ΠΏΡ€ΠΈΠ²ΠΎΠ΄ΠΈΡ‚ ΠΊ созданию этого Π²Π΅Π±-сайта — ΠΈΠ½Ρ‚ΡƒΠΈΡ‚ΠΈΠ²Π½ΠΎ понятному руководству ΠΏΠΎ уравнСниям МаксвСлла. Π― Π±ΡƒΠ΄Ρƒ ΠΈΠ·Π±Π΅Π³Π°Ρ‚ΡŒ, Ссли Π² всС Π²ΠΎΠ·ΠΌΠΎΠΆΠ½Ρ‹Π΅ матСматичСскиС трудности, ΠΊΠΎΡ‚ΠΎΡ€Ρ‹Π΅ Π²ΠΎΠ·Π½ΠΈΠΊΠ°ΡŽΡ‚, ΠΈ вмСсто этого ΠΎΠΏΠΈΡˆΠΈΡ‚Π΅, Ρ‡Ρ‚ΠΎ ΠΎΠ·Π½Π°Ρ‡Π°ΡŽΡ‚ уравнСния. А Ρ‚Π°ΠΊΠΆΠ΅ Π½Π΅ Π±ΠΎΠΉΡ‚Π΅ΡΡŒ — ΠΌΠ°Ρ‚Π΅ΠΌΠ°Ρ‚ΠΈΠΊΠ° Π½Π°ΡΡ‚ΠΎΠ»ΡŒΠΊΠΎ слоТна, Ρ‡Ρ‚ΠΎ Ρ‚Π΅, ΠΊΡ‚ΠΎ ΠΏΠΎΠ½ΠΈΠΌΠ°Π΅Ρ‚ слоТноС Π²Π΅ΠΊΡ‚ΠΎΡ€Π½ΠΎΠ΅ исчислСниС, всС Π΅Ρ‰Π΅ Π½Π΅ ΠΌΠΎΠ³ΡƒΡ‚ ΠΏΡ€ΠΈΠΌΠ΅Π½ΡΡ‚ΡŒ уравнСния МаксвСлла Π² Ρ‡Π΅ΠΌ ΡƒΠ³ΠΎΠ΄Π½ΠΎ, ΠΊΡ€ΠΎΠΌΠ΅ ΠΏΡ€ΠΎΡΡ‚Π΅ΠΉΡˆΠΈΡ… сцСнариСв. По этой ΠΏΡ€ΠΈΡ‡ΠΈΠ½Π΅ ΠΈΠ½Ρ‚ΡƒΠΈΡ‚ΠΈΠ²Π½ΠΎΠ΅ Π·Π½Π°Π½ΠΈΠ΅ УравнСния МаксвСлла Π½Π°ΠΌΠ½ΠΎΠ³ΠΎ прСвосходят знания, основанныС Π½Π° матСматичСских манипуляциях.Π§Ρ‚ΠΎΠ±Ρ‹ ΠΏΠΎΠ½ΡΡ‚ΡŒ ΠΌΠΈΡ€, Π²Ρ‹ Π΄ΠΎΠ»ΠΆΠ½Ρ‹ ΠΏΠΎΠ½ΠΈΠΌΠ°Ρ‚ΡŒ, Ρ‡Ρ‚ΠΎ ΠΎΠ·Π½Π°Ρ‡Π°ΡŽΡ‚ уравнСния, Π° Π½Π΅ просто Π·Π½Π°Ρ‚ΡŒ матСматичСскиС конструкции. Π― ΡΡ‡ΠΈΡ‚Π°ΡŽ принятым ΠΌΠ΅Ρ‚ΠΎΠ΄Ρ‹ обучСния элСктромагнСтизму ΠΈ уравнСния МаксвСлла Π½Π΅ Π΄Π°ΡŽΡ‚ понимания. И с этим, скаТСм Ρ‡Ρ‚ΠΎ-Π½ΠΈΠ±ΡƒΠ΄ΡŒ ΠΎΠ± этих уравнСниях.

УравнСния МаксвСлла — это Π·Π°ΠΊΠΎΠ½Ρ‹, ΠΏΠΎΠ΄ΠΎΠ±Π½Ρ‹Π΅ Π·Π°ΠΊΠΎΠ½Ρƒ всСмирного тяготСния. Π­Ρ‚ΠΈ уравнСния — ΠΏΡ€Π°Π²ΠΈΠ»Π°, ΠΊΠΎΡ‚ΠΎΡ€Ρ‹Π΅ ΠΈΡΠΏΠΎΠ»ΡŒΠ·ΡƒΠ΅Ρ‚ всСлСнная. ΡƒΠΏΡ€Π°Π²Π»ΡΡ‚ΡŒ ΠΏΠΎΠ²Π΅Π΄Π΅Π½ΠΈΠ΅ΠΌ элСктричСских ΠΈ ΠΌΠ°Π³Π½ΠΈΡ‚Π½Ρ‹Ρ… ΠΏΠΎΠ»Π΅ΠΉ. ΠŸΠΎΡ‚ΠΎΠΊ элСктричСского Ρ‚ΠΎΠΊΠ° ΠΏΡ€ΠΎΠΈΠ·Π²Π΅Π΄Π΅Ρ‚ ΠΌΠ°Π³Π½ΠΈΡ‚Π½ΠΎΠ΅ ΠΏΠΎΠ»Π΅.Если Ρ‚Π΅ΠΊΡƒΡ‰ΠΈΠΉ ΠΏΠΎΡ‚ΠΎΠΊ мСняСтся со Π²Ρ€Π΅ΠΌΠ΅Π½Π΅ΠΌ (ΠΊΠ°ΠΊ Π² случаС любой Π²ΠΎΠ»Π½Ρ‹ ΠΈΠ»ΠΈ пСриодичСского сигнала), ΠΌΠ°Π³Π½ΠΈΡ‚Π½ΠΎΠ΅ ΠΏΠΎΠ»Π΅ Ρ‚Π°ΠΊΠΆΠ΅ Π²Ρ‹Π·ΠΎΠ²Π΅Ρ‚ элСктричСскоС ΠΏΠΎΠ»Π΅. УравнСния МаксвСлла ΠΏΠΎΠΊΠ°Π·Ρ‹Π²Π°ΡŽΡ‚, Ρ‡Ρ‚ΠΎ Ρ€Π°Π·Π΄Π΅Π»Π΅Π½Π½Ρ‹ΠΉ заряд (ΠΏΠΎΠ»ΠΎΠΆΠΈΡ‚Π΅Π»ΡŒΠ½Ρ‹ΠΉ ΠΈ ΠΎΡ‚Ρ€ΠΈΡ†Π°Ρ‚Π΅Π»ΡŒΠ½Ρ‹ΠΉ) Π²Ρ‹Π·Ρ‹Π²Π°Π΅Ρ‚ элСктричСскоС ΠΏΠΎΠ»Π΅ — ΠΈ Ссли ΠΎΠ½ΠΎ мСняСтся со Π²Ρ€Π΅ΠΌΠ΅Π½Π΅ΠΌ Ρ‚Π°ΠΊΠΆΠ΅ Π²Ρ‹Π·ΠΎΠ²Π΅Ρ‚ Ρ€Π°ΡΠΏΡ€ΠΎΡΡ‚Ρ€Π°Π½ΡΡŽΡ‰Π΅Π΅ΡΡ элСктричСскоС ΠΏΠΎΠ»Π΅, Ρ‡Ρ‚ΠΎ Π² дальнСйшСм ΠΏΡ€ΠΈΠ²Π΅Π΄Π΅Ρ‚ ΠΊ Ρ€Π°ΡΠΏΡ€ΠΎΡΡ‚Ρ€Π°Π½Π΅Π½ΠΈΡŽ ΠΌΠ°Π³Π½ΠΈΡ‚Π½ΠΎΠ΅ ΠΏΠΎΠ»Π΅.

Π§Ρ‚ΠΎΠ±Ρ‹ ΠΏΠΎΠ½ΡΡ‚ΡŒ уравнСния МаксвСлла Π½Π° Π±ΠΎΠ»Π΅Π΅ ΠΈΠ½Ρ‚ΡƒΠΈΡ‚ΠΈΠ²Π½ΠΎΠΌ ΡƒΡ€ΠΎΠ²Π½Π΅, Ρ‡Π΅ΠΌ Ρƒ Π±ΠΎΠ»ΡŒΡˆΠΈΠ½ΡΡ‚Π²Π° Π΄ΠΎΠΊΡ‚ΠΎΡ€ΠΎΠ² тСхничСских Π½Π°ΡƒΠΊ ΠΈΠ»ΠΈ Ρ„ΠΈΠ·ΠΈΠΊΠΈ, Ρ‰Π΅Π»ΠΊΠ½ΠΈΡ‚Π΅ ΠΏΠΎ ссылкам ΠΈ опрСдСлСниям Π²Ρ‹ΡˆΠ΅.Π’Ρ‹ ΠΎΠ±Π½Π°Ρ€ΡƒΠΆΠΈΡ‚Π΅, Ρ‡Ρ‚ΠΎ слоТныС матСматичСскиС маски внутрСнняя ΡΠ»Π΅Π³Π°Π½Ρ‚Π½ΠΎΡΡ‚ΡŒ этих ΡƒΡ€Π°Π²Π½Π΅Π½ΠΈΠΉ — ΠΈ Π²Ρ‹ ΡƒΠ·Π½Π°Π΅Ρ‚Π΅, ΠΊΠ°ΠΊ ВсСлСнная управляСт элСктромагнитным Машина.

Π’ΠΎΡ‚ ΠΎΠ±ΡƒΡ‡Π°ΡŽΡ‰Π΅Π΅ Π²ΠΈΠ΄Π΅ΠΎ, ΠΈΠ½Ρ‚ΡƒΠΈΡ‚ΠΈΠ²Π½ΠΎ ΠΎΠ±ΡŠΡΡΠ½ΡΡŽΡ‰Π΅Π΅ уравнСния МаксвСлла:

Π’Π²Π΅Ρ€Ρ…Ρƒ: уравнСния МаксвСлла

уравнСния МаксвСлла | БлСстящая Π²ΠΈΠΊΠΈ ΠΏΠΎ ΠΌΠ°Ρ‚Π΅ΠΌΠ°Ρ‚ΠΈΠΊΠ΅ ΠΈ Π½Π°ΡƒΠΊΠ΅

Π—Π°ΠΊΠΎΠ½ Π›ΠΎΡ€Π΅Π½Ρ†Π°, Π³Π΄Π΅ qqq ΠΈ v \ mathbf {v} v — элСктричСский заряд ΠΈ ΡΠΊΠΎΡ€ΠΎΡΡ‚ΡŒ частицы соотвСтствСнно, опрСдСляСт элСктричСскоС ΠΏΠΎΠ»Π΅ E \ mathbf {E} E ΠΈ ΠΌΠ°Π³Π½ΠΈΡ‚Π½ΠΎΠ΅ ΠΏΠΎΠ»Π΅ B \ mathbf {B} B, задавая общая элСктромагнитная сила F \ mathbf {F} F ΠΊΠ°ΠΊ

F = qE + qv Γ— B.\ mathbf {F} = q \ mathbf {E} + q \ mathbf {v} \ times \ mathbf {B}. F = qE + qv Γ— B.

По сути, ΠΎΠ΄Π½Π° Ρ‡Π°ΡΡ‚ΡŒ элСктромагнитной силы, Π²ΠΎΠ·Π½ΠΈΠΊΠ°ΡŽΡ‰Π°Ρ ΠΏΡ€ΠΈ взаимодСйствии с двиТущимся зарядом (qv q \ mathbf {v} qv), рассматриваСтся ΠΊΠ°ΠΊ ΠΌΠ°Π³Π½ΠΈΡ‚Π½ΠΎΠ΅ ΠΏΠΎΠ»Π΅, Π° другая Ρ‡Π°ΡΡ‚ΡŒ — ΠΊΠ°ΠΊ элСктричСскоС ΠΏΠΎΠ»Π΅.

Π—Π°ΠΊΠΎΠ½ Гаусса: Π‘Π°ΠΌΡ‹ΠΌ Ρ€Π°Π½Π½ΠΈΠΌ ΠΈΠ· Ρ‡Π΅Ρ‚Ρ‹Ρ€Π΅Ρ… ΡƒΡ€Π°Π²Π½Π΅Π½ΠΈΠΉ МаксвСлла, ΠΊΠΎΡ‚ΠΎΡ€Ρ‹Π΅ Π±Ρ‹Π»ΠΈ ΠΎΡ‚ΠΊΡ€Ρ‹Ρ‚Ρ‹ (Π² эквивалСнтной Ρ„ΠΎΡ€ΠΌΠ΅ Π·Π°ΠΊΠΎΠ½Π° ΠšΡƒΠ»ΠΎΠ½Π°), Π±Ρ‹Π» Π·Π°ΠΊΠΎΠ½ Гаусса. Π’ своСй ΠΈΠ½Ρ‚Π΅Π³Ρ€Π°Π»ΡŒΠ½ΠΎΠΉ Ρ„ΠΎΡ€ΠΌΠ΅ Π² Π΅Π΄ΠΈΠ½ΠΈΡ†Π°Ρ… БИ ΠΎΠ½ заявляСт, Ρ‡Ρ‚ΠΎ ΠΏΠΎΠ»Π½Ρ‹ΠΉ заряд, содСрТащийся Π²Π½ΡƒΡ‚Ρ€ΠΈ Π·Π°ΠΌΠΊΠ½ΡƒΡ‚ΠΎΠΉ повСрхности, ΠΏΡ€ΠΎΠΏΠΎΡ€Ρ†ΠΈΠΎΠ½Π°Π»Π΅Π½ ΠΎΠ±Ρ‰Π΅ΠΌΡƒ элСктричСскому ΠΏΠΎΡ‚ΠΎΠΊΡƒ (суммС Π½ΠΎΡ€ΠΌΠ°Π»ΡŒΠ½ΠΎΠΉ ΡΠΎΡΡ‚Π°Π²Π»ΡΡŽΡ‰Π΅ΠΉ поля) Ρ‡Π΅Ρ€Π΅Π· ΠΏΠΎΠ²Π΅Ρ€Ρ…Π½ΠΎΡΡ‚ΡŒ:

∫SEβ‹…da = 1Ο΅0∫ρ dV, \ int_S \ mathbf {E} \ cdot d \ mathbf {a} = \ frac {1} {\ epsilon_0} \ int \ rho \, dV, ∫S Eβ‹…da = Ο΅0 1 ρdV,

, Π³Π΄Π΅ коэффициСнт ΠΏΡ€ΠΎΠΏΠΎΡ€Ρ†ΠΈΠΎΠ½Π°Π»ΡŒΠ½ΠΎΡΡ‚ΠΈ Ρ€Π°Π²Π΅Π½ 1 / Ο΅0, 1 / \ epsilon_0, 1 / Ο΅0, ΠΎΠ±Ρ€Π°Ρ‚Π½ΠΎΠΉ Π²Π΅Π»ΠΈΡ‡ΠΈΠ½Π΅ элСктричСской постоянной.ΠŸΠΎΠ»Π½Ρ‹ΠΉ заряд выраТаСтся ΠΊΠ°ΠΊ ΠΏΠ»ΠΎΡ‚Π½ΠΎΡΡ‚ΡŒ заряда ρ \ rho ρ, проинтСгрированная ΠΏΠΎ области.

Π—Π°ΠΊΠΎΠ½ Гаусса для ΠΌΠ°Π³Π½Π΅Ρ‚ΠΈΠ·ΠΌΠ°: Π₯отя ΠΌΠ°Π³Π½ΠΈΡ‚Π½Ρ‹Π΅ Π΄ΠΈΠΏΠΎΠ»ΠΈ ΠΌΠΎΠ³ΡƒΡ‚ ΡΠΎΠ·Π΄Π°Π²Π°Ρ‚ΡŒ Π°Π½Π°Π»ΠΎΠ³ΠΈΡ‡Π½Ρ‹ΠΉ ΠΌΠ°Π³Π½ΠΈΡ‚Π½Ρ‹ΠΉ ΠΏΠΎΡ‚ΠΎΠΊ, ΠΊΠΎΡ‚ΠΎΡ€Ρ‹ΠΉ ΠΈΠΌΠ΅Π΅Ρ‚ Π°Π½Π°Π»ΠΎΠ³ΠΈΡ‡Π½ΡƒΡŽ ΠΌΠ°Ρ‚Π΅ΠΌΠ°Ρ‚ΠΈΡ‡Π΅ΡΠΊΡƒΡŽ Ρ„ΠΎΡ€ΠΌΡƒ, Π½Π΅ сущСствуСт эквивалСнтных ΠΌΠ°Π³Π½ΠΈΡ‚Π½Ρ‹Ρ… ΠΌΠΎΠ½ΠΎΠΏΠΎΠ»Π΅ΠΉ, ΠΈ поэтому ΠΎΠ±Ρ‰ΠΈΠΉ Β«ΠΌΠ°Π³Π½ΠΈΡ‚Π½Ρ‹ΠΉ заряд» Π²ΠΎ всСм пространствС Π΄ΠΎΠ»ΠΆΠ΅Π½ Π² суммС Ρ€Π°Π²Π½ΡΡ‚ΡŒΡΡ Π½ΡƒΠ»ΡŽ. Π‘Π»Π΅Π΄ΠΎΠ²Π°Ρ‚Π΅Π»ΡŒΠ½ΠΎ, Π·Π°ΠΊΠΎΠ½ Гаусса для ΠΌΠ°Π³Π½Π΅Ρ‚ΠΈΠ·ΠΌΠ° читаСтся просто

.

∫SBβ‹…da = 0. \ int_S \ mathbf {B} \ cdot d \ mathbf {a} = 0. ∫S Bβ‹…da = 0.

Π—Π°ΠΊΠΎΠ½ ЀарадСя: ЭлСктричСскоС ΠΈ ΠΌΠ°Π³Π½ΠΈΡ‚Π½ΠΎΠ΅ поля ΠΏΠ΅Ρ€Π΅ΠΏΠ»Π΅Ρ‚Π°ΡŽΡ‚ΡΡ, ΠΊΠΎΠ³Π΄Π° поля ΠΏΡ€Π΅Ρ‚Π΅Ρ€ΠΏΠ΅Π²Π°ΡŽΡ‚ Π²Ρ€Π΅ΠΌΠ΅Π½Π½ΡƒΡŽ ΡΠ²ΠΎΠ»ΡŽΡ†ΠΈΡŽ.Π’ 1820-Ρ… Π³ΠΎΠ΄Π°Ρ… Π€Π°Ρ€Π°Π΄Π΅ΠΉ ΠΎΠ±Π½Π°Ρ€ΡƒΠΆΠΈΠ», Ρ‡Ρ‚ΠΎ ΠΈΠ·ΠΌΠ΅Π½Π΅Π½ΠΈΠ΅ ΠΌΠ°Π³Π½ΠΈΡ‚Π½ΠΎΠ³ΠΎ ΠΏΠΎΡ‚ΠΎΠΊΠ° создаСт элСктричСскоС ΠΏΠΎΠ»Π΅ ΠΏΠΎ Π·Π°ΠΌΠΊΠ½ΡƒΡ‚ΠΎΠΌΡƒ ΠΊΠΎΠ½Ρ‚ΡƒΡ€Ρƒ. Π­Ρ‚ΠΎ ΡΠΎΠΎΡ‚Π½ΠΎΡˆΠ΅Π½ΠΈΠ΅ Ρ‚Π΅ΠΏΠ΅Ρ€ΡŒ называСтся Π·Π°ΠΊΠΎΠ½ΠΎΠΌ ЀарадСя:

∫loopEβ‹…ds = βˆ’ddt∫SBβ‹…da. \ int_ \ text {loop} \ mathbf {E} \ cdot d \ mathbf {s} = — \ frac {d} {dt} \ int_S \ mathbf {B} \ cdot d \ mathbf {a}. ∫loop Eβ‹…ds = βˆ’dtd ∫S Bβ‹…da.

ΠŸΡ€ΠΈ ΠΎΡ€ΠΈΠ΅Π½Ρ‚Π°Ρ†ΠΈΠΈ ΠΏΠ΅Ρ‚Π»ΠΈ, ΠΎΠΏΡ€Π΅Π΄Π΅Π»Π΅Π½Π½ΠΎΠΉ Π² соотвСтствии с ΠΏΡ€Π°Π²ΠΈΠ»ΠΎΠΌ ΠΏΡ€Π°Π²ΠΎΠΉ Ρ€ΡƒΠΊΠΈ, ΠΎΡ‚Ρ€ΠΈΡ†Π°Ρ‚Π΅Π»ΡŒΠ½Ρ‹ΠΉ Π·Π½Π°ΠΊ ΠΎΡ‚Ρ€Π°ΠΆΠ°Π΅Ρ‚ Π·Π°ΠΊΠΎΠ½ Π›Π΅Π½Ρ†Π°.

Π—Π°ΠΊΠΎΠ½ АмпСра: НаконСц, Π·Π°ΠΊΠΎΠ½ АмпСра ΠΏΡ€Π΅Π΄ΠΏΠΎΠ»Π°Π³Π°Π΅Ρ‚, Ρ‡Ρ‚ΠΎ постоянный Ρ‚ΠΎΠΊ Ρ‡Π΅Ρ€Π΅Π· ΠΏΠΎΠ²Π΅Ρ€Ρ…Π½ΠΎΡΡ‚ΡŒ ΠΏΡ€ΠΈΠ²ΠΎΠ΄ΠΈΡ‚ ΠΊ возникновСнию ΠΌΠ°Π³Π½ΠΈΡ‚Π½ΠΎΠ³ΠΎ поля (Π²Ρ‹Ρ€Π°ΠΆΠ΅Π½Π½ΠΎΠ³ΠΎ Π² Π΅Π΄ΠΈΠ½ΠΈΡ†Π°Ρ… ΠΏΠΎΡ‚ΠΎΠΊΠ°).Π’Π΄ΠΎΠ±Π°Π²ΠΎΠΊ МаксвСлл ΠΎΠΏΡ€Π΅Π΄Π΅Π»ΠΈΠ», Ρ‡Ρ‚ΠΎ быстрыС измСнСния элСктричСского ΠΏΠΎΡ‚ΠΎΠΊΠ° (d / dt) Eβ‹…da (d / dt) \ mathbf {E} \ cdot d \ mathbf {a} (d / dt) Eβ‹…da Ρ‚Π°ΠΊΠΆΠ΅ ΠΌΠΎΠ³ΡƒΡ‚ ΠΏΡ€ΠΈΠ²ΠΎΠ΄ΠΈΡ‚ΡŒ ΠΊ ΠΊ измСнСниям ΠΌΠ°Π³Π½ΠΈΡ‚Π½ΠΎΠ³ΠΎ ΠΏΠΎΡ‚ΠΎΠΊΠ°. Π’ Ρ†Π΅Π»ΠΎΠΌ Π·Π°ΠΊΠΎΠ½ АмпСра с ΠΏΠΎΠΏΡ€Π°Π²ΠΊΠΎΠΉ МаксвСлла ΡƒΡ‚Π²Π΅Ρ€ΠΆΠ΄Π°Π΅Ρ‚, Ρ‡Ρ‚ΠΎ

∫loopBβ‹…ds = ΞΌ0∫SJβ‹…da + ΞΌ0Ο΅0ddt∫SEβ‹…da. \ int _ {\ text {loop}} \ mathbf {B} \ cdot d \ mathbf {s} = \ mu_0 \ int_S \ mathbf {J} \ cdot d \ mathbf {a} + \ mu_0 \ epsilon_0 \ frac {d} {dt} \ int_S \ mathbf {E} \ cdot d \ mathbf {a}. ∫loop Bβ‹…ds = ΞΌ0 ∫S Jβ‹…da + ΞΌ0 Ο΅0 dtd ∫S Eβ‹…da.

Π’Π°ΠΊΠΈΠΌ ΠΎΠ±Ρ€Π°Π·ΠΎΠΌ,

  • Π—Π°ΠΊΠΎΠ½ Гаусса: ∫SEβ‹…da = 1Ο΅0∫ρ dV \ int_S \ mathbf {E} \ cdot d \ mathbf {a} = \ frac {1} {\ epsilon_0} \ int \ rho \, dV ∫ S Eβ‹…da = Ο΅0 1 ρdV
  • Π—Π°ΠΊΠΎΠ½ Гаусса для ΠΌΠ°Π³Π½Π΅Ρ‚ΠΈΠ·ΠΌΠ°: ∫SBβ‹…da = 0 \ int_S \ mathbf {B} \ cdot d \ mathbf {a} = 0 ∫S Bβ‹…da = 0
  • Π—Π°ΠΊΠΎΠ½ ЀарадСя: ∫loopEβ‹…ds = βˆ’ddt∫SBβ‹…da \ int_ \ text {loop} \ mathbf {E} \ cdot d \ mathbf {s} = — \ frac {d} {dt} \ int_S \ mathbf {B} \ cdot d \ mathbf {a} ∫loop Eβ‹…ds = βˆ’dtd ∫S Bβ‹…da
  • Π—Π°ΠΊΠΎΠ½ АмпСра: ∫loopBβ‹…ds = ΞΌ0∫SJβ‹…da + Ο΅0ΞΌ0ddt∫SEβ‹…da.\ int _ {\ text {loop}} \ mathbf {B} \ cdot d \ mathbf {s} = \ mu_0 \ int_S \ mathbf {J} \ cdot d \ mathbf {a} + \ epsilon_0 \ mu_0 \ frac {d} {dt} \ int_S \ mathbf {E} \ cdot d \ mathbf {a}. ∫loop Bβ‹…ds = ΞΌ0 ∫S Jβ‹…da + Ο΅0 ΞΌ0 dtd ∫S Eβ‹…da.

Π’ своСй ΠΈΠ½Ρ‚Π΅Π³Ρ€Π°Π»ΡŒΠ½ΠΎΠΉ Ρ„ΠΎΡ€ΠΌΠ΅ уравнСния МаксвСлла ΠΌΠΎΠ³ΡƒΡ‚ ΠΈΡΠΏΠΎΠ»ΡŒΠ·ΠΎΠ²Π°Ρ‚ΡŒΡΡ для опрСдСлСния области заряда ΠΈΠ»ΠΈ Ρ‚ΠΎΠΊΠ°.

УравнСния МаксвСлла: прСдсказаниС ΠΈ наблюдСниС элСктромагнитных Π²ΠΎΠ»Π½

ЦСль обучСния

К ΠΊΠΎΠ½Ρ†Ρƒ этого Ρ€Π°Π·Π΄Π΅Π»Π° Π²Ρ‹ смоТСтС:

  • ΠŸΠ΅Ρ€Π΅Ρ„ΠΎΡ€ΠΌΡƒΠ»ΠΈΡ€ΡƒΠΉΡ‚Π΅ уравнСния МаксвСлла.

Рис. 1. ДТСймс ΠšΠ»Π΅Ρ€ΠΊ МаксвСлл, Ρ„ΠΈΠ·ΠΈΠΊ 19-Π³ΠΎ Π²Π΅ΠΊΠ°, Ρ€Π°Π·Ρ€Π°Π±ΠΎΡ‚Π°Π» Ρ‚Π΅ΠΎΡ€ΠΈΡŽ, ΠΎΠ±ΡŠΡΡΠ½ΡΡŽΡ‰ΡƒΡŽ взаимосвязь ΠΌΠ΅ΠΆΠ΄Ρƒ элСктричСством ΠΈ ΠΌΠ°Π³Π½Π΅Ρ‚ΠΈΠ·ΠΌΠΎΠΌ, ΠΈ ΠΏΡ€Π°Π²ΠΈΠ»ΡŒΠ½ΠΎ прСдсказал, Ρ‡Ρ‚ΠΎ Π²ΠΈΠ΄ΠΈΠΌΡ‹ΠΉ свСт вызываСтся элСктромагнитными Π²ΠΎΠ»Π½Π°ΠΌΠΈ. (ΠΊΡ€Π΅Π΄ΠΈΡ‚: Π“. Π”ΠΆ. Π‘Ρ‚ΠΎΠ΄Π°Ρ€Ρ‚)

Π¨ΠΎΡ‚Π»Π°Π½Π΄Π΅Ρ† ДТСймс ΠšΠ»Π΅Ρ€ΠΊ МаксвСлл (1831–1879) считаСтся Π²Π΅Π»ΠΈΡ‡Π°ΠΉΡˆΠΈΠΌ Ρ„ΠΈΠ·ΠΈΠΊΠΎΠΌ-Ρ‚Π΅ΠΎΡ€Π΅Ρ‚ΠΈΠΊΠΎΠΌ XIX Π²Π΅ΠΊΠ°. (Π‘ΠΌ. Рис. 1.) Π₯отя ΠΎΠ½ ΡƒΠΌΠ΅Ρ€ ΠΌΠΎΠ»ΠΎΠ΄Ρ‹ΠΌ, МаксвСлл Π½Π΅ Ρ‚ΠΎΠ»ΡŒΠΊΠΎ сформулировал ΠΏΠΎΠ»Π½ΡƒΡŽ ΡΠ»Π΅ΠΊΡ‚Ρ€ΠΎΠΌΠ°Π³Π½ΠΈΡ‚Π½ΡƒΡŽ Ρ‚Π΅ΠΎΡ€ΠΈΡŽ, ΠΏΡ€Π΅Π΄ΡΡ‚Π°Π²Π»Π΅Π½Π½ΡƒΡŽ уравнСниями МаксвСлла , ΠΎΠ½ Ρ‚Π°ΠΊΠΆΠ΅ Ρ€Π°Π·Ρ€Π°Π±ΠΎΡ‚Π°Π» ΠΊΠΈΠ½Π΅Ρ‚ΠΈΡ‡Π΅ΡΠΊΡƒΡŽ Ρ‚Π΅ΠΎΡ€ΠΈΡŽ Π³Π°Π·ΠΎΠ² ΠΈ внСс Π·Π½Π°Ρ‡ΠΈΡ‚Π΅Π»ΡŒΠ½Ρ‹ΠΉ Π²ΠΊΠ»Π°Π΄ Π² ΠΏΠΎΠ½ΠΈΠΌΠ°Π½ΠΈΠ΅ Ρ†Π²Π΅Ρ‚ΠΎΠ²ΠΎΠ³ΠΎ зрСния ΠΈ ΠΏΡ€ΠΈΡ€ΠΎΠ΄Ρ‹ Π‘Π°Ρ‚ΡƒΡ€Π½Π°. ΠΊΠΎΠ»ΡŒΡ†Π°.

МаксвСлл объСдинил всю Ρ€Π°Π±ΠΎΡ‚Ρƒ, ΠΏΡ€ΠΎΠ΄Π΅Π»Π°Π½Π½ΡƒΡŽ блСстящими Ρ„ΠΈΠ·ΠΈΠΊΠ°ΠΌΠΈ, Ρ‚Π°ΠΊΠΈΠΌΠΈ ΠΊΠ°ΠΊ ЭрстСд, ΠšΡƒΠ»ΠΎΠ½, Гаусс ΠΈ Π€Π°Ρ€Π°Π΄Π΅ΠΉ, ΠΈ Π΄ΠΎΠ±Π°Π²ΠΈΠ» свои собствСнныС ΠΈΠ΄Π΅ΠΈ для Ρ€Π°Π·Ρ€Π°Π±ΠΎΡ‚ΠΊΠΈ Π²ΡΠ΅ΠΎΠ±ΡŠΠ΅ΠΌΠ»ΡŽΡ‰Π΅ΠΉ Ρ‚Π΅ΠΎΡ€ΠΈΠΈ элСктромагнСтизма. УравнСния МаксвСлла здСсь ΠΏΠ΅Ρ€Π΅Ρ„Ρ€Π°Π·ΠΈΡ€ΠΎΠ²Π°Π½Ρ‹ словами, ΠΏΠΎΡ‚ΠΎΠΌΡƒ Ρ‡Ρ‚ΠΎ ΠΈΡ… матСматичСская Ρ„ΠΎΡ€ΠΌΡƒΠ»ΠΈΡ€ΠΎΠ²ΠΊΠ° Π²Ρ‹Ρ…ΠΎΠ΄ΠΈΡ‚ Π·Π° Ρ€Π°ΠΌΠΊΠΈ этого тСкста. Однако уравнСния ΠΏΠΎΠΊΠ°Π·Ρ‹Π²Π°ΡŽΡ‚, ΠΊΠ°ΠΊ простыС матСматичСскиС утвСрТдСния ΠΌΠΎΠ³ΡƒΡ‚ элСгантно ΠΎΠ±ΡŠΠ΅Π΄ΠΈΠ½ΡΡ‚ΡŒ ΠΈ Π²Ρ‹Ρ€Π°ΠΆΠ°Ρ‚ΡŒ мноТСство ΠΊΠΎΠ½Ρ†Π΅ΠΏΡ†ΠΈΠΉ — ΠΏΠΎΡ‡Π΅ΠΌΡƒ ΠΌΠ°Ρ‚Π΅ΠΌΠ°Ρ‚ΠΈΠΊΠ° являСтся языком Π½Π°ΡƒΠΊΠΈ.

УравнСния МаксвСлла

  1. Π›ΠΈΠ½ΠΈΠΈ элСктричСского поля Π±Π΅Ρ€ΡƒΡ‚ Π½Π°Ρ‡Π°Π»ΠΎ ΠΎΡ‚ ΠΏΠΎΠ»ΠΎΠΆΠΈΡ‚Π΅Π»ΡŒΠ½Ρ‹Ρ… зарядов ΠΈ Π·Π°ΠΊΠ°Π½Ρ‡ΠΈΠ²Π°ΡŽΡ‚ΡΡ ΠΎΡ‚Ρ€ΠΈΡ†Π°Ρ‚Π΅Π»ΡŒΠ½Ρ‹ΠΌΠΈ зарядами. ЭлСктричСскоС ΠΏΠΎΠ»Π΅ опрСдСляСтся ΠΊΠ°ΠΊ сила, приходящаяся Π½Π° Π΅Π΄ΠΈΠ½ΠΈΡ†Ρƒ заряда ΠΈΡΠΏΡ‹Ρ‚Π°Ρ‚Π΅Π»ΡŒΠ½ΠΎΠ³ΠΎ заряда, Π° сила силы связана с элСктричСской постоянной Ξ΅ 0 , Ρ‚Π°ΠΊΠΆΠ΅ извСстной ΠΊΠ°ΠΊ диэлСктричСская ΠΏΡ€ΠΎΠ½ΠΈΡ†Π°Π΅ΠΌΠΎΡΡ‚ΡŒ свободного пространства. Из ΠΏΠ΅Ρ€Π²ΠΎΠ³ΠΎ уравнСния МаксвСлла ΠΌΡ‹ ΠΏΠΎΠ»ΡƒΡ‡Π°Π΅ΠΌ ΠΎΡΠΎΠ±ΡƒΡŽ Ρ„ΠΎΡ€ΠΌΡƒ Π·Π°ΠΊΠΎΠ½Π° ΠšΡƒΠ»ΠΎΠ½Π°, ΠΈΠ·Π²Π΅ΡΡ‚Π½ΡƒΡŽ ΠΊΠ°ΠΊ Π·Π°ΠΊΠΎΠ½ Гаусса для элСктричСства.
  2. Π›ΠΈΠ½ΠΈΠΈ ΠΌΠ°Π³Π½ΠΈΡ‚Π½ΠΎΠ³ΠΎ поля Π½Π΅ΠΏΡ€Π΅Ρ€Ρ‹Π²Π½Ρ‹, Π½Π΅ ΠΈΠΌΠ΅ΡŽΡ‚ Π½ΠΈ Π½Π°Ρ‡Π°Π»Π°, Π½ΠΈ ΠΊΠΎΠ½Ρ†Π°.О сущСствовании ΠΌΠ°Π³Π½ΠΈΡ‚Π½Ρ‹Ρ… ΠΌΠΎΠ½ΠΎΠΏΠΎΠ»Π΅ΠΉ Π½Π΅ извСстно. Π‘ΠΈΠ»Π° ΠΌΠ°Π³Π½ΠΈΡ‚Π½ΠΎΠΉ силы связана с ΠΌΠ°Π³Π½ΠΈΡ‚Π½ΠΎΠΉ постоянной ΞΌ 0 , Ρ‚Π°ΠΊΠΆΠ΅ извСстной ΠΊΠ°ΠΊ ΠΏΡ€ΠΎΠ½ΠΈΡ†Π°Π΅ΠΌΠΎΡΡ‚ΡŒ свободного пространства. Π­Ρ‚ΠΎ Π²Ρ‚ΠΎΡ€ΠΎΠ΅ ΠΈΠ· ΡƒΡ€Π°Π²Π½Π΅Π½ΠΈΠΉ МаксвСлла извСстно ΠΊΠ°ΠΊ Π·Π°ΠΊΠΎΠ½ Гаусса для ΠΌΠ°Π³Π½Π΅Ρ‚ΠΈΠ·ΠΌΠ°.
  3. Π˜Π·ΠΌΠ΅Π½ΡΡŽΡ‰Π΅Π΅ΡΡ ΠΌΠ°Π³Π½ΠΈΡ‚Π½ΠΎΠ΅ ΠΏΠΎΠ»Π΅ ΠΈΠ½Π΄ΡƒΡ†ΠΈΡ€ΡƒΠ΅Ρ‚ ΡΠ»Π΅ΠΊΡ‚Ρ€ΠΎΠ΄Π²ΠΈΠΆΡƒΡ‰ΡƒΡŽ силу (Π­Π”Π‘) ΠΈ, ΡΠ»Π΅Π΄ΠΎΠ²Π°Ρ‚Π΅Π»ΡŒΠ½ΠΎ, элСктричСскоС ΠΏΠΎΠ»Π΅. НаправлСниС Π­Π”Π‘ противодСйствуСт измСнСнию. Π­Ρ‚Π° Ρ‚Ρ€Π΅Ρ‚ΡŒ ΡƒΡ€Π°Π²Π½Π΅Π½ΠΈΠΉ МаксвСлла являСтся Π·Π°ΠΊΠΎΠ½ΠΎΠΌ ΠΈΠ½Π΄ΡƒΠΊΡ†ΠΈΠΈ ЀарадСя ΠΈ Π²ΠΊΠ»ΡŽΡ‡Π°Π΅Ρ‚ Π² сСбя Π·Π°ΠΊΠΎΠ½ Π›Π΅Π½Ρ†Π°.
  4. ΠœΠ°Π³Π½ΠΈΡ‚Π½Ρ‹Π΅ поля ΡΠΎΠ·Π΄Π°ΡŽΡ‚ΡΡ двиТущимися зарядами ΠΈΠ»ΠΈ ΠΈΠ·ΠΌΠ΅Π½ΡΡŽΡ‰ΠΈΠΌΠΈΡΡ элСктричСскими полями. Π­Ρ‚Π° чСтвСртая Ρ‡Π°ΡΡ‚ΡŒ ΡƒΡ€Π°Π²Π½Π΅Π½ΠΈΠΉ МаксвСлла Π²ΠΊΠ»ΡŽΡ‡Π°Π΅Ρ‚ Π·Π°ΠΊΠΎΠ½ АмпСра ΠΈ добавляСт Π΅Ρ‰Π΅ ΠΎΠ΄ΠΈΠ½ источник ΠΌΠ°Π³Π½Π΅Ρ‚ΠΈΠ·ΠΌΠ° — ΠΈΠ·ΠΌΠ΅Π½Π΅Π½ΠΈΠ΅ элСктричСских ΠΏΠΎΠ»Π΅ΠΉ.

УравнСния МаксвСлла ΠΎΡ…Π²Π°Ρ‚Ρ‹Π²Π°ΡŽΡ‚ основныС Π·Π°ΠΊΠΎΠ½Ρ‹ элСктричСства ΠΈ ΠΌΠ°Π³Π½Π΅Ρ‚ΠΈΠ·ΠΌΠ°. Π§Ρ‚ΠΎ Π½Π΅ Ρ‚Π°ΠΊ ΠΎΡ‡Π΅Π²ΠΈΠ΄Π½ΠΎ, Ρ‚Π°ΠΊ это симмСтрия, ΠΊΠΎΡ‚ΠΎΡ€ΡƒΡŽ МаксвСлл Π²Π²Π΅Π» Π² свою ΠΌΠ°Ρ‚Π΅ΠΌΠ°Ρ‚ΠΈΡ‡Π΅ΡΠΊΡƒΡŽ структуру. ОсобСнно Π²Π°ΠΆΠ½ΠΎ Π΅Π³ΠΎ Π΄ΠΎΠ±Π°Π²Π»Π΅Π½ΠΈΠ΅ ΠΊ Π³ΠΈΠΏΠΎΡ‚Π΅Π·Π΅ ΠΎ Ρ‚ΠΎΠΌ, Ρ‡Ρ‚ΠΎ ΠΈΠ·ΠΌΠ΅Π½ΡΡŽΡ‰ΠΈΠ΅ΡΡ элСктричСскиС поля ΡΠΎΠ·Π΄Π°ΡŽΡ‚ ΠΌΠ°Π³Π½ΠΈΡ‚Π½Ρ‹Π΅ поля.Π­Ρ‚ΠΎ Π² точности Π°Π½Π°Π»ΠΎΠ³ΠΈΡ‡Π½ΠΎ (ΠΈ симмСтрично) Π·Π°ΠΊΠΎΠ½Ρƒ ΠΈΠ½Π΄ΡƒΠΊΡ†ΠΈΠΈ ЀарадСя ΠΈ ΠΏΠΎΠ΄ΠΎΠ·Ρ€Π΅Π²Π°Π»ΠΎΡΡŒ Π² Ρ‚Π΅Ρ‡Π΅Π½ΠΈΠ΅ Π½Π΅ΠΊΠΎΡ‚ΠΎΡ€ΠΎΠ³ΠΎ Π²Ρ€Π΅ΠΌΠ΅Π½ΠΈ, Π½ΠΎ прСкрасно вписываСтся Π² уравнСния МаксвСлла.

БиммСтрия проявляСтся Π² самых Ρ€Π°Π·Π½Ρ‹Ρ… ситуациях. Π’ соврСмСнных исслСдованиях симмСтрия ΠΈΠ³Ρ€Π°Π΅Ρ‚ Π²Π°ΠΆΠ½ΡƒΡŽ Ρ€ΠΎΠ»ΡŒ Π² поискС субатомных частиц с использованиСм массивных ΠΌΠ½ΠΎΠ³ΠΎΠ½Π°Ρ†ΠΈΠΎΠ½Π°Π»ΡŒΠ½Ρ‹Ρ… ускоритСлСй частиц, Ρ‚Π°ΠΊΠΈΡ… ΠΊΠ°ΠΊ Π½ΠΎΠ²Ρ‹ΠΉ Π‘ΠΎΠ»ΡŒΡˆΠΎΠΉ Π°Π΄Ρ€ΠΎΠ½Π½Ρ‹ΠΉ ΠΊΠΎΠ»Π»Π°ΠΉΠ΄Π΅Ρ€ Π² ЦЕРНС.

НалаТиваниС связСй: объСдинСниС сил

Полная ΠΈ симмСтричная тСория МаксвСлла ΠΏΠΎΠΊΠ°Π·Π°Π»Π°, Ρ‡Ρ‚ΠΎ элСктричСскиС ΠΈ ΠΌΠ°Π³Π½ΠΈΡ‚Π½Ρ‹Π΅ силы Π½Π΅ ΠΎΡ‚Π΄Π΅Π»ΡŒΠ½Ρ‹Π΅, Π° Ρ€Π°Π·Π½Ρ‹Π΅ проявлСния ΠΎΠ΄Π½ΠΎΠ³ΠΎ ΠΈ Ρ‚ΠΎΠ³ΠΎ ΠΆΠ΅ — элСктромагнитной силы.Π­Ρ‚ΠΎ классичСскоС объСдинСниС сил являСтся ΠΎΠ΄Π½ΠΎΠΉ ΠΈΠ· ΠΏΡ€ΠΈΡ‡ΠΈΠ½ Π½Ρ‹Π½Π΅ΡˆΠ½ΠΈΡ… ΠΏΠΎΠΏΡ‹Ρ‚ΠΎΠΊ ΠΎΠ±ΡŠΠ΅Π΄ΠΈΠ½ΠΈΡ‚ΡŒ Ρ‡Π΅Ρ‚Ρ‹Ρ€Π΅ основныС силы Π² ΠΏΡ€ΠΈΡ€ΠΎΠ΄Π΅ — Π³Ρ€Π°Π²ΠΈΡ‚Π°Ρ†ΠΈΠΎΠ½Π½ΠΎΠ΅, элСктричСскоС, сильноС ΠΈ слабоС ядСрныС взаимодСйствия.

ΠŸΠΎΡΠΊΠΎΠ»ΡŒΠΊΡƒ ΠΈΠ·ΠΌΠ΅Π½ΡΡŽΡ‰ΠΈΠ΅ΡΡ элСктричСскиС поля ΡΠΎΠ·Π΄Π°ΡŽΡ‚ ΠΎΡ‚Π½ΠΎΡΠΈΡ‚Π΅Π»ΡŒΠ½ΠΎ слабыС ΠΌΠ°Π³Π½ΠΈΡ‚Π½Ρ‹Π΅ поля, ΠΈΡ… Π±Ρ‹Π»ΠΎ Π½Π΅Π»Π΅Π³ΠΊΠΎ ΠΎΠ±Π½Π°Ρ€ΡƒΠΆΠΈΡ‚ΡŒ Π²ΠΎ врСмя Π³ΠΈΠΏΠΎΡ‚Π΅Π·Ρ‹ МаксвСлла. Однако МаксвСлл понял, Ρ‡Ρ‚ΠΎ ΠΊΠΎΠ»Π΅Π±Π»ΡŽΡ‰ΠΈΠ΅ΡΡ заряды, ΠΊΠ°ΠΊ Π² цСпях ΠΏΠ΅Ρ€Π΅ΠΌΠ΅Π½Π½ΠΎΠ³ΠΎ Ρ‚ΠΎΠΊΠ°, ΡΠΎΠ·Π΄Π°ΡŽΡ‚ ΠΈΠ·ΠΌΠ΅Π½ΡΡŽΡ‰ΠΈΠ΅ΡΡ элСктричСскиС поля. Он прСдсказал, Ρ‡Ρ‚ΠΎ эти ΠΈΠ·ΠΌΠ΅Π½ΡΡŽΡ‰ΠΈΠ΅ΡΡ поля Π±ΡƒΠ΄ΡƒΡ‚ Ρ€Π°ΡΠΏΡ€ΠΎΡΡ‚Ρ€Π°Π½ΡΡ‚ΡŒΡΡ ΠΎΡ‚ источника, ΠΊΠ°ΠΊ Π²ΠΎΠ»Π½Ρ‹, создаваСмыС ΠΏΡ€Ρ‹Π³Π°ΡŽΡ‰Π΅ΠΉ Ρ€Ρ‹Π±ΠΎΠΉ Π² ΠΎΠ·Π΅Ρ€Π΅.

Π’ΠΎΠ»Π½Ρ‹, прСдсказанныС МаксвСллом, Π±ΡƒΠ΄ΡƒΡ‚ ΡΠΎΡΡ‚ΠΎΡΡ‚ΡŒ ΠΈΠ· ΠΊΠΎΠ»Π΅Π±Π»ΡŽΡ‰ΠΈΡ…ΡΡ элСктричСских ΠΈ ΠΌΠ°Π³Π½ΠΈΡ‚Π½Ρ‹Ρ… ΠΏΠΎΠ»Π΅ΠΉ, опрСдСляСмых ΠΊΠ°ΠΊ элСктромагнитная Π²ΠΎΠ»Π½Π° (элСктромагнитная Π²ΠΎΠ»Π½Π°). Π­Π»Π΅ΠΊΡ‚Ρ€ΠΎΠΌΠ°Π³Π½ΠΈΡ‚Π½Ρ‹Π΅ Π²ΠΎΠ»Π½Ρ‹ Π±ΡƒΠ΄ΡƒΡ‚ способны Π²ΠΎΠ·Π΄Π΅ΠΉΡΡ‚Π²ΠΎΠ²Π°Ρ‚ΡŒ Π½Π° заряды Π½Π° большом расстоянии ΠΎΡ‚ ΠΈΡ… источника, ΠΈ, Ρ‚Π°ΠΊΠΈΠΌ ΠΎΠ±Ρ€Π°Π·ΠΎΠΌ, ΠΈΡ… ΠΌΠΎΠΆΠ½ΠΎ Π±ΡƒΠ΄Π΅Ρ‚ ΠΎΠ±Π½Π°Ρ€ΡƒΠΆΠΈΡ‚ΡŒ. МаксвСлл рассчитал, Ρ‡Ρ‚ΠΎ элСктромагнитныС Π²ΠΎΠ»Π½Ρ‹ Ρ€Π°ΡΠΏΡ€ΠΎΡΡ‚Ρ€Π°Π½ΡΡŽΡ‚ΡΡ со ΡΠΊΠΎΡ€ΠΎΡΡ‚ΡŒΡŽ, Π·Π°Π΄Π°Π²Π°Π΅ΠΌΠΎΠΉ ΡƒΡ€Π°Π²Π½Π΅Π½ΠΈΠ΅ΠΌ

.

[латСкс] \ displaystyle {c} = \ frac {1} {\ sqrt {\ mu_ {0} \ epsilon_0}} \\ [/ latex]

Когда значСния для ΞΌ 0 ΠΈ Ξ΅ 0 вводятся Π² ΡƒΡ€Π°Π²Π½Π΅Π½ΠΈΠ΅ для c , ΠΌΡ‹ Π½Π°Ρ…ΠΎΠ΄ΠΈΠΌ, Ρ‡Ρ‚ΠΎ

[латСкс] \ displaystyle {c} = \ frac {1} {\ sqrt {\ left (8.8 \ text {m / s} \\ [/ latex]

— ΡΠΊΠΎΡ€ΠΎΡΡ‚ΡŒ свСта. ЀактичСски, МаксвСлл ΠΏΡ€ΠΈΡˆΠ΅Π» ΠΊ Π²Ρ‹Π²ΠΎΠ΄Ρƒ, Ρ‡Ρ‚ΠΎ свСт — это элСктромагнитная Π²ΠΎΠ»Π½Π° с Ρ‚Π°ΠΊΠΎΠΉ Π΄Π»ΠΈΠ½ΠΎΠΉ Π²ΠΎΠ»Π½Ρ‹, Ρ‡Ρ‚ΠΎ Π΅Π΅ ΠΌΠΎΠΆΠ½ΠΎ ΠΎΠ±Π½Π°Ρ€ΡƒΠΆΠΈΡ‚ΡŒ Π³Π»Π°Π·ΠΎΠΌ.

Π”ΠΎΠ»ΠΆΠ½Ρ‹ ΡΡƒΡ‰Π΅ΡΡ‚Π²ΠΎΠ²Π°Ρ‚ΡŒ Π΄Ρ€ΡƒΠ³ΠΈΠ΅ Π΄Π»ΠΈΠ½Ρ‹ Π²ΠΎΠ»Π½ — Π΅Ρ‰Π΅ нСизвСстно, ΡΡƒΡ‰Π΅ΡΡ‚Π²ΡƒΡŽΡ‚ Π»ΠΈ ΠΎΠ½ΠΈ. Если Ρ‚Π°ΠΊ, тСория МаксвСлла ΠΈ Π΅Π³ΠΎ Π·Π°ΠΌΠ΅Ρ‡Π°Ρ‚Π΅Π»ΡŒΠ½Ρ‹Π΅ прСдсказания подтвСрдятся, Ρ‡Ρ‚ΠΎ станСт Π²Π΅Π»ΠΈΡ‡Π°ΠΉΡˆΠΈΠΌ Ρ‚Ρ€ΠΈΡƒΠΌΡ„ΠΎΠΌ Ρ„ΠΈΠ·ΠΈΠΊΠΈ со Π²Ρ€Π΅ΠΌΠ΅Π½ ΠΡŒΡŽΡ‚ΠΎΠ½Π°. Π­ΠΊΡΠΏΠ΅Ρ€ΠΈΠΌΠ΅Π½Ρ‚Π°Π»ΡŒΠ½Π°Ρ ΠΏΡ€ΠΎΠ²Π΅Ρ€ΠΊΠ° ΠΏΡ€ΠΎΠΈΠ·ΠΎΡˆΠ»Π° Ρ‡Π΅Ρ€Π΅Π· нСсколько Π»Π΅Ρ‚, Π½ΠΎ Π½Π΅ Ρ€Π°Π½ΡŒΡˆΠ΅ смСрти МаксвСлла.

НаблюдСния Π“Π΅Ρ€Ρ†Π°

НСмСцкий Ρ„ΠΈΠ·ΠΈΠΊ Π“Π΅Π½Ρ€ΠΈΡ… Π“Π΅Ρ€Ρ† (1857–1894) Π±Ρ‹Π» ΠΏΠ΅Ρ€Π²Ρ‹ΠΌ, ΠΊΡ‚ΠΎ Π³Π΅Π½Π΅Ρ€ΠΈΡ€ΠΎΠ²Π°Π» ΠΈ ΠΎΠ±Π½Π°Ρ€ΡƒΠΆΠΈΠ²Π°Π» ΠΎΠΏΡ€Π΅Π΄Π΅Π»Π΅Π½Π½Ρ‹Π΅ Ρ‚ΠΈΠΏΡ‹ элСктромагнитных Π²ΠΎΠ»Π½ Π² Π»Π°Π±ΠΎΡ€Π°Ρ‚ΠΎΡ€ΠΈΠΈ.Начиная с 1887 Π³ΠΎΠ΄Π°, ΠΎΠ½ ΠΏΡ€ΠΎΠ²Π΅Π» ΡΠ΅Ρ€ΠΈΡŽ экспСримСнтов, ΠΊΠΎΡ‚ΠΎΡ€Ρ‹Π΅ Π½Π΅ Ρ‚ΠΎΠ»ΡŒΠΊΠΎ ΠΏΠΎΠ΄Ρ‚Π²Π΅Ρ€Π΄ΠΈΠ»ΠΈ сущСствованиС элСктромагнитных Π²ΠΎΠ»Π½, Π½ΠΎ ΠΈ ΠΏΠΎΠ΄Ρ‚Π²Π΅Ρ€Π΄ΠΈΠ»ΠΈ, Ρ‡Ρ‚ΠΎ ΠΎΠ½ΠΈ двиТутся со ΡΠΊΠΎΡ€ΠΎΡΡ‚ΡŒΡŽ свСта.

Π“Π΅Ρ€Ρ† использовал схСму AC RLC (рСзистор-ΠΈΠ½Π΄ΡƒΠΊΡ‚ΠΎΡ€-кондСнсатор), которая Ρ€Π΅Π·ΠΎΠ½ΠΈΡ€ΡƒΠ΅Ρ‚ Π½Π° извСстной частотС [латСкс] f_0 = \ frac {1} {2 \ pi \ sqrt {LC}} \ [/ латСкс] ΠΈ ΠΏΠΎΠ΄ΠΊΠ»ΡŽΡ‡Π΅Π½Π° Π΅Π³ΠΎ ΠΊ ΠΏΡ€ΠΎΠ²ΠΎΠ»ΠΎΡ‡Π½ΠΎΠΉ ΠΏΠ΅Ρ‚Π»Π΅, ΠΊΠ°ΠΊ ΠΏΠΎΠΊΠ°Π·Π°Π½ΠΎ Π½Π° рисункС 2. ВысокоС напряТСниС, ΠΈΠ½Π΄ΡƒΡ†ΠΈΡ€ΠΎΠ²Π°Π½Π½ΠΎΠ΅ Ρ‡Π΅Ρ€Π΅Π· Π·Π°Π·ΠΎΡ€ Π² ΠΏΠ΅Ρ‚Π»Π΅, Π²Ρ‹Π·Ρ‹Π²Π°Π»ΠΎ искры, ΠΊΠΎΡ‚ΠΎΡ€Ρ‹Π΅ Π±Ρ‹Π»ΠΈ Π²ΠΈΠ΄ΠΈΠΌΡ‹ΠΌ ΡΠ²ΠΈΠ΄Π΅Ρ‚Π΅Π»ΡŒΡΡ‚Π²ΠΎΠΌ наличия Ρ‚ΠΎΠΊΠ° Π² Ρ†Π΅ΠΏΠΈ ΠΈ ΠΏΠΎΠΌΠΎΠ³Π°Π»ΠΈ Π³Π΅Π½Π΅Ρ€ΠΈΡ€ΠΎΠ²Π°Ρ‚ΡŒ элСктромагнитныС Π²ΠΎΠ»Π½Ρ‹.

Π§Π΅Ρ€Π΅Π· Π»Π°Π±ΠΎΡ€Π°Ρ‚ΠΎΡ€ΠΈΡŽ Π“Π΅Ρ€Ρ† ΠΏΠΎΠ΄ΠΊΠ»ΡŽΡ‡ΠΈΠ» Π΅Ρ‰Π΅ ΠΎΠ΄ΠΈΠ½ ΠΊΠΎΠ½Ρ‚ΡƒΡ€ ΠΊ Π΄Ρ€ΡƒΠ³ΠΎΠΌΡƒ ΠΊΠΎΠ½Ρ‚ΡƒΡ€Ρƒ RLC , ΠΊΠΎΡ‚ΠΎΡ€Ρ‹ΠΉ ΠΌΠΎΠΆΠ½ΠΎ Π±Ρ‹Π»ΠΎ Π½Π°ΡΡ‚Ρ€ΠΎΠΈΡ‚ΡŒ (ΠΊΠ°ΠΊ Ρ†ΠΈΡ„Π΅Ρ€Π±Π»Π°Ρ‚ Π½Π° Ρ€Π°Π΄ΠΈΠΎ) Π½Π° Ρ‚Ρƒ ΠΆΠ΅ Ρ€Π΅Π·ΠΎΠ½Π°Π½ΡΠ½ΡƒΡŽ частоту, Ρ‡Ρ‚ΠΎ ΠΈ ΠΏΠ΅Ρ€Π²Ρ‹ΠΉ, ΠΈ, Ρ‚Π°ΠΊΠΈΠΌ ΠΎΠ±Ρ€Π°Π·ΠΎΠΌ, ΠΌΠΎΠΆΠ½ΠΎ Π±Ρ‹Π»ΠΎ Π·Π°ΡΡ‚Π°Π²ΠΈΡ‚ΡŒ ΠΏΡ€ΠΈΠ½ΠΈΠΌΠ°Ρ‚ΡŒ элСктромагнитныС Π²ΠΎΠ»Π½Ρ‹. Π’ этой ΠΏΠ΅Ρ‚Π»Π΅ Ρ‚Π°ΠΊΠΆΠ΅ Π±Ρ‹Π» Π·Π°Π·ΠΎΡ€, Π² ΠΊΠΎΡ‚ΠΎΡ€ΠΎΠΌ Π²ΠΎΠ·Π½ΠΈΠΊΠ°Π»ΠΈ искры, Ρ‡Ρ‚ΠΎ Π΄Π°Π²Π°Π»ΠΎ Ρ‚Π²Π΅Ρ€Π΄ΠΎΠ΅ ΡΠ²ΠΈΠ΄Π΅Ρ‚Π΅Π»ΡŒΡΡ‚Π²ΠΎ ΠΏΡ€ΠΈΠ΅ΠΌΠ° элСктромагнитных Π²ΠΎΠ»Π½.

Рис. 2. Устройство, ΠΊΠΎΡ‚ΠΎΡ€ΠΎΠ΅ Π“Π΅Ρ€Ρ† использовал Π² 1887 Π³ΠΎΠ΄Ρƒ для Π³Π΅Π½Π΅Ρ€Π°Ρ†ΠΈΠΈ ΠΈ обнаруТСния элСктромагнитных Π²ΠΎΠ»Π½. Π‘Ρ…Π΅ΠΌΠ° RLC , ΠΏΠΎΠ΄ΠΊΠ»ΡŽΡ‡Π΅Π½Π½Π°Ρ ΠΊ ΠΏΠ΅Ρ€Π²ΠΎΠΌΡƒ ΠΊΠΎΠ½Ρ‚ΡƒΡ€Ρƒ, Π²Ρ‹Π·Π²Π°Π»Π° искры Ρ‡Π΅Ρ€Π΅Π· Ρ€Π°Π·Ρ€Ρ‹Π² Π² ΠΏΡ€ΠΎΠ²ΠΎΠ΄Π½ΠΎΠΌ ΠΊΠΎΠ½Ρ‚ΡƒΡ€Π΅ ΠΈ Π³Π΅Π½Π΅Ρ€ΠΈΡ€ΠΎΠ²Π°Π»Π° элСктромагнитныС Π²ΠΎΠ»Π½Ρ‹.Π˜ΡΠΊΡ€Ρ‹ Π² Ρ‰Π΅Π»ΠΈ Π²ΠΎ Π²Ρ‚ΠΎΡ€ΠΎΠΉ ΠΏΠ΅Ρ‚Π»Π΅, располоТСнной Π½Π°ΠΏΡ€ΠΎΡ‚ΠΈΠ² Π»Π°Π±ΠΎΡ€Π°Ρ‚ΠΎΡ€ΠΈΠΈ, ΡΠ²ΠΈΠ΄Π΅Ρ‚Π΅Π»ΡŒΡΡ‚Π²ΠΎΠ²Π°Π»ΠΈ ΠΎ Ρ‚ΠΎΠΌ, Ρ‡Ρ‚ΠΎ Π²ΠΎΠ»Π½Ρ‹ Π±Ρ‹Π»ΠΈ приняты.

Π“Π΅Ρ€Ρ† Ρ‚Π°ΠΊΠΆΠ΅ ΠΈΠ·ΡƒΡ‡ΠΈΠ» ΠΊΠ°Ρ€Ρ‚ΠΈΠ½Ρ‹ отраТСния, прСломлСния ΠΈ ΠΈΠ½Ρ‚Π΅Ρ€Ρ„Π΅Ρ€Π΅Π½Ρ†ΠΈΠΈ Π³Π΅Π½Π΅Ρ€ΠΈΡ€ΡƒΠ΅ΠΌΡ‹Ρ… ΠΈΠΌ элСктромагнитных Π²ΠΎΠ»Π½, провСряя ΠΈΡ… Π²ΠΎΠ»Π½ΠΎΠ²ΠΎΠΉ Ρ…Π°Ρ€Π°ΠΊΡ‚Π΅Ρ€. Он смог ΠΎΠΏΡ€Π΅Π΄Π΅Π»ΠΈΡ‚ΡŒ Π΄Π»ΠΈΠ½Ρƒ Π²ΠΎΠ»Π½Ρ‹ ΠΈΠ· ΠΈΠ½Ρ‚Π΅Ρ€Ρ„Π΅Ρ€Π΅Π½Ρ†ΠΈΠΎΠ½Π½Ρ‹Ρ… ΠΊΠ°Ρ€Ρ‚ΠΈΠ½ ΠΈ, зная ΠΈΡ… частоту, ΠΎΠ½ ΠΌΠΎΠ³ Π²Ρ‹Ρ‡ΠΈΡΠ»ΠΈΡ‚ΡŒ ΡΠΊΠΎΡ€ΠΎΡΡ‚ΡŒ распространСния, ΠΈΡΠΏΠΎΠ»ΡŒΠ·ΡƒΡ ΡƒΡ€Π°Π²Π½Π΅Π½ΠΈΠ΅ v = fΞ» (ΡΠΊΠΎΡ€ΠΎΡΡ‚ΡŒ — ΠΈΠ»ΠΈ ΡΠΊΠΎΡ€ΠΎΡΡ‚ΡŒ — Ρ€Π°Π²Π½Π° частотС, ΡƒΠΌΠ½ΠΎΠΆΠ΅Π½Π½ΠΎΠΉ Π½Π° Π΄Π»ΠΈΠ½Ρƒ Π²ΠΎΠ»Π½Ρ‹).Π’Π°ΠΊΠΈΠΌ ΠΎΠ±Ρ€Π°Π·ΠΎΠΌ, Π“Π΅Ρ€Ρ† смог Π΄ΠΎΠΊΠ°Π·Π°Ρ‚ΡŒ, Ρ‡Ρ‚ΠΎ элСктромагнитныС Π²ΠΎΠ»Π½Ρ‹ Ρ€Π°ΡΠΏΡ€ΠΎΡΡ‚Ρ€Π°Π½ΡΡŽΡ‚ΡΡ со ΡΠΊΠΎΡ€ΠΎΡΡ‚ΡŒΡŽ свСта. Π•Π΄ΠΈΠ½ΠΈΡ†Π° измСрСния частоты Π² систСмС БИ, Π³Π΅Ρ€Ρ† (1 Π“Ρ† = 1 Ρ†ΠΈΠΊΠ» / сСк), Π½Π°Π·Π²Π°Π½Π° Π² Π΅Π³ΠΎ Ρ‡Π΅ΡΡ‚ΡŒ.

Π‘Π²ΠΎΠ΄ΠΊΠ° Ρ€Π°Π·Π΄Π΅Π»Π°

  • Π­Π»Π΅ΠΊΡ‚Ρ€ΠΎΠΌΠ°Π³Π½ΠΈΡ‚Π½Ρ‹Π΅ Π²ΠΎΠ»Π½Ρ‹ состоят ΠΈΠ· ΠΊΠΎΠ»Π΅Π±Π»ΡŽΡ‰ΠΈΡ…ΡΡ элСктричСских ΠΈ ΠΌΠ°Π³Π½ΠΈΡ‚Π½Ρ‹Ρ… ΠΏΠΎΠ»Π΅ΠΉ ΠΈ Ρ€Π°ΡΠΏΡ€ΠΎΡΡ‚Ρ€Π°Π½ΡΡŽΡ‚ΡΡ со ΡΠΊΠΎΡ€ΠΎΡΡ‚ΡŒΡŽ свСта c . Π˜Ρ… прСдсказал МаксвСлл, ΠΊΠΎΡ‚ΠΎΡ€Ρ‹ΠΉ Ρ‚Π°ΠΊΠΆΠ΅ ΠΏΠΎΠΊΠ°Π·Π°Π», Ρ‡Ρ‚ΠΎ
    [латСкс] \ displaystyle {c} = \ frac {1} {\ sqrt {{\ mu} _ {0} {\ epsilon} _ {0}}} \\ [ / латСкс],
    , Π³Π΄Π΅ ΞΌ 0 — ΠΏΡ€ΠΎΠ½ΠΈΡ†Π°Π΅ΠΌΠΎΡΡ‚ΡŒ свободного пространства, Π° Ξ΅ 0 — диэлСктричСская ΠΏΡ€ΠΎΠ½ΠΈΡ†Π°Π΅ΠΌΠΎΡΡ‚ΡŒ свободного пространства.
  • ΠŸΡ€Π΅Π΄ΡΠΊΠ°Π·Π°Π½ΠΈΠ΅ МаксвСлла ΠΎΠ± элСктромагнитных Π²ΠΎΠ»Π½Π°Ρ… стало Ρ€Π΅Π·ΡƒΠ»ΡŒΡ‚Π°Ρ‚ΠΎΠΌ Π΅Π³ΠΎ Ρ„ΠΎΡ€ΠΌΡƒΠ»ΠΈΡ€ΠΎΠ²ΠΊΠΈ ΠΏΠΎΠ»Π½ΠΎΠΉ ΠΈ симмСтричной Ρ‚Π΅ΠΎΡ€ΠΈΠΈ элСктричСства ΠΈ ΠΌΠ°Π³Π½Π΅Ρ‚ΠΈΠ·ΠΌΠ°, извСстной ΠΊΠ°ΠΊ уравнСния МаксвСлла.
  • Π­Ρ‚ΠΈ Ρ‡Π΅Ρ‚Ρ‹Ρ€Π΅ уравнСния ΠΏΠ΅Ρ€Π΅Ρ„Ρ€Π°Π·ΠΈΡ€ΠΎΠ²Π°Π½Ρ‹ Π² этом тСкстС, Π° Π½Π΅ прСдставлСны Π² числовом Π²ΠΈΠ΄Π΅, ΠΈ ΠΎΡ…Π²Π°Ρ‚Ρ‹Π²Π°ΡŽΡ‚ основныС Π·Π°ΠΊΠΎΠ½Ρ‹ элСктричСства ΠΈ ΠΌΠ°Π³Π½Π΅Ρ‚ΠΈΠ·ΠΌΠ°. ΠŸΠ΅Ρ€Π²Ρ‹ΠΉ — это Π·Π°ΠΊΠΎΠ½ Гаусса для элСктричСства, Π²Ρ‚ΠΎΡ€ΠΎΠΉ — Π·Π°ΠΊΠΎΠ½ Гаусса для ΠΌΠ°Π³Π½Π΅Ρ‚ΠΈΠ·ΠΌΠ°, Ρ‚Ρ€Π΅Ρ‚ΠΈΠΉ — Π·Π°ΠΊΠΎΠ½ ΠΈΠ½Π΄ΡƒΠΊΡ†ΠΈΠΈ ЀарадСя, Π²ΠΊΠ»ΡŽΡ‡Π°Ρ Π·Π°ΠΊΠΎΠ½ Π›Π΅Π½Ρ†Π°, ΠΈ Ρ‡Π΅Ρ‚Π²Π΅Ρ€Ρ‚Ρ‹ΠΉ — Π·Π°ΠΊΠΎΠ½ АмпСра Π² симмСтричной Ρ„ΠΎΡ€ΠΌΡƒΠ»ΠΈΡ€ΠΎΠ²ΠΊΠ΅, ΠΊΠΎΡ‚ΠΎΡ€Ρ‹ΠΉ добавляСт Π΅Ρ‰Π΅ ΠΎΠ΄ΠΈΠ½ источник ΠΌΠ°Π³Π½Π΅Ρ‚ΠΈΠ·ΠΌΠ° — ΠΈΠ·ΠΌΠ΅Π½Π΅Π½ΠΈΠ΅ элСктричСских ΠΏΠΎΠ»Π΅ΠΉ.

Π—Π°Π΄Π°Ρ‡ΠΈ ΠΈ упраТнСния

  1. Π£Π±Π΅Π΄ΠΈΡ‚Π΅ΡΡŒ, Ρ‡Ρ‚ΠΎ ΠΏΡ€Π°Π²ΠΈΠ»ΡŒΠ½ΠΎΠ΅ Π·Π½Π°Ρ‡Π΅Π½ΠΈΠ΅ скорости свСта c получаСтся, ΠΊΠΎΠ³Π΄Π° числовыС значСния проницаСмости ΠΈ диэлСктричСской проницаСмости свободного пространства (ΞΌ 0 ΠΈ Ξ΅ 0 ) вводятся Π² ΡƒΡ€Π°Π²Π½Π΅Π½ΠΈΠ΅ [латСкс] c = \ Π³ΠΈΠ΄Ρ€ΠΎΡ€Π°Π·Ρ€Ρ‹Π² {1} {\ sqrt {{\ mu} _ {0} {\ epsilon} _ {0}}} \\ [/ latex].
  2. ΠŸΠΎΠΊΠ°ΠΆΠΈΡ‚Π΅, Ρ‡Ρ‚ΠΎ, ΠΊΠΎΠ³Π΄Π° вводятся Π΅Π΄ΠΈΠ½ΠΈΡ†Ρ‹ БИ для ΞΌ 0 ΠΈ Ξ΅ 0 , Π΅Π΄ΠΈΠ½ΠΈΡ†Ρ‹, Π·Π°Π΄Π°Π½Π½Ρ‹Π΅ ΠΏΡ€Π°Π²ΠΎΠΉ Ρ‡Π°ΡΡ‚ΡŒΡŽ уравнСния Π² ΠΏΡ€ΠΈΠ²Π΅Π΄Π΅Π½Π½ΠΎΠΉ Π²Ρ‹ΡˆΠ΅ Π·Π°Π΄Π°Ρ‡Π΅, Ρ€Π°Π²Π½Ρ‹ ΠΌ / с.

Глоссарий

Π²ΠΎΠ»Π½Ρ‹ элСктромагнитныС: ΠΈΠ·Π»ΡƒΡ‡Π΅Π½ΠΈΠ΅ Π² Π²ΠΈΠ΄Π΅ Π²ΠΎΠ»Π½ элСктричСской ΠΈ ΠΌΠ°Π³Π½ΠΈΡ‚Π½ΠΎΠΉ энСргии

УравнСния МаксвСлла: Набор ΠΈΠ· Ρ‡Π΅Ρ‚Ρ‹Ρ€Π΅Ρ… ΡƒΡ€Π°Π²Π½Π΅Π½ΠΈΠΉ, ΡΠΎΡΡ‚Π°Π²Π»ΡΡŽΡ‰ΠΈΡ… ΠΏΠΎΠ»Π½ΡƒΡŽ, Π²ΡΠ΅ΠΎΠ±ΡŠΠ΅ΠΌΠ»ΡŽΡ‰ΡƒΡŽ Ρ‚Π΅ΠΎΡ€ΠΈΡŽ элСктромагнСтизма

ЦСпь RLC: элСктричСская Ρ†Π΅ΠΏΡŒ, которая Π²ΠΊΠ»ΡŽΡ‡Π°Π΅Ρ‚ рСзистор, кондСнсатор ΠΈ ΠΊΠ°Ρ‚ΡƒΡˆΠΊΡƒ индуктивности

Π³Π΅Ρ€Ρ†: Π΅Π΄ΠΈΠ½ΠΈΡ†Π° БИ, ΠΎΠ±ΠΎΠ·Π½Π°Ρ‡Π°ΡŽΡ‰Π°Ρ частоту элСктромагнитной Π²ΠΎΠ»Π½Ρ‹ Π² Ρ†ΠΈΠΊΠ»Π°Ρ… Π² сСкунду

ΡΠΊΠΎΡ€ΠΎΡΡ‚ΡŒ свСта: Π² Π²Π°ΠΊΡƒΡƒΠΌΠ΅, Π½Π°ΠΏΡ€ΠΈΠΌΠ΅Ρ€ Π² космосС, ΡΠΊΠΎΡ€ΠΎΡΡ‚ΡŒ свСта постоянна 3 Γ— 10 8 ΠΌ / с

элСктродвиТущая сила (Π­Π”Π‘): энСргия, вырабатываСмая Π½Π° Π΅Π΄ΠΈΠ½ΠΈΡ†Ρƒ заряда, получСнная ΠΈΠ· источника, производящСго элСктричСский Ρ‚ΠΎΠΊ

Π›ΠΈΠ½ΠΈΠΈ элСктричСского поля: шаблон Π²ΠΎΠΎΠ±Ρ€Π°ΠΆΠ°Π΅ΠΌΡ‹Ρ… Π»ΠΈΠ½ΠΈΠΉ, ΠΊΠΎΡ‚ΠΎΡ€Ρ‹Π΅ проходят ΠΌΠ΅ΠΆΠ΄Ρƒ источником элСктричСского Ρ‚ΠΎΠΊΠ° ΠΈ заряТСнными ΠΎΠ±ΡŠΠ΅ΠΊΡ‚Π°ΠΌΠΈ Π² ΠΎΠΊΡ€ΡƒΠΆΠ°ΡŽΡ‰Π΅ΠΉ области, со стрСлками, Π½Π°ΠΏΡ€Π°Π²Π»Π΅Π½Π½Ρ‹ΠΌΠΈ ΠΎΡ‚ ΠΏΠΎΠ»ΠΎΠΆΠΈΡ‚Π΅Π»ΡŒΠ½ΠΎ заряТСнных ΠΎΠ±ΡŠΠ΅ΠΊΡ‚ΠΎΠ² ΠΊ ΠΎΡ‚Ρ€ΠΈΡ†Π°Ρ‚Π΅Π»ΡŒΠ½ΠΎ заряТСнным ΠΎΠ±ΡŠΠ΅ΠΊΡ‚Π°ΠΌ.Π§Π΅ΠΌ большС Π»ΠΈΠ½ΠΈΠΉ Π² ΡƒΠ·ΠΎΡ€Π΅, Ρ‚Π΅ΠΌ сильнСС элСктричСскоС ΠΏΠΎΠ»Π΅ Π² этой области

Π»ΠΈΠ½ΠΈΠΈ ΠΌΠ°Π³Π½ΠΈΡ‚Π½ΠΎΠ³ΠΎ поля: ΡƒΠ·ΠΎΡ€ ΠΈΠ· Π½Π΅ΠΏΡ€Π΅Ρ€Ρ‹Π²Π½Ρ‹Ρ… Π²ΠΎΠΎΠ±Ρ€Π°ΠΆΠ°Π΅ΠΌΡ‹Ρ… Π»ΠΈΠ½ΠΈΠΉ, ΠΊΠΎΡ‚ΠΎΡ€Ρ‹Π΅ выходят ΠΈΠ· ΠΏΡ€ΠΎΡ‚ΠΈΠ²ΠΎΠΏΠΎΠ»ΠΎΠΆΠ½Ρ‹Ρ… ΠΌΠ°Π³Π½ΠΈΡ‚Π½Ρ‹Ρ… полюсов ΠΈ входят Π² Π½ΠΈΡ…. ΠŸΠ»ΠΎΡ‚Π½ΠΎΡΡ‚ΡŒ Π»ΠΈΠ½ΠΈΠΉ ΡƒΠΊΠ°Π·Ρ‹Π²Π°Π΅Ρ‚ Π²Π΅Π»ΠΈΡ‡ΠΈΠ½Ρƒ ΠΌΠ°Π³Π½ΠΈΡ‚Π½ΠΎΠ³ΠΎ поля

ΡƒΡ€Π°Π²Π½Π΅Π½ΠΈΠΉ МаксвСлла ΠΈ свСт

уравнСния МаксвСлла ΠΈ свСт

УравнСния МаксвСлла ΠΈ свСт

ΠœΡ‹ Π·Π°Π²Π΅Ρ€ΡˆΠΈΠ»ΠΈ ΠΈΠ·ΡƒΡ‡Π΅Π½ΠΈΠ΅ Π·Π°ΠΊΠΎΠ½ΠΎΠ² элСктричСства ΠΈ ΠΌΠ°Π³Π½Π΅Ρ‚ΠΈΠ·ΠΌΠ°.ВсС, Ρ‡Ρ‚ΠΎ ΠΌΡ‹ ΡƒΠ·Π½Π°Π»ΠΈ ΠΌΠΎΠΆΠ½ΠΎ ΡΡƒΠΌΠΌΠΈΡ€ΠΎΠ²Π°Ρ‚ΡŒ Π² уравнСниях, извСстных ΠΊΠ°ΠΊ уравнСния МаксвСлла , вмСстС с Ρ„ΠΎΡ€ΠΌΡƒΠ»Π°ΠΌΠΈ для элСктричСских ΠΈ ΠΌΠ°Π³Π½ΠΈΡ‚Π½Ρ‹Ρ… сил Π½Π° зарядС. Один ΠΈΠ· Π½Π΅ΡΠΊΠΎΠ»ΡŒΠΊΠΈΡ… способов прСобразования ΡƒΡ€Π°Π²Π½Π΅Π½ΠΈΠΉ МаксвСлла: написано отобраТаСтся справа, Ρ‡Ρ‚ΠΎΠ±Ρ‹ Π²Ρ‹ ΠΌΠΎΠ³Π»ΠΈ ΡΠΊΠ°Π·Π°Ρ‚ΡŒ, Ρ‡Ρ‚ΠΎ Π²ΠΈΠ΄Π΅Π»ΠΈ ΠΈΡ…. ΠŸΠ΅Ρ€Π²ΠΎΠ΅ ΡƒΡ€Π°Π²Π½Π΅Π½ΠΈΠ΅ — это Π·Π°ΠΊΠΎΠ½ АмпСра (Π²ΠΊΠ»ΡŽΡ‡Π°Ρ Ρ‡Π»Π΅Π½ Π² ΠΏΡ€Π°Π²ΠΎΠΉ части, ΠΊΠΎΡ‚ΠΎΡ€Ρ‹ΠΉ ΠΌΡ‹ Π½Π΅ обсуТдали Π² этом курсС). Π’Ρ‚ΠΎΡ€ΠΎΠ΅ ΡƒΡ€Π°Π²Π½Π΅Π½ΠΈΠ΅ — это Π·Π°ΠΊΠΎΠ½ ЀарадСя Π² сочСтании с Π·Π°ΠΊΠΎΠ½ΠΎΠΌ Π›Π΅Π½Ρ†Π°. Π’Ρ€Π΅Ρ‚ΡŒΠ΅ ΡƒΡ€Π°Π²Π½Π΅Π½ΠΈΠ΅ — это Π·Π°ΠΊΠΎΠ½ Гаусса, ΠΊΠΎΡ‚ΠΎΡ€Ρ‹ΠΉ Π²Ρ‹Ρ€Π°ΠΆΠ°Π΅Ρ‚ Ρ‚ΠΎΡ‚ Ρ„Π°ΠΊΡ‚, Ρ‡Ρ‚ΠΎ силовыС Π»ΠΈΠ½ΠΈΠΈ элСктричСского поля Π½Π°Ρ‡ΠΈΠ½Π°ΡŽΡ‚ΡΡ ΠΈ Π·Π°ΠΊΠ°Π½Ρ‡ΠΈΠ²Π°ΡŽΡ‚ΡΡ Ρ‚ΠΎΠ»ΡŒΠΊΠΎ Π·Π° ΠΏΠ»Π°Ρ‚Ρƒ.Π§Π΅Ρ‚Π²Π΅Ρ€Ρ‚ΠΎΠ΅ ΡƒΡ€Π°Π²Π½Π΅Π½ΠΈΠ΅ являСтся ΠΌΠ°Π³Π½ΠΈΡ‚Π½Ρ‹ΠΌ эквивалСнтом Π·Π°ΠΊΠΎΠ½Π° Гаусса, ΠΊΠΎΡ‚ΠΎΡ€Ρ‹ΠΉ Π²Ρ‹Ρ€Π°ΠΆΠ°Π΅Ρ‚ Ρ‚ΠΎΡ‚ Ρ„Π°ΠΊΡ‚, Ρ‡Ρ‚ΠΎ ΠΌΠ°Π³Π½ΠΈΡ‚Π½Ρ‹Π΅ силовыС Π»ΠΈΠ½ΠΈΠΈ Π½ΠΈΠΊΠΎΠ³Π΄Π° Π½Π΅ Π½Π°Ρ‡ΠΈΠ½Π°ΡŽΡ‚ΡΡ ΠΈ Π½Π΅ Π·Π°ΠΊΠ°Π½Ρ‡ΠΈΠ²Π°ΡŽΡ‚ΡΡ (Ρ‚. Π΅. ΠΎΡ‚ΡΡƒΡ‚ΡΡ‚Π²ΡƒΡŽΡ‚ свободныС ΠΌΠ°Π³Π½ΠΈΡ‚Π½Ρ‹Π΅ заряды).

Π˜ΡΠΏΠΎΠ»ΡŒΠ·ΡƒΡ Π½Π΅ΠΊΠΎΡ‚ΠΎΡ€Ρ‹Π΅ Π½Π΅ ΠΎΡ‡Π΅Π½ΡŒ простыС вычислСния, уравнСния МаксвСлла ΠΌΠΎΠ³ΡƒΡ‚ Π±Ρ‹Ρ‚ΡŒ ΠΈΡΠΏΠΎΠ»ΡŒΠ·ΡƒΠ΅Ρ‚ΡΡ, Ρ‡Ρ‚ΠΎΠ±Ρ‹ ΠΏΠΎΠΊΠ°Π·Π°Ρ‚ΡŒ, Ρ‡Ρ‚ΠΎ элСктричСскоС ΠΈ ΠΌΠ°Π³Π½ΠΈΡ‚Π½ΠΎΠ΅ поля ΠΏΠΎΠ΄Ρ‡ΠΈΠ½ΡΡŽΡ‚ΡΡ Π²ΠΎΠ»Π½ΠΎΠ²Ρ‹ΠΌ уравнСниям. Π’ ΡΠΊΠΎΡ€ΠΎΡΡ‚ΡŒ c элСктромагнитной Π²ΠΎΠ»Π½Ρ‹ опрСдСляСтся константами элСктричСство ΠΈ ΠΌΠ°Π³Π½Π΅Ρ‚ΠΈΠ·ΠΌ, ΠΊΠΎΡ‚ΠΎΡ€Ρ‹Π΅ Π²Ρ‹ Ρ‚Π°ΠΊ Ρ…ΠΎΡ€ΠΎΡˆΠΎ Π·Π½Π°Π΅Ρ‚Π΅:
& nbsp c = 1 / ( e 0 м 0 ) 1/2 = 2.998 X 10 8 м / с .

Π‘Π²Π΅Ρ‚ — это элСктромагнитная Π²ΠΎΠ»Π½Π°: это понял МаксвСлл ΠΏΡ€ΠΈΠΌΠ΅Ρ€Π½ΠΎ Π² 1864 Π³ΠΎΠ΄Ρƒ, ΠΊΠ°ΠΊ Ρ‚ΠΎΠ»ΡŒΠΊΠΎ ΡƒΡ€Π°Π²Π½Π΅Π½ΠΈΠ΅ c = 1 / (e 0 ΠΌ 0 ) 1/2 = 2.998 X 10 8 ΠΌ / с Π±Ρ‹Π»ΠΎ ΠΎΠ±Π½Π°Ρ€ΡƒΠΆΠ΅Π½ΠΎ, ΠΏΠΎΡΠΊΠΎΠ»ΡŒΠΊΡƒ ΠΊ Ρ‚ΠΎΠΌΡƒ Π²Ρ€Π΅ΠΌΠ΅Π½ΠΈ ΡΠΊΠΎΡ€ΠΎΡΡ‚ΡŒ свСта Π±Ρ‹Π»Π° Ρ‚ΠΎΡ‡Π½ΠΎ ΠΈΠ·ΠΌΠ΅Ρ€Π΅Π½Π°, ΠΈ Π΅Π΅ соотвСтствиС с c вряд Π»ΠΈ Π±Ρ‹Π»ΠΎ совпадСниСм. Радиосигналы — это Ρ‚ΠΎΠΆΠ΅ элСктромагнитныС Π²ΠΎΠ»Π½Ρ‹; послС открытия ΡƒΡ€Π°Π²Π½Π΅Π½ΠΈΠΉ МаксвСлла ΠΏΠΎΡ‚Ρ€Π΅Π±ΠΎΠ²Π°Π»ΠΎΡΡŒ ΠΌΠ΅Π½Π΅Π΅ Π²Π΅ΠΊΠ°, Ρ‡Ρ‚ΠΎΠ±Ρ‹ Π²Ρ‹ΡΡΠ½ΠΈΡ‚ΡŒ, ΠΊΠ°ΠΊ ΠΏΡ€ΠΎΠΈΠ·Π²ΠΎΠ΄ΠΈΡ‚ΡŒ ΠΈ ΠΎΠ±Π½Π°Ρ€ΡƒΠΆΠΈΠ²Π°Ρ‚ΡŒ ΠΈΡ… Π΄ΠΎ Ρ‚Π°ΠΊΠΎΠΉ стСпСни, Ρ‡Ρ‚ΠΎΠ±Ρ‹ ΠΏΡ€ΠΎΠΈΠ·Π²Π΅Π» Ρ€Π΅Π²ΠΎΠ»ΡŽΡ†ΠΈΡŽ Π² нашСй тСхнологичСской Ρ†ΠΈΠ²ΠΈΠ»ΠΈΠ·Π°Ρ†ΠΈΠΈ.Π˜Π½Ρ„Ρ€Π°ΠΊΡ€Π°ΡΠ½Ρ‹ΠΉ свСт, ΡƒΠ»ΡŒΡ‚Ρ€Π°Ρ„ΠΈΠΎΠ»Π΅Ρ‚ΠΎΠ²Ρ‹ΠΉ свСт, рСнтгСновскиС Π»ΡƒΡ‡ΠΈ ΠΈ Π³Π°ΠΌΠΌΠ°-Π»ΡƒΡ‡ΠΈ Ρ‚Π°ΠΊΠΆΠ΅ ΡΠ²Π»ΡΡŽΡ‚ΡΡ ΠΏΡ€ΠΈΠΌΠ΅Ρ€Π°ΠΌΠΈ элСктромагнитных Π²ΠΎΠ»Π½; ΠΏΠΎΠ΄ΠΎΠ±Π½ΠΎ Ρ€Π°Π΄ΠΈΠΎΠ²ΠΎΠ»Π½Π°ΠΌ, ΠΎΠ½ΠΈ Π±Ρ‹Π»ΠΈ нСизвСстны Π² Ρ‚ΠΎ врСмя, ΠΊΠΎΠ³Π΄Π° Π±Ρ‹Π»ΠΈ ΠΎΡ‚ΠΊΡ€Ρ‹Ρ‚Ρ‹ уравнСния МаксвСлла.

Π”Π΅Ρ‚Π°Π»ΠΈ элСктромагнитной Π²ΠΎΠ»Π½Ρ‹ довольно слоТно. Π’Π΅ΠΊΡ‚ΠΎΡ€Ρ‹ элСктричСского ΠΈ ΠΌΠ°Π³Π½ΠΈΡ‚Π½ΠΎΠ³ΠΎ ΠΏΠΎΠ»Π΅ΠΉ ΡƒΠΊΠ°Π·Ρ‹Π²Π°ΡŽΡ‚ Π² направлСниях, ΠΊΠΎΡ‚ΠΎΡ€Ρ‹Π΅ пСрпСндикулярны Π΄Ρ€ΡƒΠ³ Π΄Ρ€ΡƒΠ³Ρƒ ΠΈ пСрпСндикулярны Π½Π°ΠΏΡ€Π°Π²Π»Π΅Π½ΠΈΡŽ распространСния Π²ΠΎΠ»Π½Ρ‹. Если Π²ΠΎΠ»Π½Π° двиТСтся Π² Π½Π°ΠΏΡ€Π°Π²Π»Π΅Π½ΠΈΠΈ z , ΠΈ элСктричСскоС ΠΏΠΎΠ»Π΅ колСблСтся Π² Π½Π°ΠΏΡ€Π°Π²Π»Π΅Π½ΠΈΠΈ x , Π·Π°Ρ‚Π΅ΠΌ ΠΌΠ°Π³Π½ΠΈΡ‚Π½ΠΎΠ΅ ΠΏΠΎΠ»Π΅ колСблСтся Π² Π½Π°ΠΏΡ€Π°Π²Π»Π΅Π½ΠΈΠΈ y .Вакая Π²ΠΎΠ»Π½Π° называСтся поляризованной Π² Π½Π°ΠΏΡ€Π°Π²Π»Π΅Π½ΠΈΠΈ , x , . (ΠŸΠΎΠ»ΡΡ€ΠΈΠ·Π°Ρ†ΠΈΡ описываСтся Π² Ρ‚Π΅Ρ€ΠΌΠΈΠ½Π°Ρ… элСктричСского поля, Π° Π½Π΅ ΠΌΠ°Π³Π½ΠΈΡ‚Π½ΠΎΠ³ΠΎ поля, ΠΏΠΎΡ‚ΠΎΠΌΡƒ Ρ‡Ρ‚ΠΎ элСктричСскоС ΠΏΠΎΠ»Π΅ Π»Π΅Π³Ρ‡Π΅ взаимодСйствуСт с Π±ΠΎΠ»ΡŒΡˆΠΈΠ½ΡΡ‚Π²ΠΎΠΌ ΠΌΠ°Ρ‚Π΅Ρ€ΠΈΠ°Π»ΠΎΠ².) На рисункС Π½ΠΈΠΆΠ΅ ΠΏΠΎΠΊΠ°Π·Π°Π½ΠΎ ΠΏΠΎΠ²Π΅Π΄Π΅Π½ΠΈΠ΅ поляризованной элСктромагнитной Π²ΠΎΠ»Π½Ρ‹. вдоль оси x ΠΈ ΠΏΠ΅Ρ€Π΅ΠΌΠ΅Ρ‰Π°ΡΡΡŒ Π² z — Π½Π°ΠΏΡ€Π°Π²Π»Π΅Π½ΠΈΠ΅.

Π€ΠΎΡ€ΠΌΡƒΠ»Ρ‹ для элСктричСского ΠΈ ΠΌΠ°Π³Π½ΠΈΡ‚Π½ΠΎΠ³ΠΎ ΠΏΠΎΠ»Π΅ΠΉ Π² этой Π²ΠΎΠ»Π½Π΅:

, Π³Π΄Π΅ частота ΠΈ Π΄Π»ΠΈΠ½Π° Π²ΠΎΠ»Π½Ρ‹ связаны ΡΠΎΠΎΡ‚Π½ΠΎΡˆΠ΅Π½ΠΈΠ΅ΠΌ c = f l .


Π­Π»Π΅ΠΊΡ‚Ρ€ΠΎΠΌΠ°Π³Π½ΠΈΡ‚Π½Ρ‹ΠΉ индСкс Π²ΠΎΠ»Π½ ΠŸΡ€ΠΈΠΌΠ΅Ρ€Ρ‹ Π£ΠΊΠ°Π·Π°Ρ‚Π΅Π»ΡŒ Π»Π΅ΠΊΡ†ΠΈΠΉ

ΠŸΠΎΠ·Π½Π°ΠΊΠΎΠΌΡŒΡ‚Π΅ΡΡŒ с уравнСниями МаксвСлла — Π²Ρ‹ ΠΈΡ… ΠΈΡΠΏΠΎΠ»ΡŒΠ·ΡƒΠ΅Ρ‚Π΅ прямо сСйчас | ΠŸΠ ΠžΠ’ΠžΠ”ΠΠžΠ™

Π’ΠΎΡ‚ Ρ‡Ρ‚ΠΎ ΠΏΡ€ΠΎΠΈΠ·ΠΎΠΉΠ΄Π΅Ρ‚, Ссли ΠΌΠ°Π³Π½ΠΈΡ‚ ΡΠ»ΠΎΠΌΠ°Ρ‚ΡŒ ΠΏΠΎΠΏΠΎΠ»Π°ΠΌ. Π”Π°, я ΠΎΠ±ΠΌΠ°Π½ΡƒΠ». На ΠΊΠ°Ρ€Ρ‚ΠΈΠ½ΠΊΠ΅ Π²Ρ‹ΡˆΠ΅ ΠΏΠΎΠΊΠ°Π·Π°Π½Ρ‹ Π΄Π²Π° стСрТнСвых ΠΌΠ°Π³Π½ΠΈΡ‚Π°. Но ΠΏΠΎΠ²Π΅Ρ€ΡŒΡ‚Π΅ ΠΌΠ½Π΅ — Ссли Ρ€Π°Π·ΠΎΠ±ΡŒΠ΅Ρ‚Π΅ ΠΌΠ°Π³Π½ΠΈΡ‚ Π½Π° Π΄Π²Π΅ части, это Π±ΡƒΠ΄Π΅Ρ‚ Π²Ρ‹Π³Π»ΡΠ΄Π΅Ρ‚ΡŒ Ρ‚Π°ΠΊ.

Rhett Allain

Π­Ρ‚ΠΎ Π΅Ρ‰Π΅ диполь. Π’Ρ‹ Π½Π΅ ΠΌΠΎΠΆΠ΅Ρ‚Π΅ Π·Π°ΡΡ‚Π°Π²ΠΈΡ‚ΡŒ ΠΌΠ°Π³Π½ΠΈΡ‚Π½ΠΎΠ΅ ΠΏΠΎΠ»Π΅ Π²Ρ‹Π³Π»ΡΠ΄Π΅Ρ‚ΡŒ ΠΊΠ°ΠΊ элСктричСскоС ΠΈΠ·-Π·Π° Ρ‚ΠΎΡ‡Π΅Ρ‡Π½ΠΎΠ³ΠΎ заряда, ΠΏΠΎΡ‚ΠΎΠΌΡƒ Ρ‡Ρ‚ΠΎ Π½Π΅Ρ‚ ΠΎΡ‚Π΄Π΅Π»ΡŒΠ½Ρ‹Ρ… ΠΌΠ°Π³Π½ΠΈΡ‚Π½Ρ‹Ρ… зарядов (Π½Π°Π·Ρ‹Π²Π°Π΅ΠΌΡ‹Ρ… ΠΌΠ°Π³Π½ΠΈΡ‚Π½Ρ‹ΠΌ ΠΌΠΎΠ½ΠΎΠΏΠΎΠ»Π΅ΠΌ).По сути, это Ρ‚ΠΎ, Ρ‡Ρ‚ΠΎ гласит Π·Π°ΠΊΠΎΠ½ Гаусса для ΠΌΠ°Π³Π½Π΅Ρ‚ΠΈΠ·ΠΌΠ° — Ρ‡Ρ‚ΠΎ Π½Π΅ сущСствуСт Ρ‚Π°ΠΊΠΎΠΉ Π²Π΅Ρ‰ΠΈ, ΠΊΠ°ΠΊ ΠΌΠ°Π³Π½ΠΈΡ‚Π½Ρ‹ΠΉ монополь. Π₯ΠΎΡ€ΠΎΡˆΠΎ, здСсь я Π΄ΠΎΠ»ΠΆΠ΅Π½ Π±Ρ‹Ρ‚ΡŒ ясным. ΠœΡ‹ Π½ΠΈΠΊΠΎΠ³Π΄Π° Π½Π΅ Π²ΠΈΠ΄Π΅Π»ΠΈ ΠΌΠ°Π³Π½ΠΈΡ‚Π½ΠΎΠ³ΠΎ монополя . Они ΠΌΠΎΠ³ΡƒΡ‚ ΡΡƒΡ‰Π΅ΡΡ‚Π²ΠΎΠ²Π°Ρ‚ΡŒ.

Π—Π°ΠΊΠΎΠ½ ЀарадСя

Π Π΅Ρ‚Ρ‚ АллСн

БупСркороткая вСрсия этого уравнСния состоит Π² Ρ‚ΠΎΠΌ, Ρ‡Ρ‚ΠΎ Π΅ΡΡ‚ΡŒ Π΅Ρ‰Π΅ ΠΎΠ΄ΠΈΠ½ способ ΡΠΎΠ·Π΄Π°Ρ‚ΡŒ элСктричСскоС ΠΏΠΎΠ»Π΅. ЭлСктричСскиС поля ΡΠΎΠ·Π΄Π°ΡŽΡ‚ Π½Π΅ Ρ‚ΠΎΠ»ΡŒΠΊΠΎ элСктричСскиС заряды. ЀактичСски, Π²Ρ‹ Ρ‚Π°ΠΊΠΆΠ΅ ΠΌΠΎΠΆΠ΅Ρ‚Π΅ ΡΠΎΠ·Π΄Π°Ρ‚ΡŒ элСктричСскоС ΠΏΠΎΠ»Π΅ с ΠΈΠ·ΠΌΠ΅Π½ΡΡŽΡ‰ΠΈΠΌΡΡ ΠΌΠ°Π³Π½ΠΈΡ‚Π½Ρ‹ΠΌ ΠΏΠΎΠ»Π΅ΠΌ. Π­Ρ‚ΠΎ ΠžΠ“Π ΠžΠœΠΠΠ― идСя, ΠΏΠΎΡΠΊΠΎΠ»ΡŒΠΊΡƒ ΠΎΠ½Π° устанавливаСт связь ΠΌΠ΅ΠΆΠ΄Ρƒ элСктричСским ΠΈ ΠΌΠ°Π³Π½ΠΈΡ‚Π½Ρ‹ΠΌ полями.

ΠŸΠΎΠ·Π²ΠΎΠ»ΡŒΡ‚Π΅ ΠΌΠ½Π΅ Π½Π°Ρ‡Π°Ρ‚ΡŒ с классичСской дСмонстрации. Π’ΠΎΡ‚ ΠΌΠ°Π³Π½ΠΈΡ‚, ΠΊΠ°Ρ‚ΡƒΡˆΠΊΠ° с ΠΏΡ€ΠΎΠ²ΠΎΠ»ΠΎΠΊΠΎΠΉ ΠΈ Π³Π°Π»ΡŒΠ²Π°Π½ΠΎΠΌΠ΅Ρ‚Ρ€ (ΠΎΠ½ Π² основном измСряСт ΠΊΡ€ΠΎΡˆΠ΅Ρ‡Π½Ρ‹Π΅ элСктричСскиС Ρ‚ΠΎΠΊΠΈ). Когда я ΠΏΠ΅Ρ€Π΅ΠΌΠ΅Ρ‰Π°ΡŽ ΠΌΠ°Π³Π½ΠΈΡ‚ Π²Π½ΡƒΡ‚Ρ€ΡŒ ΠΊΠ°Ρ‚ΡƒΡˆΠΊΠΈ ΠΈΠ»ΠΈ ΠΈΠ· Π½Π΅Π΅, я ΠΏΠΎΠ»ΡƒΡ‡Π°ΡŽ Ρ‚ΠΎΠΊ.

Π Π΅Ρ‚Ρ‚ АллСн
Rhett Allain

Если просто ΡƒΠ΄Π΅Ρ€ΠΆΠΈΠ²Π°Ρ‚ΡŒ ΠΌΠ°Π³Π½ΠΈΡ‚ Π² ΠΊΠ°Ρ‚ΡƒΡˆΠΊΠ΅, Ρ‚ΠΎΠΊ Π½Π΅ Π±ΡƒΠ΄Π΅Ρ‚. Π­Ρ‚ΠΎ Π΄ΠΎΠ»ΠΆΠ½ΠΎ Π±Ρ‹Ρ‚ΡŒ ΠΈΠ·ΠΌΠ΅Π½ΡΡŽΡ‰Π΅Π΅ΡΡ ΠΌΠ°Π³Π½ΠΈΡ‚Π½ΠΎΠ΅ ΠΏΠΎΠ»Π΅ . Ой, Π° Π³Π΄Π΅ ΠΆΠ΅ элСктричСскоС ΠΏΠΎΠ»Π΅? Π§Ρ‚ΠΎ ΠΆ, способ ΡΠΎΠ·Π΄Π°Ρ‚ΡŒ элСктричСский Ρ‚ΠΎΠΊ — это ΡΠΎΠ·Π΄Π°Ρ‚ΡŒ элСктричСскоС ΠΏΠΎΠ»Π΅ Π² Π½Π°ΠΏΡ€Π°Π²Π»Π΅Π½ΠΈΠΈ ΠΏΡ€ΠΎΠ²ΠΎΠ΄Π°.Π­Ρ‚ΠΎ элСктричСскоС ΠΏΠΎΠ»Π΅ Π²Π½ΡƒΡ‚Ρ€ΠΈ ΠΏΡ€ΠΎΠ²ΠΎΠ΄Π° Ρ‚ΠΎΠ»ΠΊΠ°Π΅Ρ‚ элСктричСскиС заряды, Ρ‡Ρ‚ΠΎΠ±Ρ‹ ΡΠΎΠ·Π΄Π°Ρ‚ΡŒ Ρ‚ΠΎΠΊ.

Но Π² этом элСктричСском ΠΏΠΎΠ»Π΅ Π΅ΡΡ‚ΡŒ Π½Π΅Ρ‡Ρ‚ΠΎ ΠΈΠ½ΠΎΠ΅. ВмСсто Ρ‚ΠΎΠ³ΠΎ, Ρ‡Ρ‚ΠΎΠ±Ρ‹ ΡƒΠΊΠ°Π·Ρ‹Π²Π°Ρ‚ΡŒ ΠΎΡ‚ ΠΏΠΎΠ»ΠΎΠΆΠΈΡ‚Π΅Π»ΡŒΠ½Ρ‹Ρ… зарядов ΠΈ ΡƒΠΊΠ°Π·Ρ‹Π²Π°Ρ‚ΡŒ Π½Π° ΠΎΡ‚Ρ€ΠΈΡ†Π°Ρ‚Π΅Π»ΡŒΠ½Ρ‹Π΅ заряды, рисунок поля просто ΠΎΠ±Ρ€Π°Π·ΡƒΠ΅Ρ‚ ΠΊΡ€ΡƒΠ³ΠΈ. Для Ρ‚Π°ΠΊΠΎΠ³ΠΎ случая я Π±ΡƒΠ΄Ρƒ ΠΈΡΠΏΠΎΠ»ΡŒΠ·ΠΎΠ²Π°Ρ‚ΡŒ Π½Π°Π·Π²Π°Π½ΠΈΠ΅ Β«Ρ„ΠΈΠ³ΡƒΡ€Π½ΠΎΠ΅ элСктричСскоС ΠΏΠΎΠ»Π΅Β» (я позаимствовал Ρ‚Π΅Ρ€ΠΌΠΈΠ½ ΠΎΡ‚ ΠΌΠΎΠΈΡ… Π»ΡŽΠ±ΠΈΠΌΡ‹Ρ… Π°Π²Ρ‚ΠΎΡ€ΠΎΠ² ΡƒΡ‡Π΅Π±Π½ΠΈΠΊΠΎΠ² Ρ„ΠΈΠ·ΠΈΠΊΠΈ). ΠŸΡ€ΠΈ этом ΠΌΡ‹ ΠΌΠΎΠΆΠ΅ΠΌ Π½Π°Π·Π²Π°Ρ‚ΡŒ элСктричСскоС ΠΏΠΎΠ»Π΅, созданноС ΠΈΠ· зарядов, «кулоновским ΠΏΠΎΠ»Π΅ΠΌΒ» (ΠΈΠ·-Π·Π° Π·Π°ΠΊΠΎΠ½Π° ΠšΡƒΠ»ΠΎΠ½Π°).

Π—Π°ΠΊΠΎΠ½Ρ‹ Гаусса, ЀарадСя ΠΈ АмпСра с ΠΏΡ€ΠΎΠΈΠ·Π²ΠΎΠ΄Π½Ρ‹ΠΌΠΈ

УравнСния МаксвСлла Π±Ρ‹Π»ΠΈ ΠΎΠΏΡƒΠ±Π»ΠΈΠΊΠΎΠ²Π°Π½Ρ‹ ΡƒΡ‡Π΅Π½Ρ‹ΠΌ Β« ДТСймс ΠšΠ»Π΅Ρ€ΠΊ МаксвСлл Β» Π² 1860 Π³ΠΎΠ΄Ρƒ. Π­Ρ‚ΠΈ уравнСния ΠΏΠΎΠΊΠ°Π·Ρ‹Π²Π°ΡŽΡ‚, ΠΊΠ°ΠΊ заряТСнныС Π°Ρ‚ΠΎΠΌΡ‹ ΠΈΠ»ΠΈ элСмСнты ΠΎΠ±Π΅ΡΠΏΠ΅Ρ‡ΠΈΠ²Π°ΡŽΡ‚ ΡΠ»Π΅ΠΊΡ‚Ρ€ΠΈΡ‡Π΅ΡΠΊΡƒΡŽ силу, Π° Ρ‚Π°ΠΊΠΆΠ΅ силу магнитная сила для ΠΊΠ°ΠΆΠ΄ΠΎΠΉ Π΅Π΄ΠΈΠ½ΠΈΡ†Ρ‹ заряда. ЭнСргия для ΠΊΠ°ΠΆΠ΄ΠΎΠ³ΠΎ Π΅Π΄ΠΈΠ½ΠΈΡ‡Π½ΠΎΠ³ΠΎ заряда называСтся ΠΏΠΎΠ»Π΅ΠΌ. Π­Π»Π΅ΠΌΠ΅Π½Ρ‚Ρ‹ ΠΌΠΎΠ³ΡƒΡ‚ Π±Ρ‹Ρ‚ΡŒ Π½Π΅ΠΏΠΎΠ΄Π²ΠΈΠΆΠ½Ρ‹ΠΌΠΈ, Π² ΠΏΡ€ΠΎΡ‚ΠΈΠ²Π½ΠΎΠΌ случаС — Π΄Π²ΠΈΠ³Π°Ρ‚ΡŒΡΡ. УравнСния МаксвСлла ΠΎΠ±ΡŠΡΡΠ½ΡΡŽΡ‚, ΠΊΠ°ΠΊ ΠΌΠ°Π³Π½ΠΈΡ‚Π½Ρ‹Π΅ поля ΠΌΠΎΠ³ΡƒΡ‚ ΠΎΠ±Ρ€Π°Π·ΠΎΠ²Ρ‹Π²Π°Ρ‚ΡŒΡΡ элСктричСскими Ρ‚ΠΎΠΊΠ°ΠΌΠΈ, Π° Ρ‚Π°ΠΊΠΆΠ΅ зарядами, ΠΈ, Π½Π°ΠΊΠΎΠ½Π΅Ρ†, ΠΎΠ½ΠΈ ΠΎΠ±ΡŠΡΡΠ½ΡΡŽΡ‚, ΠΊΠ°ΠΊ элСктричСскоС ΠΏΠΎΠ»Π΅ ΠΌΠΎΠΆΠ΅Ρ‚ ΡΠΎΠ·Π΄Π°Π²Π°Ρ‚ΡŒ ΠΌΠ°Π³Π½ΠΈΡ‚Π½ΠΎΠ΅ ΠΏΠΎΠ»Π΅ ΠΈ Ρ‚. Π”.ΠŸΠ΅Ρ€Π²ΠΈΡ‡Π½ΠΎΠ΅ ΡƒΡ€Π°Π²Π½Π΅Π½ΠΈΠ΅ позволяСт ΠΎΠΏΡ€Π΅Π΄Π΅Π»ΠΈΡ‚ΡŒ элСктричСскоС ΠΏΠΎΠ»Π΅, ΠΎΠ±Ρ€Π°Π·ΠΎΠ²Π°Π½Π½ΠΎΠ΅ зарядом. Π‘Π»Π΅Π΄ΡƒΡŽΡ‰Π΅Π΅ ΡƒΡ€Π°Π²Π½Π΅Π½ΠΈΠ΅ позволяСт Π²Π°ΠΌ ΠΎΠΏΡ€Π΅Π΄Π΅Π»ΠΈΡ‚ΡŒ ΠΌΠ°Π³Π½ΠΈΡ‚Π½ΠΎΠ΅ ΠΏΠΎΠ»Π΅, Π° ΠΎΡΡ‚Π°Π²ΡˆΠΈΠ΅ΡΡ Π΄Π²Π° ΠΎΠ±ΡŠΡΡΠ½ΡΡ‚, ΠΊΠ°ΠΊ поля ΠΏΡ€ΠΎΡ‚Π΅ΠΊΠ°ΡŽΡ‚ Π²ΠΎΠΊΡ€ΡƒΠ³ источников питания. Π’ этой ΡΡ‚Π°Ρ‚ΡŒΠ΅ обсуТдаСтся тСория МаксвСлла ΠΈΠ»ΠΈ Π·Π°ΠΊΠΎΠ½ МаксвСлла . Π’ этой ΡΡ‚Π°Ρ‚ΡŒΠ΅ обсуТдаСтся ΠΎΠ±Π·ΠΎΡ€ элСктромагнитной Ρ‚Π΅ΠΎΡ€ΠΈΠΈ МаксвСлла .


Π§Ρ‚ΠΎ Ρ‚Π°ΠΊΠΎΠ΅ уравнСния МаксвСлла?

Π’Ρ‹Π²ΠΎΠ΄ уравнСния МаксвСлла состоит ΠΈΠ· Ρ‡Π΅Ρ‚Ρ‹Ρ€Π΅Ρ… ΡƒΡ€Π°Π²Π½Π΅Π½ΠΈΠΉ, ΠΊΠ°ΠΆΠ΄ΠΎΠ΅ ΠΈΠ· ΠΊΠΎΡ‚ΠΎΡ€Ρ‹Ρ… соотвСтствСнно ΠΎΠ±ΡŠΡΡΠ½ΡΠ΅Ρ‚ ΠΎΠ΄ΠΈΠ½ Ρ„Π°ΠΊΡ‚.ВсС эти уравнСния Π½Π΅ ΠΈΠ·ΠΎΠ±Ρ€Π΅Ρ‚Π΅Π½Ρ‹ МаксвСллом; ΠΎΠ΄Π½Π°ΠΊΠΎ ΠΎΠ½ объСдинил Ρ‡Π΅Ρ‚Ρ‹Ρ€Π΅ уравнСния, составлСнныС Π€Π°Ρ€Π°Π΄Π΅Π΅ΠΌ, Гауссом ΠΈ АмпСром. Π₯отя МаксвСлл Π²ΠΊΠ»ΡŽΡ‡ΠΈΠ» ΠΎΠ΄Π½Ρƒ Ρ‡Π°ΡΡ‚ΡŒ ΠΈΠ½Ρ„ΠΎΡ€ΠΌΠ°Ρ†ΠΈΠΈ Π² Ρ‡Π΅Ρ‚Π²Π΅Ρ€Ρ‚ΠΎΠ΅ ΡƒΡ€Π°Π²Π½Π΅Π½ΠΈΠ΅, Π° ΠΈΠΌΠ΅Π½Π½ΠΎ Π·Π°ΠΊΠΎΠ½ АмпСра, это Π΄Π΅Π»Π°Π΅Ρ‚ ΡƒΡ€Π°Π²Π½Π΅Π½ΠΈΠ΅ Π·Π°ΠΊΠΎΠ½Ρ‡Π΅Π½Π½Ρ‹ΠΌ.

УравнСния МаксвСлла
  • ΠŸΠ΅Ρ€Π²Ρ‹ΠΉ Π·Π°ΠΊΠΎΠ½ Π—Π°ΠΊΠΎΠ½ Гаусса , ΠΏΡ€Π΅Π΄Π½Π°Π·Π½Π°Ρ‡Π΅Π½Π½Ρ‹ΠΉ для статичСских элСктричСских ΠΏΠΎΠ»Π΅ΠΉ
  • Π’Ρ‚ΠΎΡ€ΠΎΠΉ Π·Π°ΠΊΠΎΠ½ Ρ‚Π°ΠΊΠΆΠ΅ Π—Π°ΠΊΠΎΠ½ Гаусса , ΠΏΡ€Π΅Π΄Π½Π°Π·Π½Π°Ρ‡Π΅Π½Π½Ρ‹ΠΉ для статичСских ΠΌΠ°Π³Π½ΠΈΡ‚Π½Ρ‹Ρ… ΠΏΠΎΠ»Π΅ΠΉ
  • Π’Ρ€Π΅Ρ‚ΠΈΠΉ Π·Π°ΠΊΠΎΠ½ Π·Π°ΠΊΠΎΠ½ ЀарадСя , ΠΊΠΎΡ‚ΠΎΡ€Ρ‹ΠΉ Π³ΠΎΠ²ΠΎΡ€ΠΈΡ‚ ΠΎΠ± ΠΈΠ·ΠΌΠ΅Π½Π΅Π½ΠΈΠΈ ΠΌΠ°Π³Π½ΠΈΡ‚Π½ΠΎΠ³ΠΎ поля. field создаст элСктричСскоС ΠΏΠΎΠ»Π΅.
  • Π§Π΅Ρ‚Π²Π΅Ρ€Ρ‚Ρ‹ΠΉ Π·Π°ΠΊΠΎΠ½ Π·Π°ΠΊΠΎΠ½ МаксвСлла АмпСра , ΠΊΠΎΡ‚ΠΎΡ€Ρ‹ΠΉ гласит, Ρ‡Ρ‚ΠΎ ΠΈΠ·ΠΌΠ΅Π½Π΅Π½ΠΈΠ΅ элСктричСского поля ΠΏΡ€ΠΈΠ²Π΅Π΄Π΅Ρ‚ ΠΊ возникновСнию ΠΌΠ°Π³Π½ΠΈΡ‚Π½ΠΎΠ³ΠΎ поля.

Π”Π²Π° уравнСния 3 ΠΈ 4 ΠΌΠΎΠ³ΡƒΡ‚ ΠΎΠΏΠΈΡΡ‹Π²Π°Ρ‚ΡŒ ΡΠ»Π΅ΠΊΡ‚Ρ€ΠΎΠΌΠ°Π³Π½ΠΈΡ‚Π½ΡƒΡŽ Π²ΠΎΠ»Π½Ρƒ , которая ΠΌΠΎΠΆΠ΅Ρ‚ Ρ€Π°ΡΠΏΡ€ΠΎΡΡ‚Ρ€Π°Π½ΡΡ‚ΡŒΡΡ сама ΠΏΠΎ сСбС. Π“Ρ€ΡƒΠΏΠΏΠΈΡ€ΠΎΠ²ΠΊΠ° этих ΡƒΡ€Π°Π²Π½Π΅Π½ΠΈΠΉ Π³ΠΎΠ²ΠΎΡ€ΠΈΡ‚ ΠΎ Ρ‚ΠΎΠΌ, Ρ‡Ρ‚ΠΎ ΠΈΠ·ΠΌΠ΅Π½Π΅Π½ΠΈΠ΅ ΠΌΠ°Π³Π½ΠΈΡ‚Π½ΠΎΠ³ΠΎ поля ΠΌΠΎΠΆΠ΅Ρ‚ Π²Ρ‹Π·Π²Π°Ρ‚ΡŒ ΠΈΠ·ΠΌΠ΅Π½Π΅Π½ΠΈΠ΅ элСктричСского поля, Π° Π·Π°Ρ‚Π΅ΠΌ это ΠΏΡ€ΠΈΠ²Π΅Π΄Π΅Ρ‚ ΠΊ Π΄ΠΎΠΏΠΎΠ»Π½ΠΈΡ‚Π΅Π»ΡŒΠ½ΠΎΠΌΡƒ измСнСнию ΠΌΠ°Π³Π½ΠΈΡ‚Π½ΠΎΠ³ΠΎ поля. ΠŸΠΎΡΡ‚ΠΎΠΌΡƒ эта сСрия продолТаСтся, Ρ‚Π°ΠΊ ΠΊΠ°ΠΊ элСктромагнитный сигнал Π³ΠΎΡ‚ΠΎΠ² ΠΈ распространяСтся ΠΏΠΎ всСму пространству.


Π§Π΅Ρ‚Ρ‹Ρ€Π΅ уравнСния МаксвСлла

Π§Π΅Ρ‚Ρ‹Ρ€Π΅ уравнСния МаксвСлла ΠΎΠ±ΡŠΡΡΠ½ΡΡŽΡ‚ Π΄Π²Π° поля, Π²ΠΎΠ·Π½ΠΈΠΊΠ°ΡŽΡ‰ΠΈΠ΅ ΠΎΡ‚ источников элСктричСского Ρ‚ΠΎΠΊΠ° ΠΈ Ρ‚ΠΎΠΊΠ°. Поля Π±Ρ‹Π²Π°ΡŽΡ‚ ΠΊΠ°ΠΊ элСктричСскими, Ρ‚Π°ΠΊ ΠΈ ΠΌΠ°Π³Π½ΠΈΡ‚Π½Ρ‹ΠΌΠΈ, ΠΈ ΠΊΠ°ΠΊ ΠΎΠ½ΠΈ ΠΌΠ΅Π½ΡΡŽΡ‚ΡΡ Π²ΠΎ Π²Ρ€Π΅ΠΌΠ΅Π½ΠΈ. Π§Π΅Ρ‚Ρ‹Ρ€Π΅ уравнСния МаксвСлла Π²ΠΊΠ»ΡŽΡ‡Π°ΡŽΡ‚ ΡΠ»Π΅Π΄ΡƒΡŽΡ‰Π΅Π΅.

  • ΠŸΠ΅Ρ€Π²Ρ‹ΠΉ Π·Π°ΠΊΠΎΠ½: Π·Π°ΠΊΠΎΠ½ Гаусса для элСктричСства
  • Π’Ρ‚ΠΎΡ€ΠΎΠΉ Π·Π°ΠΊΠΎΠ½: Π·Π°ΠΊΠΎΠ½ Гаусса для ΠΌΠ°Π³Π½Π΅Ρ‚ΠΈΠ·ΠΌΠ°
  • Π’Ρ€Π΅Ρ‚ΠΈΠΉ Π·Π°ΠΊΠΎΠ½: Π·Π°ΠΊΠΎΠ½ ΠΈΠ½Π΄ΡƒΠΊΡ†ΠΈΠΈ ЀарадСя
  • Π§Π΅Ρ‚Π²Π΅Ρ€Ρ‚Ρ‹ΠΉ Π·Π°ΠΊΠΎΠ½: Π·Π°ΠΊΠΎΠ½ АмпСра

Π’Ρ‹ΡˆΠ΅ΡƒΠΏΠΎΠΌΡΠ½ΡƒΡ‚Ρ‹Π΅ Ρ‡Π΅Ρ‚Ρ‹Ρ€Π΅ уравнСния МаксвСлла ΡΠ²Π»ΡΡŽΡ‚ΡΡ уравнСниями Гаусса для элСктричСства, Гаусс для ΠΌΠ°Π³Π½Π΅Ρ‚ΠΈΠ·ΠΌΠ°, Π·Π°ΠΊΠΎΠ½ ЀарадСя для ΠΈΠ½Π΄ΡƒΠΊΡ†ΠΈΠΈ. Π—Π°ΠΊΠΎΠ½ АмпСра записываСтся ΠΏΠΎ-Ρ€Π°Π·Π½ΠΎΠΌΡƒ, Π½Π°ΠΏΡ€ΠΈΠΌΠ΅Ρ€, уравнСния МаксвСлла Π² ΠΈΠ½Ρ‚Π΅Π³Ρ€Π°Π»ΡŒΠ½ΠΎΠΉ Ρ„ΠΎΡ€ΠΌΠ΅ ΠΈ уравнСния МаксвСлла Π² Π΄ΠΈΡ„Ρ„Π΅Ρ€Π΅Π½Ρ†ΠΈΠ°Π»ΡŒΠ½ΠΎΠΉ Ρ„ΠΎΡ€ΠΌΠ΅ , ΠΊΠΎΡ‚ΠΎΡ€Ρ‹Π΅ ΠΎΠ±ΡΡƒΠΆΠ΄Π°ΡŽΡ‚ΡΡ Π½ΠΈΠΆΠ΅.

Π‘ΠΈΠΌΠ²ΠΎΠ»Ρ‹ уравнСния МаксвСлла

Π‘ΠΈΠΌΠ²ΠΎΠ»Ρ‹, ΠΈΡΠΏΠΎΠ»ΡŒΠ·ΡƒΠ΅ΠΌΡ‹Π΅ Π² ΡƒΡ€Π°Π²Π½Π΅Π½ΠΈΠΈ МаксвСлла, Π²ΠΊΠ»ΡŽΡ‡Π°ΡŽΡ‚ ΡΠ»Π΅Π΄ΡƒΡŽΡ‰ΠΈΠ΅:

  • E ΠΎΠ±ΠΎΠ·Π½Π°Ρ‡Π°Π΅Ρ‚ элСктричСскоС ΠΏΠΎΠ»Π΅
  • M ΠΎΠ±ΠΎΠ·Π½Π°Ρ‡Π°Π΅Ρ‚ ΠΌΠ°Π³Π½ΠΈΡ‚Π½ΠΎΠ΅ ΠΏΠΎΠ»Π΅
  • D ΠΎΠ±ΠΎΠ·Π½Π°Ρ‡Π°Π΅Ρ‚ элСктричСскоС смСщСниС
  • H ΠΎΠ±ΠΎΠ·Π½Π°Ρ‡Π°Π΅Ρ‚ Π½Π°ΠΏΡ€ΡΠΆΠ΅Π½Π½ΠΎΡΡ‚ΡŒ ΠΌΠ°Π³Π½ΠΈΡ‚Π½ΠΎΠ³ΠΎ поля
  • Ξ‘ ΠΎΠ±ΠΎΠ·Π½Π°Ρ‡Π°Π΅Ρ‚ ΠΏΠ»ΠΎΡ‚Π½ΠΎΡΡ‚ΡŒ заряда
  • i ΠΎΠ±ΠΎΠ·Π½Π°Ρ‡Π°Π΅Ρ‚ элСктричСский Ρ‚ΠΎΠΊ
  • Ξ΅0 ΠΎΠ±ΠΎΠ·Π½Π°Ρ‡Π°Π΅Ρ‚ Π΄ΠΈΡΠ»Π΅ΠΊΡ‚Ρ€ΠΈΡ‡Π΅ΡΠΊΡƒΡŽ ΠΏΡ€ΠΎΠ½ΠΈΡ†Π°Π΅ΠΌΠΎΡΡ‚ΡŒ
  • J ΠΎΠ±ΠΎΠ·Π½Π°Ρ‡Π°Π΅Ρ‚ ΠΏΠ»ΠΎΡ‚Π½ΠΎΡΡ‚ΡŒ Ρ‚ΠΎΠΊΠ°
  • ΞΌ0 ΠΎΠ±ΠΎΠ·Π½Π°Ρ‡Π°Π΅Ρ‚ ΠΏΡ€ΠΎΠ½ΠΈΡ†Π°Π΅ΠΌΠΎΡΡ‚ΡŒ
  • c ΠΎΠ±ΠΎΠ·Π½Π°Ρ‡Π°Π΅Ρ‚ ΡΠΊΠΎΡ€ΠΎΡΡ‚ΡŒ свСта
  • M ΠΎΠ±ΠΎΠ·Π½Π°Ρ‡Π°Π΅Ρ‚ Π½Π°ΠΌΠ°Π³Π½ΠΈΡ‡Π΅Π½Π½ΠΎΡΡ‚ΡŒ
  • P ΠΎΠ±ΠΎΠ·Π½Π°Ρ‡Π°Π΅Ρ‚ ΠΏΠΎΠ»ΡΡ€ΠΈΠ·Π°Ρ†ΠΈΡŽ

ΠŸΠ΅Ρ€Π²Ρ‹ΠΉ Π·Π°ΠΊΠΎΠ½: Π·Π°ΠΊΠΎΠ½ Гаусса для элСктричСства

ΠŸΠ΅Ρ€Π²Ρ‹ΠΉ Π·Π°ΠΊΠΎΠ½ МаксвСлла — это Π·Π°ΠΊΠΎΠ½ Гаусса , ΠΊΠΎΡ‚ΠΎΡ€Ρ‹ΠΉ ΠΈΡΠΏΠΎΠ»ΡŒΠ·ΡƒΠ΅Ρ‚ΡΡ для элСктричСства.Π—Π°ΠΊΠΎΠ½ Гаусса опрСдСляСт, Ρ‡Ρ‚ΠΎ элСктричСский ΠΏΠΎΡ‚ΠΎΠΊ ΠΎΡ‚ любой Π·Π°ΠΌΠΊΠ½ΡƒΡ‚ΠΎΠΉ повСрхности Π±ΡƒΠ΄Π΅Ρ‚ ΠΏΡ€ΠΎΠΏΠΎΡ€Ρ†ΠΈΠΎΠ½Π°Π»Π΅Π½ всСму заряду, Π·Π°ΠΊΠ»ΡŽΡ‡Π΅Π½Π½ΠΎΠΌΡƒ Π½Π° повСрхности.

Π˜Π½Ρ‚Π΅Π³Ρ€Π°Π»ΡŒΠ½Π°Ρ Ρ„ΠΎΡ€ΠΌΠ° Π·Π°ΠΊΠΎΠ½Π° Гаусса Π½Π°Ρ…ΠΎΠ΄ΠΈΡ‚ ΠΏΡ€ΠΈΠΌΠ΅Π½Π΅Π½ΠΈΠ΅ ΠΏΡ€ΠΈ расчСтС элСктричСских ΠΏΠΎΠ»Π΅ΠΉ Π² области заряТСнных ΠΎΠ±ΡŠΠ΅ΠΊΡ‚ΠΎΠ². ΠŸΡ€ΠΈΠΌΠ΅Π½ΡΡ этот Π·Π°ΠΊΠΎΠ½ ΠΊ Ρ‚ΠΎΡ‡Π΅Ρ‡Π½ΠΎΠΌΡƒ заряду Π² элСктричСском ΠΏΠΎΠ»Π΅, ΠΌΠΎΠΆΠ½ΠΎ ΠΏΠΎΠΊΠ°Π·Π°Ρ‚ΡŒ, Ρ‡Ρ‚ΠΎ ΠΎΠ½ зависит ΠΎΡ‚ Π·Π°ΠΊΠΎΠ½Π° ΠšΡƒΠ»ΠΎΠ½Π°.

Π₯отя пСрвичная ΠΎΠ±Π»Π°ΡΡ‚ΡŒ элСктричСского поля обСспСчиваСт ΠΌΠ΅Ρ€Ρƒ Π²ΠΊΠ»ΡŽΡ‡Π΅Π½Π½ΠΎΠ³ΠΎ чистого заряда, ΠΎΡ‚ΠΊΠ»ΠΎΠ½Π΅Π½ΠΈΠ΅ элСктричСского поля Π΄Π°Π΅Ρ‚ ΠΌΠ΅Ρ€Ρƒ компактности источников, Π° Ρ‚Π°ΠΊΠΆΠ΅ Π²ΠΊΠ»ΡŽΡ‡Π°Π΅Ρ‚ Π·Π½Π°Ρ‡Π΅Π½ΠΈΠ΅, ΠΈΡΠΏΠΎΠ»ΡŒΠ·ΡƒΠ΅ΠΌΠΎΠ΅ для Π·Π°Ρ‰ΠΈΡ‚Ρ‹ заряда.

Π’Ρ‚ΠΎΡ€ΠΎΠΉ Π·Π°ΠΊΠΎΠ½: Π·Π°ΠΊΠΎΠ½ Гаусса для ΠΌΠ°Π³Π½Π΅Ρ‚ΠΈΠ·ΠΌΠ°

Π—Π°ΠΊΠΎΠ½ МаксвСлла сСкунды — это Π·Π°ΠΊΠΎΠ½ Гаусса , ΠΊΠΎΡ‚ΠΎΡ€Ρ‹ΠΉ ΠΈΡΠΏΠΎΠ»ΡŒΠ·ΡƒΠ΅Ρ‚ΡΡ для ΠΌΠ°Π³Π½Π΅Ρ‚ΠΈΠ·ΠΌΠ°. Π—Π°ΠΊΠΎΠ½ Гаусса гласит, Ρ‡Ρ‚ΠΎ ΠΎΡ‚ΠΊΠ»ΠΎΠ½Π΅Π½ΠΈΠ΅ ΠΌΠ°Π³Π½ΠΈΡ‚Π½ΠΎΠ³ΠΎ поля Ρ€Π°Π²Π½ΠΎ Π½ΡƒΠ»ΡŽ. Π­Ρ‚ΠΎΡ‚ Π·Π°ΠΊΠΎΠ½ ΠΏΡ€ΠΈΠΌΠ΅Π½ΠΈΠΌ ΠΊ ΠΌΠ°Π³Π½ΠΈΡ‚Π½ΠΎΠΌΡƒ ΠΏΠΎΡ‚ΠΎΠΊΡƒ Ρ‡Π΅Ρ€Π΅Π· Π·Π°ΠΌΠΊΠ½ΡƒΡ‚ΡƒΡŽ ΠΏΠΎΠ²Π΅Ρ€Ρ…Π½ΠΎΡΡ‚ΡŒ. Π’ этом случаС Π²Π΅ΠΊΡ‚ΠΎΡ€ ΠΏΠ»ΠΎΡ‰Π°Π΄ΠΈ Π½Π°ΠΏΡ€Π°Π²Π»Π΅Π½ ΠΎΡ‚ повСрхности.

ΠœΠ°Π³Π½ΠΈΡ‚Π½ΠΎΠ΅ ΠΏΠΎΠ»Π΅ ΠΈΠ·-Π·Π° ΠΌΠ°Ρ‚Π΅Ρ€ΠΈΠ°Π»ΠΎΠ² Π±ΡƒΠ΄Π΅Ρ‚ Π³Π΅Π½Π΅Ρ€ΠΈΡ€ΠΎΠ²Π°Ρ‚ΡŒΡΡ Ρ‡Π΅Ρ€Π΅Π· ΡƒΠ·ΠΎΡ€, Π½Π°Π·Π²Π°Π½Π½Ρ‹ΠΉ Π΄ΠΈΠΏΠΎΠ»Π΅ΠΌ. Π­Ρ‚ΠΈ полюса Π»ΡƒΡ‡ΡˆΠ΅ всСго ΠΎΠ±ΠΎΠ·Π½Π°Ρ‡Π°ΡŽΡ‚ΡΡ пСтлями Ρ‚ΠΎΠΊΠ°, ΠΎΠ΄Π½Π°ΠΊΠΎ ΠΎΠ½ΠΈ ΠΏΠΎΡ…ΠΎΠΆΠΈ Π½Π° ΠΏΠΎΠ»ΠΎΠΆΠΈΡ‚Π΅Π»ΡŒΠ½Ρ‹Π΅ ΠΈ ΠΎΡ‚Ρ€ΠΈΡ†Π°Ρ‚Π΅Π»ΡŒΠ½Ρ‹Π΅ ΠΌΠ°Π³Π½ΠΈΡ‚Π½Ρ‹Π΅ заряды, ΠΊΠΎΡ‚ΠΎΡ€Ρ‹Π΅ Π½Π΅Π²ΠΈΠ΄ΠΈΠΌΠΎ ΠΎΡ‚Ρ€Π°ΠΆΠ°ΡŽΡ‚ΡΡ Π΄Ρ€ΡƒΠ³ ΠΎΡ‚ Π΄Ρ€ΡƒΠ³Π°.Π’ условиях силовых Π»ΠΈΠ½ΠΈΠΉ этот Π·Π°ΠΊΠΎΠ½ гласит, Ρ‡Ρ‚ΠΎ силовыС Π»ΠΈΠ½ΠΈΠΈ ΠΌΠ°Π³Π½ΠΈΡ‚Π½ΠΎΠ³ΠΎ поля Π½Π΅ Π½Π°Ρ‡ΠΈΠ½Π°ΡŽΡ‚ΡΡ ΠΈ Π½Π΅ Π·Π°ΠΊΠ°Π½Ρ‡ΠΈΠ²Π°ΡŽΡ‚ΡΡ, Π° ΡΠΎΠ·Π΄Π°ΡŽΡ‚ ΠΏΠ΅Ρ‚Π»ΠΈ, Π² ΠΏΡ€ΠΎΡ‚ΠΈΠ²Π½ΠΎΠΌ случаС ΠΎΠ½ΠΈ Ρ€Π°ΡΡˆΠΈΡ€ΡΡŽΡ‚ΡΡ Π΄ΠΎ бСсконСчности ΠΈ Ρ€Π΅Π²Π΅Ρ€ΡΠΈΡ€ΡƒΡŽΡ‚ΡΡ. Π”Ρ€ΡƒΠ³ΠΈΠΌΠΈ словами, любая линия ΠΌΠ°Π³Π½ΠΈΡ‚Π½ΠΎΠ³ΠΎ поля, которая ΠΏΡ€ΠΎΡ…ΠΎΠ΄ΠΈΡ‚ Ρ‡Π΅Ρ€Π΅Π· Π΄Π°Π½Π½Ρ‹ΠΉ ΡƒΡ€ΠΎΠ²Π΅Π½ΡŒ, Π΄ΠΎΠ»ΠΆΠ½Π° Π³Π΄Π΅-Ρ‚ΠΎ Π²Ρ‹Ρ…ΠΎΠ΄ΠΈΡ‚ΡŒ ΠΈΠ· этого объСма.

Π­Ρ‚ΠΎΡ‚ Π·Π°ΠΊΠΎΠ½ ΠΌΠΎΠΆΠ΅Ρ‚ Π±Ρ‹Ρ‚ΡŒ записан Π² Π΄Π²ΡƒΡ… Ρ„ΠΎΡ€ΠΌΠ°Ρ…: ΠΈΠ½Ρ‚Π΅Π³Ρ€Π°Π»ΡŒΠ½ΠΎΠΉ ΠΈ Π΄ΠΈΡ„Ρ„Π΅Ρ€Π΅Π½Ρ†ΠΈΠ°Π»ΡŒΠ½ΠΎΠΉ. Π­Ρ‚ΠΈ Π΄Π²Π΅ Ρ„ΠΎΡ€ΠΌΡ‹ Ρ€Π°Π²Π½Ρ‹ ΠΏΠΎ Ρ‚Π΅ΠΎΡ€Π΅ΠΌΠ΅ ΠΎ расходимости.

Π’Ρ€Π΅Ρ‚ΠΈΠΉ Π·Π°ΠΊΠΎΠ½: Π—Π°ΠΊΠΎΠ½ ΠΈΠ½Π΄ΡƒΠΊΡ†ΠΈΠΈ ЀарадСя

Π’Ρ€Π΅Ρ‚ΠΈΠΉ Π·Π°ΠΊΠΎΠ½ МаксвСлла — это Π·Π°ΠΊΠΎΠ½ ЀарадСя , ΠΊΠΎΡ‚ΠΎΡ€Ρ‹ΠΉ ΠΈΡΠΏΠΎΠ»ΡŒΠ·ΡƒΠ΅Ρ‚ΡΡ для ΠΈΠ½Π΄ΡƒΠΊΡ†ΠΈΠΈ.Π—Π°ΠΊΠΎΠ½ ЀарадСя гласит, ΠΊΠ°ΠΊ ΠΈΠ·ΠΌΠ΅Π½ΡΡŽΡ‰Π΅Π΅ΡΡ Π²ΠΎ Π²Ρ€Π΅ΠΌΠ΅Π½ΠΈ ΠΌΠ°Π³Π½ΠΈΡ‚Π½ΠΎΠ΅ ΠΏΠΎΠ»Π΅ создаСт элСктричСскоС ΠΏΠΎΠ»Π΅. Π’ ΠΈΠ½Ρ‚Π΅Π³Ρ€Π°Π»ΡŒΠ½ΠΎΠΉ Ρ„ΠΎΡ€ΠΌΠ΅ ΠΎΠ½ опрСдСляСт, Ρ‡Ρ‚ΠΎ усилиС для ΠΊΠ°ΠΆΠ΄ΠΎΠ³ΠΎ Π΅Π΄ΠΈΠ½ΠΈΡ‡Π½ΠΎΠ³ΠΎ заряда Π½Π΅ΠΎΠ±Ρ…ΠΎΠ΄ΠΈΠΌΠΎ для пСрСмСщСния заряда Π² области Π·Π°ΠΌΠΊΠ½ΡƒΡ‚ΠΎΠ³ΠΎ ΠΊΠΎΠ½Ρ‚ΡƒΡ€Π°, Ρ‡Ρ‚ΠΎ равняСтся скорости ΡƒΠΌΠ΅Π½ΡŒΡˆΠ΅Π½ΠΈΡ ΠΌΠ°Π³Π½ΠΈΡ‚Π½ΠΎΠ³ΠΎ ΠΏΠΎΡ‚ΠΎΠΊΠ° Π½Π° Π·Π°ΠΌΠΊΠ½ΡƒΡ‚ΠΎΠΉ повСрхности.

Подобно ΠΌΠ°Π³Π½ΠΈΡ‚Π½ΠΎΠΌΡƒ полю, элСктричСски ΠΈΠ½Π΄ΡƒΡ†ΠΈΡ€ΠΎΠ²Π°Π½Π½ΠΎΠ΅ элСктричСскоС ΠΏΠΎΠ»Π΅ Π²ΠΊΠ»ΡŽΡ‡Π°Π΅Ρ‚ Π·Π°ΠΌΠΊΠ½ΡƒΡ‚Ρ‹Π΅ силовыС Π»ΠΈΠ½ΠΈΠΈ, Ссли Π½Π° Π½ΠΈΡ… Π½Π΅ дСйствуСт статичСскоС элСктричСскоС ΠΏΠΎΠ»Π΅. Π­Ρ‚Π° функция элСктромагнитной ΠΈΠ½Π΄ΡƒΠΊΡ†ΠΈΠΈ являСтся ΠΏΡ€ΠΈΠ½Ρ†ΠΈΠΏΠΎΠΌ Ρ€Π°Π±ΠΎΡ‚Ρ‹ Π½Π΅ΡΠΊΠΎΠ»ΡŒΠΊΠΈΡ… элСктричСских Π³Π΅Π½Π΅Ρ€Π°Ρ‚ΠΎΡ€ΠΎΠ² : Π½Π°ΠΏΡ€ΠΈΠΌΠ΅Ρ€, ΠΌΠ°Π³Π½ΠΈΡ‚ с Π²Ρ€Π°Ρ‰Π°ΡŽΡ‰ΠΈΠΌΡΡ стСрТнСм создаСт ΠΈΠ·ΠΌΠ΅Π½Π΅Π½ΠΈΠ΅ ΠΌΠ°Π³Π½ΠΈΡ‚Π½ΠΎΠ³ΠΎ поля, ΠΊΠΎΡ‚ΠΎΡ€ΠΎΠ΅, Π² свою ΠΎΡ‡Π΅Ρ€Π΅Π΄ΡŒ, создаСт элСктричСскоС ΠΏΠΎΠ»Π΅ Π² блиТайшСм ΠΏΡ€ΠΎΠ²ΠΎΠ΄Π΅.

Π§Π΅Ρ‚Π²Π΅Ρ€Ρ‚Ρ‹ΠΉ Π·Π°ΠΊΠΎΠ½: Π—Π°ΠΊΠΎΠ½ АмпСра

Π§Π΅Ρ‚Π²Π΅Ρ€Ρ‚Ρ‹ΠΉ Π·Π°ΠΊΠΎΠ½ МаксвСлла — это Π·Π°ΠΊΠΎΠ½ АмпСра . Π—Π°ΠΊΠΎΠ½ АмпСра гласит, Ρ‡Ρ‚ΠΎ созданиС ΠΌΠ°Π³Π½ΠΈΡ‚Π½Ρ‹Ρ… ΠΏΠΎΠ»Π΅ΠΉ ΠΌΠΎΠΆΠ΅Ρ‚ ΠΎΡΡƒΡ‰Π΅ΡΡ‚Π²Π»ΡΡ‚ΡŒΡΡ двумя способами, Π° ΠΈΠΌΠ΅Π½Π½ΠΎ с ΠΏΠΎΠΌΠΎΡ‰ΡŒΡŽ элСктричСского Ρ‚ΠΎΠΊΠ°, Π° Ρ‚Π°ΠΊΠΆΠ΅ с ΠΏΠΎΠΌΠΎΡ‰ΡŒΡŽ ΠΈΠ·ΠΌΠ΅Π½ΡΡŽΡ‰ΠΈΡ…ΡΡ элСктричСских ΠΏΠΎΠ»Π΅ΠΉ. Π’ ΠΈΠ½Ρ‚Π΅Π³Ρ€Π°Π»ΡŒΠ½ΠΎΠΌ Ρ‚ΠΈΠΏΠ΅ ΠΈΠ½Π΄ΡƒΡ†ΠΈΡ€ΠΎΠ²Π°Π½Π½ΠΎΠ΅ ΠΌΠ°Π³Π½ΠΈΡ‚Π½ΠΎΠ΅ ΠΏΠΎΠ»Π΅ Π² области любого Π·Π°ΠΌΠΊΠ½ΡƒΡ‚ΠΎΠ³ΠΎ ΠΊΠΎΠ½Ρ‚ΡƒΡ€Π° Π±ΡƒΠ΄Π΅Ρ‚ ΠΏΡ€ΠΎΠΏΠΎΡ€Ρ†ΠΈΠΎΠ½Π°Π»ΡŒΠ½ΠΎ элСктричСскому Ρ‚ΠΎΠΊΡƒ ΠΈ Ρ‚ΠΎΠΊΡƒ смСщСния ΠΏΠΎ всСй Π·Π°ΠΌΠΊΠ½ΡƒΡ‚ΠΎΠΉ повСрхности.

Π—Π°ΠΊΠΎΠ½ АмпСра МаксвСлла сдСлаСт систСму ΡƒΡ€Π°Π²Π½Π΅Π½ΠΈΠΉ Ρ‚ΠΎΡ‡Π½ΠΎ Π½Π°Π΄Π΅ΠΆΠ½ΠΎΠΉ для нСстатичСских ΠΏΠΎΠ»Π΅ΠΉ Π±Π΅Π· измСнСния Π·Π°ΠΊΠΎΠ½ΠΎΠ² АмпСра, Π° Ρ‚Π°ΠΊΠΆΠ΅ Π·Π°ΠΊΠΎΠ½ΠΎΠ² Гаусса для фиксированных ΠΏΠΎΠ»Π΅ΠΉ.Но Π² Ρ€Π΅Π·ΡƒΠ»ΡŒΡ‚Π°Ρ‚Π΅ оТидаСтся, Ρ‡Ρ‚ΠΎ ΠΈΠ·ΠΌΠ΅Π½Π΅Π½ΠΈΠ΅ ΠΌΠ°Π³Π½ΠΈΡ‚Π½ΠΎΠ³ΠΎ поля Π²Ρ‹Π·ΠΎΠ²Π΅Ρ‚ элСктричСскоС ΠΏΠΎΠ»Π΅. Π’Π°ΠΊΠΈΠΌ ΠΎΠ±Ρ€Π°Π·ΠΎΠΌ, эти матСматичСскиС уравнСния позволят самодостаточной элСктромагнитной Π²ΠΎΠ»Π½Π΅ ΠΏΠ΅Ρ€Π΅ΠΌΠ΅Ρ‰Π°Ρ‚ΡŒΡΡ Ρ‡Π΅Ρ€Π΅Π· пустоС пространство. Π‘ΠΊΠΎΡ€ΠΎΡΡ‚ΡŒ элСктромагнитных Π²ΠΎΠ»Π½ ΠΌΠΎΠΆΠ½ΠΎ ΠΈΠ·ΠΌΠ΅Ρ€ΠΈΡ‚ΡŒ, ΠΈ этого ΠΌΠΎΠΆΠ½ΠΎ ΠΎΠΆΠΈΠ΄Π°Ρ‚ΡŒ ΠΎΡ‚ Ρ‚ΠΎΠΊΠΎΠ², Π° Ρ‚Π°ΠΊΠΆΠ΅ ΠΎΡ‚ экспСримСнтов с зарядом, ΠΊΠΎΡ‚ΠΎΡ€Ρ‹Π΅ ΡΠΎΠΎΡ‚Π²Π΅Ρ‚ΡΡ‚Π²ΡƒΡŽΡ‚ скорости свСта, ΠΈ это ΠΎΠ΄ΠΈΠ½ ΠΈΠ· Ρ‚ΠΈΠΏΠΎΠ² элСктромагнитного излучСния.

βˆ‡ x B = J / Ξ΅0c2 + 1 / c2 βˆ‚E / βˆ‚t

Π’Π°ΠΊΠΈΠΌ ΠΎΠ±Ρ€Π°Π·ΠΎΠΌ, это всС уравнСния МаксвСлла.Из ΠΏΡ€ΠΈΠ²Π΅Π΄Π΅Π½Π½Ρ‹Ρ… Π²Ρ‹ΡˆΠ΅ ΡƒΡ€Π°Π²Π½Π΅Π½ΠΈΠΉ, Π½Π°ΠΊΠΎΠ½Π΅Ρ†, ΠΌΡ‹ ΠΌΠΎΠΆΠ΅ΠΌ ΡΠ΄Π΅Π»Π°Ρ‚ΡŒ Π²Ρ‹Π²ΠΎΠ΄, Ρ‡Ρ‚ΠΎ эти уравнСния Π²ΠΊΠ»ΡŽΡ‡Π°ΡŽΡ‚ Ρ‡Π΅Ρ‚Ρ‹Ρ€Π΅ Π·Π°ΠΊΠΎΠ½Π°, ΠΊΠΎΡ‚ΠΎΡ€Ρ‹Π΅ связаны с элСктричСским (E), Π° Ρ‚Π°ΠΊΠΆΠ΅ с ΠΌΠ°Π³Π½ΠΈΡ‚Π½Ρ‹ΠΌ (B) ΠΏΠΎΠ»Π΅ΠΌ, ΠΊΠΎΡ‚ΠΎΡ€Ρ‹Π΅ ΠΎΠ±ΡΡƒΠΆΠ΄Π°Π»ΠΈΡΡŒ Π²Ρ‹ΡˆΠ΅. УравнСния МаксвСлла ΠΌΠΎΠ³ΡƒΡ‚ Π±Ρ‹Ρ‚ΡŒ записаны ΠΊΠ°ΠΊ Π² Ρ„ΠΎΡ€ΠΌΠ΅ эквивалСнтного ΠΈΠ½Ρ‚Π΅Π³Ρ€Π°Π»Π°, Ρ‚Π°ΠΊ ΠΈ Π² Ρ„ΠΎΡ€ΠΌΠ΅ Π΄ΠΈΡ„Ρ„Π΅Ρ€Π΅Π½Ρ†ΠΈΠ°Π»Π°. Π’ΠΎΡ‚ Π²Π°ΠΌ вопрос, ΠΊΠ°ΠΊΠΎΠ²Ρ‹ прилоТСния ΡƒΡ€Π°Π²Π½Π΅Π½ΠΈΠΉ МаксвСлла?

16.2: УравнСния МаксвСлла ΠΈ элСктромагнитныС Π²ΠΎΠ»Π½Ρ‹

Π¦Π΅Π»ΠΈ обучСния

К ΠΊΠΎΠ½Ρ†Ρƒ этого Ρ€Π°Π·Π΄Π΅Π»Π° Π²Ρ‹ смоТСтС:

  • ΠžΠ±ΡŠΡΡΠ½ΠΈΡ‚Π΅ исправлСниС МаксвСллом Π·Π°ΠΊΠΎΠ½Π° АмпСра, Π²ΠΊΠ»ΡŽΡ‡ΠΈΠ² Π² Π½Π΅Π³ΠΎ Ρ‚ΠΎΠΊ смСщСния
  • Π‘Ρ„ΠΎΡ€ΠΌΡƒΠ»ΠΈΡ€ΡƒΠΉΡ‚Π΅ ΠΈ ΠΏΡ€ΠΈΠΌΠ΅Π½ΠΈΡ‚Π΅ уравнСния МаксвСлла Π² ΠΈΠ½Ρ‚Π΅Π³Ρ€Π°Π»ΡŒΠ½ΠΎΠΉ Ρ„ΠΎΡ€ΠΌΠ΅
  • ΠžΠΏΠΈΡˆΠΈΡ‚Π΅, ΠΊΠ°ΠΊ симмСтрия ΠΌΠ΅ΠΆΠ΄Ρƒ ΠΈΠ·ΠΌΠ΅Π½ΡΡŽΡ‰ΠΈΠΌΡΡ элСктричСским ΠΈ ΠΈΠ·ΠΌΠ΅Π½ΡΡŽΡ‰ΠΈΠΌΡΡ ΠΌΠ°Π³Π½ΠΈΡ‚Π½Ρ‹ΠΌ полями ΠΎΠ±ΡŠΡΡΠ½ΡΠ΅Ρ‚ прСдсказаниС МаксвСлла ΠΎΠ± элСктромагнитных Π²ΠΎΠ»Π½Π°Ρ….
  • ΠžΠΏΠΈΡˆΠΈΡ‚Π΅, ΠΊΠ°ΠΊ Π“Π΅Ρ€Ρ† ΠΏΠΎΠ΄Ρ‚Π²Π΅Ρ€Π΄ΠΈΠ» прСдсказаниС МаксвСлла ΠΎΠ± элСктромагнитных Π²ΠΎΠ»Π½Π°Ρ….

ДТСймс ΠšΠ»Π΅Ρ€ΠΊ МаксвСлл (1831–1879) Π±Ρ‹Π» ΠΎΠ΄Π½ΠΈΠΌ ΠΈΠ· основных Π°Π²Ρ‚ΠΎΡ€ΠΎΠ² Ρ„ΠΈΠ·ΠΈΠΊΠΈ дСвятнадцатого Π²Π΅ΠΊΠ° (рисунок \ (\ PageIndex {1} \)).Π₯отя ΠΎΠ½ ΡƒΠΌΠ΅Ρ€ ΠΌΠΎΠ»ΠΎΠ΄Ρ‹ΠΌ, ΠΎΠ½ внСс большой Π²ΠΊΠ»Π°Π΄ Π² Ρ€Π°Π·Π²ΠΈΡ‚ΠΈΠ΅ кинСтичСской Ρ‚Π΅ΠΎΡ€ΠΈΠΈ Π³Π°Π·ΠΎΠ², ΠΏΠΎΠ½ΠΈΠΌΠ°Π½ΠΈΠ΅ Ρ†Π²Π΅Ρ‚ΠΎΠ²ΠΎΠ³ΠΎ зрСния ΠΈ ΠΏΡ€ΠΈΡ€ΠΎΠ΄Ρƒ ΠΊΠΎΠ»Π΅Ρ† Π‘Π°Ρ‚ΡƒΡ€Π½Π°. Он, вСроятно, Π½Π°ΠΈΠ±ΠΎΠ»Π΅Π΅ извСстСн Ρ‚Π΅ΠΌ, Ρ‡Ρ‚ΠΎ объСдинил ΡΡƒΡ‰Π΅ΡΡ‚Π²ΡƒΡŽΡ‰ΠΈΠ΅ знания ΠΎ Π·Π°ΠΊΠΎΠ½Π°Ρ… элСктричСства ΠΈ ΠΌΠ°Π³Π½Π΅Ρ‚ΠΈΠ·ΠΌΠ° с собствСнными открытиями Π² ΠΏΠΎΠ»Π½ΡƒΡŽ Π²ΡΠ΅ΠΎΠ±ΡŠΠ΅ΠΌΠ»ΡŽΡ‰ΡƒΡŽ ΡΠ»Π΅ΠΊΡ‚Ρ€ΠΎΠΌΠ°Π³Π½ΠΈΡ‚Π½ΡƒΡŽ Ρ‚Π΅ΠΎΡ€ΠΈΡŽ, ΠΏΡ€Π΅Π΄ΡΡ‚Π°Π²Π»Π΅Π½Π½ΡƒΡŽ уравнСниями МаксвСлла .

Рисунок \ (\ PageIndex {1} \): ДТСймс ΠšΠ»Π΅Ρ€ΠΊ МаксвСлл, Ρ„ΠΈΠ·ΠΈΠΊ дСвятнадцатого Π²Π΅ΠΊΠ°, Ρ€Π°Π·Ρ€Π°Π±ΠΎΡ‚Π°Π» Ρ‚Π΅ΠΎΡ€ΠΈΡŽ, ΠΎΠ±ΡŠΡΡΠ½ΡΡŽΡ‰ΡƒΡŽ взаимосвязь ΠΌΠ΅ΠΆΠ΄Ρƒ элСктричСством ΠΈ ΠΌΠ°Π³Π½Π΅Ρ‚ΠΈΠ·ΠΌΠΎΠΌ, ΠΈ ΠΏΡ€Π°Π²ΠΈΠ»ΡŒΠ½ΠΎ прСдсказал, Ρ‡Ρ‚ΠΎ Π²ΠΈΠ΄ΠΈΠΌΡ‹ΠΉ свСт состоит ΠΈΠ· элСктромагнитных Π²ΠΎΠ»Π½.

ΠŸΠΎΠΏΡ€Π°Π²ΠΊΠ° МаксвСлла ΠΊ Π·Π°ΠΊΠΎΠ½Π°ΠΌ элСктричСства ΠΈ ΠΌΠ°Π³Π½Π΅Ρ‚ΠΈΠ·ΠΌΠ°

Π§Π΅Ρ‚Ρ‹Ρ€Π΅ основных Π·Π°ΠΊΠΎΠ½Π° элСктричСства ΠΈ ΠΌΠ°Π³Π½Π΅Ρ‚ΠΈΠ·ΠΌΠ° Π±Ρ‹Π»ΠΈ ΠΎΡ‚ΠΊΡ€Ρ‹Ρ‚Ρ‹ ΡΠΊΡΠΏΠ΅Ρ€ΠΈΠΌΠ΅Π½Ρ‚Π°Π»ΡŒΠ½ΠΎ благодаря Ρ€Π°Π±ΠΎΡ‚Π΅ Ρ‚Π°ΠΊΠΈΡ… Ρ„ΠΈΠ·ΠΈΠΊΠΎΠ², ΠΊΠ°ΠΊ ЭрстСд, ΠšΡƒΠ»ΠΎΠ½, Гаусс ΠΈ Π€Π°Ρ€Π°Π΄Π΅ΠΉ. МаксвСлл ΠΎΠ±Π½Π°Ρ€ΡƒΠΆΠΈΠ» логичСскиС нСсоотвСтствия Π² этих Π±ΠΎΠ»Π΅Π΅ Ρ€Π°Π½Π½ΠΈΡ… Ρ€Π΅Π·ΡƒΠ»ΡŒΡ‚Π°Ρ‚Π°Ρ… ΠΈ ​​назвал ΠΈΡ… ΠΏΡ€ΠΈΡ‡ΠΈΠ½ΠΎΠΉ Π½Π΅ΠΏΠΎΠ»Π½ΠΎΡ‚Ρƒ Π·Π°ΠΊΠΎΠ½Π° АмпСра.

Напомним, Ρ‡Ρ‚ΠΎ согласно Π·Π°ΠΊΠΎΠ½Ρƒ АмпСра, ΠΈΠ½Ρ‚Π΅Π³Ρ€Π°Π» ΠΌΠ°Π³Π½ΠΈΡ‚Π½ΠΎΠ³ΠΎ поля Π²ΠΎΠΊΡ€ΡƒΠ³ Π·Π°ΠΌΠΊΠ½ΡƒΡ‚ΠΎΠ³ΠΎ ΠΊΠΎΠ½Ρ‚ΡƒΡ€Π° C ΠΏΡ€ΠΎΠΏΠΎΡ€Ρ†ΠΈΠΎΠ½Π°Π»Π΅Π½ Ρ‚ΠΎΠΊΡƒ I , проходящСму Ρ‡Π΅Ρ€Π΅Π· Π»ΡŽΠ±ΡƒΡŽ ΠΏΠΎΠ²Π΅Ρ€Ρ…Π½ΠΎΡΡ‚ΡŒ, Π³Ρ€Π°Π½ΠΈΡ†Π° ΠΊΠΎΡ‚ΠΎΡ€ΠΎΠΉ прСдставляСт собой сам ΠΊΠΎΠ½Ρ‚ΡƒΡ€ C :

\ [\ oint \ vec {B} \ cdot d \ vec {s} = \ mu_0 I.\ label {Eq1} \]

БущСствуСт бСсконСчно ΠΌΠ½ΠΎΠ³ΠΎ повСрхностСй, ΠΊΠΎΡ‚ΠΎΡ€Ρ‹Π΅ ΠΌΠΎΠΆΠ½ΠΎ ΠΏΡ€ΠΈΡΠΎΠ΅Π΄ΠΈΠ½ΠΈΡ‚ΡŒ ΠΊ любой ΠΏΠ΅Ρ‚Π»Π΅, ΠΈ Π·Π°ΠΊΠΎΠ½ АмпСра, сформулированный Π² ΡƒΡ€Π°Π²Π½Π΅Π½ΠΈΠΈ \ ref {Eq1}, Π½Π΅ зависит ΠΎΡ‚ Π²Ρ‹Π±ΠΎΡ€Π° повСрхности.

Рассмотрим установку, ΠΏΠΎΠΊΠ°Π·Π°Π½Π½ΡƒΡŽ Π½Π° рисункС \ (\ PageIndex {2} \). Π˜ΡΡ‚ΠΎΡ‡Π½ΠΈΠΊ Π­Π”Π‘ Ρ€Π΅Π·ΠΊΠΎ ΠΏΠΎΠ΄ΠΊΠ»ΡŽΡ‡Π°Π΅Ρ‚ΡΡ ΠΊ кондСнсатору с ΠΏΠ°Ρ€Π°Π»Π»Π΅Π»ΡŒΠ½Ρ‹ΠΌΠΈ пластинами, Ρ‚Π°ΠΊ Ρ‡Ρ‚ΠΎ Π² ΠΏΡ€ΠΎΠ²ΠΎΠ΄Π΅ Π²ΠΎΠ·Π½ΠΈΠΊΠ°Π΅Ρ‚ зависящий ΠΎΡ‚ Π²Ρ€Π΅ΠΌΠ΅Π½ΠΈ Ρ‚ΠΎΠΊ I . ΠŸΡ€Π΅Π΄ΠΏΠΎΠ»ΠΎΠΆΠΈΠΌ, ΠΌΡ‹ примСняСм Π·Π°ΠΊΠΎΠ½ АмпСра ΠΊ ΠΊΠΎΠ½Ρ‚ΡƒΡ€Ρƒ C , ΠΏΠΎΠΊΠ°Π·Π°Π½Π½ΠΎΠΌΡƒ ΠΏΠ΅Ρ€Π΅Π΄ Ρ‚Π΅ΠΌ, ΠΊΠ°ΠΊ кондСнсатор ΠΏΠΎΠ»Π½ΠΎΡΡ‚ΡŒΡŽ заряТСн, Ρ‚Π°ΠΊ Ρ‡Ρ‚ΠΎ \ (I \ neq 0 \).ΠŸΠΎΠ²Π΅Ρ€Ρ…Π½ΠΎΡΡ‚ΡŒ \ (S_1 \) Π΄Π°Π΅Ρ‚ Π½Π΅Π½ΡƒΠ»Π΅Π²ΠΎΠ΅ Π·Π½Π°Ρ‡Π΅Π½ΠΈΠ΅ для Π·Π°ΠΌΠΊΠ½ΡƒΡ‚ΠΎΠ³ΠΎ Ρ‚ΠΎΠΊΠ° I , Ρ‚ΠΎΠ³Π΄Π° ΠΊΠ°ΠΊ ΠΏΠΎΠ²Π΅Ρ€Ρ…Π½ΠΎΡΡ‚ΡŒ \ (S_2 \) Π΄Π°Π΅Ρ‚ ноль для Π·Π°ΠΌΠΊΠ½ΡƒΡ‚ΠΎΠ³ΠΎ Ρ‚ΠΎΠΊΠ°, ΠΏΠΎΡ‚ΠΎΠΌΡƒ Ρ‡Ρ‚ΠΎ Ρ‡Π΅Ρ€Π΅Π· Π½Π΅Π³ΠΎ Π½Π΅ ΠΏΡ€ΠΎΡ…ΠΎΠ΄ΠΈΡ‚ Ρ‚ΠΎΠΊ:

\ [\ underbrace {\ oint_C \ vec {B} \ cdot d \ vec {s} = \ mu_0 I} _ {\ text {Ссли ΠΈΡΠΏΠΎΠ»ΡŒΠ·ΡƒΠ΅Ρ‚ΡΡ ΠΏΠΎΠ²Π΅Ρ€Ρ…Π½ΠΎΡΡ‚ΡŒ} S_1 \ text {}} \]

\ [\ underbrace {\, = 0} _ {\ text {Ссли ΠΈΡΠΏΠΎΠ»ΡŒΠ·ΡƒΠ΅Ρ‚ΡΡ ΠΏΠΎΠ²Π΅Ρ€Ρ…Π½ΠΎΡΡ‚ΡŒ} S_2 \ text {}} \]

Ясно, Ρ‡Ρ‚ΠΎ Π·Π°ΠΊΠΎΠ½ АмпСра Π² Π΅Π³ΠΎ ΠΎΠ±Ρ‹Ρ‡Π½ΠΎΠΌ Π²ΠΈΠ΄Π΅ здСсь Π½Π΅ Ρ€Π°Π±ΠΎΡ‚Π°Π΅Ρ‚. Π­Ρ‚ΠΎ ΠΌΠΎΠΆΠ΅Ρ‚ Π±Ρ‹Ρ‚ΡŒ Π½Π΅ΡƒΠ΄ΠΈΠ²ΠΈΡ‚Π΅Π»ΡŒΠ½ΠΎ, ΠΏΠΎΡ‚ΠΎΠΌΡƒ Ρ‡Ρ‚ΠΎ Π·Π°ΠΊΠΎΠ½ АмпСра, ΠΏΡ€ΠΈΠΌΠ΅Π½ΡΠ²ΡˆΠΈΠΉΡΡ Π² ΠΏΡ€Π΅Π΄Ρ‹Π΄ΡƒΡ‰ΠΈΡ… Π³Π»Π°Π²Π°Ρ…, Ρ‚Ρ€Π΅Π±ΠΎΠ²Π°Π» постоянного Ρ‚ΠΎΠΊΠ°, Ρ‚ΠΎΠ³Π΄Π° ΠΊΠ°ΠΊ Ρ‚ΠΎΠΊ Π² этом экспСримСнтС мСняСтся со Π²Ρ€Π΅ΠΌΠ΅Π½Π΅ΠΌ ΠΈ совсСм Π½Π΅ постоянный.

Рисунок \ (\ PageIndex {2} \): Ρ‚ΠΎΠΊΠΈ Ρ‡Π΅Ρ€Π΅Π· ΠΏΠΎΠ²Π΅Ρ€Ρ…Π½ΠΎΡΡ‚ΡŒ \ (S_1 \) ΠΈ ΠΏΠΎΠ²Π΅Ρ€Ρ…Π½ΠΎΡΡ‚ΡŒ \ (S_2 \) Π½Π΅ Ρ€Π°Π²Π½Ρ‹, нСсмотря Π½Π° Π½Π°Π»ΠΈΡ‡ΠΈΠ΅ ΠΎΠ΄Π½ΠΎΠ³ΠΎ ΠΈ Ρ‚ΠΎΠ³ΠΎ ΠΆΠ΅ Π³Ρ€Π°Π½ΠΈΡ‡Π½ΠΎΠ³ΠΎ ΠΊΠΎΠ½Ρ‚ΡƒΡ€Π° C .

Как ΠΌΠΎΠΆΠ½ΠΎ ΠΈΠ·ΠΌΠ΅Π½ΠΈΡ‚ΡŒ Π·Π°ΠΊΠΎΠ½ АмпСра, Ρ‡Ρ‚ΠΎΠ±Ρ‹ ΠΎΠ½ Ρ€Π°Π±ΠΎΡ‚Π°Π» Π²ΠΎ всСх ситуациях? МаксвСлл ΠΏΡ€Π΅Π΄Π»ΠΎΠΆΠΈΠ» Π²ΠΊΠ»ΡŽΡ‡ΠΈΡ‚ΡŒ Π΄ΠΎΠΏΠΎΠ»Π½ΠΈΡ‚Π΅Π»ΡŒΠ½Ρ‹ΠΉ Π²ΠΊΠ»Π°Π΄, Π½Π°Π·Ρ‹Π²Π°Π΅ΠΌΡ‹ΠΉ Ρ‚ΠΎΠΊΠΎΠΌ смСщСния \ (I_d \), Π² Ρ€Π΅Π°Π»ΡŒΠ½Ρ‹ΠΉ Ρ‚ΠΎΠΊ I ,

\ [\ Π² ΠΊΠΎΡ€ΠΎΠ±ΠΊΠ΅ {\ oint_S \ vec {B} \ cdot d \ vec {s} = \ mu_0 (I + I_d)} \ label {EQ4} \]

, Π³Π΄Π΅ Ρ‚ΠΎΠΊ смСщСния ΠΎΠΏΡ€Π΅Π΄Π΅Π»Π΅Π½ ΠΊΠ°ΠΊ

\ [\ Π² ΠΊΠΎΡ€ΠΎΠ±ΠΊΠ΅ {I_d = \ epsilon_0 \ dfrac {d \ Phi_E} {dt}.} \ label {EQ5} \]

Π—Π΄Π΅ΡΡŒ \ (\ epsilon_0 \) — диэлСктричСская ΠΏΡ€ΠΎΠ½ΠΈΡ†Π°Π΅ΠΌΠΎΡΡ‚ΡŒ свободного пространства , Π° \ (\ Phi_E \) — элСктричСский ΠΏΠΎΡ‚ΠΎΠΊ, опрСдСляСмый ΠΊΠ°ΠΊ

.

\ [\ Phi_E = \ iint_ {Surface \, S} \ vec {E} \ cdot d \ vec {A}. \]

Π’ΠΎΠΊ смСщСния Π°Π½Π°Π»ΠΎΠ³ΠΈΡ‡Π΅Π½ Ρ€Π΅Π°Π»ΡŒΠ½ΠΎΠΌΡƒ Ρ‚ΠΎΠΊΡƒ Π² Π·Π°ΠΊΠΎΠ½Π΅ АмпСра, Ρ‚Π°ΠΊΠΈΠΌ ΠΆΠ΅ ΠΎΠ±Ρ€Π°Π·ΠΎΠΌ входя Π² Π·Π°ΠΊΠΎΠ½ АмпСра. Однако ΠΎΠ½ создаСтся ΠΈΠ·ΠΌΠ΅Π½ΡΡŽΡ‰ΠΈΠΌΡΡ элСктричСским ΠΏΠΎΠ»Π΅ΠΌ. Он ΡƒΡ‡ΠΈΡ‚Ρ‹Π²Π°Π΅Ρ‚ ΠΈΠ·ΠΌΠ΅Π½ΡΡŽΡ‰Π΅Π΅ΡΡ элСктричСскоС ΠΏΠΎΠ»Π΅, ΡΠΎΠ·Π΄Π°ΡŽΡ‰Π΅Π΅ ΠΌΠ°Π³Π½ΠΈΡ‚Π½ΠΎΠ΅ ΠΏΠΎΠ»Π΅, ΠΊΠ°ΠΊ ΠΈ настоящий Ρ‚ΠΎΠΊ, Π½ΠΎ Ρ‚ΠΎΠΊ смСщСния ΠΌΠΎΠΆΠ΅Ρ‚ ΡΠΎΠ·Π΄Π°Π²Π°Ρ‚ΡŒ ΠΌΠ°Π³Π½ΠΈΡ‚Π½ΠΎΠ΅ ΠΏΠΎΠ»Π΅ Π΄Π°ΠΆΠ΅ Ρ‚Π°ΠΌ, Π³Π΄Π΅ Π½Π΅Ρ‚ Ρ€Π΅Π°Π»ΡŒΠ½ΠΎΠ³ΠΎ Ρ‚ΠΎΠΊΠ°.Когда этот Π΄ΠΎΠΏΠΎΠ»Π½ΠΈΡ‚Π΅Π»ΡŒΠ½Ρ‹ΠΉ Ρ‡Π»Π΅Π½ Π²ΠΊΠ»ΡŽΡ‡Π΅Π½, ΠΌΠΎΠ΄ΠΈΡ„ΠΈΡ†ΠΈΡ€ΠΎΠ²Π°Π½Π½ΠΎΠ΅ ΡƒΡ€Π°Π²Π½Π΅Π½ΠΈΠ΅ Π·Π°ΠΊΠΎΠ½Π° АмпСра ΠΏΡ€ΠΈΠ½ΠΈΠΌΠ°Π΅Ρ‚ Π²ΠΈΠ΄

.

\ [\ oint_C \ vec {B} \ cdot d \ vec {s} = \ mu_0 I + \ epsilon_0 \ mu_0 \ dfrac {d \ Phi_E} {dt} \]

ΠΈ Π½Π΅ зависит ΠΎΡ‚ повСрхности S , Ρ‡Π΅Ρ€Π΅Π· ΠΊΠΎΡ‚ΠΎΡ€ΡƒΡŽ измСряСтся Ρ‚ΠΎΠΊ I .

Π’Π΅ΠΏΠ΅Ρ€ΡŒ ΠΌΡ‹ ΠΌΠΎΠΆΠ΅ΠΌ ΠΈΡΡΠ»Π΅Π΄ΠΎΠ²Π°Ρ‚ΡŒ эту ΠΌΠΎΠ΄ΠΈΡ„ΠΈΡ†ΠΈΡ€ΠΎΠ²Π°Π½Π½ΡƒΡŽ Π²Π΅Ρ€ΡΠΈΡŽ Π·Π°ΠΊΠΎΠ½Π° АмпСра, Ρ‡Ρ‚ΠΎΠ±Ρ‹ ΠΏΠΎΠ΄Ρ‚Π²Π΅Ρ€Π΄ΠΈΡ‚ΡŒ, Ρ‡Ρ‚ΠΎ ΠΎΠ½ выполняСтся нСзависимо ΠΎΡ‚ Ρ‚ΠΎΠ³ΠΎ, Π²Ρ‹Π±Ρ€Π°Π½Π° Π»ΠΈ ΠΏΠΎΠ²Π΅Ρ€Ρ…Π½ΠΎΡΡ‚ΡŒ \ (S_1 \) ΠΈΠ»ΠΈ ΠΏΠΎΠ²Π΅Ρ€Ρ…Π½ΠΎΡΡ‚ΡŒ \ (S_2 \) Π½Π° рисункС \ (\ PageIndex {2} \). ЭлСктричСскоС ΠΏΠΎΠ»Π΅ \ (\ vec {E} \), ΡΠΎΠΎΡ‚Π²Π΅Ρ‚ΡΡ‚Π²ΡƒΡŽΡ‰Π΅Π΅ ΠΏΠΎΡ‚ΠΎΠΊΡƒ \ (\ Phi_E \) Π² ΡƒΡ€Π°Π²Π½Π΅Π½ΠΈΠΈ \ ref {EQ5}, находится ΠΌΠ΅ΠΆΠ΄Ρƒ пластинами кондСнсатора.Π‘Π»Π΅Π΄ΠΎΠ²Π°Ρ‚Π΅Π»ΡŒΠ½ΠΎ, ΠΏΠΎΠ»Π΅ \ (\ vec {E} \) ΠΈ Ρ‚ΠΎΠΊ смСщСния Ρ‡Π΅Ρ€Π΅Π· ΠΏΠΎΠ²Π΅Ρ€Ρ…Π½ΠΎΡΡ‚ΡŒ \ (S_1 \) Ρ€Π°Π²Π½Ρ‹ Π½ΡƒΠ»ΡŽ, ΠΈ ΡƒΡ€Π°Π²Π½Π΅Π½ΠΈΠ΅ \ ref {EQ4} ΠΏΡ€ΠΈΠ½ΠΈΠΌΠ°Π΅Ρ‚ Ρ„ΠΎΡ€ΠΌΡƒ

\ [\ oint_C \ vec {B} \ cdot d \ vec {s} = \ mu_0 I. \ label {16.5} \]

Π’Π΅ΠΏΠ΅Ρ€ΡŒ ΠΌΡ‹ Π΄ΠΎΠ»ΠΆΠ½Ρ‹ ΠΏΠΎΠΊΠ°Π·Π°Ρ‚ΡŒ, Ρ‡Ρ‚ΠΎ для повСрхности \ (S_2 \), Ρ‡Π΅Ρ€Π΅Π· ΠΊΠΎΡ‚ΠΎΡ€ΡƒΡŽ фактичСски Π½Π΅ ΠΏΡ€ΠΎΡ‚Π΅ΠΊΠ°Π΅Ρ‚ Ρ‚ΠΎΠΊ, Ρ‚ΠΎΠΊ смСщСния ΠΏΡ€ΠΈΠ²ΠΎΠ΄ΠΈΡ‚ ΠΊ Ρ‚ΠΎΠΌΡƒ ΠΆΠ΅ Π·Π½Π°Ρ‡Π΅Π½ΠΈΡŽ \ (\ mu_0 I \) для ΠΏΡ€Π°Π²ΠΎΠΉ части уравнСния Π·Π°ΠΊΠΎΠ½Π° АмпСра. Для повСрхности \ (S_2 \) ΡƒΡ€Π°Π²Π½Π΅Π½ΠΈΠ΅ ΠΏΡ€ΠΈΠ½ΠΈΠΌΠ°Π΅Ρ‚ Π²ΠΈΠ΄

\ [\ oint_C \ vec {B} \ cdot d \ vec {s} = \ mu_0 \ dfrac {d} {dt} \ left [\ epsilon_0 \ iint_ {Surface \, S_2} \ vec {E} \ cdot d \ vec {A} \ right].\]

Π—Π°ΠΊΠΎΠ½ Гаусса для элСктричСского заряда Ρ‚Ρ€Π΅Π±ΡƒΠ΅Ρ‚ Π·Π°ΠΌΠΊΠ½ΡƒΡ‚ΠΎΠΉ повСрхности ΠΈ ΠΎΠ±Ρ‹Ρ‡Π½ΠΎ Π½Π΅ ΠΌΠΎΠΆΠ΅Ρ‚ Π±Ρ‹Ρ‚ΡŒ ΠΏΡ€ΠΈΠΌΠ΅Π½Π΅Π½ ΠΊ повСрхности Π²Ρ€ΠΎΠ΄Π΅ \ (S_1 \) ΠΎΡ‚Π΄Π΅Π»ΡŒΠ½ΠΎ ΠΈΠ»ΠΈ Ρ‚ΠΎΠ»ΡŒΠΊΠΎ \ (S_2 \). Но Π΄Π²Π΅ повСрхности \ (S_1 \) ΠΈ \ (S_2 \) ΠΎΠ±Ρ€Π°Π·ΡƒΡŽΡ‚ Π·Π°ΠΌΠΊΠ½ΡƒΡ‚ΡƒΡŽ ΠΏΠΎΠ²Π΅Ρ€Ρ…Π½ΠΎΡΡ‚ΡŒ Π½Π° рисункС \ (\ PageIndex {2} \) ΠΈ ΠΌΠΎΠ³ΡƒΡ‚ ΠΈΡΠΏΠΎΠ»ΡŒΠ·ΠΎΠ²Π°Ρ‚ΡŒΡΡ Π² Π·Π°ΠΊΠΎΠ½Π΅ Гаусса. ΠŸΠΎΡΠΊΠΎΠ»ΡŒΠΊΡƒ элСктричСскоС ΠΏΠΎΠ»Π΅ Ρ€Π°Π²Π½ΠΎ Π½ΡƒΠ»ΡŽ Π½Π° \ (S_1 \), Π²ΠΊΠ»Π°Π΄ ΠΏΠΎΡ‚ΠΎΠΊΠ° Ρ‡Π΅Ρ€Π΅Π· \ (S_1 \) Ρ€Π°Π²Π΅Π½ Π½ΡƒΠ»ΡŽ. Π­Ρ‚ΠΎ Π΄Π°Π΅Ρ‚ Π½Π°ΠΌ

\ [\ begin {align} \ oint_ {Surface \, S_1 + S_2} \ vec {E} \ cdot d \ vec {A} & = \ iint_ {Surface \, S_1} \ vec {E} \ cdot d \ vec {A} + \ iint_ {Surface \, S_2} \ vec {E} \ cdot d \ vec {A} \\ [4pt] & = 0 + \ iint_ {Surface \, S_2} \ vec {E} \ cdot d \ vec {A} \\ [4pt] & = \ iint_ {Surface \, S_2} \ vec {E} \ cdot d \ vec {A}.\ end {align} \]

Π‘Π»Π΅Π΄ΠΎΠ²Π°Ρ‚Π΅Π»ΡŒΠ½ΠΎ, ΠΌΡ‹ ΠΌΠΎΠΆΠ΅ΠΌ Π·Π°ΠΌΠ΅Π½ΠΈΡ‚ΡŒ ΠΈΠ½Ρ‚Π΅Π³Ρ€Π°Π» ΠΏΠΎ \ (S_2 \) Π² ΡƒΡ€Π°Π²Π½Π΅Π½ΠΈΠΈ \ ref {16.5} Π½Π° Π·Π°ΠΌΠΊΠ½ΡƒΡ‚ΡƒΡŽ гауссову ΠΏΠΎΠ²Π΅Ρ€Ρ…Π½ΠΎΡΡ‚ΡŒ \ (S_1 + S_2 \) ΠΈ ΠΏΡ€ΠΈΠΌΠ΅Π½ΠΈΡ‚ΡŒ Π·Π°ΠΊΠΎΠ½ Гаусса, Ρ‡Ρ‚ΠΎΠ±Ρ‹ ΠΏΠΎΠ»ΡƒΡ‡ΠΈΡ‚ΡŒ

\ [\ oint_ {S_1} \ vec {B} \ cdot d \ vec {s} = \ mu_0 \ dfrac {dQ_ {in}} {dt} = \ mu_0 I. \]

Π’Π°ΠΊΠΈΠΌ ΠΎΠ±Ρ€Π°Π·ΠΎΠΌ, ΠΌΠΎΠ΄ΠΈΡ„ΠΈΡ†ΠΈΡ€ΠΎΠ²Π°Π½Π½ΠΎΠ΅ ΡƒΡ€Π°Π²Π½Π΅Π½ΠΈΠ΅ Π·Π°ΠΊΠΎΠ½Π° АмпСра Ρ‚Π°ΠΊΠΎΠ΅ ΠΆΠ΅, ΠΊΠ°ΠΊ ΠΈ для повСрхности \ (S_2 \), Π³Π΄Π΅ правая Ρ‡Π°ΡΡ‚ΡŒ являСтся Ρ€Π΅Π·ΡƒΠ»ΡŒΡ‚Π°Ρ‚ΠΎΠΌ Ρ‚ΠΎΠΊΠ° смСщСния, ΠΊΠ°ΠΊ ΠΈ для повСрхности \ (S_1 \), Π³Π΄Π΅ Π²ΠΊΠ»Π°Π΄ происходит ΠΎΡ‚ фактичСского ΠΏΠΎΡ‚ΠΎΠΊ элСктричСского заряда.

Π’ΠΎΠΊ смСщСния Π² зарядном кондСнсаторС

ΠšΠΎΠ½Π΄Π΅Π½ΡΠ°Ρ‚ΠΎΡ€ с ΠΏΠ°Ρ€Π°Π»Π»Π΅Π»ΡŒΠ½Ρ‹ΠΌΠΈ пластинами Π΅ΠΌΠΊΠΎΡΡ‚ΡŒΡŽ C, , пластины ΠΊΠΎΡ‚ΠΎΡ€ΠΎΠ³ΠΎ ΠΈΠΌΠ΅ΡŽΡ‚ ΠΏΠ»ΠΎΡ‰Π°Π΄ΡŒ A, ΠΈ расстояниС ΠΌΠ΅ΠΆΠ΄Ρƒ Π½ΠΈΠΌΠΈ d , ΠΏΠΎΠ΄ΠΊΠ»ΡŽΡ‡Π΅Π½ ΠΊ рСзистору R ΠΈ Π±Π°Ρ‚Π°Ρ€Π΅Π΅ с напряТСниСм Π’, . Π’ΠΎΠΊ Π½Π°Ρ‡ΠΈΠ½Π°Π΅Ρ‚ Ρ‚Π΅Ρ‡ΡŒ ΠΏΡ€ΠΈ \ (t = 0 \).

  1. НайдитС Ρ‚ΠΎΠΊ смСщСния ΠΌΠ΅ΠΆΠ΄Ρƒ пластинами кондСнсатора Π² ΠΌΠΎΠΌΠ΅Π½Ρ‚ Π²Ρ€Π΅ΠΌΠ΅Π½ΠΈ t .
  2. Из свойств кондСнсатора Π½Π°ΠΉΠ΄ΠΈΡ‚Π΅ ΡΠΎΠΎΡ‚Π²Π΅Ρ‚ΡΡ‚Π²ΡƒΡŽΡ‰ΠΈΠΉ Ρ€Π΅Π°Π»ΡŒΠ½Ρ‹ΠΉ Ρ‚ΠΎΠΊ \ (I = \ dfrac {dQ} {dt} \) ΠΈ сравнитС ΠΎΡ‚Π²Π΅Ρ‚ с ΠΎΠΆΠΈΠ΄Π°Π΅ΠΌΡ‹ΠΌ Ρ‚ΠΎΠΊΠΎΠΌ Π² ΠΏΡ€ΠΎΠ²ΠΎΠ΄Π°Ρ… ΡΠΎΠΎΡ‚Π²Π΅Ρ‚ΡΡ‚Π²ΡƒΡŽΡ‰Π΅ΠΉ Ρ†Π΅ΠΏΠΈ RC .{-t / RC}. \ nonumber \] Π­Ρ‚ΠΎΡ‚ Ρ‚ΠΎΠΊ Ρ‚Π°ΠΊΠΎΠΉ ΠΆΠ΅, ΠΊΠ°ΠΊ \ (I_d \), Π½Π°ΠΉΠ΄Π΅Π½Π½Ρ‹ΠΉ Π² (a).

УравнСния МаксвСлла

Π‘ ΠΏΠΎΠΏΡ€Π°Π²ΠΊΠΎΠΉ Π½Π° Ρ‚ΠΎΠΊ смСщСния уравнСния МаксвСлла ΠΏΡ€ΠΈΠ½ΠΈΠΌΠ°ΡŽΡ‚ Π²ΠΈΠ΄

\ [\ begin {align} \ oint \ vec {E} \ cdot d \ vec {A} & = \ dfrac {Q_ {in}} {\ epsilon_0} \, (\ text {Π·Π°ΠΊΠΎΠ½ Гаусса}) \ label { eq1} \\ [4pt] \ oint \ vec {B} \ cdot d \ vec {A} & = 0 \, (\ text {Π·Π°ΠΊΠΎΠ½ Гаусса для ΠΌΠ°Π³Π½Π΅Ρ‚ΠΈΠ·ΠΌΠ°}) \ label {eq2} \\ [4pt] \ oint \ vec {E} \ cdot d \ vec {s} & = — \ dfrac {d \ Phi_m} {dt} \, (\ text {Π·Π°ΠΊΠΎΠ½ ЀарадСя}) \ label {eq3} \\ [4pt] \ oint \ vec { B} \ cdot d \ vec {s} & = \ mu_0 I + \ epsilon_0I + \ epsilon_0 \ mu_0 \ dfrac {d \ Phi_E} {dt} (\ text {Π·Π°ΠΊΠΎΠ½ АмпСра-МаксвСлла}).\ label {eq4} \ end {align} \]

ПослС Ρ‚ΠΎΠ³ΠΎ, ΠΊΠ°ΠΊ поля Π±Ρ‹Π»ΠΈ рассчитаны с использованиСм этих Ρ‡Π΅Ρ‚Ρ‹Ρ€Π΅Ρ… ΡƒΡ€Π°Π²Π½Π΅Π½ΠΈΠΉ, ΡƒΡ€Π°Π²Π½Π΅Π½ΠΈΠ΅ силы Π›ΠΎΡ€Π΅Π½Ρ†Π°

\ [\ vec {F} = q \ vec {E} + q \ vec {v} \ times \ vec {B} \]

Π΄Π°Π΅Ρ‚ силу, ΠΊΠΎΡ‚ΠΎΡ€ΡƒΡŽ поля Π΄Π΅ΠΉΡΡ‚Π²ΡƒΡŽΡ‚ Π½Π° частицу с зарядом q , Π΄Π²ΠΈΠΆΡƒΡ‰ΡƒΡŽΡΡ со ΡΠΊΠΎΡ€ΠΎΡΡ‚ΡŒΡŽ \ (\ vec {v} \). Π£Ρ€Π°Π²Π½Π΅Π½ΠΈΠ΅ силы Π›ΠΎΡ€Π΅Π½Ρ†Π° ΠΎΠ±ΡŠΠ΅Π΄ΠΈΠ½ΡΠ΅Ρ‚ силу элСктричСского поля ΠΈ ΠΌΠ°Π³Π½ΠΈΡ‚Π½ΠΎΠ³ΠΎ поля Π½Π° двиТущийся заряд. ΠœΠ°Π³Π½ΠΈΡ‚Π½Ρ‹Π΅ ΠΈ элСктричСскиС силы ΠΈΠ·ΡƒΡ‡Π°Π»ΠΈΡΡŒ Π² ΠΏΡ€Π΅Π΄Ρ‹Π΄ΡƒΡ‰ΠΈΡ… модулях.Π­Ρ‚ΠΈ Ρ‡Π΅Ρ‚Ρ‹Ρ€Π΅ уравнСния МаксвСлла, соотвСтствСнно:

УравнСния МаксвСлла

1. Π—Π°ΠΊΠΎΠ½ Гаусса

ЭлСктричСский ΠΏΠΎΡ‚ΠΎΠΊ Ρ‡Π΅Ρ€Π΅Π· Π»ΡŽΠ±ΡƒΡŽ Π·Π°ΠΌΠΊΠ½ΡƒΡ‚ΡƒΡŽ ΠΏΠΎΠ²Π΅Ρ€Ρ…Π½ΠΎΡΡ‚ΡŒ Ρ€Π°Π²Π΅Π½ элСктричСскому заряду \ (Q_ {in} \), Π·Π°ΠΊΠ»ΡŽΡ‡Π΅Π½Π½ΠΎΠΌΡƒ Π² повСрхности. Π—Π°ΠΊΠΎΠ½ Гаусса (ΡƒΡ€Π°Π²Π½Π΅Π½ΠΈΠ΅ \ ref {eq1}) описываСт связь ΠΌΠ΅ΠΆΠ΄Ρƒ элСктричСским зарядом ΠΈ создаваСмым ΠΈΠΌ элСктричСским ΠΏΠΎΠ»Π΅ΠΌ. Π­Ρ‚ΠΎ часто ΠΈΠ·ΠΎΠ±Ρ€Π°ΠΆΠ°ΡŽΡ‚ Π² Π²ΠΈΠ΄Π΅ Π»ΠΈΠ½ΠΈΠΉ элСктричСского поля, исходящих ΠΎΡ‚ ΠΏΠΎΠ»ΠΎΠΆΠΈΡ‚Π΅Π»ΡŒΠ½Ρ‹Ρ… зарядов ΠΈ Π·Π°ΠΊΠ°Π½Ρ‡ΠΈΠ²Π°ΡŽΡ‰ΠΈΡ…ΡΡ ΠΎΡ‚Ρ€ΠΈΡ†Π°Ρ‚Π΅Π»ΡŒΠ½Ρ‹ΠΌΠΈ зарядами, ΠΈ ΡƒΠΊΠ°Π·Ρ‹Π²Π°ΡŽΡ‰ΠΈΡ… Π½Π°ΠΏΡ€Π°Π²Π»Π΅Π½ΠΈΠ΅ элСктричСского поля Π² ΠΊΠ°ΠΆΠ΄ΠΎΠΉ Ρ‚ΠΎΡ‡ΠΊΠ΅ пространства.

2. Π—Π°ΠΊΠΎΠ½ Гаусса для ΠΌΠ°Π³Π½Π΅Ρ‚ΠΈΠ·ΠΌΠ°

ΠŸΠΎΡ‚ΠΎΠΊ ΠΌΠ°Π³Π½ΠΈΡ‚Π½ΠΎΠ³ΠΎ поля Ρ‡Π΅Ρ€Π΅Π· Π»ΡŽΠ±ΡƒΡŽ Π·Π°ΠΌΠΊΠ½ΡƒΡ‚ΡƒΡŽ ΠΏΠΎΠ²Π΅Ρ€Ρ…Π½ΠΎΡΡ‚ΡŒ Ρ€Π°Π²Π΅Π½ Π½ΡƒΠ»ΡŽ (ΡƒΡ€Π°Π²Π½Π΅Π½ΠΈΠ΅ \ ref {eq2}). Π­Ρ‚ΠΎ эквивалСнтно ΡƒΡ‚Π²Π΅Ρ€ΠΆΠ΄Π΅Π½ΠΈΡŽ, Ρ‡Ρ‚ΠΎ силовыС Π»ΠΈΠ½ΠΈΠΈ ΠΌΠ°Π³Π½ΠΈΡ‚Π½ΠΎΠ³ΠΎ поля Π½Π΅ΠΏΡ€Π΅Ρ€Ρ‹Π²Π½Ρ‹, Π½Π΅ ΠΈΠΌΠ΅ΡŽΡ‚ Π½ΠΈ Π½Π°Ρ‡Π°Π»Π°, Π½ΠΈ ΠΊΠΎΠ½Ρ†Π°. Π›ΡŽΠ±Π°Ρ линия ΠΌΠ°Π³Π½ΠΈΡ‚Π½ΠΎΠ³ΠΎ поля, входящая Π² ΠΎΠ±Π»Π°ΡΡ‚ΡŒ, ΠΎΠ³Ρ€Π°Π½ΠΈΡ‡Π΅Π½Π½ΡƒΡŽ ΠΏΠΎΠ²Π΅Ρ€Ρ…Π½ΠΎΡΡ‚ΡŒΡŽ, Ρ‚Π°ΠΊΠΆΠ΅ Π΄ΠΎΠ»ΠΆΠ½Π° ΠΏΠΎΠΊΠΈΠ½ΡƒΡ‚ΡŒ Π΅Π΅. НС извСстно ΠΎ сущСствовании ΠΌΠ°Π³Π½ΠΈΡ‚Π½Ρ‹Ρ… ΠΌΠΎΠ½ΠΎΠΏΠΎΠ»Π΅ΠΉ, Π½Π° ΠΊΠΎΡ‚ΠΎΡ€Ρ‹Ρ… Π»ΠΈΠ½ΠΈΠΈ ΠΌΠ°Π³Π½ΠΈΡ‚Π½ΠΎΠ³ΠΎ поля Π·Π°ΠΊΠ°Π½Ρ‡ΠΈΠ²Π°Π»ΠΈΡΡŒ Π±Ρ‹ (см. Π Π°Π·Π΄Π΅Π» Β«ΠœΠ°Π³Π½ΠΈΡ‚Π½Ρ‹Π΅ поля ΠΈ Π»ΠΈΠ½ΠΈΠΈΒ»).

3. Π—Π°ΠΊΠΎΠ½ ЀарадСя

Π˜Π·ΠΌΠ΅Π½ΡΡŽΡ‰Π΅Π΅ΡΡ ΠΌΠ°Π³Π½ΠΈΡ‚Π½ΠΎΠ΅ ΠΏΠΎΠ»Π΅ ΠΈΠ½Π΄ΡƒΡ†ΠΈΡ€ΡƒΠ΅Ρ‚ ΡΠ»Π΅ΠΊΡ‚Ρ€ΠΎΠ΄Π²ΠΈΠΆΡƒΡ‰ΡƒΡŽ силу (Π­Π”Π‘) ΠΈ, ΡΠ»Π΅Π΄ΠΎΠ²Π°Ρ‚Π΅Π»ΡŒΠ½ΠΎ, элСктричСскоС ΠΏΠΎΠ»Π΅.НаправлСниС Π­Π”Π‘ противодСйствуСт измСнСнию. Π£Ρ€Π°Π²Π½Π΅Π½ΠΈΠ΅ \ ref {eq3} являСтся Π·Π°ΠΊΠΎΠ½ΠΎΠΌ ΠΈΠ½Π΄ΡƒΠΊΡ†ΠΈΠΈ ЀарадСя ΠΈ Π²ΠΊΠ»ΡŽΡ‡Π°Π΅Ρ‚ Π·Π°ΠΊΠΎΠ½ Π›Π΅Π½Ρ†Π°. ЭлСктричСскоС ΠΏΠΎΠ»Π΅ ΠΎΡ‚ ΠΈΠ·ΠΌΠ΅Π½ΡΡŽΡ‰Π΅Π³ΠΎΡΡ ΠΌΠ°Π³Π½ΠΈΡ‚Π½ΠΎΠ³ΠΎ поля ΠΈΠΌΠ΅Π΅Ρ‚ силовыС Π»ΠΈΠ½ΠΈΠΈ, ΠΊΠΎΡ‚ΠΎΡ€Ρ‹Π΅ ΠΎΠ±Ρ€Π°Π·ΡƒΡŽΡ‚ Π·Π°ΠΌΠΊΠ½ΡƒΡ‚Ρ‹Π΅ ΠΊΠΎΠ½Ρ‚ΡƒΡ€Ρ‹ Π±Π΅Π· Π½Π°Ρ‡Π°Π»Π° ΠΈ ΠΊΠΎΠ½Ρ†Π°.

4. Π—Π°ΠΊΠΎΠ½ АмпСра-МаксвСлла

ΠœΠ°Π³Π½ΠΈΡ‚Π½Ρ‹Π΅ поля ΡΠΎΠ·Π΄Π°ΡŽΡ‚ΡΡ двиТущимися зарядами ΠΈΠ»ΠΈ ΠΈΠ·ΠΌΠ΅Π½ΡΡŽΡ‰ΠΈΠΌΠΈΡΡ элСктричСскими полями. Π§Π΅Ρ‚Π²Π΅Ρ€Ρ‚ΠΎΠ΅ ΠΈΠ· ΡƒΡ€Π°Π²Π½Π΅Π½ΠΈΠΉ МаксвСлла, Equation \ ref {eq4}, Π²ΠΊΠ»ΡŽΡ‡Π°Π΅Ρ‚ Π·Π°ΠΊΠΎΠ½ АмпСра ΠΈ добавляСт Π΅Ρ‰Π΅ ΠΎΠ΄ΠΈΠ½ источник ΠΌΠ°Π³Π½ΠΈΡ‚Π½Ρ‹Ρ… ΠΏΠΎΠ»Π΅ΠΉ, Π° ΠΈΠΌΠ΅Π½Π½ΠΎ ΠΈΠ·ΠΌΠ΅Π½ΡΡŽΡ‰ΠΈΠ΅ΡΡ элСктричСскиС поля.

УравнСния МаксвСлла ΠΈ Π·Π°ΠΊΠΎΠ½ силы Π›ΠΎΡ€Π΅Π½Ρ†Π° вмСстС ΠΎΡ…Π²Π°Ρ‚Ρ‹Π²Π°ΡŽΡ‚ всС Π·Π°ΠΊΠΎΠ½Ρ‹ элСктричСства ΠΈ ΠΌΠ°Π³Π½Π΅Ρ‚ΠΈΠ·ΠΌΠ°. БиммСтрия, ΠΊΠΎΡ‚ΠΎΡ€ΡƒΡŽ МаксвСлл Π²Π²Π΅Π» Π² свою ΠΌΠ°Ρ‚Π΅ΠΌΠ°Ρ‚ΠΈΡ‡Π΅ΡΠΊΡƒΡŽ структуру, ΠΌΠΎΠΆΠ΅Ρ‚ Π±Ρ‹Ρ‚ΡŒ Π½Π΅ сразу ΠΎΡ‡Π΅Π²ΠΈΠ΄Π½Π°. Π—Π°ΠΊΠΎΠ½ ЀарадСя описываСт, ΠΊΠ°ΠΊ ΠΈΠ·ΠΌΠ΅Π½ΡΡŽΡ‰ΠΈΠ΅ΡΡ ΠΌΠ°Π³Π½ΠΈΡ‚Π½Ρ‹Π΅ поля ΡΠΎΠ·Π΄Π°ΡŽΡ‚ элСктричСскиС поля. Π’ΠΎΠΊ смСщСния, Π²Π²Π΅Π΄Π΅Π½Π½Ρ‹ΠΉ МаксвСллом, являСтся Ρ€Π΅Π·ΡƒΠ»ΡŒΡ‚Π°Ρ‚ΠΎΠΌ измСнСния элСктричСского поля ΠΈ ΡƒΡ‡ΠΈΡ‚Ρ‹Π²Π°Π΅Ρ‚ ΠΈΠ·ΠΌΠ΅Π½ΡΡŽΡ‰Π΅Π΅ΡΡ элСктричСскоС ΠΏΠΎΠ»Π΅, ΡΠΎΠ·Π΄Π°ΡŽΡ‰Π΅Π΅ ΠΌΠ°Π³Π½ΠΈΡ‚Π½ΠΎΠ΅ ΠΏΠΎΠ»Π΅. УравнСния для эффСктов ΠΊΠ°ΠΊ ΠΈΠ·ΠΌΠ΅Π½ΡΡŽΡ‰ΠΈΡ…ΡΡ элСктричСских ΠΏΠΎΠ»Π΅ΠΉ, Ρ‚Π°ΠΊ ΠΈ ΠΈΠ·ΠΌΠ΅Π½ΡΡŽΡ‰ΠΈΡ…ΡΡ ΠΌΠ°Π³Π½ΠΈΡ‚Π½Ρ‹Ρ… ΠΏΠΎΠ»Π΅ΠΉ ΠΎΡ‚Π»ΠΈΡ‡Π°ΡŽΡ‚ΡΡ ΠΏΠΎ Ρ„ΠΎΡ€ΠΌΠ΅ Ρ‚ΠΎΠ»ΡŒΠΊΠΎ Ρ‚Π°ΠΌ, Π³Π΄Π΅ отсутствиС ΠΌΠ°Π³Π½ΠΈΡ‚Π½Ρ‹Ρ… ΠΌΠΎΠ½ΠΎΠΏΠΎΠ»Π΅ΠΉ ΠΏΡ€ΠΈΠ²ΠΎΠ΄ΠΈΡ‚ ΠΊ ΠΎΡ‚ΡΡƒΡ‚ΡΡ‚Π²ΠΈΡŽ Ρ‡Π»Π΅Π½ΠΎΠ².Π­Ρ‚Π° симмСтрия ΠΌΠ΅ΠΆΠ΄Ρƒ эффСктами измСнСния ΠΌΠ°Π³Π½ΠΈΡ‚Π½ΠΎΠ³ΠΎ ΠΈ элСктричСского ΠΏΠΎΠ»Π΅ΠΉ Π²Π°ΠΆΠ½Π° для объяснСния ΠΏΡ€ΠΈΡ€ΠΎΠ΄Ρ‹ элСктромагнитных Π²ΠΎΠ»Π½.

Π‘ΠΎΠ»Π΅Π΅ ΠΏΠΎΠ·Π΄Π½Π΅Π΅ ΠΏΡ€ΠΈΠΌΠ΅Π½Π΅Π½ΠΈΠ΅ Ρ‚Π΅ΠΎΡ€ΠΈΠΈ ΠΎΡ‚Π½ΠΎΡΠΈΡ‚Π΅Π»ΡŒΠ½ΠΎΡΡ‚ΠΈ Π­ΠΉΠ½ΡˆΡ‚Π΅ΠΉΠ½Π° ΠΊ ΠΏΠΎΠ»Π½ΠΎΠΉ ΠΈ симмСтричной Ρ‚Π΅ΠΎΡ€ΠΈΠΈ МаксвСлла ΠΏΠΎΠΊΠ°Π·Π°Π»ΠΎ, Ρ‡Ρ‚ΠΎ элСктричСскиС ΠΈ ΠΌΠ°Π³Π½ΠΈΡ‚Π½Ρ‹Π΅ силы Π½Π΅ Ρ€Π°Π·Π΄Π΅Π»Π΅Π½Ρ‹, Π° ΡΠ²Π»ΡΡŽΡ‚ΡΡ Ρ€Π°Π·Π½Ρ‹ΠΌΠΈ проявлСниями ΠΎΠ΄Π½ΠΎΠΉ ΠΈ Ρ‚ΠΎΠΉ ΠΆΠ΅ Π²Π΅Ρ‰ΠΈ — элСктромагнитной силы. ЭлСктромагнитная сила ΠΈ слабая ядСрная сила Ρ‚Π°ΠΊ ΠΆΠ΅ ΠΎΠ±ΡŠΠ΅Π΄ΠΈΠ½Π΅Π½Ρ‹, ΠΊΠ°ΠΊ ΠΈ элСктрослабая сила. Π­Ρ‚ΠΎ объСдинСниС сил Π±Ρ‹Π»ΠΎ ΠΎΠ΄Π½ΠΎΠΉ ΠΈΠ· ΠΏΡ€ΠΈΡ‡ΠΈΠ½ ΠΏΠΎΠΏΡ‹Ρ‚ΠΎΠΊ ΠΎΠ±ΡŠΠ΅Π΄ΠΈΠ½ΠΈΡ‚ΡŒ всС Ρ‡Π΅Ρ‚Ρ‹Ρ€Π΅ основныС силы Π² ΠΏΡ€ΠΈΡ€ΠΎΠ΄Π΅ — Π³Ρ€Π°Π²ΠΈΡ‚Π°Ρ†ΠΈΠΎΠ½Π½ΠΎΠ΅, элСктричСскоС, сильноС ΠΈ слабоС ядСрныС взаимодСйствия (см. Π€ΠΈΠ·ΠΈΠΊΠ° элСмСнтарных частиц ΠΈ космология).

ΠœΠ΅Ρ…Π°Π½ΠΈΠ·ΠΌ распространСния элСктромагнитных Π²ΠΎΠ»Π½

Π§Ρ‚ΠΎΠ±Ρ‹ ΡƒΠ²ΠΈΠ΄Π΅Ρ‚ΡŒ, ΠΊΠ°ΠΊ симмСтрия, ввСдСнная МаксвСллом, ΠΎΠ±ΡŠΡΡΠ½ΡΠ΅Ρ‚ сущСствованиС ΠΊΠΎΠΌΠ±ΠΈΠ½ΠΈΡ€ΠΎΠ²Π°Π½Π½Ρ‹Ρ… элСктричСских ΠΈ ΠΌΠ°Π³Π½ΠΈΡ‚Π½Ρ‹Ρ… Π²ΠΎΠ»Π½, Ρ€Π°ΡΠΏΡ€ΠΎΡΡ‚Ρ€Π°Π½ΡΡŽΡ‰ΠΈΡ…ΡΡ Π² пространствС, ΠΏΡ€Π΅Π΄ΡΡ‚Π°Π²ΡŒΡ‚Π΅ ΠΈΠ·ΠΌΠ΅Π½ΡΡŽΡ‰Π΅Π΅ΡΡ Π²ΠΎ Π²Ρ€Π΅ΠΌΠ΅Π½ΠΈ ΠΌΠ°Π³Π½ΠΈΡ‚Π½ΠΎΠ΅ ΠΏΠΎΠ»Π΅ \ (\ vec {B} _0 (t) \), создаваСмоС высокочастотным ΠΏΠ΅Ρ€Π΅ΠΌΠ΅Π½Π½Ρ‹ΠΉ Ρ‚ΠΎΠΊ ΠΏΠΎΠΊΠ°Π·Π°Π½ Π½Π° рисункС \ (\ PageIndex {3} \). ΠœΡ‹ прСдставляСм \ (\ vec {B} _0 (t) \) Π½Π° Π΄ΠΈΠ°Π³Ρ€Π°ΠΌΠΌΠ΅ ΠΎΠ΄Π½ΠΎΠΉ ΠΈΠ· Π΅Π³ΠΎ силовых Π»ΠΈΠ½ΠΈΠΉ. Богласно Π·Π°ΠΊΠΎΠ½Ρƒ ЀарадСя, ΠΈΠ·ΠΌΠ΅Π½ΡΡŽΡ‰Π΅Π΅ΡΡ ΠΌΠ°Π³Π½ΠΈΡ‚Π½ΠΎΠ΅ ΠΏΠΎΠ»Π΅ Ρ‡Π΅Ρ€Π΅Π· ΠΏΠΎΠ²Π΅Ρ€Ρ…Π½ΠΎΡΡ‚ΡŒ ΠΈΠ½Π΄ΡƒΡ†ΠΈΡ€ΡƒΠ΅Ρ‚ ΠΈΠ·ΠΌΠ΅Π½ΡΡŽΡ‰Π΅Π΅ΡΡ Π²ΠΎ Π²Ρ€Π΅ΠΌΠ΅Π½ΠΈ элСктричСскоС ΠΏΠΎΠ»Π΅ \ (\ vec {E} _0 (t) \) Π½Π° Π³Ρ€Π°Π½ΠΈΡ†Π΅ этой повСрхности.Π˜ΡΡ‚ΠΎΡ‡Π½ΠΈΠΊ Ρ‚ΠΎΠΊΠ° смСщСния для элСктричСского поля, ΠΊΠ°ΠΊ ΠΈ источник Π·Π°ΠΊΠΎΠ½Π° ЀарадСя для ΠΌΠ°Π³Π½ΠΈΡ‚Π½ΠΎΠ³ΠΎ поля, создаСт Ρ‚ΠΎΠ»ΡŒΠΊΠΎ Π·Π°ΠΌΠΊΠ½ΡƒΡ‚Ρ‹Π΅ ΠΊΠΎΠ½Ρ‚ΡƒΡ€Ρ‹ силовых Π»ΠΈΠ½ΠΈΠΉ ΠΈΠ·-Π·Π° матСматичСской симмСтрии, Π²ΠΊΠ»ΡŽΡ‡Π΅Π½Π½ΠΎΠΉ Π² уравнСния для ΠΈΠ½Π΄ΡƒΡ†ΠΈΡ€ΠΎΠ²Π°Π½Π½ΠΎΠ³ΠΎ элСктричСского ΠΈ ΠΈΠ½Π΄ΡƒΡ†ΠΈΡ€ΠΎΠ²Π°Π½Π½ΠΎΠ³ΠΎ ΠΌΠ°Π³Π½ΠΈΡ‚Π½ΠΎΠ³ΠΎ ΠΏΠΎΠ»Π΅ΠΉ. Показано Π»ΠΈΠ½Π΅ΠΉΠ½ΠΎΠ΅ прСдставлСниС \ (\ vec {E} _0 (t) \). Π’ свою ΠΎΡ‡Π΅Ρ€Π΅Π΄ΡŒ, ΠΈΠ·ΠΌΠ΅Π½ΡΡŽΡ‰Π΅Π΅ΡΡ элСктричСскоС ΠΏΠΎΠ»Π΅ \ (\ vec {E} _0 (t) \) создаСт ΠΌΠ°Π³Π½ΠΈΡ‚Π½ΠΎΠ΅ ΠΏΠΎΠ»Π΅ \ (\ vec {B} _1 (t) \) согласно ΠΌΠΎΠ΄ΠΈΡ„ΠΈΡ†ΠΈΡ€ΠΎΠ²Π°Π½Π½ΠΎΠΌΡƒ Π·Π°ΠΊΠΎΠ½Ρƒ АмпСра. Π­Ρ‚ΠΎ ΠΈΠ·ΠΌΠ΅Π½ΡΡŽΡ‰Π΅Π΅ΡΡ ΠΏΠΎΠ»Π΅ ΠΈΠ½Π΄ΡƒΡ†ΠΈΡ€ΡƒΠ΅Ρ‚ \ (\ vec {E} _1 (t) \), ΠΊΠΎΡ‚ΠΎΡ€ΠΎΠ΅ ΠΈΠ½Π΄ΡƒΡ†ΠΈΡ€ΡƒΠ΅Ρ‚ \ (\ vec {B} _2 (t) \) ΠΈ Ρ‚Π°ΠΊ Π΄Π°Π»Π΅Π΅.Π—Π°Ρ‚Π΅ΠΌ Ρƒ нас Π΅ΡΡ‚ΡŒ ΡΠ°ΠΌΠΎΠΏΡ€ΠΎΠ΄ΠΎΠ»ΠΆΠ°ΡŽΡ‰ΠΈΠΉΡΡ процСсс, ΠΊΠΎΡ‚ΠΎΡ€Ρ‹ΠΉ ΠΏΡ€ΠΈΠ²ΠΎΠ΄ΠΈΡ‚ ΠΊ созданию ΠΈΠ·ΠΌΠ΅Π½ΡΡŽΡ‰ΠΈΡ…ΡΡ Π²ΠΎ Π²Ρ€Π΅ΠΌΠ΅Π½ΠΈ элСктричСских ΠΈ ΠΌΠ°Π³Π½ΠΈΡ‚Π½Ρ‹Ρ… ΠΏΠΎΠ»Π΅ΠΉ Π² областях всС дальшС ΠΈ дальшС ΠΎΡ‚ O . Π­Ρ‚ΠΎΡ‚ процСсс ΠΌΠΎΠΆΠ½ΠΎ ΠΏΡ€Π΅Π΄ΡΡ‚Π°Π²ΠΈΡ‚ΡŒ сСбС ΠΊΠ°ΠΊ распространСниС элСктромагнитной Π²ΠΎΠ»Π½Ρ‹ Π² пространствС.

Рисунок \ (\ PageIndex {3} \): Как ΠΈΠ·ΠΌΠ΅Π½ΡΡŽΡ‰ΠΈΠ΅ΡΡ поля \ (\ vec {E} \) ΠΈ \ (\ vec {B} \) Ρ€Π°ΡΠΏΡ€ΠΎΡΡ‚Ρ€Π°Π½ΡΡŽΡ‚ΡΡ Π² пространствС.

Π’ ΡΠ»Π΅Π΄ΡƒΡŽΡ‰Π΅ΠΌ Ρ€Π°Π·Π΄Π΅Π»Π΅ ΠΌΡ‹ ΠΏΠΎΠΊΠ°ΠΆΠ΅ΠΌ Π² Π±ΠΎΠ»Π΅Π΅ Ρ‚ΠΎΡ‡Π½Ρ‹Ρ… матСматичСских Ρ‚Π΅Ρ€ΠΌΠΈΠ½Π°Ρ…, ΠΊΠ°ΠΊ уравнСния МаксвСлла приводят ΠΊ ΠΏΡ€Π΅Π΄ΡΠΊΠ°Π·Π°Π½ΠΈΡŽ элСктромагнитных Π²ΠΎΠ»Π½, ΠΊΠΎΡ‚ΠΎΡ€Ρ‹Π΅ ΠΌΠΎΠ³ΡƒΡ‚ ΠΏΠ΅Ρ€Π΅ΠΌΠ΅Ρ‰Π°Ρ‚ΡŒΡΡ Π² пространствС Π±Π΅Π· ΠΌΠ°Ρ‚Π΅Ρ€ΠΈΠ°Π»ΡŒΠ½ΠΎΠΉ срСды, подразумСвая ΡΠΊΠΎΡ€ΠΎΡΡ‚ΡŒ элСктромагнитных Π²ΠΎΠ»Π½, Ρ€Π°Π²Π½ΡƒΡŽ скорости свСта.

Π”ΠΎ Ρ€Π°Π±ΠΎΡ‚Ρ‹ МаксвСлла экспСримСнты ΡƒΠΆΠ΅ ΠΏΠΎΠΊΠ°Π·Π°Π»ΠΈ, Ρ‡Ρ‚ΠΎ свСт — это Π²ΠΎΠ»Π½ΠΎΠ²ΠΎΠ΅ явлСниС, хотя ΠΏΡ€ΠΈΡ€ΠΎΠ΄Π° Π²ΠΎΠ»Π½ Π΅Ρ‰Π΅ Π±Ρ‹Π»Π° нСизвСстна. Π’ 1801 Π³ΠΎΠ΄Ρƒ Вомас Π―Π½Π³ (1773–1829) ΠΏΠΎΠΊΠ°Π·Π°Π», Ρ‡Ρ‚ΠΎ ΠΊΠΎΠ³Π΄Π° свСтовой Π»ΡƒΡ‡ раздСлялся двумя ΡƒΠ·ΠΊΠΈΠΌΠΈ щСлями ΠΈ Π·Π°Ρ‚Π΅ΠΌ снова объСдинялся, Π½Π° экранС формировался ΡƒΠ·ΠΎΡ€, состоящий ΠΈΠ· ярких ΠΈ Ρ‚Π΅ΠΌΠ½Ρ‹Ρ… полос. Π―Π½Π³ объяснил это ΠΏΠΎΠ²Π΅Π΄Π΅Π½ΠΈΠ΅, ΠΏΡ€Π΅Π΄ΠΏΠΎΠ»ΠΎΠΆΠΈΠ², Ρ‡Ρ‚ΠΎ свСт состоит ΠΈΠ· Π²ΠΎΠ»Π½, ΠΊΠΎΡ‚ΠΎΡ€Ρ‹Π΅ конструктивно ΡΠΊΠ»Π°Π΄Ρ‹Π²Π°ΡŽΡ‚ΡΡ Π² ΠΎΠ΄Π½ΠΈΡ… Ρ‚ΠΎΡ‡ΠΊΠ°Ρ… ΠΈ Ρ€Π°Π·Ρ€ΡƒΡˆΠΈΡ‚Π΅Π»ΡŒΠ½ΠΎ Π² Π΄Ρ€ΡƒΠ³ΠΈΡ… (см. Π˜Π½Ρ‚Π΅Ρ€Ρ„Π΅Ρ€Π΅Π½Ρ†ΠΈΡ).ВпослСдствии Π–Π°Π½ Π€ΡƒΠΊΠΎ (1819–1868) с измСрСниями скорости свСта Π² Ρ€Π°Π·Π»ΠΈΡ‡Π½Ρ‹Ρ… срСдах ΠΈ ΠžΠ³ΡŽΡΡ‚Π΅Π½ Π€Ρ€Π΅Π½Π΅Π»ΡŒ (1788–1827) с ΠΏΠΎΠ΄Ρ€ΠΎΠ±Π½Ρ‹ΠΌΠΈ экспСримСнтами, Π²ΠΊΠ»ΡŽΡ‡Π°ΡŽΡ‰ΠΈΠΌΠΈ ΠΈΠ½Ρ‚Π΅Ρ€Ρ„Π΅Ρ€Π΅Π½Ρ†ΠΈΡŽ ΠΈ Π΄ΠΈΡ„Ρ€Π°ΠΊΡ†ΠΈΡŽ свСта, прСдоставили Π΄ΠΎΠΏΠΎΠ»Π½ΠΈΡ‚Π΅Π»ΡŒΠ½Ρ‹Π΅ ΡƒΠ±Π΅Π΄ΠΈΡ‚Π΅Π»ΡŒΠ½Ρ‹Π΅ Π΄ΠΎΠΊΠ°Π·Π°Ρ‚Π΅Π»ΡŒΡΡ‚Π²Π° Ρ‚ΠΎΠ³ΠΎ, Ρ‡Ρ‚ΠΎ свСт Π±Ρ‹Π» Π²ΠΎΠ»Π½ΠΎΠΉ. Π˜Ρ‚Π°ΠΊ, свСт Π±Ρ‹Π» извСстСн ΠΊΠ°ΠΊ Π²ΠΎΠ»Π½Π°, ΠΈ МаксвСлл прСдсказал сущСствованиС элСктромагнитных Π²ΠΎΠ»Π½, ΠΊΠΎΡ‚ΠΎΡ€Ρ‹Π΅ Ρ€Π°ΡΠΏΡ€ΠΎΡΡ‚Ρ€Π°Π½ΡΡŽΡ‚ΡΡ со ΡΠΊΠΎΡ€ΠΎΡΡ‚ΡŒΡŽ свСта. Π’Ρ‹Π²ΠΎΠ΄ казался Π½Π΅ΠΈΠ·Π±Π΅ΠΆΠ½Ρ‹ΠΌ: свСт Π΄ΠΎΠ»ΠΆΠ΅Π½ Π±Ρ‹Ρ‚ΡŒ Ρ„ΠΎΡ€ΠΌΠΎΠΉ элСктромагнитного излучСния. Но тСория МаксвСлла ΠΏΠΎΠΊΠ°Π·Π°Π»Π°, Ρ‡Ρ‚ΠΎ для элСктромагнитных Π²ΠΎΠ»Π½ Π²ΠΎΠ·ΠΌΠΎΠΆΠ½Ρ‹ Π΄Ρ€ΡƒΠ³ΠΈΠ΅ Π΄Π»ΠΈΠ½Ρ‹ Π²ΠΎΠ»Π½ ΠΈ частоты, ΠΎΡ‚Π»ΠΈΡ‡Π½Ρ‹Π΅ ΠΎΡ‚ свСта.Он ΠΏΠΎΠΊΠ°Π·Π°Π», Ρ‡Ρ‚ΠΎ элСктромагнитноС ΠΈΠ·Π»ΡƒΡ‡Π΅Π½ΠΈΠ΅ с Ρ‚Π΅ΠΌΠΈ ΠΆΠ΅ Ρ„ΡƒΠ½Π΄Π°ΠΌΠ΅Π½Ρ‚Π°Π»ΡŒΠ½Ρ‹ΠΌΠΈ свойствами, Ρ‡Ρ‚ΠΎ ΠΈ Π²ΠΈΠ΄ΠΈΠΌΡ‹ΠΉ свСт, Π΄ΠΎΠ»ΠΆΠ½ΠΎ ΡΡƒΡ‰Π΅ΡΡ‚Π²ΠΎΠ²Π°Ρ‚ΡŒ Π½Π° любой частотС. ΠžΡΡ‚Π°Π»ΡŒΠ½Ρ‹ΠΌ ΠΎΡΡ‚Π°Π²Π°Π»ΠΎΡΡŒ ΠΏΡ€ΠΎΠ²Π΅Ρ€ΠΈΡ‚ΡŒ ΠΈ ΠΏΠΎΠ΄Ρ‚Π²Π΅Ρ€Π΄ΠΈΡ‚ΡŒ это прСдсказаниС.

Π£ΠΏΡ€Π°ΠΆΠ½Π΅Π½ΠΈΠ΅ \ (\ PageIndex {1} \)

Когда Π­Π”Π‘ Π½Π° кондСнсаторС Π²ΠΊΠ»ΡŽΡ‡Π΅Π½Π° ΠΈ кондСнсатору ΠΏΠΎΠ·Π²ΠΎΠ»ΡΡŽΡ‚ Π·Π°Ρ€ΡΠΆΠ°Ρ‚ΡŒΡΡ, ΠΊΠΎΠ³Π΄Π° ΠΌΠ°Π³Π½ΠΈΡ‚Π½ΠΎΠ΅ ΠΏΠΎΠ»Π΅, ΠΈΠ½Π΄ΡƒΡ†ΠΈΡ€ΠΎΠ²Π°Π½Π½ΠΎΠ΅ Ρ‚ΠΎΠΊΠΎΠΌ смСщСния, ΠΈΠΌΠ΅Π΅Ρ‚ Π½Π°ΠΈΠ±ΠΎΠ»ΡŒΡˆΡƒΡŽ Π²Π΅Π»ΠΈΡ‡ΠΈΠ½Ρƒ?

Максимально сразу послС Π²ΠΊΠ»ΡŽΡ‡Π΅Π½ΠΈΡ Ρ‚ΠΎΠΊΠ°.Π’ΠΎΠΊ смСщСния ΠΈ создаваСмоС ΠΈΠΌ ΠΌΠ°Π³Π½ΠΈΡ‚Π½ΠΎΠ΅ ΠΏΠΎΠ»Π΅ ΠΏΡ€ΠΎΠΏΠΎΡ€Ρ†ΠΈΠΎΠ½Π°Π»ΡŒΠ½Ρ‹ скорости измСнСния элСктричСского поля ΠΌΠ΅ΠΆΠ΄Ρƒ пластинами, которая максимальна, ΠΊΠΎΠ³Π΄Π° пластины Π²ΠΏΠ΅Ρ€Π²Ρ‹Π΅ Π½Π°Ρ‡ΠΈΠ½Π°ΡŽΡ‚ Π·Π°Ρ€ΡΠΆΠ°Ρ‚ΡŒΡΡ.

НаблюдСния Π“Π΅Ρ€Ρ†Π°

НСмСцкий Ρ„ΠΈΠ·ΠΈΠΊ Π“Π΅Π½Ρ€ΠΈΡ… Π“Π΅Ρ€Ρ† (1857–1894) Π±Ρ‹Π» ΠΏΠ΅Ρ€Π²Ρ‹ΠΌ, ΠΊΡ‚ΠΎ Π³Π΅Π½Π΅Ρ€ΠΈΡ€ΠΎΠ²Π°Π» ΠΈ ΠΎΠ±Π½Π°Ρ€ΡƒΠΆΠΈΠ²Π°Π» ΠΎΠΏΡ€Π΅Π΄Π΅Π»Π΅Π½Π½Ρ‹Π΅ Ρ‚ΠΈΠΏΡ‹ элСктромагнитных Π²ΠΎΠ»Π½ Π² Π»Π°Π±ΠΎΡ€Π°Ρ‚ΠΎΡ€ΠΈΠΈ. Начиная с 1887 Π³ΠΎΠ΄Π°, ΠΎΠ½ ΠΏΡ€ΠΎΠ²Π΅Π» ΡΠ΅Ρ€ΠΈΡŽ экспСримСнтов, ΠΊΠΎΡ‚ΠΎΡ€Ρ‹Π΅ Π½Π΅ Ρ‚ΠΎΠ»ΡŒΠΊΠΎ ΠΏΠΎΠ΄Ρ‚Π²Π΅Ρ€Π΄ΠΈΠ»ΠΈ сущСствованиС элСктромагнитных Π²ΠΎΠ»Π½, Π½ΠΎ ΠΈ ΠΏΠΎΠ΄Ρ‚Π²Π΅Ρ€Π΄ΠΈΠ»ΠΈ, Ρ‡Ρ‚ΠΎ ΠΎΠ½ΠΈ двиТутся со ΡΠΊΠΎΡ€ΠΎΡΡ‚ΡŒΡŽ свСта.

Π“Π΅Ρ€Ρ† использовал схСму ΠΏΠ΅Ρ€Π΅ΠΌΠ΅Π½Π½ΠΎΠ³ΠΎ Ρ‚ΠΎΠΊΠ° RLC (рСзистор-ΠΈΠ½Π΄ΡƒΠΊΡ‚ΠΎΡ€-кондСнсатор), которая Ρ€Π΅Π·ΠΎΠ½ΠΈΡ€ΡƒΠ΅Ρ‚ Π½Π° извСстной частотС \ (f_0 = \ dfrac {1} {2 \ pi \ sqrt {LC}} \), ΠΈ ΠΏΠΎΠ΄ΠΊΠ»ΡŽΡ‡ΠΈΠ» Π΅Π΅ ΠΊ пСтля ΠΏΡ€ΠΎΠ²ΠΎΠ΄Π°, ΠΊΠ°ΠΊ ΠΏΠΎΠΊΠ°Π·Π°Π½ΠΎ Π½Π° рисункС \ (\ PageIndex {4} \). ВысокоС напряТСниС, ΠΈΠ½Π΄ΡƒΡ†ΠΈΡ€ΠΎΠ²Π°Π½Π½ΠΎΠ΅ Ρ‡Π΅Ρ€Π΅Π· Π·Π°Π·ΠΎΡ€ Π² ΠΏΠ΅Ρ‚Π»Π΅, Π²Ρ‹Π·Ρ‹Π²Π°Π»ΠΎ искры, ΠΊΠΎΡ‚ΠΎΡ€Ρ‹Π΅ Π±Ρ‹Π»ΠΈ Π²ΠΈΠ΄ΠΈΠΌΡ‹ΠΌ ΡΠ²ΠΈΠ΄Π΅Ρ‚Π΅Π»ΡŒΡΡ‚Π²ΠΎΠΌ наличия Ρ‚ΠΎΠΊΠ° Π² Ρ†Π΅ΠΏΠΈ ΠΈ ΠΏΠΎΠΌΠΎΠ³Π°Π»ΠΈ Π³Π΅Π½Π΅Ρ€ΠΈΡ€ΠΎΠ²Π°Ρ‚ΡŒ элСктромагнитныС Π²ΠΎΠ»Π½Ρ‹.

Π§Π΅Ρ€Π΅Π· всю Π»Π°Π±ΠΎΡ€Π°Ρ‚ΠΎΡ€ΠΈΡŽ Π“Π΅Ρ€Ρ† размСстил Π΅Ρ‰Π΅ ΠΎΠ΄Π½Ρƒ ΠΏΠ΅Ρ‚Π»ΡŽ, ΠΏΡ€ΠΈΡΠΎΠ΅Π΄ΠΈΠ½Π΅Π½Π½ΡƒΡŽ ΠΊ Π΄Ρ€ΡƒΠ³ΠΎΠΉ схСмС RLC , ΠΊΠΎΡ‚ΠΎΡ€ΡƒΡŽ ΠΌΠΎΠΆΠ½ΠΎ Π±Ρ‹Π»ΠΎ Π½Π°ΡΡ‚Ρ€ΠΎΠΈΡ‚ΡŒ (ΠΊΠ°ΠΊ Ρ†ΠΈΡ„Π΅Ρ€Π±Π»Π°Ρ‚ Π½Π° Ρ€Π°Π΄ΠΈΠΎ) Π½Π° Ρ‚Ρƒ ΠΆΠ΅ Ρ€Π΅Π·ΠΎΠ½Π°Π½ΡΠ½ΡƒΡŽ частоту, Ρ‡Ρ‚ΠΎ ΠΈ пСрвая, ΠΈ, Ρ‚Π°ΠΊΠΈΠΌ ΠΎΠ±Ρ€Π°Π·ΠΎΠΌ, ΠΌΠΎΠΆΠ½ΠΎ Π±Ρ‹Π»ΠΎ Π·Π°ΡΡ‚Π°Π²ΠΈΡ‚ΡŒ Π΅Π΅ ΠΏΡ€ΠΈΠ½ΠΈΠΌΠ°Ρ‚ΡŒ элСктромагнитныС Π²ΠΎΠ»Π½Ρ‹.Π’ этой ΠΏΠ΅Ρ‚Π»Π΅ Ρ‚Π°ΠΊΠΆΠ΅ Π±Ρ‹Π» Π·Π°Π·ΠΎΡ€, Π² ΠΊΠΎΡ‚ΠΎΡ€ΠΎΠΌ Π²ΠΎΠ·Π½ΠΈΠΊΠ°Π»ΠΈ искры, Ρ‡Ρ‚ΠΎ Π΄Π°Π²Π°Π»ΠΎ Ρ‚Π²Π΅Ρ€Π΄ΠΎΠ΅ ΡΠ²ΠΈΠ΄Π΅Ρ‚Π΅Π»ΡŒΡΡ‚Π²ΠΎ ΠΏΡ€ΠΈΠ΅ΠΌΠ° элСктромагнитных Π²ΠΎΠ»Π½.

Рисунок \ (\ PageIndex {4} \): устройство, ΠΊΠΎΡ‚ΠΎΡ€ΠΎΠ΅ Π“Π΅Ρ€Ρ† использовал Π² 1887 Π³ΠΎΠ΄Ρƒ для Π³Π΅Π½Π΅Ρ€Π°Ρ†ΠΈΠΈ ΠΈ обнаруТСния элСктромагнитных Π²ΠΎΠ»Π½.

Π“Π΅Ρ€Ρ† Ρ‚Π°ΠΊΠΆΠ΅ ΠΈΠ·ΡƒΡ‡ΠΈΠ» ΠΊΠ°Ρ€Ρ‚ΠΈΠ½Ρ‹ отраТСния, прСломлСния ΠΈ ΠΈΠ½Ρ‚Π΅Ρ€Ρ„Π΅Ρ€Π΅Π½Ρ†ΠΈΠΈ Π³Π΅Π½Π΅Ρ€ΠΈΡ€ΡƒΠ΅ΠΌΡ‹Ρ… ΠΈΠΌ элСктромагнитных Π²ΠΎΠ»Π½, ΠΏΠΎΠ΄Ρ‚Π²Π΅Ρ€Π΄ΠΈΠ² ΠΈΡ… Π²ΠΎΠ»Π½ΠΎΠ²ΠΎΠΉ Ρ…Π°Ρ€Π°ΠΊΡ‚Π΅Ρ€. Он смог ΠΎΠΏΡ€Π΅Π΄Π΅Π»ΠΈΡ‚ΡŒ Π΄Π»ΠΈΠ½Ρ‹ Π²ΠΎΠ»Π½ ΠΏΠΎ ΠΈΠ½Ρ‚Π΅Ρ€Ρ„Π΅Ρ€Π΅Π½Ρ†ΠΈΠΎΠ½Π½Ρ‹ΠΌ ΠΊΠ°Ρ€Ρ‚ΠΈΠ½Π°ΠΌ ΠΈ, зная ΠΈΡ… частоты, ΠΎΠ½ ΠΌΠΎΠ³ Ρ€Π°ΡΡΡ‡ΠΈΡ‚Π°Ρ‚ΡŒ ΡΠΊΠΎΡ€ΠΎΡΡ‚ΡŒ распространСния, ΠΈΡΠΏΠΎΠ»ΡŒΠ·ΡƒΡ ΡƒΡ€Π°Π²Π½Π΅Π½ΠΈΠ΅ \ (v = f \ lambda \), Π³Π΄Π΅ v — ΡΠΊΠΎΡ€ΠΎΡΡ‚ΡŒ Π²ΠΎΠ»Π½Ρ‹, f — Π΅Π³ΠΎ частота, Π° \ (\ lambda \) — Π΅Π³ΠΎ Π΄Π»ΠΈΠ½Π° Π²ΠΎΠ»Π½Ρ‹.Π’Π°ΠΊΠΈΠΌ ΠΎΠ±Ρ€Π°Π·ΠΎΠΌ, Π“Π΅Ρ€Ρ† смог Π΄ΠΎΠΊΠ°Π·Π°Ρ‚ΡŒ, Ρ‡Ρ‚ΠΎ элСктромагнитныС Π²ΠΎΠ»Π½Ρ‹ Ρ€Π°ΡΠΏΡ€ΠΎΡΡ‚Ρ€Π°Π½ΡΡŽΡ‚ΡΡ со ΡΠΊΠΎΡ€ΠΎΡΡ‚ΡŒΡŽ свСта. Π•Π΄ΠΈΠ½ΠΈΡ†Π° измСрСния частоты Π² систСмС БИ, Π³Π΅Ρ€Ρ† \ ((1 \, Π“Ρ† = 1 \, Ρ†ΠΈΠΊΠ» / сСкунда) \), Π½Π°Π·Π²Π°Π½Π° Π² Π΅Π³ΠΎ Ρ‡Π΅ΡΡ‚ΡŒ.

Π£ΠΏΡ€Π°ΠΆΠ½Π΅Π½ΠΈΠ΅ \ (\ PageIndex {2} \)

ΠœΠΎΠΆΠ΅Ρ‚ Π»ΠΈ чисто элСктричСскоС ΠΏΠΎΠ»Π΅ Ρ€Π°ΡΠΏΡ€ΠΎΡΡ‚Ρ€Π°Π½ΡΡ‚ΡŒΡΡ ΠΊΠ°ΠΊ Π²ΠΎΠ»Π½Π° Ρ‡Π΅Ρ€Π΅Π· Π²Π°ΠΊΡƒΡƒΠΌ Π±Π΅Π· ΠΌΠ°Π³Π½ΠΈΡ‚Π½ΠΎΠ³ΠΎ поля? ΠžΠ±ΠΎΡΠ½ΠΎΠ²Π°Ρ‚ΡŒ ΠΎΡ‚Π²Π΅Ρ‚.

НСт. Π˜Π·ΠΌΠ΅Π½ΡΡŽΡ‰Π΅Π΅ΡΡ элСктричСскоС ΠΏΠΎΠ»Π΅ Π² соотвСтствии с ΠΌΠΎΠ΄ΠΈΡ„ΠΈΡ†ΠΈΡ€ΠΎΠ²Π°Π½Π½ΠΎΠΉ вСрсиСй Π·Π°ΠΊΠΎΠ½Π° АмпСра ΠΎΠ±ΡΠ·Π°Ρ‚Π΅Π»ΡŒΠ½ΠΎ Π²Ρ‹Π·ΠΎΠ²Π΅Ρ‚ ΠΈΠ·ΠΌΠ΅Π½ΡΡŽΡ‰Π΅Π΅ΡΡ ΠΌΠ°Π³Π½ΠΈΡ‚Π½ΠΎΠ΅ ΠΏΠΎΠ»Π΅.

Авторы ΠΈ авторство

  • Бэмюэл Π”ΠΆ. Π›ΠΈΠ½Π³ (ГосударствСнный унивСрситСт Врумэна), Π”ΠΆΠ΅Ρ„Ρ„ Π‘Π°Π½Π½ΠΈ (УнивСрситСт Π›ΠΎΠΉΠΎΠ»Π° ΠœΡΡ€ΠΈΠΌΠ°ΡƒΠ½Ρ‚) ΠΈ Π‘ΠΈΠ»Π» Мобс со ΠΌΠ½ΠΎΠ³ΠΈΠΌΠΈ Π°Π²Ρ‚ΠΎΡ€Π°ΠΌΠΈ. Π­Ρ‚Π° Ρ€Π°Π±ΠΎΡ‚Π° Π»ΠΈΡ†Π΅Π½Π·ΠΈΡ€ΠΎΠ²Π°Π½Π° OpenStax University Physics Π² соотвСтствии с Π»ΠΈΡ†Π΅Π½Π·ΠΈΠ΅ΠΉ Creative Commons Attribution License (4.

alexxlab

Π”ΠΎΠ±Π°Π²ΠΈΡ‚ΡŒ ΠΊΠΎΠΌΠΌΠ΅Π½Ρ‚Π°Ρ€ΠΈΠΉ

Π’Π°Ρˆ адрСс email Π½Π΅ Π±ΡƒΠ΄Π΅Ρ‚ ΠΎΠΏΡƒΠ±Π»ΠΈΠΊΠΎΠ²Π°Π½. ΠžΠ±ΡΠ·Π°Ρ‚Π΅Π»ΡŒΠ½Ρ‹Π΅ поля ΠΏΠΎΠΌΠ΅Ρ‡Π΅Π½Ρ‹ *